Download as pdf or txt
Download as pdf or txt
You are on page 1of 303

2023

FINANCIAL ACCOUNTING AND


REPORTING - I
F ICAP ATTEMPTWISE PAST PAPERS
(Updated with Spring-2023 QP)
By the Grace of Almighty Allah, I am pleased to present the attemptwise past
papers of Financial Accounting & Reporting-I for CAF-01.

A R This volume contains;


 ICAP papers of last 18 attempts.
 Suggested Answers by ICAP (Autumn14-Autumn22)
 Examiner Comments (Autumn14-Autumn22)
 Marking plan (Autumn15-Autumn22)

1
INTRODUCTION

The Directorate of Education and Training is continually endeavoring to assist the students of

Chartered Accountancy to prepare for their examinations through high quality study material and

suggested answers of past ICAP examinations.

The suggested answers are prepared on the principle of hints to answers, rather than detailed

theory and description and are based on International Standards and laws applicable at that time.

The answers are not updated subsequently for any changes in law.

We hope that the students will make the most of these suggested answers and use it as a study aid.

Users are encouraged to provide their feedback to the Directorate to enhance the quality of the

answers. The Directorate may, however, not be able to respond to individual queries from users.

ICAP
Disclaimer

The suggested answers to examination questions have been developed by the Directorate of Education

and Training of ICAP based on standards, laws, rules, regulations, theories and practice as applicable on

the date of examination, except as stated otherwise. These answers are not meant to provide the

assessment criteria against the particular examination questions. The purpose of these suggested answers

is only to guide the students in their future studies for ICAP’s examinations, without seeking to suggest a

solution for the present incumbents, in any way. However, there are alternative solution(s) to the

questions which are also considered by the Examination Department while marking the answer scripts.

Although reasonable care has been taken to ensure correctness in the preparation of these answers, the

Directorate does not take responsibility for any deviation of views, opinion or answers suggested by any

other person or persons. Similarly, the Council of the Institute of Chartered Accountants of Pakistan

assumes no responsibility for the errors or omissions in the suggested answers. Nevertheless, if any error

or omission is noticed, it should be brought to the notice of the Senior Director Education and Training for

information.

If you are not the intended addressee, you are notified that dissemination, copying, distributing,

commenting or printing of these answers is strictly prohibited.

ICAP
Certificate in Accounting and Finance Stage Examinations
The Institute of 8 September 2014
Chartered Accountants 3 hours – 100 marks
of Pakistan Additional reading time – 15 minutes

Financial Accounting and Reporting-I


Q.1 Following is the summarised trial balance of ABC Limited as at 30 June 2014:

Rs. in million
Sales 737
Stock at 1 July 2013 75
Purchases 301
Manufacturing expenses 240
Selling and marketing expenses 28
Administrative expenses 51
Factory building – cost at 1 July 2013 200
Machines – cost at 1 July 2013 280
Factory building – accumulated depreciation at 1 July 2013 50
Machines – accumulated depreciation at 1 July 2013 87
Advance income tax 4
Debtors 117
Cash and bank 51
Creditors 83
Share capital 300
Unappropriated profit at 1 July 2013 90
1,347 1,347

Additional information:
(i) Depreciation on factory building and machines are provided on reducing balance
method @ 10% and 15% per annum respectively. 60% depreciation on factory
building and 100% depreciation on machines are charged to cost of sales. The balance
depreciation is charged to administrative expenses.
(ii) On 31 May 2014, a fully depreciated machine was sold for Rs. 3 million. The sale
proceeds were received on 5 July 2014. No entries have been made in respect of these
transactions.
(iii) Debtors include an amount of Rs. 28 million owed by a customer who experienced
cash flow problems prior to the year-end. The company has agreed to accept
Rs. 18 million in full and final settlement of the debt. Four other debtors aggregating
Rs. 5 million are required to be written off.
(iv) Income tax liability for the year ended 30 June 2014 is estimated at Rs. 25 million.
(v) On 20 June 2014 an advance of Rs. 12 million was received under a contract for
supply of goods in August 2014. The advance was credited to sales.
(vi) Closing stock at 30 June 2014 amounted to Rs. 114 million. It included stock costing
Rs. 20 million whereas the related invoice was booked on 4 July 2014.
(vii) In June 2014, a competitor developed a new product which has affected ABC’s ability
to sell one of its products at its normal price of Rs. 160. It is estimated that to sell the
product, the company needs to offer a discount of 25%. 150,000 units of that product
were in hand as on 30 June 2014 at a cost of Rs. 120 per unit. Its selling costs are
estimated at Rs. 20 per unit.

Required:
Prepare the statement of comprehensive income for the year ended 30 June 2014 and the
statement of financial position as at that date in accordance with International Financial
Reporting Standards. (20)
Financial Accounting and Reporting-I Page 2 of 4

Q.2 Zeeshan Enterprise invoice goods to its Islamabad branch at cost plus 20 percent. The
expenses of the branch are paid by the head office. The branch has supplied the following
information for the year ended 30 June 2014:

Rupees
Opening stock - at invoice price 240,000
Closing stock - at invoice price 180,000
Cash sales 175,000
Credit sales 410,000
Collection from debtors 378,000
Debtors as on 30 June 2014 91,600
Goods received from head office - at invoice price 300,000
Goods returned to head office 30,000
Goods in transit from head office as on 30 June 2014 - at invoice price 36,000
Branch expenses paid by the head office 104,000

Required:
Show the Branch Account as it would appear in the books of head office for the year ended
30 June 2014 showing the profit made by the branch. (10)

Q.3 (a) List the conditions which are necessary to be fulfilled for recognizing revenue from sale
of goods under IAS 18 ‘Revenue’. (04)

(b) Attire Limited (AL) is a manufacturer of kids’ garments which are supplied to large
departmental stores. Following are some of the transactions which were carried out in
August 2014:

(i) AL delivered 2,000 garment pieces to Elegant Mart (EM). According to the terms
of sale, at the expiry of three months from the date of delivery, EM would have
the right to return the unsold garments to AL. All garments sold during this
period or retained by EM would be invoiced after three months of delivery and
would thereafter be paid within seven days.

EM has agreed to display AL’s garments at a prominent place at all its stores and
in return AL has agreed to allow a discount of 2%. (03)

(ii) AL sold 10,000 pieces of garments to Salam Garments on lay away basis. The
payment is to be made in 12 monthly instalments of Rs. 1,000,000 each. (03)

Required:
Describe how the above transactions would be accounted for in AL’s books of account.

Q.4 Shahzad Textile Mills Limited (STML) purchased a plant for Rs. 500 million on 1 July
2010. The plant has an estimated useful life of 10 years and no residual value.

STML uses revaluation model for subsequent measurement of its property, plant and
equipment and accounts for revaluations on net replacement value method. The details of
revaluations performed by an independent firm of valuers are as follows:

Revaluation date Fair value


1 July 2011 Rs. 575 million
1 July 2012 Rs. 390 million
1 July 2013 Rs. 380 million

Required:
Prepare journal entries to record the above transactions from the date of acquisition of the
plant to the year ended 30 June 2014. (Ignore tax implications) (15)
Financial Accounting and Reporting-I Page 3 of 4

Q.5 Hammad Limited (HL) imports and supplies three products, Alpha, Gamma and Beta. The
opening balances and transactions for the month of June 2014 are as follows:

Units purchased Units Sold during the


Opening balance
during the month month
Items
Invoice
Qty. Value (Rs.) Qty. Qty. Value (Rs.)
value (Rs.)
Alpha 20 60,000 360 920,000 350 1,820,000
Gamma 100 4,800,000 50 2,375,000 70 4,060,000
Beta 30 120,000 490 1,820,000 400 1,640,000

The following information is also available:

(i) HL’s bank charges a commission of 0.5% of invoice value for opening the letter of
credit.
(ii) Import taxes and duties were 23% of the invoice value out of which 40% are
refundable/adjustable.
(iii) The transportation charges are Rs. 1,500 per trip. 20 units of Alpha, 2 units of Gamma
or 15 units of Beta can be transported in each trip.
(iv) All goods are repacked after import. The cost of packing per unit was Rs. 300,
Rs. 1,500 and 700 respectively.
(v) HL values its stock on first-in, first-out basis.
(vi) Average selling costs per unit are Rs. 700, Rs. 1,500 and Rs. 400 respectively.

Required:
Compute the value of stock of each product as at 30 June 2014 in accordance with IAS-2
‘Inventories’. (15)

Q.6 Following information has been extracted from the financial statements of Full Speed
Enterprises (FSE) for the year ended 30 June 2013:

Rupees
Vehicles – cost 65,201,300
Less: Accumulated depreciation (24,450,500)
WDV of vehicles 40,750,800

FSE provides depreciation on vehicles @ 15% per annum on written down values.
Depreciation on addition/deletion is provided in proportion to the period of use.

Other related information is as follows:

(i) On 1 August 2013, a vehicle which was acquired at a cost of Rs. 850,000 on
1 July 2011 was exchanged for a new vehicle. The balance was settled with a cheque
for Rs. 350,000. The list price of the new vehicle was Rs. 900,000.
(ii) Three new vehicles were purchased on 1 December 2013 for Rs. 1,250,000 each.
(iii) On 1 February 2014, a vehicle having written down value of Rs. 550,000 was repaired
at a cost of Rs. 250,000. It is expected that the repairs would improve the efficiency of
the vehicle significantly.
(iv) On 30 June 2014, a vehicle purchased on 1 January 2012 at a cost of Rs. 1,500,000 was
sold for Rs. 1,350,000.

Required:
Prepare the following ledger accounts for the year ended 30 June 2014:
(a) Vehicles account
(b) Accumulated depreciation on vehicles
(c) Loss/gain on sale of vehicles (10)
Financial Accounting and Reporting-I Page 4 of 4

Q.7 Following is the balance sheet of Ashfaq as at 30 June 2013:


Owner's equity / Liabilities Rupees Assets Rupees
Ashfaq’s capital 4,396,600 Motor car 2,000,000
Creditors 1,102,000 Furniture 1,000,000
Accrued rent 20,000 Stock-in-trade 1,805,000
Loan taken from a friend 27,900 Debtors 350,000
Prepaid insurance 15,000
Balance at bank 360,600
Cash in hand 15,900
5,546,500 5,546,500

Ashfaq needs to submit his Trading and Profit and Loss Account for the year ended 30 June
2014 and Balance Sheet as of that date to his bankers in order to obtain an overdraft facility.
He has not maintained proper books of account of the business but has provided you the
following information:

(i) He purchased goods from a single supplier who allows a discount of 3% on goods
purchased in excess of Rs. 3,000,000 in a year. The discount for the year ended
30 June 2014 amounts to Rs. 265,800 and would be received in August 2014.
(ii) All goods are sold at cost plus 60%.
(iii) All cash received against sale of goods has been banked with the exception of the
following weekly average cash expenses/drawings:

Rupees
Drawings 30,000
Carriage outward 5,000
Petrol 3,000
Misc. expenses 2,500

(iv) Cash in hand on 30 June 2014 amounted to Rs. 26,700.


(v) An analysis of Ashfaq’s bank statement revealed the following information:

Receipts Rupees Payments Rupees


Collection from debtors 464,400 Purchase of goods 9,850,700
Cash deposited into bank 13,717,800 Car expenses (for business) 73,000
Rent 42,000
Repayment of loan to friend 27,900
Salaries 1,600,000
Purchase of freehold land 2,500,000
Travelling expenses 40,000
Printing & stationery 46,000
Advertisement 125,000
Insurance 50,000
Truck hire charges 657,000
Misc. expenses 362,300
14,182,200 15,373,900

(vi) Depreciation on motor car and furniture is to be provided @ 30% and 15%
respectively under the reducing balance method.
(vii) Stock-in-trade on 30 June 2014 amounted to Rs. 702,000.

Required:
Prepare Trading and Profit and Loss Account for the year ended 30 June 2014 and Balance
Sheet as on 30 June 2014. (20)

(THE END)
Financial Accounting and Reporting-I
Suggested Answers
Certificate in Accounting and Finance – Autumn 2014

Ans.1 (a) ABC Limited


Statement of comprehensive income for the year ended 30 June 2014
Rs. in million
Sales (737 - 12) 725
Cost of sales (W-1) (563)
Gross profit 162
Selling and marketing expense (28)
Administrative expenses [(51+6(W-3)]+(28-18)+5 (72)
Other income 3
Profit before tax 65
Income tax expense (25)
Profit for the year 40

(b) Statement of financial position as at 30 June 2014


ASSETS
Non-current assets
Property, plant and equipment (343 - 44) 299

Current assets
Stock 111
Debtors [117-(28-18)-5] 102
Other receivable 3
Cash and bank 51
267
Total assets 566

EQUITY AND LIABILITIES


Owner's equity
Share capital 300
Unappropriated profit (90+40) 130
Total equity 430

Current liabilities
Creditors (83+20) 103
Income tax payable (25-4) 21
Advance from customer 12
136
Total equity and liabilities 566

Workings
W-1: Cost of Sales
Opening stock 75
Purchases (301+20) 321
Manufacturing expenses 240
Depreciation (W-3) 38
Closing stock (114-3) (W-2) (111)
563
W-2: Inventory adjustment
Cost of product (150,000 x Rs. 120) 18
NRV of product (150,000 x [((Rs. 160×75%) - Rs. 20)] (15)
3

W-3: Depreciation:
Chargeable to
Depreciation
Cost of sales Administration
Factory building [(200-50]*10%) (60:40) 15 9 6
Machinery [((280-87)*15%)] 29 29 -
44 38 6

Page 1 of 8
Financial Accounting and Reporting-I
Suggested Answers
Certificate in Accounting and Finance – Autumn 2014

Ans.2 Branch Account


Rupees Rupees
Opening stock 240,000 Opening stock reserve (240,000*20/120) 40,000
Opening debtors (W) 59,600 Cash sales 175,000
Goods sent to branch (300,000+36,000) 336,000 Collection from debtors 378,000
Branch expenses 104,000 Goods returned to HO 30,000
Goods in transit 36,000
Goods sent to branch a/c (loading on
net goods sent to branch) [(336,000-
Closing stock reserve (180,000*20/120) 30,000 36,000-30,000) × 20/120] 45,000
Profit and loss a/c (Branch profit Closing stock 180,000
transferred) (balancing) 206,000
Closing debtors 91,600

975,600 975,600

Working:
Debtors as on 30 June 2013
91,600 + 378,000 – 410,000 = 59,600

Ans.3 (a) Revenue recognition from sale of goods


IAS 18 says that an entity may recognise revenue from the sale of goods only when
all of the following conditions have been met:

 The entity has transferred to the buyer the ‘significant risks and rewards of
ownership of the goods’. This normally occurs when legal title to the goods or
possession of the goods passes to the buyer.
 The entity does not retain effective control over the goods sold, nor retains a
continuing management involvement to the degree usually associated with
ownership.
 The amount of revenue can be measured reliably.
 It is probable that economic benefits associated with the transaction will flow
to the entity.
 The costs incurred (or to be incurred) for the transaction can be measured
reliably.

(b) (i) The garments remain the property of AL and EM bears none of the risks of
ownership. When EM sells the garments or decides to keep them at the end of
three months, it records the purchases at that point from AL. This is therefore
the point at which the risks and rewards pass to EM. Up to that point there is
no sale and the garments should appear in inventory of AL.

The 2% display charge should be accounted for as marketing expense.

(ii) A lay away sale does not involve financing and the revenue from the lay away
sale may be recognized in AL’s financial statements when the goods are
delivered. However, if experience indicates that most such sales are
consummated, revenue may be recognized when a significant deposit is
received provided the goods are on hand, identified and ready for delivery to
the buyer.

Page 2 of 8
Financial Accounting and Reporting-I
Suggested Answers
Certificate in Accounting and Finance – Autumn 2014

Ans.4 Debit Credit


Date Particulars
(Rs. in million) (Rs. in million)
1-Jul-10 Plant 500.00
Bank 500.00
(Record purchase of plant)

30-Jun-11 Depreciation 50.00


Accumulated depreciation - Plant 50.00
(Record depreciation for the year 2010-11)
Working: Rs. 500m ÷ 10 = Rs. 50m

1-Jul-11 Accumulated depreciation - Plant 50.00


Plant 50.00
(Reversal of prior years depreciation)

1-Jul-11 Plant 125.00


Surplus on revaluation of fixed assets 125.00
(Increase in value through revaluation)
Working: Rs. 575m - Rs. 450m = Rs. 125m

30-Jun-12 Depreciation 63.89


Accumulated depreciation - Plant 63.89
(Record depreciation for the year 2011-12)
Working: Rs. 575m ÷ 9 = Rs. 63.89m

30-Jun-12 Surplus on revaluation of fixed assets 13.89


Retained earnings 13.89
(Reversal for excess depreciation)
Working: Rs. 125m ÷ 9 = Rs. 13.89m

1-Jul-12 Accumulated depreciation - Plant 63.89


Plant 63.89
(Reversal of prior year depreciation)

1-Jul-12 Surplus on revaluation of fixed assets 111.11


Revaluation expense (balancing) 10.00
Plant 121.11
(Decrease in value through revaluation)
Working:
Surplus on rev. = Rs. 125m - Rs. 13.89m = Rs. 111.11
Building: [Rs. 575m - Rs. 63.89m] - Rs. 390m =Rs. 121.11

30-Jun-13 Depreciation 48.75


Accumulated depreciation - Plant 48.75
(Record depreciation for the year 2012-13)
Working: Rs. 390m ÷ 8 = Rs. 48.75m

1-Jul-13 Accumulated depreciation - Plant 48.75


Plant 48.75
(Reversal of prior year depreciation)

1-Jul-13 Plant 38.75


Revaluation income 8.75
Surplus on revaluation of fixed assets (balancing) 30.00
(Reversal of prior year depreciation)
Working:
Revaluation income = Rs. 10m-[Rs. 50m - Rs. 48.75m] = Rs.
8.75m
Plant: Rs. 380m - [Rs. 390m - Rs. 48.75m] = Rs. 38.75m

Page 3 of 8
Financial Accounting and Reporting-I
Suggested Answers
Certificate in Accounting and Finance – Autumn 2014

30-Jun-14 Depreciation 54.29


Accumulated depreciation - Plant 54.29
(Record depreciation for the year 2013-14)
Working: Rs. Rs. 380m ÷ 7 = Rs. 54.29m

30-Jun-14 Surplus on revaluation of fixed assets 4.29


Retained earnings 4.29
(Reversal for excess depreciation)
Working: Rs. 30m ÷ 7 = Rs. 4.29m

Page 4 of 8
Financial Accounting and Reporting-I
Suggested Answers
Certificate in Accounting and Finance – Autumn 2014

Ans.5
Alpha Gamma Beta
Total
Cost Cost Cost
Units Cost (Rs.) Units Cost (Rs.) Units Cost (Rs.) Cost
P/U P/U P/U
Opening stock 20 3,000 60,000 100 48,000 4,800,000 30 4,000 120,000

Purchased during the month


Invoice value 920,000 2,375,000 1,820,000
LC opening charges
(0.5% of invoice value) 4,600 11,875 9,100
Import duties to be added
(23%×60%=13.8%) × Invoice value 126,960 327,750 251,160
Transportation cost (360/20)×1,500 27,000 (50/2)×1,500 37,500 (490/15)×1,500 49,000
Wrapping cost (360 × 300) 108,000 (50 ×1,500) 75,000 (490 ×700) 343,000
360 *3,296 1,186,560 50 *56,543 2,827,125 490 *5,045 2,472,260
Closing stock 30 80 120
(20+360-350) (100+50-70) (30+490-400)

Value of closing stock on FIFO basis


From opening stock - - - 30 48,000 1,440,000 - - -
From purchases 30 3,296 98,880 50 56,543 2,827,125 120 5,045 605,451
98,880 4,267,125 605,451
NRV of closing stock
Selling price 5,200 58,000 4,100
Less: Selling cost per unit (700) (1,500) (400)
30 4,500 135,000 80 56,500 4,520,000 120 3,700 444,000

Closing stock at lower of cost and NRV 98,880 4,267,125 444,000 4,810,005

Page 5 of 8
Financial Accounting and Reporting-I
Suggested Answers
Certificate in Accounting and Finance – Autumn 2014

Ans.6 Vehicle
Rupees Rupees
1/7/2013 Opening balance 65,201,300 1/8/2013 Cost of vehicle exchanged 850,000
New vehicle in exchange
1/8/2013 for old car 900,000 6/30/2014 Vehicle sold 1,500,000
1/12/2013 3 car @ 1,250,000 each 3,750,000 30/6/2014 Closing balance 67,751,300
1/2/2014 Repair to a vehicle 250,000
70,101,300 70,101,300

Accumulated Depreciation - Vehicle


Rupees Rupees
1/8/2013 Acc dep for vehicle 1/7/2013 Opening balance 24,450,500
exchanged
(127,500+108,375+7,677) 243,552
6/30/2014 Acc dep for vehicle sold 30/6/2014 Depreciation for the year 6,496,733
(112,500+208,125+176,906) 497,531 (W-1)
30/6/2014 Closing balance 30,206,150
30,947,233 30,947,233

Loss / gain on sale of vehicles


Rupees Rupees
1/8/2013 Loss on exchange of 6/30/2014 Gain on sale of vehicle 347,531
vehicle [1,350,000 - (1,500,000-
350,000 +(850,000- 497,531)]
243,552-900,000) 56,448
6/30/2014 Transfer to P&L 291,083
Net gain on disposal
of assets
347,531 347,531

W-1: Depreciation for the year


Depreciation on opening written down value (65,201,300 - 24,450,500) ×15% 6,112,620
Less: Depreciation on exchanged assets from 1/8/13 to 30/6/14 (850,000-243,552) × 15%
× 11/12 (83,387)
Add: Depreciation on addition - Exchanged asset (900,000 × 15% × 11/12) 123,750
Add: Depreciation on addition (3,750,000 × 15% × 7/12) 328,125
Add: Depreciation on major repair (250,000 × 5/12 × 15% ) 15,625
6,496,733

Page 6 of 8
Financial Accounting and Reporting-I
Suggested Answers
Certificate in Accounting and Finance – Autumn 2014

Ans.7 TRADING ACCOUNT


For the year ended 30 June 2014
Rupees Rupees
Opening stock 1,805,000 Sales – Cash (W-2) 15,834,600
Purchases (W-1) 11,860,000 - Credit (W-3) 4,480,920
Purchase discount 265,800
Gross profit c/d 7,618,320 Closing stock 702,000
21,283,320 21,283,320

PROFIT & LOSS ACCOUNT


For the year ended 30 June 2014
Rupees Rupees
Carriage outward 260,000 Gross profit b/d 7,618,320
Petrol 156,000
Car expenses 73,000
Rent (42,000-20,000) 22,000
Salaries 1,600,000
Traveling expenses 40,000
Printing & stationary 46,000
Advertisement 125,000
Insurance (50,000 + 15,000) 65,000
Depreciation (600,000+150,000) 750,000
Truck hire charges 657,000
Misc. expense (362,300+130,000) 492,300
Profit for the year 3,332,020
7,618,320 7,618,320

Balance Sheet
As at 30 June 2014
Rupees Rupees
Owner's equity Fixed assets
Ashfaq's capital Freehold land 2,500,000
(4,396,600+3,332,020-1,560,000) 6,168,620 Motor car (2,000,000-600,000) 1,400,000
Furniture (1,000,000 - 150,000) 850,000

Liabilities Current assets


Creditors (W-1) 2,845,500 Stock 702,000
Bank overdraft (W-5) 831,100 Debtors (W-4) 4,366,520
Cash in hand 26,700
9,845,220 9,845,220

WORKINGS
W-1 : Creditors
Rupees Rupees
Bank 9,850,700 Balance b/d 1,102,000
Purchase discount 265,800 Purchases (W-1.1) 11,860,000
Balance c/d 2,845,500
12,962,000 12,962,000

W-1.1: Rs. 3,000,000 + (Rs.265,800/0.03) = Rs. 11,860,000


W-2 : Cash
Rupees Rupees
Balance b/d 15,900 Bank 13,717,800
Cash sales (bal) 15,834,600 Drawings (30,000 × 52) 1,560,000
Carriage outward (5,000 × 52) 260,000
Petrol (3,000 × 52) 156,000
Misc. expense (2,500 × 52) 130,000
Balance c/d 26,700
15,850,500 15,850,500

Op. Stock + Net purchases – Closing Stock


W-3: Total sales = (Rs. 1,805,000 + 11,594,200 – 702,000)× 1.6 = Rs. 20,315,520
Credit sales = 20,315,520 - 15,834,600 = Rs. 4,480,920

Page 7 of 8
Financial Accounting and Reporting-I
Suggested Answers
Certificate in Accounting and Finance – Autumn 2014

W-4 : Debtors
Rupees Rupees
Balance b/d 350,000 Bank 464,400
Credit sales (W-3) 4,480,920 Balance c/d (balancing) 4,366,520

4,830,920 4,830,920

W-5: (360,600+14,182,200-15,373,900=831,100 (Bank over draft)

(THE END)

Page 8 of 8
INSTITUTE OF CHARTERED ACCOUNTANTS OF PAKISTAN

EXAMINERS’ COMMENTS

SUBJECT SESSION
Financial Accounting and Reporting-I Certificate in Accounting and Finance
– Autumn 2014

General:

The overall result in this paper was 35%. The following general issues were observed
during the assessment process:

 Students did not carefully read the style of amounts mentioned in the question like
Rupees, “Rs in 000” or “Rs in million”. Consequently, they arrived at abnormal
figures which confused them and resulted in errors.

 The exact requirement of the question was not read. Consequently, either important
calculations were missed or additional working was carried out which resulted in
waste of precious time.

Question 1

This question required basic knowledge for preparing Statement of Comprehensive


Income and Statement of Financial Position. Unadjusted trial balance and related data for
the purpose of adjustments were provided in the question.

Generally the performance was good; however, the following mistakes were observed:

 Receivable from sale of machine was not incorporated in the statement of financial
position.

 Rs. 20 million relating to invoice booked after year end was adjusted from stock
instead of purchases made during the year.

 Net realizable value (NRV) of specified products was not correctly calculated.
Discount of 25% which should have been deducted from total sale in order to arrive
at NRV, was ignored. Many students ignored the selling costs that were required to
sell the related stock.

 Many students did not include depreciation in cost of sales and included the entire
depreciation in administrative expenses.

 Most of the students were not familiar with the treatment of tax. Instead of showing
tax as a deduction from profit before tax, many students included tax in
Administration Expenses. Further, most students did not adjust advance tax from the
income tax liability. Many students ignored it altogether.

Page 1 of 4
Examiners’ Comments on Financial Accounting and Reporting-I - Autumn 2014

Question 2

This question required preparation of branch account showing the profit made by the
branch. An average performance was observed because many students made errors even
in the treatment of simple items. Some of the common mistakes are enumerated below:

 Goods in transit were included in the amount of Goods sent to Branch.

 Many students ignored the stock reserve specially the reserve on opening stock.
Further, in computing reserve on closing stocks most of the students calculated it on
GIT also.

 Opening debtors were ignored in most cases.

Question 3

(a) Conditions for revenue recognition as specified in IAS 18 were required. Majority
of the students missed one or two conditions specially those related to non-
retention of control by the seller and the condition that economic benefits
associated with the transaction should flow to the seller were missed.

(b) In this part, two transactions were described and the candidates were required to
specify the accounting treatment thereof. In such questions, the recommended
accounting treatment is required to be justified on the basis of relevant accounting
principles/rules. Many students ignored explanation altogether. Other common
mistakes were as follows:

(i) Majority of the students clearly knew that sales will be recorded on
completion of 3 months. However, only few were able to discuss the reasons
thereof in a proper way.

(ii) Majority of the students were unaware of the concept of sales on lay away
basis as has been discussed in IAS 18 and assumed it as Installment sales.
They are advised to refer to the ICAP’s suggested answer and to study the
relevant paragraphs of IAS 18 to understand this concept.

Question 4

This question required preparation of Journal Entries to record the transactions pertaining
to subsequent measurement of fixed assets under the Revaluation Model using Net
Replacement Value Method. Generally, the students did not have command over the
topic and were further confused between the Net Replacement Value Method and the
Gross Replacement Value Method. A number of errors were observed which are
discussed below:

 Most of the students did not understand that the balance in the accumulated
depreciation account has to be reversed at the time of each revaluation.

 Excess depreciation due to revaluation was not transferred from surplus on


revaluation to Retained Earnings.

Page 2 of 4
Examiners’ Comments on Financial Accounting and Reporting-I - Autumn 2014

 In the 2nd revaluation, many students debited the entire reduction in value of plant to
the surplus account. The correct procedure is to debit the surplus account only to the
extent of credit balance in the account and to debit the remaining loss to Retained
Earnings.

 Similarly in the 3rd revaluation the entire difference between the carrying value of the
plant and the fair value was credited to Surplus account. For correct procedure the
students may refer to the ICAP’s suggested answer.

Question 5

The requirement of this question was simply to calculate the value of 3 (imported)
products under the FIFO method. However, the candidates made many errors while
performing simple calculations and missed an easy chance of scoring high marks.
Commonly observed errors are described below:

 The refundable portion of import duty was also included in the cost.

 Transportation cost per trip was taken as the transportation cost for entire purchases,
whereas many students were too confused and multiplied the cost per trip with the
units per trip to arrive at the total transportation cost for that product.

 Some students ignored wrapping costs and commented that these are marketing
expenses.

 Many students ignored the fact that closing inventory of Gamma included 30 units
from the opening balance.

 In the computation of NRV, selling costs were ignored.

Further, a number of students made separate calculations (on separate pages) for each
product which resulted in wastage of time.

Question 6

This question required preparation of ledger accounts of vehicle, accumulated


depreciation on vehicle and gain/loss on disposal of fixed assets.

Majority of the students managed to score good marks in this question. The mistakes
observed were as follows:

(a) Some students prepared a single fixed asset account and merged depreciation and
cost in the same account.

(b) The depreciation on purchases during the year was mostly provided correctly based
on the period of use but no depreciation was provided on cars sold during the year.

(c) The concept of trade-in was misunderstood by many students as they debited the net
amount paid to the vehicle account.

Page 3 of 4
Examiners’ Comments on Financial Accounting and Reporting-I - Autumn 2014

Question 7

This was a question on single entry in which the requirement was to prepare Trading and
Profit and Loss Account and Balance Sheet. Majority of the students seemed well versed
with techniques involved and secured good marks. Most of the errors were in respect of
the following:

 Cash sales were not calculated and cash deposited was considered as cash sales.

 Sale was required to be calculated by multiplying cost of sales with 1.6. Instead,
many students calculated it by dividing cost of sales by 0.6.

 Many students wasted time in preparing bank account which was not required as all
the information was already given in the question.

 Purchase discount was shown in profit and loss account instead of trading account.

 Carriage outward was taken to trading account instead of profit and loss account.

THE END

Page 4 of 4
Certificate in Accounting and Finance Stage Examinations
The Institute of 2 March 2015
Chartered Accountants 3 hours – 100 marks
of Pakistan Additional reading time – 15 minutes

Financial Accounting and Reporting-I


Q.1 Babar had purchased a running business from Razi on 1 January 2014 at a total agreed price
of Rs. 960,000. Babar died on 16 June 2014 and his son Sami took over the business. Sami
wants to assess the profitability of the business and for that purpose he has collected the
following information from the records maintained by him and his father:

(i) Correspondence between Babar and Razi has revealed that they had agreed to value
the inventory and other assets of the business at Rs. 600,000 and Rs. 120,000
respectively. However, in view of Razi’s standing in the market, the deal had been
finalised at a lump sum price of Rs. 960,000 payable in two equal instalments. The
first instalment was paid by Babar from his personal account.

(ii) Babar had opened a bank account in the name of the business. An analysis of the
bank statement revealed the following details:

Receipts Rupees
Amount deposited by Babar on 1 January 2014 from his personal account 2,000,000
Day to day collections banked at day end 3,800,000

Payments
Second instalment to Mr. Razi on 31 January 2014 480,000
Purchases 3,150,000
Lease rent 120,000
Electricity 22,000
Furniture purchased on 1 July 2014 25,000

(iii) Babar and Sami kept a notebook which shows that the following payments were
made out of daily sale proceeds before depositing them in the bank:

Rupees
Salaries and EOBI payments 184,300
Purchases 49,500
Sundry shop expenses 35,600
Drawings 192,500

(iv) On 31 August 2014, there was a burglary at the warehouse and inventory costing
Rs. 50,000 was stolen. Due to defect in the insurance policy, the insurance company
acknowledged the claim of Rs. 20,000 only, which was received on 5 November 2014.
(v) On 31 December 2014, stock on hand costed Rs. 450,000. Cash in hand, trade
creditors and accrued expenses (electricity) amounted to Rs. 34,500, Rs. 82,500 and
Rs. 5,200 respectively.
(vi) Depreciation on fixtures and fittings is to be provided at the rate of 10% per annum.

Required:
Prepare Trading and Profit and Loss Account for the year ended 31 December 2014 and
Balance Sheet as on 31 December 2014. (20)
Financial Accounting and Reporting-I Page 2 of 5

Q.2 Trade Link Enterprises opened a branch at Lahore on 1 January 2014. The branch has
provided the following summary of transactions carried out by it during the year 2014 :

Rupees
Goods received from head office 21,732,000
Sales made during the year, of which 40% on credit 15,846,250
Realized from credit customers 4,753,875
Trade discount allowed to customers 36,220
Sales return by customers 108,660
Bad debts 9,055
Petty expenses incurred 70,629
Closing stock 6,385,000
Goods in transit from head office at year-end 250,000
Purchase of fixed assets, bills discharged by head office 1,448,800
Expenses incurred and reimbursed by head office:
 Rent and utilities 537,100
 Sales promotion 144,880
 Payroll 724,400
Other information:
(i) Head Office invoices goods to branch at cost plus 25 percent.
(ii) The branch maintains an imprest of Rs. 100,000 and a balance of Rs. 500,000 in its
bank account. All other takings are transferred to head office.
(iii) Depreciation on fixed assets is to be charged at 15% per annum.

Required:
Prepare Lahore Branch Account in the books of Trade Link Enterprises for the year ended
31 December 2014 showing the profit made by the branch. (12)

Q.3 (a) HCL had agreed to provide services to NPL. The total contract price was Rs. 800,000
and HCL had initially expected to earn 25% profit on the contract. 50% of the work
had been completed at year end at the cost of Rs. 320,000. Soon thereafter, a dispute
arose on the quality of work and further work has been stopped pending settlement of
the dispute. HCL is however very confident of recovering the cost incurred on the
contract plus a margin of 10% above cost.

Required:
Discuss how much revenue should be recognised at the year end? (02)

(b) Saleem owns 10,000 shares in a listed company on 3 December 2014. On the same
date, the company declared a dividend of Rs. 2 per share on the basis of shares held
on 31 December 2014.

The dividend was paid by the company on 15 January 2015.

Required:
Prepare necessary journal entries relating to the dividend in the books of Saleem. (02)

(c) A company sold equipment to a customer on 1 September 2014 for Rs. 15 million. As
per market norms the company has agreed to provide free support services for the next
two years. The cost of providing the support services is estimated at Rs. 250,000 per
annum. On such services, the company usually earns a profit of 20% of cost.

Required:
Prepare journal entries relating to this transaction for the year ended
31 December 2014. (04)

(d) In the sale of goods how should the revenue be recognised when goods are shipped
subject to installation and inspection? (04)
Financial Accounting and Reporting-I Page 3 of 5

Q.4 (a) List the elements of financial statements. (02)

(b) Following is the draft balance sheet of XYZ Limited as at 31 December 2014 which
was prepared by its accountant:
Rs. in Rs. in
Assets Equities and liabilities
million million
Leasehold land – cost 250 Capital 1,000
Leasehold land – accumulated amortisation (200) Accumulated profit 1,816
Building – cost 1,000 Long term bank loan 200
Building – accumulated depreciation (500) Trade payables 228
Machinery – cost 1,750 Income tax payable 85
Machinery – accumulated depreciation (1,150) Accrued interest 13
Long term deposit 70
Stocks 910
Account receivables – net of provision 361
Cash and bank 851
3,342 3,342

Additional information:
(i) Profit before tax and income tax expenses for the year amounted to Rs. 275
million and Rs. 13 million respectively.
(ii) Balances as at 31 December 2013 were as under:
Rs. in million
Stock 703
Account receivables – net of provision 418
Cash and bank 243
Trade payables 150
Income tax payable 80
Long term deposit 70

The company follows a policy of maintaining provision for bad debts equal to
5% of account receivables.

(iii) The bank loan was obtained on 1 January 2014 and carries interest @ 9% per
annum.
(iv) XYZ uses straight line method for depreciation. Rates of depreciation are as
under:
Leasehold land 2%
Building 5%
Machinery 10%

Full month’s depreciation is provided in the month of acquisition but no


depreciation is charged in the month of disposal. Depreciation for the year 2014
has already been provided.

On review the CFO has discovered the following:


 A machine with list price of Rs. 50 million was purchased on
1 January 2014. An amount of Rs. 30 million had been paid in cash
whereas Rs. 20 million were adjusted against trade-in of a machine costing
Rs. 40 million and having a book value of Rs. 25 million. The transaction
was recorded by debiting the plant and machinery account by Rs. 30
million i.e. the net amount paid to the supplier.
 One of the company's customers became bankrupt during the year. Rs. 5
million out of total debt of Rs. 25 million were recovered from him.
Balance has to be written off.

Required:
Prepare a statement of cash flow as at 31 December 2014. (20)
Financial Accounting and Reporting-I Page 4 of 5

Q.5 (a) List the particulars that are required to be disclosed in the financial statements in
respect of inventories, according to IAS 2. (03)

(b) Soya Fry Limited manufactures Cooking Oil. Following information is available with
respect to purchases and overheads for the year ended 31 December 2014.
Details of purchases: Rs. in ‘000’
Raw material purchased (including 17% sales tax which
is refundable) 60,500
Packing material purchased 2,050
Settlement discount received on raw material purchases 400
Transportation cost relating to raw material (70%) and
packing material (30%) 300
Details of overheads:
Rent 2,700
Salaries and wages 2,500
Other variable overheads 5,000
Other fixed overheads 1,500

Other information:
(i) The break-up of rent is as follows:
Rs. in ‘000’
Factory 2,000
Warehouse (50% for raw material, 10% for
packing material and 40% for finished goods) 500
Shelf spacing in super markets 200

(ii) Break-up of salaries and wages, other variable and fixed overheads is as follows:
Allocation between
Manufacturing Administration
Salaries and wages *60% 40%
Other variable overheads 80% 20%
Other fixed overheads 60% 40%
*Manufacturing salaries includes 70% direct wages to labourers
working in the factory which vary with the level of production.

(iii) Normal production level is 45,000 units per annum. Actual production during
the year was 40,000 units.
(iv) Opening and closing inventories are as follows:

1-Jan-2014 31-Dec-2014
--------- Rs. in ‘000’---------
Packing material 700 285
Raw material 5,000 7,780
Finished goods 2,962 4,162
Work in process 1,950 3,000
Goods costing Rs. 200,000 (2013: Rs. 300,000) are considered as obsolete and
have been fully provided. Further, closing stock of finished goods include goods
costing Rs. 75,000 which were damaged due to flood and can only be sold at
60% of its cost.

Required:
Disclose the above information in the note on ‘Cost of goods sold’ as would appear in
the profit and loss account for the year ended 31 December 2014. (17)
Financial Accounting and Reporting-I Page 5 of 5

Q.6 You have recently been appointed as chief financial officer of Al-Hafeez Limited (AHL).
While finalizing the company’s financial statements for the year ended 31 December 2014,
you have observed the following issues:

(a) Plant and equipment includes Machine A-31 at a carrying amount of Rs. 918,400
which was fabricated in-house by AHL in February 2014 by using existing plant and
machinery. The details are as follows:
Rupees
Direct material and labour 656,000
Depreciation – existing plant and machinery 24,000
Administration costs 140,000
20% profit (normally charged to its customers) 164,000
984,000
Less: Depreciation for the year (10% of the cost for 8 months) (65,600)
Carrying value of the machine at year-end 918,400

Direct material includes material lost due to fire amounting to Rs. 40,000.

The fabricated machine was transferred and available for use on 1 March 2014 and
was brought into commercial production on 1 May 2014. (07)

(b) AHL provides transportation services to its factory workers through its fleet of six
buses. The buses are depreciated on straight line basis. At the end of last year, the
buses had carrying value of Rs. 7 million and remaining useful life of 5 years.

On 1 July 2014, the local government promulgated a new legislation whereby all
public transport buses were required to undergo regular major inspection after a
period of three years. An inspection exercise of the fleet of buses was undertaken on
1 September 2014 at a cost of Rs. 1.8 million and this amount was capitalized in the
carrying amount of buses. (04)

(c) On 31 December 2014, AHL acquired a used specialized machine which has no
active market, by exchange of Machine X. The newly acquired machine was booked
at the carrying value of Machine X which was Rs. 9.5 million. However, the fair value
of Machine X on the date of sale was Rs. 8 million but no adjustment was made on
the premise that the acquisition of this specialised machine would increase efficiency
and consequently save approximately Rs. 1.5 million over its useful life. (03)

Required:
Explain the correct accounting treatment of the transactions by AHL and substantiate your
point of view with references to International Accounting Standards – 16 ‘Property, Plant
and Equipment’. Also prepare the necessary journal entries.

(THE END)
Financial Accounting and Reporting-I
Suggested Answers
Certificate in Accounting and Finance – Spring 2015

Ans.1 TRADING ACCOUNT


For the year ended 31 December 2014
Rupees Rupees
Stock received from Razi 600,000 Sales (3800000+461900+34500) 4,296,400
Purchases (3,150,000 – 50,000) 3,100,000 Closing stock 450,000
Creditors for Purchases 82,500
Cash Purchases 49,500
Gross profit c/d 914,400
4,746,400 4,746,400

PROFIT & LOSS ACCOUNT


For the year ended 31 December 2014
Rupees Rupees
Salaries 184,300 Gross profit b/d 914,400
Sundry shop expenses 35,600
Lease rentals 120,000
Loss on burglary 30,000
Electricity(22000+5200) 27,200
Depreciation (5% × 25,000) 1,250
Profit for the year 516,050
914,400 914,400

BALANCE SHEET
As at 31 December 2014
Rupees Rupees
Owner's equity Fixed assets
Babar’s capital (2,000,000+480,000) Goodwill
2,480,000 (960,000–600,000–120,000) 240,000
Profit for the year 516,050 Furniture (25000 – 1250) 23,750
Drawings (192,500)
2,803,550 263,750
Liabilities Current assets
Creditors 82,500 Stock 450,000
Accrued expenses (Electricity) 5,200 Other assets* 120,000
Cash at bank (W-1) 2,023,000
Cash in hand 34,500
2,891,250 2,891,250

WORKING:
W-1 : Cash at Bank
Rupees Rupees
Capital introduced 2,000,000 Payment of 2nd installment to 480,000
Razi
Cash deposited 3,800,000 Payment for purchases 3,150,000
Cash received from insurance 20,000 Lease payment 120,000
Electricity 22,000
Furniture & Fixtures 25,000
Balance at bank 2,023,000
5,820,000 5,820,000

Page 1 of 6
Financial Accounting and Reporting-I
Suggested Answers
Certificate in Accounting and Finance – Spring 2015

Ans. 2 Lahore branch account in the head office books


Particulars Rs. Particulars Rs.
Goods received from head office 21,732,000 Cash sales less imprest and 8,907,750
*Goods in transit 250,000 amount retained by bank (W-1)
Fixed assets 1,448,800 Credit sales realized 4,753,875
Petty expenses disbursements 70,629 *Goods in transit 250,000
Rent & Utilities 537,100 Goods sent to branch – loading 4,346,400
Sales promotion 144,880 (21,732,000×25/125)
Salaries 724,400 Branch stock a/c 6,385,000
Stock reserve - (6,385,000×25/125) 1,277,000 Branch debtors (W-1) 1,430,690
Profit transferred to P & L A/c 1,720,386 Balance of fixed assets – net 1,231,480
Balance at bank 500,000
Petty cash imprest 100,000
27,905,195 27,905,195

W-1: Branch Debtors


Sales 6,338,500 Bank 4,753,875
Sales discount 36,220
Sales return 108,660
Bad debts 9,055
Balance c/d (Balancing fig.) 1,430,690
6,338,500 6,338,500

W-1: Sales made during the year Rs.


Sales during the year 15,846,250
Sales to credit customers 6,338,500
Cash sales 9,507,750
Less: Petty cash imprest 100,000
Bank balance 500,000
Cash sales deposited into bank 8,907,750

Ans.3 (a) If the outcome of a services transaction cannot be estimated reliably, revenue should only be
recognized to the extent that expenses incurred are recoverable from the customer.
Therefore, HCL may recognize revenue to the extent of Rs. 320,000 only.

(b) Date Description Debit Credit


---------- Rupees ----------
31-Dec-2014 Dividend receivable 20,000
Dividend income 20,000

15-Jan-2015 Bank 20,000


Dividend receivable 20,000

(c) Date Description Debit Credit


---------- Rupees ----------
31-Dec-2014 Bank/Receivable 15,000,000
Sales 14,400,000
Deferred support service revenue (500,000+20%) 600,000
31-Dec-2014 Deferred support service revenue 100,000
Cost of support services 83,333
Sales support service - (600,000×50%×4/12) 100,000
Bank/payables 83,333
Revenue and cost of services for 4 months

Page 2 of 6
Financial Accounting and Reporting-I
Suggested Answers
Certificate in Accounting and Finance – Spring 2015

(d) Revenue is normally recognised when the buyer accepts delivery, and installation
and inspection are complete. However, revenue is recognised immediately upon the
buyer’s acceptance of delivery when:
(i) the installation process is simple in nature, for example the installation of a
factory tested television receiver which only requires unpacking and
connection of power and antennae; or
(ii) the inspection is performed only for purposes of final determination of
contract prices, for example, shipments of iron ore, sugar or soya beans.

Ans.4 (a) The element of financial statement are as under:


(i) The elements directly related to the measurement of financial position in the
statement of financial position are assets, liabilities and equity.
(ii) The elements directly related to the measurement of performance in the
statement of comprehensive income are income and expenses.

(b) XYZ Limited


Statement of Cash flow
For the year ended 31 December 2014

Rs in million
Cash flow from operating activities
Profit before taxation as revised (W-1) 253
Adjustments for non-cash items and other changes:
Depreciation (W-3) 228
Loss on disposal of machine (W-2) 5
Interest expense (200×9%) 18
(Increase) / decrease in stock (703–910) (207)
(Increase) / decrease in account receivables (418–342)(W-4) 76
Increase / (decrease) in trade payables (228–150) 78
198
Finance cost paid (18–13) (5)
Income tax paid (80+13–85) (8)
(13)
Net cash flow from operating activities 438

Cash flow from investing activities


Capital expenditure- machine purchased amount paid (30)

Cash flow from financing activities


Loan 200
Net increase/decrease in cash & cash equivalents 608
Cash & cash equivalents at beginning of the year 243
Cash & cash equivalents at end of the year 851

WORKINGS:
W-1: Profit before tax Rs. in million
Profit before tax (as given) 275
Depreciation on addition of machine (2)
Reversal of depreciation excess provided 4
Loss on disposal (W-2) (5)
Additional provision for bad debts (19)
Profit before tax 253
Page 3 of 6
Financial Accounting and Reporting-I
Suggested Answers
Certificate in Accounting and Finance – Spring 2015

W-2: Gain/loss on disposal of fixed assets


Rs. in million Rs. in million
Cost 40 Trade-in allowance 20
Accumulated depreciation 15
Loss on disposal - Bal fig 5
40 40

W-3: Depreciation expense


Rs. in million Rs. in million
Depreciation on:
Land 5 Disposal (Rs. 40 million×10%) 4
Building 50 Transfer to P & L account 228
Plant & Machinery 175
Addition (20m ×10%) 2
232 232
W-4: Account receivable 2014 2013
----- Rs. in million -----
Closing balance 361 418
Add: Allowance for bad debts [2014: 361×5/95; 2013: 418×5/95] 19 22
380 440
Less: write off (20) -
Closing balance - Gross 360 440
Less: Provision for bad debts (5%) (18) (22)
Closing balance – Net of provision 342 418

W-5: Provision for bad debts


Rs. in million Rs. in million
Write off during the year 20 Opening balance 22
Other adjustment 3 Provision for the year 19
Closing balance (W-4) 18
41 41

Ans.5 (a) Disclosure requirements for inventory


IAS 2 requires the following disclosures in notes to the financial statements:

 The accounting policy adopted for measuring inventories, including the cost
measurement method used.
 The total carrying amount of inventories, classified appropriately. (For a
manufacturer, appropriate classifications will be raw materials, work-in-
progress and finished goods)
 The amount of inventories carried at net realizable value or NRV.
 The amount of inventories written down in value, and so recognized as an
expense during the period.
 Details of any circumstances that have led to the write-down of inventories to
NRV. OR Reasons for write down of inventories.
 The amount of any reversal of any write-down that is recognized as a
reduction in the amount of inventories recognized as expense in the period.
 The circumstances or events that led to the reversal of a write-down of
inventories. OR Reasons for reversal of write down of inventories.
 The carrying amount of inventories pledged as security for liabilities.

Page 4 of 6
Financial Accounting and Reporting-I
Suggested Answers
Certificate in Accounting and Finance – Spring 2015

(b) Cost of Goods Sold: Rs. in ‘000


Opening stock of raw material and packing material (5,000 + 700) 5,700
Add: Purchases [(60,500×100/117+2,050) OR (60,500–8,791+2,050)] 53,759
Transportation cost 300
Available for consumption 59,759
Less: Closing stock (285 + 7780) (8,065)
Raw and packing materials consumed 51,694
Salaries and wages (2,500×60%×70%) 1,050
Manufacturing overheads (W-1) 7,650
Prime cost 60,394
Work in process- opening 1,950
Less: Closing work in progress (3,000)
Cost of goods manufactured 59,344
Opening stock of finished goods (2962-300) 2,662
Cost of goods available for sale 62,006
Less: Closing stock [4162-200-(75×40%)] (3,932)
58,074

W-1: Manufacturing overheads


Variable overheads (5,000×80%) 4,000
Rent factory 2,000
Rent warehouse (500×60%) 300
Salaries (2,500×60%×30%) 450
Other fixed overheads (1,500×60%) 900
7,650

Following items were to be ignored:


Warehouse rent for finished goods
Rent for shelf spacing
Settlement discount on raw material purchases

Ans.6 (a) The capitalisation of the raw materials, labour and depreciation of plant &
machinery is correct as these costs were necessarily incurred in bringing the asset to
a location and condition enabling it to be used. However, the following costs
should not be capitalized:
 The materials destroyed – all unnecessary wastage is expressly not allowed to
be capitalised per IAS 16.
 Administration overheads of Rs. 140,000 – unless it can be proved that these
costs were directly linked to the manufacture of the machine.
 IAS 16 doesn’t allow to capitalize internal profit

Property, plant and equipment must be depreciated from the date on which it first
becomes available for use. AHL provided depreciation expense for eight months
i.e. from the date of commercial production, which is not in accordance with the
requirement of IAS-16. This machine should be depreciated for 10 months i.e. from
1 March 2014.

Page 5 of 6
Financial Accounting and Reporting-I
Suggested Answers
Certificate in Accounting and Finance – Spring 2015

Journal entry:
Date Description Debit Credit
31-Dec-2014 Loss due to fire 40,000
Administration expense 140,000
Other income/retained earning 164,000
Accumulated depreciation 12,267
Machine 344,000
Depreciation 12,267

(b) AHL’s decision to capitalize the cost of inspection into buses account is correct.
However, buses are depreciated over the useful life whereas major inspection
carried out by AHL should be depreciated over three years (next inspection date).
Therefore, AHL should amortize the inspection cost for 4 months of this year.

Date Description Debit Credit


01-Sep-2014 Buses – major inspection 1,800,000
Bank 1,800,000

31-Dec-2014 Depreciation expense (7/5)+(6/3) 1,600,000


Accumulated depreciation 1,600,000

(c) A newly acquired asset should be brought into the accounting records at the fair
value. Where this fair value is not available, the fair value of the exchanged asset
should be used instead (in this case, Rs. 8.0 million).

Further, IFRS doesn’t allow future savings to be recognized as an asset.

Consequently, the carrying amount of Machine X should first be reduced to Rs. 8


million, being its true fair value and record the impairment loss of Rs. 1.5 million.

Date Description Debit Credit


31-Dec-2014 Impairment expense 1,500,000
New machine 1,500,000

(THE END)

Page 6 of 6
INSTITUTE OF CHARTERED ACCOUNTANTS OF PAKISTAN

EXAMINERS’ COMMENTS

SUBJECT SESSION
Financial Accounting and Reporting-I Certificate in Accounting and Finance
– Spring 2015

General:

The performance in this attempt was almost the same as the last two attempts. Poor
performances were witnessed in question 3 and 6 which required knowledge of
accounting standards. Good performance was witnessed in the remaining questions.

Question-wise comments:

Question 1

This was a simple question on preparation of trading and profit and loss account and
balance sheet for a recently acquired sole proprietorship business. It was the best
attempted question. The errors observed were as follows:

 While calculating the purchases:

o Stolen stock of Rs. 50,000 was not deducted.


o Creditors closing balance of Rs. 82,500 and accrued purchases of Rs. 5,200
were not taken into consideration.

 Depreciation on furniture purchased on July 1, was charged for full year instead of 6
months.

 Goodwill was not taken into account.

 Receipt of Rs. 20,000 from insurance company was not recorded.

 In capital account, Rs. 480,000 which were paid by Baber, the owner, from his
personal account, were ignored.

Question 2

This was a simple question relating to maintenance of branch account in Head Office
books. It was the second best attempted question and majority of the students were able
to secure passing marks.

Page 1 of 5
Examiners’ Comments on Financial Accounting and Reporting-I - Spring 2015

The commonly observed errors were as follows:

 The question specifically required Branch Account in the books of Head Office,
showing the profit made by the branch. Some students mis-understood the
requirement and tried to prepare Profit and Loss Account.

 Trade discount, sales returns and bad debts related to branch debtors were charged to
Branch Account instead of Branch Debtors Account.

 Adjustment for Goods in Transit was taken on debit side only.

 Adjustment for loading of profit was calculated as 25/100 of the value of goods
received by the branch instead of 25/125 of the value. The main reason for such a
mistake was that candidates failed to appreciate that according to the question the
value of goods transferred had been taken from branch’s records which meant that
profit was already loaded on it.

 Depreciation expenses were debited to branch account. The correct entry was to debit
the cost of fixed assets purchased and credit the net book value at year-end.

 While calculating the cash transferred by branch, cash in hand and at the bank were
not deducted.

Question 3

This question tested the concept of revenue recognition. It consisted of four parts. In parts
(a) and (d) the candidates were required to explain the accounting treatment in the given
situations whereas in part (b) and (c) they were required to pass journal entries. The
overall performance was quite poor. Part-wise performance is discussed below:

(a) In this part, most of the students recommended that no revenue should be
recorded. However, in the given situations, IAS allows recording of that portion of
the revenue which is recoverable.

(b) Majority of the students passed the journal entry correctly. The most common
error was that dividend income was recorded on 3rd December instead of 31st
December.

(c) This part proved difficult and it was quite evident that majority of the students had
not studied this aspect. Many students did not attempt it altogether. Among those
who did attempt, only few could pass fully correct entries. Mostly, there were
three types of mistakes as discussed below:

 Sales were recorded at Rs. 14.5 million and deferred services revenue at
Rs. 0.5 million i.e. profit and cost was ignored.

 Sales were recorded at Rs. 15 million and provision was made for expenses.

Page 2 of 5
Examiners’ Comments on Financial Accounting and Reporting-I - Spring 2015

 Sales and deferred revenue were recorded correctly but entry to record the cost
and revenue for the four months i.e. September to December 2014 was
ignored.

(d) This was a poorly attempted question. Most of the students replied in a single line
that revenue would be recognized when installation and inspection is complete.
The exceptions allowed by the IAS were rarely mentioned. Further, many students
narrated lengthy general revenue recognition criteria which were not relevant in
the context of this part of the question.

Question 4 (a)

This part of the question was quite easy and majority of the students secured full marks.
Some of them were however confused and mentioned the components of financial
statements instead of the Elements.

Question 4(b)

This question was quite simple but majority of the students were unable to understand the
non-routine adjustments/situations though the routine calculations were performed well.

The common errors were as follows:

 Only few students worked out revised profit before tax after taking into account the
adjustments identified by the CFO.

 Additions and disposals of machinery were ignored, while calculating the


depreciation.

 Reconciliation of cash and cash equivalents at beginning and end of the year were
missed in many cases.

 Provision for bad debt could not be worked out correctly as most of the students did
not seem to understand the whole process of calculating the required balance of the
provision for bad debts and the impact of write off thereon. They may seek guidance
from ICAP’s suggested answers.

Question 5 (a)

This part of the question was well attempted by most of the students. However, the
following requirements were commonly missed by the students:

 reasons for write down; and


 reasons for reversal of write down.

Moreover, a number of students only mentioned one requirement i.e. that inventory
should be shown under the heads raw material, work-in-progress and finished goods.

Page 3 of 5
Examiners’ Comments on Financial Accounting and Reporting-I - Spring 2015

Question 5(b)

This question required preparation of a note on cost of goods sold as it would appear in
the profit and loss account. The performance was average.

Commonly observed errors were as follows:

 Various items forming part of the note were not arranged in proper sequence.

 While calculating purchases, amount inclusive of sales tax was taken. Further, some
students calculated net purchases using 17/117 of the gross amount instead of
100/117 of the gross amount.

 In opening stock of finished goods, effect of obsolescence of Rs. 300,000 was


ignored.

 Purchases of packing material and its stock were not taken into consideration.

 Salaries of manufacturing staff were not allocated correctly between direct labour and
manufacturing overheads.

 Some students tried to prepare income statement which was not required.

Question 6

This question was based on IAS-16. Three situations were given and in each case the
candidates were required to suggest appropriate accounting treatment.

It was the most poorly attempted question. Many students did not read the question
carefully and failed to realize that they have to pass the necessary entry and also to
substantiate their point of view with reference to IAS-16. Most of them passed the entry
but did not give any explanation.

Performance in each part is discussed below:

(a) According to the scenario, a machine had been fabricated in-house. The details of
cost at which the machine was capitalized were given in the question. The
performance was average. The common mistakes were as follows:

 Very few students could highlight the fact that depreciation should be charged
on new assets when these are available for use and accordingly, in this case,
depreciation should have been charged for 10 months instead of 8 months.

 Many students replied incorrectly that depreciation on assets used in the


fabrication cannot be capitalized.

 Majority of the students were able to specify correctly that administration


expenses should not be capitalized but they could not explain further that even
administration expenses can be capitalized if it is established that these costs
were directly linked to the manufacturing of the machine.

Page 4 of 5
Examiners’ Comments on Financial Accounting and Reporting-I - Spring 2015

 Instead of passing the correcting entry, many students passed the complete
entry as if no entry had previously been made in the accounts.

(b) A poor performance was witnessed in this question as most of the students were
of the view that inspection costs should be written off immediately. In fact,
inspection costs are required to be deferred and amortized over three years i.e. the
next inspection date.

(c) Many students passed the journal entry correctly but only few could explain their
point of view with reference to IAS-16. For proper explanation, they may seek
guidance from ICAP’s suggested answers.

THE END

Page 5 of 5
Certificate in Accounting and Finance Stage Examinations
The Institute of 9 September 2015
Chartered Accountants 3 hours – 100 marks
of Pakistan Additional reading time – 15 minutes

Financial Accounting and Reporting-I


Q.1 Mr. Razi, a sole proprietor, runs a small business. On 30 June 2015, he realized that his cash
and bank balances have reduced considerably. He suspected that one of his employees is
involved in misappropriation. He has provided you the following information:

Opening balances on 1 July 2014 Rs. in ‘000’


Cash and bank 389
Debtors 1,560
Stock 856
Land 450
Equipment – WDV (purchased on 1 April 2014 at a cost of Rs. 600,000) 585
Creditors 1,348
Accrued expenses: Marketing 30
Utilities 25
Salaries 48
Other miscellaneous 15

Receipts and payments for the period from 1 July 2014 to 30 June 2015 Rs. in ‘000’
Receipts from cash sales 1,728
Receipts from debtors 4,475
Payments made to creditors 4,774
Payments for marketing expenses 205
Payments for utility expenses 240
Payments for salaries 600
Payments for other miscellaneous expenses 107
Equipment (purchased on 1 October 2014) 250
Withdrew by Razi for his personal expenditures 125

Other information:
(i) Razi makes 35% margin on gross sales price. However, during the year, he offered 5%
discount on credit sales and 10% discount on cash sales. 70% of his total sales were on
credit.
(ii) Actual bills for the year were as follows:

Rs. in ‘000’
Marketing expenses 200
Utility expenses 250
Other misc. expenses 100

(iii) Salary of the staff was Rs. 52,000 per month.


(iv) Balances of debtors and creditors as on 30 June 2015 were Rs. 1,091,000 and
Rs. 1,195,000 respectively.
(v) Closing stock at 30 June 2015 was Rs. 1,167,000. It included 150 units costing
Rs. 1,500 each which were damaged and Razi incurred Rs. 900 per unit in July 2015
to bring them into saleable condition.
(vi) Razi depreciates equipment on straight line basis at the rate of 10% per annum.

Required:
Prepare income statement for the year ended 30 June 2015 and balance sheet as at
30 June 2015. Also compute the amount of cash shortage, if any. (19)
Financial Accounting and Reporting-I Page 2 of 5

Q.2 Following is the summarised trial balance of Eagles Limited (EL) as at 30 June 2015:
Debit Rs. in ‘000’ Credit Rs. in ‘000’
Plant 2,500 Accumulated depreciation at 1 July 2014
Equipment 700 – Plant 1,000
Stock as on 1 July 2014 1,500 – Equipment 270
Trade debtors 1,300 Provision for obsolete stock at 1 July 2014 45
Cash and bank 1,759 Provision for bad debts at 1 July 2014 48
Purchases 6,987 Capital 2,500
Salaries & wages 843 Accumulated profits 960
Warehouse rent 740 Trade creditors 1,545
Repair and maintenance 500 Revenue 10,706
Utilities expenses 400 Other income 425
Insurance expenses 300 Accruals at 1 July 2014
Bad debt written off 30 – Repairs & maintenance 45
Obsolete inventory written off 40 – Utilities expenses 55
17,599 17,599

Additional Information:
(i) The sales include goods supplied on 27 June 2015 to a customer at a price of
Rs. 390,000 on a sale or return basis. The goods were returnable by 15 July 2015. EL
sells such goods at a mark-up of 30% on cost.
(ii) Other income includes proceed from sale of an equipment amounting to Rs. 100,000
received on 31 December 2014. The cost and written down value of the equipment at
1 July 2014 were Rs. 200,000 and Rs. 70,000 respectively.
(iii) Plant and equipment are depreciated at the rate of 10% and 15% respectively on
straight line basis.
(iv) Cost of stock on 30 June 2015 was Rs. 1,400,000, having net realizable value of
Rs. 1,450,000.
(v) The management estimates that:
 5% of trade debts would not be recovered.
 3% of the stock is obsolete.
(vi) Current warehouse rent is Rs. 600,000 per annum which was paid in advance on
1 October 2014.
(vii) Following bills for expenses were received but not entered in books:
Rs. in ‘000’
Repair and maintenance 56
Utilities expenses 67
(viii) The company revalued its non-current assets on 31 December 2014. Valuer has
suggested following fair values:

Rs. in ‘000’
Plant 1,650
Equipment 175
(ix) The tax charge for the current year after making all related adjustments is estimated at
Rs. 200,000.
(x) No entry has been made in respect of disposal, revaluation and depreciation of fixed
assets.

Required:
Prepare statement of financial position as at 30 June 2015 and statement of comprehensive
income for the year ended 30 June 2015. (Deferred tax implication is to be ignored) (19)

Q.3 (a) When a company follows revaluation model for subsequent measurement of its
Property, Plant and Equipment, it is required to provide certain additional disclosures
(as compared to cost model). Specify such disclosures as have been mentioned in
IAS 16 ‘Property, Plant and Equipment’. (03)
Financial Accounting and Reporting-I Page 3 of 5

(b) PQR Enterprises was incorporated on 1 July 2012. The company depreciates its
property, plant and equipment on straight line basis over their useful life. It uses
revaluation model for subsequent measurement of the property, plant and equipment
and has a policy of revaluing these after every two years.

Following information pertains to its property, plant and equipment:

Value as determined Useful life in years


Cost as on WDV as on
by professional valuer Original at Remaining as
Assets 01-07-2013 01-07-2013
on 30-06-2014 acquisition determined by
---------------- Rs. in million ---------------- valuer
Office building 6,000 5,500 5,750 12 8
Factory building 4,400 3,960 3,320 10 9
Warehouse 4,500 4,050 3,350 10 8

During the year there were no addition or deletion in the above assets.

As per policy, PQR transfers the maximum possible amount from the revaluation
surplus to retained earnings on an annual basis.

Required:
Prepare necessary journal entries for the year ended 30 June 2014 and 2015. (12)

Q.4 Diamond Limited has its head office in Karachi and two branches in Lahore and Quetta.
Balances of its head office and branch operations for the year ended 30 June 2015 are as
under:

Head office Lahore Quetta


Karachi branch branch
-------------- Rs. in million --------------
Inventory as at 1 July 2014 400 30 48
Sales 4,800 1,550 1,198
Purchases 3,800 230 200
Expenses 500 276 202
Head office – current account (Cr.) - 200 178
Lahore branch – current account (Dr.) 230 - -
Quetta branch – current account (Dr.) 235 - -
Goods sent to branches 1,760 - -
Goods received from head office - 1,070 618
Provision for unrealized profit - 1 July 2014 13 - -

Additional information:
(i) Head office transfers goods to branches at cost plus 20%.
(ii) Inventory as at 30 June 2015:

Rs. in million
Head office 375
Lahore branch 28
Quetta branch 150

(iii) Goods worth Rs. 20 million and Rs. 52 million sent to Lahore branch and Quetta
branch respectively were in transit at year-end.
(iv) Cash transfers to head office by Lahore branch and Quetta branch amounting to Rs. 10
million and Rs. 5 million respectively were in transit at year-end.

Required:
(a) Prepare statement of comprehensive income for the year ended 30 June 2015 showing
the total profit/loss as well as profit/loss earned by the head office and two branches. (10)
(b) Reconcile the balances between the head office and the two branches. (02)
Financial Accounting and Reporting-I Page 4 of 5

Q.5 (a) Describe the term ‘revenue’ and state when and how revenue shall be recognised in the
case of royalties and dividend. (03)
(b) Adnan Limited (AL) is a supplier of machinery and spare parts. The machines supplied
are installed by AL. Following transactions took place in the last week of the
accounting year i.e. 30 June 2015:
(i) A machine was delivered to a customer. The invoiced amount was Rs. 500,000.
In accordance with the Operating Manual, the customer had to arrange a voltage
stabiliser before connecting the machine to the power supply. Machine became
operational on 1 July 2015. (02)
(ii) A specialised machine was sold to Sun Technologies (ST) for Rs. 800,000. ST
agrees to make the payment on 7 July 2015. However, ST informed AL that it
would accept the delivery in the month of August 2015. (03)

Required:
Applying the principles of IAS 18, explain when revenue from the sale of above
machines may be recorded.

(c) (i) On 31 March 2015 a machine was sold under a package deal. The package
includes a machine with free after sale service for 2 years at a total price of
Rs. 50,000. Selling price of standalone unit is Rs. 40,000. Cost of providing after
sales service is estimated at Rs. 4,000 per year. (03)

(ii) A machine was delivered to the customer on 1 July 2014. However, the invoice
was raised on 30 September 2014. According to the invoice, the total price of
Rs. 300,000 is to be paid in 2 half yearly installments of Rs. 150,000 each,
commencing from 1 January 2015. Appropriate discount rate is 10% per annum.
The present value of these two half yearly installments is to be taken as
Rs. 278,912. (03)

Required:
Prepare necessary journal entries to record the above transactions in the books of
Adnan Limited for the year ended 30 June 2015.

Q.6 A company deals in Solar Panels which are imported from China. The company follows a
perpetual inventory system and values its inventory on weighted average basis. Details of
sales and purchases during the year ended 30 June 2015 are as follows:
(i) Opening inventory on 1 July 2014 amounted to Rs. 49,000,000 and consisted of 2,450
solar panels.
(ii) Purchases during the year were as follows:
Date Quantity (Units) Price (Rs. in ‘000’)
30-Sep-2014 4,200 78,120
31-Mar-2015 4,350 87,000
Costs related to imports were 29% of purchase cost, of which 17% is refundable.
(iii) Sales during the year were as follows:
Date Quantity (Units) Price (Rs. in ‘000’)
31-Jul-2014 2,100 52,500
31-Oct-2014 2,050 48,750
28-Feb-2015 2,300 55,200
15-May-2015 2,260 53,110
(iv) Sale on 31 October 2014 includes 100 solar panels which were damaged during the
year and sold at Rs. 12,000 per unit.
(v) On 31 May 2015, 50 solar panels were totally damaged and were written off.
(vi) On 30 June 2015 there was a significant decline in the prices of solar panels as a new
type of solar panel was introduced in the market. Selling prices are now Rs. 18,500 per
unit. However, the company has made some modification in its product which will
enable it to sell it at Rs. 22,000 per unit. Cost of modification is Rs. 2,500 per unit.
Financial Accounting and Reporting-I Page 5 of 5

Required:
Prepare disclosure of inventories in the financial position as at 30 June 2015 in accordance
with the requirements of IAS-2 ‘Inventories’. (Note: Accounting policy note and comparative
figures are not required) (13)

Q.7 A manager is interested in knowing the relationship between machine hours and production
expenses. Data collected for January 2015 to August 2015 is as follows:

Production expenses
Months Machine hours
(Rs. in million)
January 264 50
February 390 90
March 280 70
April 355 85
May 375 100
June 330 75
July 300 70
August 290 60

Required:
Develop relationship between production expenses and machine hours and predict
production expenses if machine works for 365 hours. (08)

(THE END)
Financial Accounting and Reporting-I
Suggested Answers
Certificate in Accounting and Finance – Autumn 2015

A.1 Mr. Razi


Income Statement
For the year ended 30 June 2015
Rupees
Net sales (W-1) 5,984,000
Opening stock 856,000
Purchases (balancing) 4,471,000
Closing stock (1,167,000)
Cost of goods sold [6,400,000(W-1)65%] 4,160,000
Adjustment for NRV on damaged stock (W-2) 13,800
4,173,800
Gross profit 1,810,200
Marketing expenses (W-6) (200,000)
Utility expenses (W-6) (250,000)
Salaries (W-6) (624,000)
Other misc. expenses [100,000(W-6)+150,000(W-5)+250,000(W-3)] (500,000)
Depreciation expense (600,00010%+250,00010%9÷12) (78,750)
Net profit 157,450

Mr. Razi
Balance Sheet
As at 30 June 2015
Rupees
Non-Current Assets
Land 450,000
Office equipment
Cost (600,000+250,000) 850,000
Accumulated depreciation (15,000+78,750) (93,750)
756,250
1,206,250
Current Assets
Stock [1,167,000-13,800(W-2)] 1,153,200
Debtors (W-3) 1,091,000
Bank (W-4) 291,000
2,535,200
Total Assets 3,741,450

Equity
Razi's capital opening 2,374,000
Profit for the year 157,450
Drawings (125,000)
2,406,450
Current Liabilities
Creditors (W-5) 1,195,000
Accrued expenses (W-6) 140,000
Total Equity and Liabilities 3,741,450

Rupees
Cash misappropriated in debtors (W-3) 250,000
Cash misappropriated in creditors (W-5) 150,000
Cash shortage 400,000
Page 1 of 8
Financial Accounting and Reporting-I
Suggested Answers
Certificate in Accounting and Finance – Autumn 2015

Workings:

W-1: Determination of gross sales revenue and discount allowed


Net sales Discount allowed Gross sales
Cash sales 1,728,000 192,000 1,920,000
(Given) (1,728,0000.1/0.9 (1,728,000+192,000)
Credit sales 4,256,000 224,000 4,480,000
(4,480,000-224,000) (4,480,0005%) (1,920,00070÷30)
Total 5,984,000 416,000 6,400,000

W-2: Adjustment for NRV on damaged stock Rupees


Selling price of damaged stock (1,500 ÷ 0.65) 2,308
Net realizable value (2,308 – 900) 1,408
NRV expense per unit (1,500 – 1408) 92

Total NRV expense (92  150) 13,800

W-3: Debtors
Rupees Rupees
Opening balance 1,560,000 Sales discount (W-1) 224,000
Gross sales (W-1) 4,480,000 Receipts 4,475,000
Cash misappropriated 250,000
Closing balance 1,091,000
6,040,000 6,040,000

W-4: Bank
Rupees Rupees
Opening balance 389,000 Payments made to creditors 4,774,000
Receipts from cash sales 1,728,000 Payment for marketing exp. 205,000
Receipts from debtors 4,475,000 Payment for utility expenses 240,000
Payment for salaries 600,000
Payment for other misc. exp. 107,000
Drawing 125,000
Office equipment 250,000
Closing balance 291,000
6,592,000 6,592,000

W-5: Creditors
Rupees Rupees
Payments 4,774,000 Opening balance 1,348,000
Closing balance 1,195,000 Purchases (income statement) 4,471,000
Cash misappropriated 150,000
5,969,000 5,969,000

W-6: Accrued expenses


A B C A+B-C
Expense for Accruals Payment during Accruals
the year 01-07-2014 the year 30-06-2015
--------------------------- Rupees ---------------------------
Marketing expenses 200,000 30,000 205,000 25,000
Utility expenses 250,000 25,000 240,000 35,000
Salaries (52,000  12) 624,000 48,000 600,000 72,000
Other misc. expenses 100,000 15,000 107,000 8,000
118,000 140,000

Page 2 of 8
Financial Accounting and Reporting-I
Suggested Answers
Certificate in Accounting and Finance – Autumn 2015

A.2 Eagles Limited


Statement of Comprehensive Income
For the year 30 June 2015
Rupees
Revenue (10,706,000 – 390,000) 10,316,000
Cost of goods sold
Opening stock 1,500,000
Purchases 6,987,000
Closing stock (1,400,000 + 300,000) (1,700,000)
Cost of goods sold 6,787,000
Gross profit 3,529,000

Salaries & wages (843,000)


Repair and maintenance (500,000 + 56,000 – 45,000) (511,000)
Utilities expenses (400,000 + 67,000 – 55000) (412,000)
Insurance expenses (300,000)
Provision for stocks (40,000 + 51,000 – 45,000) (46,000)
Warehouse rent (740,000 – 150,000) (590,000)
Bad debt expense (30,000 + 910,000*5% – 48,000) (27,500)
Depreciation expense (W-1) (273,125)
Impairment loss - equipment (W-2) (147,500)
(3,153,125)
375,875
Other income (425–100+ 30+15) or (425-100)+(100-55) 370,000
Net profit before tax 745,875
Income tax expense (200,000)
Profit after tax 548,875

Eagles Limited
Statement of Financial Position
As at 30 June 2015
Rupees
Non-current assets
Plant (1,650 – 825) 1,567,500
Office equipment (175 – 13.125) 161,875

Current assets
Stock (1,400,000 + 300,0000 – 51,000) 1,649,000
Debtors (1,300,000 – 390,000 – 45,500) 864,500
Prepaid rent 150,000
Cash & Bank 1,759,000
Total assets 6,151,875
Equity
Capital 2,500,000
Accumulated profits (960,000 + 548,875) 1,508,875
Revaluation surplus (W-2) 275,000
Current liabilities
Creditors 1,545,000
Provision for income tax 200,000
Accrued expenses (56 + 67) 123,000
Total equities and liabilities 6,151,875

Page 3 of 8
Financial Accounting and Reporting-I
Suggested Answers
Certificate in Accounting and Finance – Autumn 2015

W-1: Depreciation
Before revaluation After revaluation
Total
(6 months) (6 months)
Depreciation
Rate Cost less Depreciation Revalued Depreciation
(A+B)
disposal (A) amount (B)
--------------------------------- Rupees in ‘000’ ---------------------------------
Plant 10% 2,500 125.0 1,650 82.500 207.500
Equipment 15% 500 52.5 175 13.125 65.625
273.125

W-2: Revaluation surplus/impairment loss ------- Rs. in ‘000’-------


Revalued amount 1,650 175
Less: WDV at revaluation date
[2,500-1,000-125(W-1)]; [(700-200)-{270-(130+15)}-52.5] (1,375) (322.5)
Revaluation / (impairment) 275 (147.5)

A.3 (a) When items of property, plant and equipment are stated at revalued amounts, the following
additional disclosure should be made:
 the effective date of the revaluation;
 whether an independent valuer was involved;
 for each revalued class of property, plant and equipment, the carrying amount that
would have been recognised had the assets been carried under the cost model; and
 the revaluation surplus, indicating the change for the period and any restrictions on the
distribution of the balance to shareholders.

(b) Journal entries


Date Particulars Debit Credit
Rs. in million
30-Jun-14 Depreciation for the year (W-1) 1,390
Accumulated depreciation – Office building 500
Accumulated depreciation – Factory building 440
Accumulated depreciation – Warehouse 450
30-Jun-14 Accumulated depreciation – Office building (W-1) 1,000
Accumulated depreciation – Factory building (W-1) 880
Accumulated depreciation – Warehouse (W-1) 900
Office building 1,000
Factory building 880
Warehouse 900
30-Jun-14 Office building (W-1) 750
Loss on impairment – buildings and warehouse 450
Surplus on revaluation 750
Factory building (W-1) 200
Warehouse (W-1) 250
30-Jun-15 Depreciation expense (W-1) 1,507
Accumulated depreciation – Office building 719
Accumulated depreciation – Factory buildings 369
Accumulated depreciation – Warehouse 419

30-Jun-15 Surplus on revaluation (750÷8) 94


Retained earnings (incremental depreciation) 94

Page 4 of 8
Financial Accounting and Reporting-I
Suggested Answers
Certificate in Accounting and Finance – Autumn 2015

W-1:
Dep. for Revaluation
Revalued
Cost WDV the year Acc. Dep. surplus/ Dep. for the
amount
Assets (A) (B) 2014 D=A-B+C (impairment) year 2015
(E)
(C) F=E-(A-D)
---------------------------------- Rupees in million ----------------------------------
Office building 6,000 5,500 500 1,000 5,750 750 719
Factory building 4,400 3,960 440 880 3,320 (200) 369
Warehouse 4,500 4,050 450 900 3,350 (250) 419
1,390 1,507

A.4 (a) Diamond Limited


Statement of comprehensive Income
For the year ended 30 June 2015
Head Lahore Quetta
Adjustment Combined
office branch branch
----------------------Rs. in '000'----------------------
Sales 4,800 1,550 1,198 - - 7,548
Goods sent to branches 1,760 - - (1,760) - -
6,560 1,550 1,198 (1,760) - 7,548
Cost of sales
Inventories as at 1-Jul-2014 400 30 48 - (13) 465
Purchases 3,800 230 200 - - 4,230
Goods received from HO - 1,070 618 1,688 - -
4,200 1,330 866 1,688 (13) 4,695
Closing inventory (375) (28) (150) 72 42 583
3,825 1,302 716 1,760 29 4,112
Gross profit 2,735 248 482 - (29) 3,436
Expenses (500) (276) (202) - - (978)
Profit for the year 2,235 (28) 280 - (29) 2,458
Unrealized profit (29) - - - 29 -
2,206 (28) 280 - - 2,458

(b) Reconciliation of Head Office and Branch Balances


Head office books Lahore branch Quetta branch
Lahore branch Quetta branch
Head office current account
current account current account
----------------------Rs. in '000'----------------------
Opening balance 230 235 200 178
Less: Goods in transit - - 20 52
Less: Cash in transit (10) (5) - -
Branch Profit / loss (28) 280 (28) 280
192 510 192 510

Page 5 of 8
Financial Accounting and Reporting-I
Suggested Answers
Certificate in Accounting and Finance – Autumn 2015

A.5 (a) IAS 18 defines revenue as the gross inflow of economic benefits in a period arising in the
course of the ordinary activities of an entity when those inflows result in an increase in equity,
other than increases relating to contributions from equity participants.
In case of royalties, revenue shall be recognised on an accrual basis in accordance with the
substance of the relevant agreement.

In case of dividends, revenue shall be recognised when the shareholder’s right to receive
payment is established.

(b) (i) Where goods are subject to installation and inspection, revenue is normally recognized
only when installation and inspection are complete. However, where the installation
process is simple in nature, revenue is recognised immediately upon the buyer
accepting the goods.

This means that revenue of Rs. 500,000 from sale of machine can be recognized in the
year ended 30 June 2015.

(ii) AL should recognizes the revenue in the year ended 30 June 2015 as:
 ST takes the title;
 It is probable that delivery will be made in August 2015;
 The item is on hand, identified and ready for delivery to ST at the time the sale is
recognized;
 ST specifically acknowledges the deferred delivery instructions; and
 The usual payment terms apply and ST agrees to make the payment on 7 July
2015.

(c) (i) Date Particulars Debit (Rs.) Credit (Rs.)


31 March 2015 Bank/Receivable 50,000
Sale 40,000
Deferred revenue 10,000
30 June 2015 Deferred Revenue (10,000÷2) × 3 ÷ 12 1,250
Service fee income 1,250
30 June 2015 Cost of sales – service (4,000  1÷ 4) 1,000
Bank/Payable 1,000

(ii) Date Particulars Debit (Rs.) Credit (Rs.)


1 July 2014 Receivable (W-1) 278,912
Sales 278,912
1 January 2015 Bank 150,000
Receivable (W-1) 136,054
Interest income 13,946
30 June 2015 Accrued income 7,142
Interest income 7,142

W-1: Determination of interest amount


Present Interest for 6
Installment Due on Payment Balance
Value months at 5%
1st Installment 1 January 2015 278,912 13,946 150,000 142,858
2nd Installment 30 June 2015 142,858 7,142 150,000 -
Page 6 of 8
Financial Accounting and Reporting-I
Suggested Answers
Certificate in Accounting and Finance – Autumn 2015

A.6 2015
Rs. in ‘000’
20 – Closing inventory
Finished goods (W-1) 43,680
20.1 Closing inventory includes items costing Rs. 50,015,000 valued at net realisable value of
Rs. 43,680,000.
20.2 The inventory expenses (cost of sales) for the year is Rs. 190,254,000(W-4)
20.3 Damaged inventory of Rs. 1,116,000(W-1) has been written off.

W-1: Determination of value of closing inventory under perpetual inventory system


Value
Date Description QTY Price/unit
(Rs. in '000')
1-Jul-14 Opening 2,450 20,000 49,000
31-Jul-14 Issue 2,100 20,000 42,000
Balance 350 20,000 7,000
30-Sep-14 Purchase 4,200 20,832 (W-2) 87,494
Balance 4,550 20,768 94,494
31-Oct-14 Issue 2,050 20,768 42,574
28-Feb-15 Issue 2,300 20,768 47,766
Balance 200 20,768 4,154
31-Mar-15 Purchase 4,350 22,400 (W-2) 97,440
Balance 4,550 22,328 101,594
15-May-15 Issue 2,260 22,328 50,462
Balance 2,290 22,328 51,132
30-Jun-15 Units w/off 50 22,328 1,116
2,240 22,328 50,015
30-Jun-15 NRV adjustment (W-3) - - 6,335
30-Jun-15 2,240 19,500 43,680

W-2: Purchase cost per unit September March


Purchase price/unit 18,600 20,000
Non-refundable import costs - 12% (29 – 17) 2,232 2,400
20,832 22,400

W-3: NRV of SP Rs. in ‘000’


Cost 22,328
Selling price 22,000
Less: Cost of modification (2,500)
NRV per unit 19500
Decline in value (22,328 – 19,500) 2,828

Expense (2,240  2,828) 6,335

W-4: Cost of sales Rs. in ‘000’


Opening stock 49,000
Purchases 184,934
233,934
Less closing stock (43,680)
190,254

Page 7 of 8
Financial Accounting and Reporting-I
Suggested Answers
Certificate in Accounting and Finance – Autumn 2015

A.7 Machine Rs. in


hours million (xy) (x2)
(x) (y)

264 50 13,200 69,696


390 90 35,100 152,100
280 70 19,600 78,400
355 85 30,175 126,025
375 100 37,500 140,625
330 75 24,750 108,900
300 70 21,000 90,000
290 60 17,400 84,100
2584 600 198,725 849,846

∑ ∑ ∑

∑ ∑

Estimated regression equation is

If machine works 365 hours, then production expense would approximate:

(THE END)

Page 8 of 8
INSTITUTE OF CHARTERED ACCOUNTANTS OF PAKISTAN

EXAMINERS’ COMMENTS

SUBJECT SESSION
Financial Accounting and Reporting-I Certificate in Accounting and Finance
– Autumn 2015

General:

The overall performance in this paper was poor mainly because of extremely poor
performances in question 2, 5 and 6. Majority of the students made mistakes even in the
simpler areas. The candidates also seemed to lack in practice as it was observed that
many students displayed good understanding of a concept in one question but made
mistakes on similar issues in the other question(s). For example, many students addressed
the issue of accrued expenses in question 1 correctly but made errors in similar type of
adjustments in question 2 and vice versa.

Question-wise comments:

Question 1

This question required computation of cash defalcated and preparation of income


statement and balance sheet for a sole proprietorship business. The opening balance sheet
and summary of receipts and payments along with other necessary information was
provided in the question.

Though it was a simple and routine question, the overall performance was average
because many students made errors on the easier aspect of the question also.

The errors observed are discussed below:

 Majority of the students were unable to understand the calculation of cash shortage
because they computed credit sales on the basis of movement in debtors account and
purchases on the basis of movement in creditors account; whereas, the other
information which was provided in the question i.e. the discount on the cash and the
credit sales and the ratio of credit sales as a percentage of total sales and margin on
gross sales, was ignored.
 Net cash sales i.e. after deducting the discount was given and the cash sale before
discount should have been computed by working back the discount as 0.1/0.9 of the
net sales. Many students calculated the discount incorrectly as 10% of net sales.
 While computing the cost of goods sold, many students applied the percentage of
gross margin on the net sales instead of gross sales.
 Adjustment on account of NRV of damaged goods was mostly ignored whereas many
students deducted the expenses incurred in July from cost, to arrive at the NRV.

Page 1 of 5
Examiners’ Comments on Financial Accounting and Reporting-I - Autumn 2015

 Many students added the opening balance of accrued expenses to the amount of
payments and deducted the closing balance to arrive at the amount of expenses.
 Many students calculated depreciation on equipment on opening WDV instead of
cost.
 Many students arrived at the closing balance of cash by including amount of expenses
in their workings, instead of the amount of payments.

Question 2

This was a straight forward question on preparation of statement of financial position and
statement of comprehensive income of a company. The only difficult item in the whole
question was the effect of revaluation. However, a poor response was witnessed. Some of
the common mistakes were as follows:

 Effects of goods shipped on sale or return basis was correctly adjusted from sales but
no adjustment was made in closing stocks whereas some students added the invoice
price to the cost of ending stock instead of adding the cost of the goods.
 Many students reported closing stocks at realizable value which was higher than cost.
 Most of the students did not seem to have appropriate understanding of how expenses
are recorded in the ledger. Consequently, in calculating repairs and maintenance and
utilities expenses for the period, many students wrongly added opening accruals and
deducted closing accruals from the recorded expenses whereas many students only
added the closing accruals to recorded expenses and ignored the opening accruals.
 In the statement of comprehensive income, actual write-offs were included as
expenses i.e. opening and closing provisions were ignored. Many students added the
closing balance of the provisions and the write-offs, to arrive at the amount of
expenses and ignored the opening balance.
 Most of the students failed to understand that the amount paid as advance warehouse
rent must have been included in the warehouse rent expenses account. They
considered it as an additional adjustment and added Rs. 450 thousand to the amount
recorded in the ledger instead of deducting Rs. 150 thousand therefrom.
 Most of the students were unable to correctly calculate the adjustment required to
properly record the loss on sale of equipment. Many students ignored it altogether
whereas a large number of students ignored the depreciation for the year i.e. upto the
date of disposal.
 Most of the students failed to calculate the depreciation correctly mainly because of
the following reasons:
o Ignored the depreciation for the year on equipment disposed of.
o Ignored the impact of revaluation in their calculations.
 Most of the students made adjustment for income tax in Statement of Comprehensive
Income but did not adjust it in the amount of liability disclosed in the Statement of
Financial Position.
 Many students did not show accruals and accumulated profit in the statement of
financial position.
 Many students appeared to waste time in trying to reconcile the balance sheet which
does not contain any mark.

Page 2 of 5
Examiners’ Comments on Financial Accounting and Reporting-I - Autumn 2015

Question 3

This question on revaluation of fixed assets was better attempted and the overall
performance was relatively better. However, about 13% of the students did not attempt it
altogether. Performance in each part is discussed below:

Question 3(a)

The performance in this part was mixed. About 15% of the students were well prepared
for this type of question and scored full marks. Among the rest, majority could only point
out the requirement to disclose the revaluation surplus and the amount thereof. Some of
the students also mentioned about the simultaneous disclosures of carrying amounts
under the cost model. However, only few could mention the remaining two conditions i.e.
about the effective date of revaluation and the involvement of independent valuer.

Question 3(b)

The performance in this part was good except in the case of very poor students who either
did not attempt it altogether or did not seem to know anything. The average students
made one or more of the following mistakes:
 Entry related to transfer of surplus to retained earnings was ignored.
 Majority of the students calculated incremental depreciation as difference of
depreciation before revaluation and after revaluation rather than allocating the re-
valuation surplus over the remaining useful lives of the assets.
 The loss on impairment and revaluation surplus were not recorded separately as the
net amount was recorded as surplus.
 Depreciation for 2013-14 was ignored.
 Depreciation for 2015 was computed on the original life instead of remaining life.
 The entry to close the accumulated depreciation balances against the cost of assets
was ignored.
 Impairment and surplus on revaluation were calculated on the basis of cost instead of
written down value.
 Many students did not read the question carefully. The revalued amount pertained to
year-end whereas they assumed that it pertained to the beginning of the year.

Question 4(a)

In this question on branch accounting, the candidates were required to compute the
separate profit and loss of a head office and its two branches and also the combined profit
and loss. The overall performance was quite good. Some of the common mistakes are
discussed below:

 Provision for unrealized profit on 1 July 2014 was either ignored or deducted from
the opening inventory of the head office instead of adjusting it for the purpose of
opening inventory in the combined profit and loss account.
 Unrealized profit at year-end was ignored.
 Many students prepared individual profit and loss statements for head office and
branches but either ignored the combined profit or loss calculation or committed
mistakes in adjusting the impact of mark-up on stocks and goods in transit.

Page 3 of 5
Examiners’ Comments on Financial Accounting and Reporting-I - Autumn 2015

Question 4(b)

This part was quite easy and most of the students were able to reconcile the balances as
required.

Question 5

This question on revenue recognition proved too difficult for majority of the students and
the overall performance was very poor. Performance in each part is discussed below:

Question 5(a)

This was a theory based question and at least in this part about 25% of the students
scored passing marks and a reasonable number of students scored full marks also.

Question 5(b)

(i) In the context of the overall performance in Question 5, the performance in this
sub-part was relatively better. About 15-20% of the students were able to identify
the key issue i.e. where the installation process is simple in nature, the revenue may
be recognized when the buyer accepts the goods.

(ii) This proved to be the most difficult part. In IAS 18 this situation is termed as bill
and hold but very few students were able to analyze the situation correctly.

Question 5(c)

(i) A very important logic was tested in this part i.e. when an item is sold as a package
deal, the revenues and costs associated with various components of the deal are to
be recorded separately. It was satisfying to note that at least half the students
seemed to understand the broader principle though difficulty was faced in its
practical application.

Deferred revenue of Rs. 10,000 was correctly recorded at the time of sale.
However, adjustment at year-end related to recognition of services income and
provision of cost thereof was generally missed or the amounts were calculated
incorrectly.

(ii) This part was based on principle of revenue recognition in situations where
recovery of the amount of sale is deferred. As in part c(i), generally the students
seemed to understand the principle but failed to apply it correctly. The common
mistakes were as follows:

 Most of the candidates correctly calculated the interest element as the difference
between sale price and the present value. However, they recorded the entire
interest income at the end of the year instead of recording it separately for the
two half yearly periods. Many candidates divided the interest income equally
between the two half yearly periods. The correct method was to allocate it on
the basis of amount outstanding.

Page 4 of 5
Examiners’ Comments on Financial Accounting and Reporting-I - Autumn 2015

Question 6

In this question, the candidates were required to compute the value of inventory based on
weighted average method under perpetual inventory system and prepare relevant
disclosures as per IAS-2.

Quite surprisingly, the performance in this part remained very poor mainly because very
few candidates seemed to be aware of perpetual inventory system and went on to
compute the value of inventory under the periodic inventory system. There were many
other types of mistakes which indicated a serious lack of conceptual understanding.
These are described below:

 Majority of the students failed to understand that non-refundable portion of import


cost has to be included in the cost of inventories. Some assumed purchase price as the
total cost and deducted refundable import cost from it.
 Some students used FIFO method instead of weighted average method.
 Many candidates showed the issue of inventory at selling price instead of cost.
 Majority of the students considered sale of damaged units as additional transaction in
that month whereas it was clearly mentioned in the question that sales for the month
includes the sale of damaged units.
 Many students took the cost of stocks written off as zero.
 In NRV calculation, significant number of students considered the reduced selling
prices as NRV instead of the expected sales value less modification cost.
 Very few students made appropriate disclosures.

Question 7

This was a straightforward question on developing relationship between production


expenses and machine hours and predicting production expenses for given machine
hours. The overall performance in this question was very good as about 77% of the
students secured passing marks whereas 29% students secured full marks.

Most of the errors were on account of incorrect formula. Selective study was quite
evident in this case as 13% of the students did not attempt it altogether.

THE END

Page 5 of 5
Financial Accounting and Reporting-I
Summary of Marking Key
Certificate in Accounting and Finance – Autumn 2015

Note regarding marking scheme:


The marking scheme is given as a guide. However, markers were also advised to award marks for
alternative approaches to a question and relevant/well-reasoned comments/explanations.

A.1 Mark(s)
 Income Statement
 Sales 3.0
 Cost of sales 3.0
 Expenses 2.5
 Presentation 0.5
 Balance sheet
 Non-current assets 0.5
 Current Assets
o Stock 1.0
o Debtors 1.0
o Bank 2.0
o Creditors 1.0
o Accrued expenses 1.0
o Razi’s capital 1.0
o Presentation 0.5
 Determination of cash shortage 2.0

A.2 Mark(s)
 Statement of comprehensive income
 Sales 1.0
 Cost of sales 1.5
 Expenses other than depreciation and impairment 4.5
 Depreciation and impairment 4.5
 Presentation 0.5
 Statement of financial position
 Non-current assets 1.0
 Current assets 1.5
 Equity including revaluation surplus 3.0
 Current liabilities 1.0
 Presentation 0.5

A.3 (a) Mark(s)


 Up to 01 mark for each disclosure 3.0

(b) Mark(s)
 Preparation of accounting entries relating to:
 Depreciation expense for the year 2014 and 2015 2.0
 Reversal of accumulated depreciation at 30 June 2014 1.5
 Recording of effects of revaluation at 30 June 2014 5.5
 Adjustment of incremental depreciation from surplus 3.0

Page 1 of 2
Financial Accounting and Reporting-I
Summary of Marking Key
Certificate in Accounting and Finance – Autumn 2015

A.4 (a) Mark(s)


 Statement of comprehensive income
 Sales 0.5
 Adjustment of goods sent to branch/goods received from Head office 3.0
 Opening and closing inventories 4.0
 Purchases 0.5
 Expenses 0.5
 Adjustment of unrealized profit 1.0
 Presentation 0.5

(b) Mark(s)
 Reconciliation of branch balances with Head office
 Adjustment relating to:
o Goods in transit 0.5
o Cash in transit 0.5
o Branch profit/loss 0.5
 Opening and closing balances of branch and Head office 0.5

A.5 (a) Mark(s)


 Definition of revenue 1.0
 Recognition of revenue in case of royalty 1.0
 Recognition of revenue in case of dividend 1.0

(b) Mark(s)
 Explanation of recognition of revenue if goods delivered are subject to
installment 2.0
 Explanation of recognition of revenue in the case of deferred delivery 3.0

(c) Mark(s)
 Preparing journal entries relating to:
 recording of deferred revenue on the date of transaction 1.0
 recognizing the service fee income at year end 1.0
 recognizing the cost of service fee at year end 1.0
 Preparing journal entries relating to:
 recognizing the credit sales 1.0
 recording the installment received along with interest income 1.0
 recording the accrual at year end 1.0

A.6 Mark(s)
 Determination of purchase cost 1.5
 Determination of NRV adjustment 2.0
 Determination of value of closing inventory under perpetual system 6.0
 Presentation and disclosure 3.5

A.7 Mark(s)
 Computation of fixed costs 3.0
 Computation of variable cost per unit 3.0
 Developing the relationship between production expense and hours 0.5
 Predicting the production expenses if machine works for 365 hours 1.5

(THE END)

Page 2 of 2
Certificate in Accounting and Finance Stage Examinations
The Institute of 9 March 2016
Chartered Accountants 3 hours – 100 marks
of Pakistan Additional reading time – 15 minutes

Financial Accounting and Reporting-I


Q.1 Seaview Club started its operations on 1 February 2015. Sponsor of the club contributed
Rs. 50 million towards general fund for the start of operations and placed the amount in the
bank. Following is the receipts and payments summary for the period from 1 February 2015
to 31 December 2015:
Receipts Rs. in ‘000 Payments Rs. in ‘000
Sponsor's contribution 50,000 Furniture & fixtures 1,200
Joining fees 20,800 Van 1,500
Subscription from members 29,952 Salaries 1,000
Sale of beverages 1,500 Rent 3,600
Utilities 570
Insurance 120
Repairs and maintenance 275
Purchase of beverages 1,367
Advance for plot of land 65,000
Balance 27,620
102,252 102,252
Additional information:
(i) The joining fee for award of membership is Rs. 50,000. Annual subscription is
Rs. 24,000. All new members pay three years’ subscription in advance. The
memberships were awarded as follows:
Month March June September December
No. of members 112 98 101 105
(ii) The club sells beverages at a gross profit margin of 20%. All sales are billed in the first
week of the next month and the payment is received in the same month. Sale of
beverages during December 2015 amounted to Rs. 150,000.
(iii) 25% of total purchases of beverages made during the year remained unsold at
year-end.
(iv) Salaries are paid on the first day of next month. The amount of salaries includes an
advance amounting to Rs. 10,000 paid to an employee on 1 December 2015. The
advance is repayable on 1 February 2016.
(v) Rent for three years was paid in advance on 1 February 2015.
(vi) Presently the club is operating on rental premises. However, a plot of land has been
purchased on which construction would commence shortly. Title of land would be
transferred after completion of legal formalities.
(vii) Payments for utilities include security deposit paid to utility companies amounting to
Rs. 20,000. Utility bills are paid on the 7th day of the next month.
(viii) Insurance premium was paid on 1 February 2015 covering a period of 12 months.
(ix) Repairs and maintenance include an advance of Rs. 100,000 paid to a contractor for
construction of a parking shed. Repair bills amounting to Rs. 7,000 were outstanding
at year-end.
(x) Furniture & fixtures and van were purchased on 1 February 2015. Depreciation on
these assets is to be charged at 10% and 20% respectively.
Required:
Prepare statement of financial position as at 31 December 2015 and income & expenditure
account of Seaview Club for the period ended 31 December 2015. (20)
Financial Accounting and Reporting-I Page 2 of 5

Q.2 AK Limited follows a perpetual inventory system. Following information is available from
the accounting records for the month of January 2016:

Quantity Amount (Rs.)


Inventory as at 31 December 2015 3,500 35,000
Purchase on 7 January 2016 3,700 44,400
Purchase on 13 January 2016 4,200 58,800
Purchase on 31 January 2016 2,000 26,000
Sale on 12 January 2016 3,000 60,000
Sale on 25 January 2016 5,500 115,500

Additional information:
(i) 100 units out of 4,200 units purchased on 13 January 2016 were found defective and
returned to supplier on 28 January 2016.
(ii) Inventory count conducted on 31 January 2016 revealed that 4,820 units were
physically available.

Required:
(a) Prepare inventory ledger cards for the month of January 2016 under the perpetual
system showing quantity, unit cost and value under each of the following basis of
inventory valuation:
 FIFO (07)
 Weighted average (06)

(b) Under weighted average method, prepare journal entries to record the defective items
returned to supplier and surplus/shortfall in the inventory due to physical count. (02)

Q.3 (a) In respect of sale of goods, give any two examples of each of the following situations:
(i) Legal title passes but the risks and rewards are retained.
(ii) Legal title does not pass but the risks and rewards are passed on to the customer. (03)

(b) State how revenue should be recognised in the following cases:

(i) Karim Industries Limited (KIL) has sold a machine on credit to Yawar
Engineering (YE). The machine would be used by YE if it is able to secure a
contract for providing services to AMZ & Company. KIL has agreed that the
machine may be returned at 90% of the price, if YE fails to secure the contract. (02)

(ii) Asif Electronics (AE) is about to sell a new type of food factory. Since customer
demand is high, AE is taking advance against orders. The selling price has been
fixed at Rs. 7,000 per unit and so far 175 customers have paid the initial 25%
deposit which is non-refundable. (02)

(iii) Nazir Engineering Limited (NEL) entered into a contract for the provision of
services over a period of two years. The total contract price was Rs. 25 million
and NEL had initially expected to earn a profit of Rs. 5 million on the contract.
However, the contract had not progressed as expected. In the first year, costs of
Rs. 12 million were incurred. Management is not sure of the ultimate outcome
but believes that at least the costs on the contract would be recovered from the
customer. (02)

(c) Abid Textile Mills Limited (ATML) sold a property to a financial institution for
Rs. 90 million when the fair value and carrying value of the property was
Rs. 100 million and Rs. 95 million respectively. However, there is an agreement
between the parties whereby ATML could repurchase the property after one year for
Rs. 99 million.

State how the above transaction should be recorded in ATML’s records. (03)
Financial Accounting and Reporting-I Page 3 of 5

Q.4 (a) What conditions must be satisfied if an item has to be recognised as property, plant
and equipment? Also state at what amount such item shall be carried after the initial
recognition if the entity is following the revaluation model. (03)
(b) On 1 January 2013 Delta acquired a specialized machine for its production
department. The available information is as follows:
Rupees
List price of machine 9,200,000
Freight charges 263,000
Electrical installation cost 245,000
Staff training for use of machine 351,000
Pre-production testing 193,000
Purchase of a three-year maintenance contract 528,000
Estimated residual value 175,000

Trade discount on list price 5%


Early settlement discount taken 3%
Estimated life (in machine hours) 12,000
Machine hours used during the years ended 31 December 2013, 2014 and 2015 were
2000, 3200 and 1400 respectively.
On 1 January 2015 Delta decided to upgrade the machine by adding new components
at a cost of Rs. 1,753,000. This upgrade led to a reduction in the production time per
unit of goods being manufactured by the machine. The upgrade also increased the
estimated remaining life of the machine at 1 January 2015 to 8,000 machine hours and
its estimated residual value to Rs. 350,000.

Required:
For the years ended 31 December 2013, 2014 and 2015, compute the relevant
amounts to be included (under each head) in the income statement and statement of
financial position. Notes to the financial statements are not required. (10)

Q.5 Maqsood Enterprises has its head office in Karachi and ten branches all over Pakistan.
Following are the details of balances related to the Peshawar branch in the books of head
office:
31-Dec-15 31-Dec-14
------ Rupees ------
Non-current assets 750,000 700,000
Inventory 250,000 200,000
Receivables 120,000 90,000
Cash 35,000 25,000
Goods returned by the branch 29,700 -
Other relevant information is as under:
(i) Goods invoiced to Peshawar branch during the year amounted to Rs. 330,000. Goods
are sent to Peshawar branch at cost plus 10%. Branch sells these goods at a further
mark-up of 15%.
(ii) During the year, Peshawar branch sent goods which were appearing in its books at
Rs. 27,500 to Lahore branch.
(iii) During the year, certain goods were sold by the branch on 30 days credit and invoiced
at Rs. 35,420 to a customer. However, the goods were returned by the customer before
the due date of payment directly to the head office. No entry has been made in respect
of return of goods.
(iv) Branch expenses amounted to Rs. 50,000 which were paid in cash.
(v) Non-current assets are net of depreciation. During the year, head office purchased
non-current assets on behalf of Peshawar branch amounting to Rs. 62,000.
Financial Accounting and Reporting-I Page 4 of 5

Required:
Prepare Peshawar branch account in the books of head office for the year ended
31 December 2015 showing profit/(loss) made by the branch. (12)

Q.6 Following are the extracts from income statement of Quality Enterprises (QE) for the year
ended 31 December 2015 and its statement of financial position as at that date, together with
some additional information:

Income statement for the year ended 31 December 2015


Rs. in ‘000
Profit from operations 6,402
Other income 1,357
Interest expense (100)
Profit before tax 7,659
Income tax expense (1,376)
Profit for the year 6,283

Statement of financial position as at 31 December 2015


2015 2014 2015 2014
Equity and liabilities Assets
--- Rs. in ‘000 --- --- Rs. in ‘000 ---
Non-current assets
Owner’s capital 14,219 10,703 Property, plant and equipment 19,628 11,845
Unappropriated profit 10,652 6,697 Investments 7,645 6,498
27,273 18,343
Revaluation surplus 2,676 1,911
10% bank loan 6,000 -
Current liabilities Current assets
Trade and other payables 3,337 4,953 Inventories 4,642 3,073
Income tax payable 1,300 994 Trade and other receivables 2,273 3,865
Bank overdraft - 27 Cash and bank 3,996 4
4,637 5,974 10,911 6,942
38,184 25,285 38,184 25,285

Additional information:
(i) During the year, movements in property, plant and equipment include:
 Depreciation amounting to Rs. 5,280,000.
 Machinery having a carrying amount of Rs. 2,481,000 was sold for Rs. 3,440,000.
 Factory building was revalued from a carrying amount of Rs. 5,963,000 to
Rs. 8,000,000.
 An office building which had previously been revalued, was sold at its carrying
amount of Rs. 2,599,000.

(ii) The owner of QE withdrew Rs. 300,000 per month. The amounts were debited to
unappropriated profit.
(iii) Trade debts written off during the year amounted to Rs. 200,000. The provision for
bad debts as at 31 December 2015 was Rs. 400,000 (2014: Rs. 550,000)
(iv) The interest on bank loan is payable on 30th June every year. The bank loan was
received on 1 November 2015. Interest for two months has been accrued and included
in trade and other payables.
(v) Other income includes investment income of Rs. 398,000. As at 31 December 2015,
trade and other receivables included investment income receivable amounting to
Rs. 96,000 (2014: Rs. 80,000).

Required:
Prepare a statement of cash flows for Quality Enterprises for the year ended
31 December 2015, using the indirect method. (18)
Financial Accounting and Reporting-I Page 5 of 5

Q.7 The following particulars/projections pertain to a well-maintained medium-sized car:


Rupees
Cost of car 1,200,000
Salvage value after 100,000 kilometres (km) 300,000
Maintenance cost:
– Service after every 5,000 km 6,000
– Replacement of spares/parts (per 2,000 km) 4,000
Vehicle tax per annum (20% adjustable against
income tax payable by the owner) 7,500
Insurance per annum 36,000
Cost of petrol per litre 75
Cost of tyres replacement after 25,000 km 20,000

On an average, the car consumes one litre for every 15 km.

Required:
For three different levels of use i.e. 10,000, 20,000 and 30,000 km per annum, prepare a
schedule showing:
 Variable, fixed and total costs
 Variable, fixed and total costs per km

In respect of each type of cost, give appropriate justification for treating it as a variable or a
fixed cost. (10)

(THE END)
Financial Accounting and Reporting-I
Suggested Answers
Certificate in Accounting and Finance – Spring 2016

A.1 Seaview Club


Income & Expenditure Account
For the period ended 31 December 2015

Expenditure Rs. in ‘000 Income Rs. in ‘000


Salaries and wages (1000–10+99) 1,089 Joining fees 20,800
Rent (3600/3×11/12) 1,100 Subscription income (W-1) 4,630
Utilities (570–20+55) 605 Profit on sale of beverages (W-2) 330
Insurance (120/12×11) 110
Repairs and maintenance (275–100+7) 182
Depreciation expense
(1,200×10%×11/12+1,500×20%×11/12) 385
Excess of income over expenditure 22,289
25,760 25,760

Seaview club
Statement of Financial Position
As at 31 December 2015

General Fund & Liabilities Rs. in ‘000 Assets Rs. in ‘000


General fund 50,000 Non-Current Assets
Excess of income over expenditure 22,289 Land/Capital advance 65,000
72,289 Furniture & fixtures (1,200110) 1,090
Van (1,500275) 1,225
Deferred income/long term advance Advance for parking shed 100
(W-1) 15,338 Long term deposits 20
Long term prepayment 1,300
Current Liabilities 68,735
Creditors (1,760 – 1,367) 393 Current Assets
Accrued expenses (7+55+99) 161 Stock (W-2) 440
Advance subscription (W-1) 9,984 Debtors for beverages (credit sale) 150
10,538 Advance & prepayments (W-3) 1,220
Bank 27,620
29,430
Total General Fund & Liabilities 98,165 Total Assets 98,165

W-1: Subscription income


Subscription for 3 years is Rs. 72,000 so subscription for 1 year is Rs. 24,000 or Rs. 2,000 per month

No. of No. of Subscription income for Deferred subscription


members months the year income
Month A×B×2,000 A×(36B)×2,000
A B
--------------- Rupees ---------------
March 112 10 2,240,000 5,824,000
June 98 7 1,372,000 5,684,000
September 101 4 808,000 6,464,000
December 105 1 210,000 7,350,000
4,630,000 25,322,000
Less: Short term [(112+98+101+105)×24,000] (9,984,000)
Long term 15,338,000

Page 1 of 6
Financial Accounting and Reporting-I
Suggested Answers
Certificate in Accounting and Finance – Spring 2016

W-2: Beverage sale results Rs. in ‘000


Sales (1,500 + 150) 1,650
Less: Cost of sales 1,760
Purchases (1,320/0.75) (440) 1,320
Closing stock (1,760×25%) 330

W-3: Advance & prepayments Rs. in ‘000


Rent [(3,600–1,100)–1,300(long term)] 1,200
Insurance (120 – 110) 10
Advance salary 10
1,220

A.2 (a) Ledger card - FIFO Method


Transaction Balance
Dates Description Unit Unit
Quantity Rupees Quantity Rupees
cost cost
1/1/2015 Opening inventory 3,500 10 35,000
7/1/2015 Purchase 3,700 12 44,400 3,500 10 35,000
3,700 12 44,400
12/1/2015 Sale (3,000) 10 30,000 500 10 5,000
3,700 12 44,400
13/1/2015 Purchase 4,200 14 58,800 500 10 5,000
3,700 12 44,400
4,200 14 58,800
25/1/2015 Sale (500) 10 5,000
(3,700) 12 44,400
(1,300) 14 18,200 2,900 14 40,600
28/1/2015 Purchase return (100) 14 1,400 2,800 14 39,200
31/1/2015 Purchase 2,000 13 26,000 2,800 14 39,200
2,000 13 26,000
31/1/2015 Surplus inventory 20 14 280 2,820 14 39,480
2,000 13 26,000

Ledger card -Weighted average method


Transaction Balance
Dates Description Unit Unit
Quantity Rupees Quantity Rupees
cost cost
1/1/2015 Opening inventory 3,500 10.00 35,000
7/1/2015 Purchase 3,700 12.00 44,400 7,200 11.03 79,400
12/1/2015 Sale (3,000) 11.03 (33,083) 4,200 11.03 46,317
13/1/2015 Purchase 4,200 14.00 58,800 8,400 12.51 105,117
25/1/2015 Sale (5,500) 12.51 (68,826) 2,900 12.51 36,290
28/1/2015 Purchase return (100) 14.00 (1,400) 2,800 12.46 34,890
31/1/2015 Purchase 2,000 13.00 26,000 4,800 12.69 60,890
31/1/2015 Surplus/closing inventory 20 12.69 254 4,820 12.69 61,144

(b) Journal entries


Debit Credit
Date Description
---------- Rupees ----------
28/1/2015 Payables (58,800/4,200×100) 1,400
Purchase returns/stock/purchases 1,400

31/1/2015 Inventory (4,820-4,800)×12.69 254


Miscellaneous income 254

Page 2 of 6
Financial Accounting and Reporting-I
Suggested Answers
Certificate in Accounting and Finance – Spring 2016

A.3 (a) Examples of the situations where legal title passes but risk and rewards are retained
 An entity may retain obligations for unsatisfactory performance not covered by normal warranty
provisions;
 The receipt of revenue may be contingent on derivation of revenue by the buyer for its sale of
goods.

Examples of the situations where legal title does not pass but the risks and rewards are transferred
 A seller may retain the legal title to the goods to protect the collectability of the amount due but
transfer the significant risks and rewards of ownership.
 In retail sale, a seller may offer a refund if the customer is not satisfied.

(b) (i) The completion of the sale transaction is uncertain because it is contingent upon purchaser
securing the contract with another company. Therefore, KIL should only recognize the revenue
when it is certain that YE will secure the contract. 10% revenue may be recognized if and when it
is confirmed that YE would not be able to secure the contract.

(ii) Revenue should be recognized when the food factory is delivered to the customer. Until then no
revenue should be recognized and the deposit should be carried forward as deferred income. 25%
advance may be transferred to other income if the parties do not claim the asset.

(iii) If the outcome of a service transaction cannot be estimated reliably, revenue should only be
recognized to the extent that expenses incurred are recoverable from the customer. Thus revenue
to the extent of Rs. 12 million may be recognised.

(c) Since the sale and repurchase prices are lower than the fair values, the substance of the arrangement
appears to be that the financial institution has granted ATML a one year loan secured on the property,
charging interest of Rs. 9 million.

The transaction should be accounted for in ATML’s books as follows:


 continue to recognise the property as an asset at the carrying amount.
 credit the Rs. 90 million received to a liability account.
 recognise finance cost of Rs. 9 million over a period of one year.
 debit the liability when Rs. 99 million cash is paid out.

A.4 (a) The cost of an item of property, plant and equipment shall be recognized as an asset if, and only if:

(i) It is probable that future economic benefits associated with the item will flow to the entity; and
(ii) The cost of the item can be measured reliably.

After recognition as an asset, an item of property, plant and equipment whose fair value can be
measured reliably shall be carried at a revalued amount, being its fair value at the date of the
revaluation less any subsequent accumulated depreciation and subsequent accumulated impairment
losses.

(b) Year ended 31-Dec-13 31-Dec-14 31-Dec-15


Income statement: --------------- Amount in Rs. ---------------
Cost of sales
Cost of sales (W-3) 1,720,333 2,646,934 1,371,028
Other income (Discount received i.e.
(8,740,000×3%)) (262,200) - -
Administration expenses
(Staff training) 351,000 - -

As at 31-Dec-13 31-Dec-14 31-Dec-15


Statement of financial position: --------------- Amount in Rs. ---------------
Property, plant and equipment (W-4) 7,896,667 5,425,733 5,983,705
Long term deposit 176,000
Short term deposit 176,000 176,000

Page 3 of 6
Financial Accounting and Reporting-I
Suggested Answers
Certificate in Accounting and Finance – Spring 2016

Workings
W-1: Cost price of machine Rupees
List price 9,200,000
Less: Trade discount (9,200,000×5%) (460,000)
8,740,000
Add: Freight charges 263,000
Electrical installation cost 245,000
Pre-production testing 193,000
9,441,000

W-2: Valuation after upgrade Rupees


Original cost (W-1) 9,441,000
Depreciation upto 31 December 2014 [1,544,333(W-3)+2,470,934(W-3)] (4,015,267)
Carrying amount on 1 January 2015 5,425,733
Capitalization of Upgrade 1,753,000
Value after capitalization 7,178,733

W-3: Costs of sales 2013 2014 2015


------------------ Rupees ------------------
Depreciation
[9,441,000(W-1)175,000]×2,000/12,000 1,544,333
[9,441,000(W-1)175,000]×3,200/12,000 2,470,934
[7,178,733(W-2)350,000]×1,400/8,000 1,195,028
Maintenance cost (528,000/3) 176,000 176,000 176,000
1,720,333 2,646,934 1,371,028

W-4: Property, plant and equipment:


Cost (W-1) 9,441,000 9,441,000 *11,194,000
Accumulated depreciation (1,544,333) (4,015,267) (5,210,295)
7,896,667 5,425,733 5,983,705
* [9,441,000+1,753,000]

A.5 Peshawar Branch Account


Balance b/d Rs. Balance b/d Rs.
Non-current assets 700,000 Inventory reserve (1/11 of 200,000) 18,182
Inventory 200,000 Mark-up on goods sent to br. 30,000
Receivable 90,000 Goods returned to HO by customers 29,700
Cash 25,000 Goods returned to HO directly
Goods sent to br. 330,000 (35,420/1.15) 30,800
Mark up on goods returned by br. 2,700 Goods transferred to Lahore br. 27,500
Mark up on transfer to Lahore br. 2,500 Cash sent to HO (W-1) 166,220
Mark up on goods returned directly to Balance c/d
HO 2,800 Non-current assets 750,000
Non-current assets purchased by HO 62,000 Inventory 250,000
Balance c/d Receivable (W-1) 84,580
Inventory reserve 22,727 Cash 35,000
Branch loss 15,745
1,437,727 1,437,727

Page 4 of 6
Financial Accounting and Reporting-I
Suggested Answers
Certificate in Accounting and Finance – Spring 2016

W-1: Cash Remitted to Head Office by Peshawar branch Rupees


Opening cash 25,000
Opening debtors 90,000
Add: Total sales at branch price (192,000 (W-1.1) ×1.15) 220,800
Less: Closing debtors (120,00035,420) (84,580)
Expenses incurred (50,000)
Closing cash (35,000)
Cash remitted to head office 166,220

W-1.1: Peshawar branch cost of sales Rupees


Opening stock 200,000
Goods received from head office 330,000
Goods returned to head office (29,700)
Goods transferred to Lahore (27,500)
Goods returned by customers (30,800)
Closing stock (250,000)
Total branch sales at HO price 192,000

A.6 Quality Enterprises


Statement of cash flows
For the year ended 31 December 2015
Rupees
Cash flow from operating activities
Profit before tax 7,659,000
Non-cash adjustments
Investment income (398,000)
Interest expense 100,000
Depreciation charge 5,280,000
Bad debt expense [(200,000 +(400,000 – 550,000)] 50,000
Profit on disposal of property, plant and equipment (3,440,000–2,481,000) (959,000)

Changes in working capital


Increase in inventories (3,073,000–4,642,000) (1,569,000)
Decrease in trade and other receivables 1,558,000
[(3,865,000–80,000–2,273,000+96,000+(550,000– 400,000– 200,000))]
Decrease in trade and other payables [(4,953,000–(3,337,000–100,000))] (1,716,000)
Net changes in working capital (cash generated from operations) 10,005,000
Income tax paid (W-2) (1,070,000)
Net cash from operating activities 8,935,000

Cash flow from investing activities


Purchase of property, plant and equipment (W-3) (16,106,000)
Proceeds from sale of property, plant and equipment (3,440,000+2,599,000) 6,039,000
Investment income received (W-1) 382,000
Purchase of investments (7,645,000 – 6,498,000) (1,147,000)
Net cash used in investing activities (10,832,000)

Cash flow from financing activities


Obtain bank loan 6,000,000
Additional capital (14,219,000 – 10,703,000) 3,516,000
Drawings (3,600,000)
Net cash from financing activities 5,916,000
Net increase in cash and cash equivalents 4,019,000
Cash and cash equivalents at beginning of period (4,000–27,000) (23,000)
Cash and cash equivalents at end of period 3,996,000

Page 5 of 6
Financial Accounting and Reporting-I
Suggested Answers
Certificate in Accounting and Finance – Spring 2016

W-1: Investment income received Amount in Rs.


Balance b/d 80,000 Cash (balancing) 382,000
Income Statement 398,000 Balance c/d 96,000
478,000 478,000

W-2: Tax paid Amount in Rs.


Taxes paid (balancing) 1,070,000 Balance b/d 994,000
Balance c/d 1,300,000 Income Statement 1,376,000
2,370,000 2,370,000

W-3: Property, plant and Equipment Amount in Rs.


Balance b/d 11,845,000 Disposals (2,481,000+2,599,000) 5,080,000
Revaluation surplus
(8,000,0005,963,000) 2,037,000 Depreciation 5,280,000
Additions (balancing) 16,106,000 Balance c/d 19,628,000
29,988,000 29,988,000

A.7 (a) -------------- Kilometers --------------


A 10,000 20,000 30,000
Variable costs: -------------- Amount in Rs. --------------
Maintenance - service after every 5000 km 12,000 24,000 36,000
Spares 20,000 40,000 60,000
Petrol 50,000 100,000 150,000
Provision for replacement of tyres
(20,000÷25,000×A) 8,000 16,000 24,000
Depreciation
[(1,200,000300,000)/100,000×A] 90,000 180,000 270,000
B 180,000 360,000 540,000
Fixed costs:
Vehicle tax 6,000 6,000 6,000
Insurance 36,000 36,000 36,000
C 42,000 42,000 42,000
Total cost 222,000 402,000 582,000

Variable cost per km (B÷A) 18.00 18.00 18.00


Fixed cost per km (C÷A) 4.20 2.10 1.40
Total cost per km 22.20 20.10 19.40

(b) (i) Spares and petrol:


Spares and petrol are variable costs being dependent on the usage of car.
(ii) Depreciation:
The depreciation in this case is variable because it is being charged on the basis of actual
running/usage and not on the basis of time.
(iii) Maintenance:
Service is to be done after each 5,000 kms and is therefore a variable cost.
(iv) Vehicle tax:
Vehicle tax is payable per year irrespective of actual running and is therefore a fixed cost.
(v) Insurance:
Insurance is payable per year irrespective of usage and is therefore a fixed cost.
(vi) Tyres:
If the cost of tyres is accrued on the basis of usage, it would be a variable cost.

(THE END)

Page 6 of 6
THE INSTITUTE OF CHARTERED ACCOUNTANTS OF PAKISTAN

EXAMINERS’ COMMENTS

SUBJECT SESSION
Financial Accounting and Reporting-I Certificate in Accounting and Finance
– Spring 2016

General:

The overall performance of the examinees was above average and much better than the
previous attempt. Surprisingly, extremely poor performance was witnessed in question 7
although apparently it was an easy question. The only probable reason for it was that the
topic had been introduced in the syllabus only recently. Performances in question 1, 3 and
4 were also below average.

Question-wise comments.

Question 1

This question required statement of financial position and income & expenditure account
of a club from the Receipt and Payment Summary and other available information.
Overall it was a simple question which mostly contained routine type of adjustments and
disclosures. However, the performance was below average as the following mistakes
were commonly observed:

 Very few students were able to bifurcate the amount of subscription correctly
between subscription income and deferred income on the basis of number of months
passed till year-end. Many students did not carry out this calculation altogether
whereas many students took 1/3rd of the amount as subscription income. Further, even
fewer students bifurcated the deferred income between the short-term and long-term
portion.
 Most of the examinees failed to accrue revenue related to sale of beverages in
December 2015 amounting to Rs. 150,000.
 Very few students seemed to understand the calculation of purchases of beverages
and the closing stock correctly. Most of them failed to realise that according to the
question, 25% of the purchases remained unsold and therefore cost of sales
represented 75% of purchases and closing stock must be equal to 1/3rd of cost of sales
or 1/4th of total purchases. Many students presumed the total purchases as Rs. 1,367
thousand which in fact represented the amount paid for purchases till year end.
Consequently, they also failed to recognise the amount payable against purchases.
 Rent expense, insurance and depreciation were calculated based on 12 months instead
of 11 months.

Page 1 of 5
Examiners’ Comments on Financial Accounting and Reporting-I - Spring 2016

 Advance for land, advance for parking shed and utility deposit were included in
current assets instead of non-current assets by majority of the examinees.
 Prepaid rent was not bifurcated with current and non-current.
 Sponsor’s contribution was incorrectly included in income by some candidates.
 Salaries and utilities were paid in subsequent months; therefore accruals of such
expenses were required but were missed generally.

Question 2

This question required determination of cost of inventory under the perpetual system
using FIFO and weighted average valuation methods and passing of journal entries to
record defective items returned to supplier and surplus/shortfall determined as a result of
stock check.

The performance in this question was very good as more than 80% students secured
passing marks and a good number scored full marks. Among the rest, the following errors
were observed:

 Periodic method was used instead of perpetual method.


 In weighted average method, purchase return was deducted at average rate rather than
the rate of purchase.
 Value of surplus inventory was not calculated and no journal entry passed.
 Many students carelessly used the gross value as the unit price.

Question 3

This question tested the concept of revenue recognition and consisted of three parts. The
performance in each part is discussed below:

(a) The question required two examples each of situations where (i) legal title passes
but risk and rewards are retained and ii) legal title does not pass but risk and reward
are passed on to the customer.

The overall performance was average. Generally the students were able to give
proper examples in the first case but not in the second case. Many candidates gave
only one example in each case. Surprisingly, many students repeated the same
examples in both the scenarios.

(b) This part contained three situations and in each case the candidates were required
to explain when the revenue would be recognized. The key observations in each
case are discussed below:

(i) Most of the students correctly mentioned that revenue will be recognized
when contract is secured; however they could not explain as to what would
happen if the buyer is unable to secure the contract. Many students
incorrectly mentioned that 10% revenue shall be recognized immediately on
sale.

Page 2 of 5
Examiners’ Comments on Financial Accounting and Reporting-I - Spring 2016

(ii) About 50% of the candidates mentioned correctly that revenue shall be
recognized on delivery; however, only few could specify the treatment if the
customer paying the security deposit does not take delivery. Rest of the
candidates mostly mentioned that revenue shall be recognised on receipt of
security deposit as the demand was high and no one would like the security
deposit to be forfeited. Like in sub-part (i) a number of candidates mentioned
incorrectly that 25% revenue shall be recognized on receipt as the security
deposit was not refundable.

(iii) Majority answered this sub-part correctly although it didn’t seem to be easy.
Most of those who erred either suggested proportionate recognition on the
basis of time or recommended that no revenue should be recognized.

(c) The performance in this part was poor. The candidates were unable to understand
that since sale and repurchase prices were lower than the fair value, under the
principle of substance over form, it was a loan rather than a sale. Most students
suggested that sale should be recorded and the asset should be credited.

Question 4

(a) This part of the question required the condition that must be met for an item to be
recognized as property plant and equipment and the amount at which such an
amount would be stated after the initial recognition if an entity follows the
revaluation model.

The overall performance was poor as instead of answering on the basis of


recognition criteria as given in IAS 16, the candidates mostly answered on the
basis of their general perception. While discussing subsequent recognition most
students only mentioned revalued amount without mentioning anything about fair
value or subsequent depreciation or impairment.

(b) In this part of the question the candidates were required to ascertain from the data
available in the question, the amounts which would appear under each head, in the
income statement and the statement of financial position. The overall performance
was average. The common mistakes were as follows:

 Staff training cost was included in cost of the asset.


 Cost of maintenance contract was included in the cost of the asset.
 Cost of the asset was adjusted by 3% early settlement discount.
 Depreciation was calculated without taking into account the residual value.
 For year-2, depreciation was calculated by using WDV of year-1.
 In year-3, instead of adding the amount of upgrade (Rs. 1,753,000) to the WDV
of the asset as on 1st January 2015, the same was added to the original cost of
asset.
 Very few examinees were able to correctly bifurcate and present the cost of
maintenance contract in income statement and statement of financial position
with further breakup in short-term and long-term prepayment.
 Very few examinees showed early settlement discount in income statement and
many among those who did, computed it incorrectly on the basis of list price
instead of list price less trade discount.

Page 3 of 5
Examiners’ Comments on Financial Accounting and Reporting-I - Spring 2016

Question 5

Above average performance was witnessed in this question which required preparation of
branch account in the books of head office showing profit or loss made by the branch.
However, most of the students were able to secure marks on easier aspects of the account
such as opening and closing balances and transfer of goods. Most of the students were
unable to correctly calculate cash remitted to head office because of the following
mistakes:

 Very few students had overall understanding of how the amount would be calculated
i.e. that first they would have to calculate the sale by branch at head office price.
 Many of those who did try to calculate branch sale at head office price, used the price
before 10% mark-up instead of the price after the mark-up while some of them
ignored the goods transferred to Lahore Branch.
 Branch debtors amounting to Rs. 35,420 were ignored in the calculation.

Question 6

Above average performance was witnessed in this question also which required
preparation of cash flow statement using the indirect method. Some of the common errors
are described below:

 Majority of the students made mistakes in classification of items between operating,


investing and financing activities.

 Difference in income tax payable was mentioned along with changes in working
capital instead of mentioning income tax paid separately. Some of the candidates
started the cash flow from Profit after Tax instead of Profit before Tax.

 Various types of errors were seen in the calculation of amounts related to property
plant and equipment. Many students ignored the profit on sale whereas many students
ignored the amount of revaluation in arriving at the amount of purchases thereof.

 Many students treated increase/decrease in inventory, debtors and creditors


incorrectly i.e. treated inflows as outflows and vice versa. Surprisingly, it included
many such students who had correctly performed many of the more complex
calculations.

Question 7

This was an easy question requiring computation of variable, fixed and total costs but
somehow proved to be the worst attempted question. Some of the common mistakes are
listed below:

 Cost of the car was taken as fixed whereas depreciation was ignored.
 Salvage value was considered as a deduction against cost of car whereas it should
have been used for the calculation of depreciation only.
 Entire vehicle tax was included as an expense i.e. the fact that 20% of the amount was
adjustable against owner’s income tax was ignored.

Page 4 of 5
Examiners’ Comments on Financial Accounting and Reporting-I - Spring 2016

 An important requirement of the question i.e. proper justification/reasoning for


considering an item as fixed/variable cost was not provided.
 Per km cost was calculated only for the total cost whereas the requirement was to
calculate it for fixed costs and variable costs also.

THE END

Page 5 of 5
Financial Accounting and Reporting - I
Summary of Marking Key
Certificate in Accounting and Finance – Spring 2016

Note regarding marking scheme:


The marking scheme is given as a guide. However, markers also award marks for alternative
approaches to a question and relevant/well-reasoned comments/explanations.

A.1 Mark(s)
Income and Expenditure Account
 Subscription income and joining fees 2.0
 Profit on sale of beverages 2.0
 Expenses 6.0

Statement of financial position


 Non-current assets
 Land, furniture and van 2.0
 Advance for parking shed, long term deposit and prepayments 1.5
 Current assets
 Stock and debtors for beverages 1.0
 Advance, prepayments and bank 0.5
 General fund and long term liabilities
 Sponsor's contribution, excess of income over expenditure 0.5
 Deferred subscription income 1.25
 Current liabilities
 Creditors and accrued expenses 2.0
 Advance subscription 1.25

A.2 (a) Inventory ledger cards:


 FIFO method:
 0.5 mark for detailing each transaction (including purchase return and
surplus inventory) and computing balance 7.0
 Weighted average method:
 01 mark for recording each purchase / purchase return transaction and
computing balance and new unit cost 4.0
 0.5 mark each for other transactions 2.0

(b) Journal entries for:


 returning the defective items 1.0
 recording the surplus stock 1.0

A.3 (a)  0.75 mark for each example under (i) and (ii) 3.0

(b) (i)  Recognition of revenue when it is certain that YE will secure the
contract 1.5
 Treatment of 10% amount withheld by KIL, if YE is not able to secure
the contract 0.5

(ii)  Recognition of revenue when food factory is delivered to customers 1.0


 Treatment of advances from customers 1.0

(iii)  Recognition of revenue when outcome cannot be estimated reliably 2.0

Page 1 of 2
Financial Accounting and Reporting - I
Summary of Marking Key
Certificate in Accounting and Finance – Spring 2016

Mark(s)
(c)  Explaining the substance of the transaction 1.0
 0.5 mark for each of the four points about how transaction should be
accounted for in the books 2.0

A.4 (a)  0.5 mark for each condition to recognise an item as PPE 1.0
 0.5 mark for explanation of revalued amount at which an item of PPE is to
be carried subsequent to its initial recognition 2.0

(b)  Calculating the cost price of machine 2.0


 Valuation of machine after upgrade 1.0
 Depreciation of machine 3.0
 Maintenance cost including long / short term deposit 2.75
 Property, plant and equipment net of depreciation 1.25

A.5  Preparing Peshawar branch account 8.0


 Determining branch cost of sales 2.0
 Computing cash remitted by branch to head office 2.0

A.6  Cash flows from operating activities:


 Profit before tax 0.5
 Adjustment for non-cash items 3.0
 Changes in working capital 4.0
 Income tax paid 1.0
 Cash flows from investing activities 4.0
 Cash flows from financing activities 2.5
 Cash and cash equivalents at the beginning and end of period 1.0
 Presentation and disclosure 2.0

A.7  Preparation of schedule showing:


 total variable cost and variable cost per kilometer 4.0
 total fixed cost and fixed cost per kilometer 1.5
 total cost and total cost per kilometer 0.5
 Justification for treating cost as variable or fixed 4.0

(THE END)

Page 2 of 2
Certificate in Accounting and Finance Stage Examinations
The Institute of 7 September 2016
Chartered Accountants 3 hours – 100 marks
of Pakistan Additional reading time – 15 minutes

Financial Accounting and Reporting-I


Q.1 Rahil runs a retail business. He appointed a cashier at a monthly salary of Rs. 13,000 on
1 April 2016. The cashier did not report for work on 1 July 2016 and it was found that he
had left, taking with him the balance in the till.

It had been Rahil's practice to deposit on each weekend the available balance in the till
after retaining a float of Rs. 5,000. He maintains record of sales on credit and a file of
unpaid invoices in respect of goods purchased by him.

The following information has been ascertained from the available records:

(i) Balance Sheet as on 31 March 2016 was as follows:

Rupees Rupees
Rahil’s capital 233,000 Fixtures and fittings – WDV 161,000
Creditors for goods 159,000 Inventory 111,000
Creditors for expenses 16,000 Debtors 55,000
Cash at bank 76,000
Cash in hand 5,000
408,000 408,000

(ii) Following is a summary of the bank statement from 1 April to 30 June 2016:

Rupees Rupees
Balance on 1 April 2016 76,000 Payment to suppliers for goods 604,000
Cheques received from customers 29,000 Rent & other expenses 37,000
Cash deposited 627,000 Balance on 30 June 2016 91,000
732,000 732,000

(iii) The following amounts were paid from the till:


Rs. per month
Salary to cashier 13,000
Rahil’s drawings 26,000
Petty expenses 5,000

(iv) Fixtures and fittings are depreciated at 10% per annum using reducing balance
method.
(v) Inventory on 1 July 2016 was Rs. 58,000.
(vi) Credit sales during the quarter ended 30 June 2016 amounted to Rs. 64,000 whereas
the debtors balances as on 30 June 2016 amounted to Rs. 66,000. However, direct
confirmations from debtors showed that receivables in fact totalled Rs. 54,000.
(vii) Creditors for goods and expenses had always been paid by cheque. Unpaid invoices
for goods on 30 June 2016 totalled Rs. 181,000 and creditors for expenses
amounted to Rs. 13,000. Detailed scrutiny of records revealed that a cash receipt of
Rs. 8,000 which had been received against goods returned to a supplier had not
been recorded.
(viii) Rahil sells goods at a gross profit margin of 20% on sales.

Required:
(a) Prepare a statement showing calculation of the amount of defalcation. (11)
(b) Prepare a balance sheet as on 30 June 2016. (09)
Financial Accounting and Reporting-I Page 2 of 4

Q.2 Khan Limited opened a new branch at Lahore on 1 January 2016. Goods are invoiced to
branch at 25% above cost and branch sells the goods on the invoice price. Expenses of
branch are met from branch cash and the balance amount is remitted to head office (HO).
Following information is available for the year ended 30 June 2016:
Rupees
Cost of goods sent to branch 460,400
Goods received by branch till 30 June 2016 (at invoice price) 454,000
Credit sales 328,000
Debtors on 30 June 2016 35,000
Cash remitted to HO 315,000
Cash at branch on 30 June 2016 14,000
Expenses by branch 40,000

Required:
Prepare following ledger accounts:
(a) Branch Cash Account (04)
(b) Branch Stock Account (04)
(c) Branch Stock Adjustment Account (04)

Q.3 The output and production costs of a garment factory for the last 10 months are given
below:
Output Production costs
Months
(units in million) (Rs. in billion)
1 1 2.05
2 2 2.82
3 4 4.33
4 8 7.31
5 6 5.80
6 5 5.08
7 8 7.29
8 9 8.10
9 7 6.52
10 6 5.82

Required:
Determine the regression line for output and production costs. Also estimate production
costs for next month if required output is 3 million units. (08)

Q.4 Salman Limited (SL) closes its books on 30th June each year. Due to an administrative
problem, SL carried out the stock-taking on 10 July 2016. The cost of stock as verified on
10 July 2016 was Rs. 812,500.
Details of transactions from 1 July to 10 July are given below:
(i) Total sales amounted to Rs. 326,000. The goods were sold in the normal course of
business at cost plus 25% except the following:
 a sale of Rs. 25,000 was made at 40% of normal selling price.
 a sale of Rs. 60,000 was made at normal selling price but the goods were
slightly damaged and an expenditure of Rs. 15,000 was incurred on these
goods to bring them to saleable condition.
(ii) Purchases amounted to Rs. 246,000. All such purchases were included in stock as
on 10 July 2016.
(iii) Sales returns and purchase returns amounted to Rs. 11,000 and Rs. 6,000
respectively.
(iv) Goods with customers on sale or return basis were Rs. 50,000 (at invoice value).
The goods had been sent to the customers on 15 June 2016. The customers have the
right to return the goods within four weeks. One of the customers informed SL on
29 June 2016 that goods worth Rs. 20,000 had been destroyed in fire.
Financial Accounting and Reporting-I Page 3 of 4

Required:
Calculate the value of stock as at 30 June 2016. (11)

Q.5 Following is the trial balance of Mateen as at 30 June 2016 :

Debit Credit
-------- Rupees --------
Sales 6,892,000
Purchases 4,124,000
Administrative expenses 1,855,000
Distribution costs 549,000
Property, plant and equipment
Cost 1,750,000
Accumulated depreciation at 30 June 2015 350,000
Inventories at 30 June 2015 344,000
Unappropriated profit at 30 June 2015 330,000
Mateen’s capital 2,000,000
Cash in hand 22,000
Cash at bank 14,000
Bank loan 500,000
Trade receivables 2,255,000
Trade and other payables 826,000
Provision for bad debts at 30 June 2015 15,000
10,913,000 10,913,000

The following additional information is available:


(i) Sales include an amount of Rs. 70,000, made to a customer on sale or return basis.
The goods had a cost of Rs. 47,000. The customer paid the amount on 5 July 2016.
(ii) Cost of inventories at 30 June 2016 amounted to Rs. 365,000. The net realizable
value of the inventories was Rs. 350,000.
(iii) Administrative expenses include rent of office building amounting to Rs. 700,000.
70% of the rental amount should be allocated to cost of sales and 30% to
administrative expenses.
(iv) Prepaid administrative expenses and accrued distribution costs at 30 June 2016
amounted to Rs. 131,000 and Rs. 176,000 respectively.
(v) Property, plant and equipment are depreciated at 10% per annum using reducing
balance method. Depreciation on addition is provided from the month in which the
asset is acquired while no depreciation is charged in the month in which the asset is
disposed of. Depreciation should be allocated between cost of sales and
administrative expenses in the ratio of 80:20 respectively.

On 10 January 2016, a generator which was purchased on 1 July 2012 for


Rs. 100,000 was traded-in for a new generator. The disposal was not recorded and
the generator was capitalized at Rs. 500,000 being the net amount paid to supplier
after adjusting trade-in allowance of Rs. 35,000. The cost of installation of the
generator amounting to Rs. 30,000 was debited to administrative expenses.
(vi) Bank loan was taken on 1 October 2015 and carries interest at 8% per annum. The
loan is repayable on 30 September 2016.
(vii) Trade receivables amounting to Rs. 5,000 are required to be written off. Bad debts
are estimated at 4% of the trade receivables.
(viii) Income tax liability for the year ended 30 June 2016 is estimated at Rs. 40,000.

Required:
Prepare the following:
(a) Statement of comprehensive income for the year ended 30 June 2016; and (10)
(b) Statement of financial position as at 30 June 2016. (10)
Financial Accounting and Reporting-I Page 4 of 4

Q.6 (a) Car World sells new cars on deferred payment basis whereby 40% deposit is
received on sale and the balance payment is received at the end of two years. The
appropriate discount rate is 10%.

On 1 July 2014 a car was sold to a customer for Rs. 2,000,000.

Required:
Prepare necessary journal entries to record the above transaction in the books of
Car World for the years ended 30 June 2015 and 2016. (07)

(b) Saleem Engineering (SE) is a supplier of various types of industrial machines. It


also provides services for the maintenance of these machines. Following
transactions were carried out by SE during the year ended 30 June 2016:

(i) Five machines were sold on a lay away basis to one of its frequent customers.
Three out of a total of five instalments had been received till the year end. (03)

(ii) A service contract for maintenance of a machine for a period of one year was
signed and SE received a non-refundable annual fee amounting to Rs. 45,000
as advance on 15 April 2016. (02)

Required:
Discuss when it will be appropriate for SE to recognise revenue in each of the above
situations.

Q.7 Kamran Enterprises (KE) provides depreciation on plant and machines at 10% on
written-down value. Depreciation is charged from the month the asset is available for use
in operations up to the month prior to its disposal. Cost of its plant & machines and the
accumulated depreciation as on 1 July 2015 were Rs. 75 million and Rs. 17 million
respectively.

The following information is available in respect of its plant & machines, for the year
ended 30 June 2016:

(i) On 1 October 2015, a second-hand machine was acquired from a Chinese company
for Rs. 15 million. The machine was renovated and overhauled at a cost of
Rs. 3 million. 25% of this expenditure was in respect of purchase of consumables.
(ii) On 1 November 2015, KE transferred a machine having a list price of
Rs. 10 million from its stock-in-trade to its Engineering Department. KE sells such
machines at cost plus 25%.
(iii) On 1 January 2016, certain worn-out parts of a plant were replaced at a cost of
Rs. 4 million. The replaced parts neither enhanced the useful life of the plant nor its
operating efficiency. The old parts were sold for Rs. 0.75 million. The plant was
purchased for Rs. 25 million on 1 January 2015.

On 1 May 2016, the plant was damaged and remained in-operative for one month.
KE spent an amount of Rs. 3 million on repairs to restore the plant in working
condition.

(iv) On 1 April 2016, a machine which was purchased on 1 July 2012 for Rs. 12 million
was completely damaged and was sold for Rs. 1.2 million.

Required:
Prepare accounting entries to record the above transactions in KE’s books for the year
ended 30 June 2016. (17)

(THE END)
Financial Accounting and Reporting-I
Suggested Answers
Certificate in Accounting and Finance – Autumn 2016

A.1 (a) Statement of amount of defalcation


Rupees
Cash in hand 30/3/16 5,000
Cash sales [838,750 (W-1)  64,000] 774,750
Receipts from customers as per debtor’s account (W-3) 24,000
Less: Cash utilized
Bank lodgments (627,000)
Assistant's salary (13,000×3) (39,000)
Petty expenses (5,000×3) (15,000)
Drawings (26,000×3) (78,000)
(759,000)
Defalcation against cash sales 44,750
Difference in debtors balance 12,000
Defalcation from amount received from supplier against purchase return 8,000
Total defalcation amount 64,750

Working Notes:

W-1: Sales Rupees


Stock on 1 April 2016 111,000
Add: Purchases for 3 months (626,000 – 8000) (W-2) 618,000
729,000
Less: Stock on 30 June 2016 (58,000)
671,000
Add: Gross profit 20% on sales (671,000 × 20÷80) 167,750
Total sales 838,750

W-2: Purchases Rupees


Cash paid to creditors against goods supplied 604,000
Add: Creditors on 30 June 2016 181,000
785,000
Less: Creditors on 31 March 2016 (159,000)
Cash received for returns (8,000)
Total purchases 618,000

W-3: Cash received from credit customers Rupees


Balance on 1 April 2016 55,000
Add: Credit sales for 3 months 64,000
119,000
Less: Balances outstanding on 30 June 2016 as per books (66,000)
53,000
Less: Receipts (cheques) shown in cash book (29,000)
Amount presumably received in cash 24,000

(b) Balance Sheet of Rahil


As on 30 June 2016
Liabilities Rs. Assets Rs.
Sundry creditors 181,000 Fixtures and fittings (net) 156,975
Expenses owing 13,000 Stock in trade 58,000
Capital: Sundry debtors 54,000
Balance on 1 April 2016 233,000 Balance at bank 91,000
Add: Net profit (W-1) 10,975
Less: Drawings (78,000)
165,975
359,975 359,975

Page 1 of 6
Financial Accounting and Reporting-I
Suggested Answers
Certificate in Accounting and Finance – Autumn 2016

W-1: Net profit for 3 months Rupees


Gross profit - 20% on sales (671,000 × 20÷80) 167,750
Less:
Depreciation (161,000×10%×3÷12) 4,025
Assistant's salary and petty cash expenses 54,000
Rent & other expenses 37,000
Decrease in creditors for expenses (16,000  13,000) (3,000)
Loss due to defalcation 64,750
(156,775)
10,975

A.2 In the books of Khan Limited

Dr. Branch Cash Account Cr.


Date Particulars Rs. Date Particulars Rs.
Branch debtors a/c - cash received 293,000 31-12-16 Branch expenses a/c 40,000
Branch stock a/c - cash sales (bal. fig.) 76,000 Remittance to H.O. 315,000
Balance c/d 14,000

369,000 369,000

Dr. Branch Stock Account Cr.


Date Particulars Rs. Date Particulars Rs.
31-12-16 Goods sent to branch a/c 575,500 31-12-16 Cash sales 76,000
(460,400×125%/100) Branch debtors A/c – Credit
sales 328,000
Stock-in-transit (575,500454,000) 121,500
Balance c/d 50,000
575,500 575,500

Dr. Branch Stock Adjustment Account Cr.


Date Particulars Rs. Date Particulars Rs.
Stock-in-transit A/c – Loading 31-12-16 Goods sent to branch A/c –
(121,500×25/125) 24,300 Loading (460,400×25%) 115,100
Balance c/d – Loading on closing
stock (50,000×25/125) 10,000
31-12-16 Branch P & L A/c – G.P. transferred
(bal. fig) 80,800
115,100 115,100

A.3 (a) Output Production costs


(Rs. in million) (Rs. in billion) (xy) (x2)
(x) (y)
1 2.05 2.05 1.00
2 2.82 5.64 4.00
4 4.33 17.32 16.00
8 7.31 58.48 64.00
6 5.80 34.80 36.00
5 5.08 25.40 25.00
8 7.29 58.32 64.00
9 8.10 72.90 81.00
7 6.52 45.64 49.00
6 5.82 34.92 36.00
56 55.12 355.47 376

Page 2 of 6
Financial Accounting and Reporting-I
Suggested Answers
Certificate in Accounting and Finance – Autumn 2016

∑ ∑ ∑

∑ ∑

Estimated regression equation is

(b) The estimated production costs for next month if required output is 3 million units will be:

A.4 ------ Rupees ------


Cost of stock on 10 July 2016 812,500
Less:
Purchases from 01 July to 10 July 2016 (246,000)
Sales returns, at cost (Rs. 11,000 × 100/125) (8,800)
557,700
Add:
Cost of stock sold for Rs. (326,000  25,000)/1.25 240,800
Cost of good sold at 40% of invoice price (25,000/0.4)×100/125 50,000
Goods on sale or return basis (50,000*20,000)× 100/125 24,000
Purchase returns 6,000
320,800
Cost of stock on 30 June 2016 (A) 878,500

Less: NRV Adjustments


Stock on which repair is done (60,00015,000) 45,000
Cost (60,000/1.25) 48,000 (3,000)

Stock sold at 40% of selling price


Normal cost (25,000/0.4/1.25) 50,000
Selling price 25,000 (25,000)

NRV of stock (B) 850,500

Value of stock as on 30 June 2016 [lower of cost (A) and NRV(B)] 850,500
*Goods destroyed in fire (Rs. 20,000) taken as sold

A.5 Income statement for the year ended 30 June 2016


Rupees
Revenue (6,892,000 – 70,000) 6,822,000
Cost of sales (W-1) (4,652,684)
Gross profit 2,169,316
Administrative expenses (W-1) (1,338,376)
Distribution costs (W-1) (725,000)
Profit from operations 105,940
Finance cost (500,000×8%×9÷12) (30,000)
Profit before tax 75,940
Income tax expense (40,000)
Profit for the period 35,940

Page 3 of 6
Financial Accounting and Reporting-I
Suggested Answers
Certificate in Accounting and Finance – Autumn 2016

Statement of financial position


As at 30 June 2016
Rupees Rupees

Equity and liabilities Assets


Capital 2,000,000 Non-current assets
Retained earnings (330,000+35,940) 365,940 Property plant and equipment (W-2) 1,281,140
2,365,940

Current liabilities Current assets


Bank loan 500,000 Inventories (W-1) 397,000
Trade and other payables Trade receivables
(826,000+176,000) 1,002,000 (2,250,000 – 70,000) – 87,200
Taxation 40,000 2,092,800
Prepaid admin expenses 131,000
Bank interest payable 30,000 Cash and bank (22,000+14,000) 36,000

3,937,940 3,937,940

Workings:
W-1: Allocation of expenses Administrative Distribution
Cost of sales
expenses costs
--------------- Rupees ---------------
Balances as per trial balance 1,855,000 549,000
Opening inventories as per trial balance 344,000
Purchases as per trial balance 4,124,000
Adjustments:
Closing stock (350,000 + 47,000) [(ii) & (i)] (397,000)
Transfer of 70% rent to cost of sales (iii) 490,000 (490,000)
Prepaid and accrued expenses (iv) (131,000) 176,000
Installation charges incorrectly expensed out (v) (30,000)
Depreciation expenses (W-2) (vi) 91,684 22,921
Loss on disposal (W-2) (vi) 34,255
Bad debts expenses
[(4% × (2,250,000 70,000))  15,000+5,000] 77,200
4,652,684 1,338,376 725,000

W-2: Depreciation and loss on disposal Rupees Dep. for the year
Property, plant & equipment as per trial balance 1,750,000
Less: Cost of generator disposed of (A) (100,000)
Less: Cost of generator purchased during the year (500,000)
Cost of PPE used throughout the year 1,150,000
Less: Opening balance of Acc. Dep. (350,000)
Add: Opening balance of Acc. Dep. relating to disposed of generator
[100,000 – (100,000 × 0.9 × 0.9 × 0.9)] (B) 27,100
WDV of PPE used throughout the year 827,100
Depreciation for the year (827,100×10%) (82,710) 82,710
Addition:
New generator 500,000
Old generator – trade-in-allowance 35,000
Installation charges 30,000
565,000
Depreciation on additions (565,000 ×10% × 6/12) (28,250) 28,250
536,750
Depreciation on disposed of generator
[(100,000 – 27,100) × 10% × 6/12] (C) - 3,645
1,281,140 114,605

Page 4 of 6
Financial Accounting and Reporting-I
Suggested Answers
Certificate in Accounting and Finance – Autumn 2016

Loss on disposal [(A– B– C) – 35,000] 34,255

A.6 (a) Debit (Rs.) Credit (Rs.)


Date Particulars
--------- Rupees ---------
1/7/2014 Cash (40%×2,000,000) 800,000
Receivables (2,000,000  800,000) 1,200,000
Car sales [800,000 + 991,736(W-1)] 1,791,736
Deferred income 208,264

30/6/15 Deferred income (10% × 991,736) 99,174


Finance income 99,174

30/6/16 Cash 1,200,000


Deferred income (208,264  99,174) 109,090
Finance income 109,090
Receivables 1,200,000

W-1: Present value of future payments Rupees


Amount receivable 1,200,000

991,736

(b) (i) Revenue from lay away sales is recognized when the goods are delivered against full payment.
However, based on experience, such revenue may be recognized earlier e.g. when a significant
deposit is received provided the goods are on hand, identified and ready for delivery to the buyer.
Hence the sale may be recognized in this case provided the machines are ready for delivery because
the sale is to a frequent customer and a significant portion of the sale proceeds has been received.
(ii) Although the fee is non-refundable, it will be recognized as income on the basis of matching
principle i.e. 1/12th of the annual fee will be taken to income each month.

A.7 Debit Credit


Date Description
------- Rupees -------
1-10-2015 Machine B (15+3×75%) 17,250,000
Cost of sales/Repair and maintenance / Profit & loss a/c 750,000
Bank 18,000,000

1-11-2015 Machine a/c (10,000,000×100/125) 8,000,000


Stock-in-trade 8,000,000

1-1-2016 Plant and Machine 4,000,000


Bank 4,000,000

1-1-2016 Bank 750,000


Cost of sales/Other income 750,000

1-5-2016 Cost of sales/Repair and maintenance / Profit & loss a/c 3,000,000
Bank/payable 3,000,000

1-4-2016 Bank 1,200,000


Accumulated depreciation – Machine [12-(12×0.9×0.9×0.9×0.925)] 3,908,100
Loss on sale of machine (balancing) 6,891,900
Machine 12,000,000

30-1-2016 Depreciation expense (W-1) 7,608,383


Accumulated depreciation – Plant and machine 7,608,383

Page 5 of 6
Financial Accounting and Reporting-I
Suggested Answers
Certificate in Accounting and Finance – Autumn 2016

W-1:
Written down Depreciation for the
value year
-------------Rupees-------------
Opening balance (75,000,000-17,000,000 58,000,000
Less: Disposal [12,000,000×(0.9)3] (8,748,000)
49,252,000 4,925,200

Addition
Addition on 01 October 2015 (17,250,000×10%×9/12) 17,250,000 1,293,750
Addition on 01 November 2015 (8,000,000×10%×8/12) 8,000,000 533,333
On 1 January 2015 4,000,000 200,000
Disposal
Depreciation on machine sold during the year (8,748,000×0.1×9/12) 656,100

Total depreciation 7,608,383

(THE END)

Page 6 of 6
THE INSTITUTE OF CHARTERED ACCOUNTANTS OF PAKISTAN

EXAMINERS’ COMMENTS

SUBJECT SESSION
Financial Accounting and Reporting-I Certificate in Accounting and Finance
– Autumn 2016

General:

This was a well attempted paper and the passing percentage increased significantly.
Below average performances were witnessed in Questions 4, 6 and 7. Questions 6 and 7
required conceptual understanding of relevant IFRSs and majority of the students seemed
unprepared for the same.

Question-wise comments.

Question 1

This question required the students to prepare balance sheet and determine the amount of
cash stolen by the cashier of a retail business which maintained records on single entry
basis. The question was quite straight forward however the overall performance of the
students was average. Majority of the students prepared some basic workings correctly
which secured them some marks but could not correlate them to arrive at the amount of
defalcation. Other common errors were as follows:
 While calculating the amount of defalcation, most students did not compute cash
misappropriated from credit sales and/or from amount received from supplier against
purchase return.
 In the calculation of purchases, cash received against purchase returns was ignored.
 While preparing Profit and loss statement for the quarter, expenses paid were
considered whereas decrease in expenses payable was ignored. Many students
ignored the depreciation also or calculated it incorrectly.

Question 2

In this question the candidates were required to prepare Branch Cash account, Branch
Stock account and Branch Stock Adjustment account. It was an easy question with simple
data and no apparent complexity. Consequently, many students were able to secure full
marks. However the overall performance was average as most of the students could only
make the basic entries correctly.
The common mistakes were as follows:
 Majority of the students could not understand that the balancing amount in Branch
Stock account represented Cash sales.

Page 1 of 3
Examiners’ Comments on Financial Accounting and Reporting-I - Autumn 2016

 In Branch Stock account many students were unable to work out the stock in transit
correctly. Further value of goods sent to branch was not grossed up.
 Majority of the students seemed unaware of the use of Branch Stock Adjustment
account and made various mistakes/omissions. Many students ignored it altogether.

Question 3

This was an easy question in which the requirement was to determine the regression line
for output and production cost and to calculate the estimated production cost based on an
output of 3 million units.
Most of the students performed well in this question and many of them secured full
marks. However, some students ignored the requirement of the question and tried to solve
it using high / low method. Some students determined the regression line using output as
the dependent variable instead of cost.

Question 4

In this question the candidates were required to compute the value of stock at year-end
based on stock taking carried out 10 days after year-end and details given in the question
as regards transactions carried out during those ten days.

The overall performance was below average as only 27% students could secure passing
marks. Most of the students found it difficult to calculate NRV of stock items on which
repair cost was incurred and the stock was sold at 40% of selling price.

Other common errors were as follows:


 Most of the students found it difficult to properly deal with the issue of goods sold on
return basis which were destroyed by fire.
 Many students did not unload profit from sale/sales return.

Question 5

In this question, trail balance of a sole proprietor and adjustment data were given and the
candidates were required to prepare Statement of Comprehensive Income and Statement
of financial position. The overall performance was good as about 53% students secured
passing marks and about 15% of the students scored more than 75% marks. The errors
observed were as follows:
 Majority of the students could not correctly adjust the error related to recording of
disposal of old generator. Most students ignored it altogether whereas the others made
various types of errors including failure to calculate correct WDV of old generator
and failure to understand that the trade-in allowance of Rs. 35,000 represented sale
price thereof.
 Many students did not record loss on disposal
 Adjustments related to admin expenses and allocation of expenses among cost of
sales, admin expenses and distribution was not carried out carefully, resulting in the
following:
o Adjustment for prepaid admin expenses was mostly ignored
o 70% rent was charged to cost of sales but was not deducted from admin expenses

Page 2 of 3
Examiners’ Comments on Financial Accounting and Reporting-I - Autumn 2016

o Installation charges which were wrongly included in administrative expenses


were included in the cost of new generator correctly but the same were not
deducted from admin expenses.

 While calculating bad debt expenses, sales made on return basis was not deducted
from trade receivable before applying the bad debt percentage. Further, adjustment of
written off trade debts was often ignored.
 Income tax liability and/or interest expenses/payable were not recorded.

Question 6(a)

In this part of the question, the candidates were required to prepare journal entries for sale
on deferred payment basis. A large number of students were totally ignorant of the
correct accounting treatment and simply booked the entire amount as sales. Many
students who had some idea, found it difficult to calculate present value of future
payments. Further, many students recorded income by crediting finance income but
didn’t understand what to debit.

Question 6(b)

In this part of the question, the candidates were required to suggest appropriate timing for
recognizing revenue in case of sales on lay away basis and also in case of a maintenance
service contract where the entire fee was received in advance.

With regard to lay away sales, most of the students were only able to mention one
condition for recognition of sales i.e. significant portion of payments has been received
but did not mention the other condition i.e. delivery of goods or goods being ready for
delivery. However, the question pertaining to receipt of maintenance fee in advance was
quite easy and most of the students answered it correctly.

Question 7

This question required preparation of accounting entries for various types of transactions
relating to Plant and Machinery. The overall response was quite poor. Depreciation on
already existing assets was calculated incorrectly by most of the students as they made
errors which could easily have been avoided at this level, by being a bit more careful.
Other Common errors were as follows:

 In sub part (i), machine renovation and overhaul cost was fully expensed out whereas
only the consumables should have been charged off and the remaining amount should
have been capitalised.
 In sub part (ii), instead of capitalizing the machine, a large number of students debited
engineering department.

THE END

Page 3 of 3
Financial Accounting and Reporting - I
Summary of Marking Key
Certificate in Accounting and Finance – Autumn 2016

Note regarding marking scheme:


The marking scheme is given as a guide. However, markers also award marks for alternative
approaches to a question and relevant/well-reasoned comments/explanations.

Mark(s)
A.1 (a) Statement showing amount of defalcation against:
 cash sales 5.0
 credit sales 5.0
 purchase return and cash in hand 1.0

(b) Balance sheet:


 Fixed assets 0.5
 Current assets 1.5
 Liabilities 1.0
 Equities 6.0

A.2 (a) Up to 1.5 marks for each entry in branch cash account 4.0

(b) Up to 01 mark for each entry in branch stock account 4.0

(c) Up to 01 mark for each entry in branch stock adjustment account 4.0

A.3  Determination of ‘b’ 3.0


 Determination of ‘a’ 3.0
 Determination of regression line for output and production costs 1.0
 Estimation of production costs for the next month 1.0

A.4 Up to 02 marks for each adjustment made in determination of value of the closing
stocks 11.0

A.5 (a) Statement of comprehensive income:


 Revenue 1.0
 Cost of sales 4.0
 Administrative expenses 3.5
 Distribution costs 0.5
 Finance cost and income tax expense 0.5
 Income tax expense 0.5

(b) Statement of financial position:


 Non-current assets 5.0
 Current assets:
 Inventories 0.5
 Trade debtors 1.0
 Prepaid admin expenses 0.5
 Equity 1.0
 Current liabilities:
 Bank loan and trade payables 1.0
 Taxation and bank interest payable 1.0

Page 1 of 2
Financial Accounting and Reporting - I
Summary of Marking Key
Certificate in Accounting and Finance – Autumn 2016

Mark(s)
A.6 (a)  Up to 02 marks for each journal entry 6.0
 01 mark for present value calculation 1.0

(b) (i)  Explanation of lay away sales 1.0


 Discussion on how the sale is to be recognized in the books 2.0
(ii) Discussion on how the fees is to be recognized 2.0

A.7  Up to 03 marks for each journal entry 11.0


 Computation of total depreciation for the year 6.0

(THE END)

Page 2 of 2
Certificate in Accounting and Finance Stage Examinations
The Institute of 8 March 2017
Chartered Accountants 3 hours – 100 marks
of Pakistan Additional reading time – 15 minutes

Financial Accounting and Reporting-I


Q.1 The accountant of Leisure Club was terminated on account of charges of fraud on
31 December 2016 and Mr. Emad has been appointed in his place. Emad has gathered the
following information in respect of the year ended 31 December 2016:
(i) The club has 3,300 members and the membership fee is Rs. 10,000 per annum. The
fee payable by each member becomes due on the first day of the quarter in which he
became a member. The fee received in each quarter was as follows:
Quarter First Second Third Fourth
Subscription received (Rs.) 9,900,000 8,250,000 5,500,000 9,350,000

Last year the fee was Rs. 9,000 per annum. However, the number of members was the
same.
(ii) A summary of the bank account for the year is shown below:
Deposits Rupees Withdrawals Rupees
Balance as at 1 Jan. 2016 3,700,500 Insurance 175,000
Cash deposited into bank 37,848,500 Rent and rates 4,200,000
Written off amount recovered 1,860,000 Utilities 4,365,000
Disposal of fixed assets 750,000 Freehold land purchased 17,000,000
Members subscription received Cash withdrawals from bank 6,120,000
directly in bank account 19,800,000 Payment to creditors 18,155,000
Repairs and maintenance 700,000
Exercise equipment 7,350,000
Balance as at 31 Dec. 2016 5,894,000
63,959,000 63,959,000

(iii) Amounts paid from petty cash were as follows:


Rupees
Salaries 2,300,000
Sundry expenses 640,000

(iv) The club has a tuck shop which earns a profit margin of 20% of sales. All sales of tuck
shop are made on cash. During the year, stock costing Rs. 500,000 was destroyed by
fire.
(v) The opening WDV of fixed assets was Rs. 28,000,000. Exercise equipment was
purchased on 1 October 2016. Fixed assets having opening WDV of Rs. 800,000 were
disposed off on 31 March 2016. Fixed assets are depreciated @ 20% under the
reducing balance method.
(vi) The opening and closing balances of cash in hand were Rs. 300,000 and Rs. 25,000
respectively.
(vii) The following balances have been extracted through a scrutiny of the available
records:
2016 2015
------- Rupees -------
Creditors 3,330,000 2,500,000
Prepaid rent 175,000 168,000
Stock- tuck shop 2,500,000 2,300,000
Financial Accounting and Reporting-I Page 2 of 5

Required:
(a) Determine the amount of loss incurred by the club due to fraud committed by the
previous accountant. (09)
(b) An income and expenditure account for the year ended 31 December 2016. (05)
(c) Statement of financial position as at 31 December 2016. (06)

Q.2 (a) Define the term ‘performance obligation’ and state the criteria which should be met if
goods or services promised to a customer are to be considered as distinct. (04)
(b) (i) ECL has entered into a contract with Kashif Builders for construction of a
residential project, including supply of construction material, architectural
services, engineering and site clearance. ECL and its competitors provide such
services separately also. (03)
(ii) eSolutions Limited, a software developer, entered into a two year contract with a
customer to provide software license including future software updates and post
implementation support services. The software license would remain functional
even if the updates and post implementation support services are discontinued. (03)

Required:
In view of the requirements of IFRS 15 ‘Revenue from Contracts with Customers’,
discuss whether goods and services provided in each of the above contracts represent a
single performance obligation.

(c) State the disclosure requirements for assets carried at revalued amounts, as referred to
in IAS – 16 ‘Property, Plant and Equipment’. (04)

Q.3 Nawaz Manufacturing Limited (NML) deals in various products. One of its product B2 is
produced using raw material A1. Production is carried out after receiving confirmed sales
order. Following information is available for the month of January 2017:
(i) Opening inventory of A1 was 200 kg @ Rs. 3,000 per kg.
(ii) Details of purchases made during the month ended 31 January 2017 are as follows:
Date Quantity (kg) Price per kg (Rs.)
1-Jan-17 250 2,800
15-Jan-17 250 2,900
50 kg of A1 purchased on 15 January 2017 were returned to the supplier on 16
January 2017 due to inferior quality of material supplied.
(iii) On 18 January 2017, 100 kg of A1 were destroyed. They had no scrap value.
(iv) Under normal circumstances 500 kg of A1 produce 400 liters of B2.
(v) Labour cost per liter of B2 was Rs. 700.
(vi) Overheads are estimated at 120% of labour cost. The actual overheads for the month
were Rs. 275,000.
(vii) There is no opening and closing work in progress.
(viii) Sales of B2 during the month of January were as follows:
Quantity Sales price per
Sale order date Delivery date
(liters) liter (Rs.)
2-Jan-17 4-Jan-17 100 7,000
26-Jan-17 28-Jan-17 160 6,250
(ix) NML uses weighted average method for valuation of inventory.

Required:
Prepare cost of goods sold statement for the month of January 2017 under each of the
following methods:
(a) Perpetual inventory method (10)
(b) Periodic inventory method (05)
Financial Accounting and Reporting-I Page 3 of 5

Q.4 Complex Industries (CI) is engaged in the manufacturing of a specialized product.


Maximum production capacity of CI is 480,000 units per month.
The number of units produced and total production cost, during the past six months were as
follows:
Production cost
Months Units in ‘000’
Rs. in ‘000’
January 280 6,056
February 232 5,080
March 305 6,552
April 320 6,840
May 230 5,064
June 200 4,800

The management is considering to increase the capacity utilization to 85%, 90% or 95%. It
is estimated that if capacity utilization is increased to 90% or more, the fixed costs would
increase by Rs. 100,000 per month.

Required:
Determine the expected cost at each of the three desired levels, using regression analysis and
identify the most beneficial option. (12)

Q.5 A & B are partners in a firm sharing profits and losses in the ratio of their capital i.e. 3:2.
The statement of financial position of the firm as at 31 December 2016 was as under:

Statement of financial position


Equity and liabilities Rupees Assets Rupees
Capital – A 1,440,000 Fixed assets 1,300,000
Capital – B 960,000 Debtors - net of 5% provision 1,500,000
General reserve 800,000 Other current assets 1,800,000
Loan from C 600,000
Creditors 800,000
4,600,000 4,600,000

Profits of the firm for the last three years were as follows:

2016 2015 2014


---------------- Rupees ----------------
1,250,000 800,000 950,000

On 1 January 2017, C who is the son of A, was admitted as a partner under the following
terms and conditions:
(i) Goodwill is to be valued at two years purchase of average profit of the last three years.
However, it was agreed that following adjustments would have to be incorporated
before the computation of goodwill.

− A sale return of Rs. 200,000 on 1 January 2014 was debited to fixed assets. The
firm charges depreciation @ 20% on written down value of fixed assets.
− A debtor balance of Rs. 300,000 was settled against the amount due to the same
customer, in the year 2016. This adjustment was not recorded in the books.
(ii) C’s share of profit would be 20% of which 5% share would be ceded to him by A. The
remaining share would be purchased by C from B.
(iii) Loan from C would be treated as his capital injection.
(iv) The total capital of the new firm will be Rs. 3,500,000. Any excess or shortage will be
settled through cash.

Required:
Prepare partner’s capital account. (12)
Financial Accounting and Reporting-I Page 4 of 5

Q.6 The statement of financial position of Liaquat Industries as at 31 December 2016 is as


follows:
2016 2015 2016 2015
Equity and liabilities Assets
-------- Rupees -------- -------- Rupees --------
Owner’s capital 13,938,060 13,665,280 Freehold land 4,778,400 6,600,000
Long-term loan 1,000,000 1,000,000 Building – WDV 5,057,600 4,171,200
Short term loan 1,331,200 1,531,200 Vehicle – WDV 600,000 800,000
Accounts payable 417,120 694,320 Equipment – WDV 1,643,100 2,112,000
Accrued interest 105,600 63,360 Capital work in progress 1,478,400 1,821,600
Long-term deposits 580,800 448,800
Inventory 685,608 320,628
Accounts receivable 1,273,272 595,452
Cash 694,800 84,480
16,791,980 16,954,160 16,791,980 16,954,160

The following information has been extracted from income statement:


Rupees
Depreciation expenses 932,500
Finance cost 141,872
Gain on sale of fixed assets (net) 98,960
Net profit before tax 1,525,948

Additional information:
(i) Details of gain on sale of fixed assets are as follows:

Rupees
Gain on sale of freehold land 168,960
Loss on disposal of equipment due to fire (70,000)
98,960

The loss on disposal of equipment represents the WDV of the equipment. The
amount of insurance claim received, amounting to Rs. 30,000 was erroneously
credited to accumulated depreciation.

(ii) Repairs to building amounting to Rs. 50,000 were erroneously debited to building
account on 31 December 2016.
(iii) Transfers from capital work in progress to building amounted to Rs. 1,200,000.
(iv) The owner withdrew Rs. 150,000 per month.

Required:
Prepare statement of cash flows for the year ended 31 December 2016, in accordance with
IAS – 7 using indirect method. (12)

Q.7 (a) The following information has been gathered by an analyst, in respect of Dairy Foods
Limited (DFL) which specializes in various dairy products.

Industry
Ratio 2016 2015 2014
average
Profit margin % 11% 10% 8% 10.45%
Quick ratio 1.38 1.40 1.42 1.52
Current ratio 1.84 1.67 1.59 1.73
Days purchases in payables 80 91 89 82

In the latest annual report to the shareholders, Directors of DFL have claimed that
liquidity position of the Company has improved significantly.

Required:
Critically analyze and discuss whether you agree with the claim. (03)
Financial Accounting and Reporting-I Page 5 of 5

(b) Extracts from latest financial statements of two companies are as follows:

Extracts from statements of financial position


A B A B
Equity and liabilities Assets
Rs. in million Rs. in million
Equity and reserves 51,690 72,114 Fixed assets 34,460 48,076
Long term loan - 36,057 Stock in trade 21,700 20,000
Trade creditors 35,790 45,135 Trade debtors 24,470 44,030
Other payables 12,000 8,500 Cash and bank 18,850 49,700
99,480 161,806 99,480 161,806

Extracts from statements of comprehensive income


A B
------ Rs. in million ------
Revenue 161,600 220,150
Cost of sales (135,160) (180,520)
Gross profit 26,440 39,630
Operating expenses (9,840) (13,870)
Interest expense (720) (2,313)
Profit before tax 15,880 23,447
Income tax (333) (409)
Profit after tax 15,547 23,038

Required:
Analyze the profitability, liquidity and working capital ratios of both the companies. (12)

(THE END)
Financial Accounting and Reporting-I
Suggested Answer
Certificate in Accounting and Finance – Spring 2017

A.1 (a) Determination of amount of loss incurred due to fraud Rupees


Opening cash balance 300,000

Cash receipts
Collection from members [(3,300×10,000) – 19,800,000] 13,200,000
Bank withdrawals 6,120,000
Tuck shop sales (W-2) 22,856,250
42,176,250
Cash payments
Salaries (2,300,000)
Sundry expenses (640,000)
Cash deposited into bank (37,848,500)
(40,788,500)
Closing cash should have been 1,687,750
Closing cash-actual (25,000)
Loss due to fraud 1,662,750

(b) Income and expenditure account


Income Rupees
Subscription income (W-1) 31,817,500
Income from tuck shop (22,856,250(W-2) – 18,285,000 (W-2)) 4,571,250
Other income – Bad debts recovered 1,860,000
38,248,750
Expenditures
Salaries 2,300,000
Insurance 175,000
Rent expense (168,000 + 4,200,000 – 175,000) 4,193,000
Utilities 4,365,000
Repair and maintenance 700,000
Depreciation (W-3.1) 5,847,500
Sundry expenses 640,000
Loss on disposal [750,000 – (800,000 – 40,000)] 10,000
Loss of inventory due to fire 500,000
Loss due to fraud 1,662,750
20,393,250
Excess of income over expenditures 17,855,500

(c) Statement of Financial Position


2016 2016
Fund and Liabilities Assets
Rupees Rupees
Accumulated fund (Balancing) 39,181,500 Fixed asset - WDV (W-3) 45,742,500
Creditors 3,330,000 Stock 2,500,000
Unearned subscription (W-1) 11,825,000 Prepaid rent 175,000
Cash and bank (5,894,000+25,000 ) 5,919,000
Total fund and liabilities 54,336,500 Total assets 54,336,500

W-1: Determination of subscription income 2015


Rupees
Opening unearned subscription income (11,825,000(W-1.1)×9/10) 10,642,500
Add: Receipts for the year (3,300×10,000) 33,000,000
Less: Closing unearned subscription income (W-1.1) (11,825,000)
Subscription income for the year 31,817,500
W-1.1: Closing unearned subscription income
Page 1 of 8
Financial Accounting and Reporting-I
Suggested Answer
Certificate in Accounting and Finance – Spring 2017

Quarter – 1 -
Quarter – 2 (8,250,000×3/12) 2,062,500
Quarter – 3 (5,500,000×6/12) 2,750,000
Quarter – 4 (9,350,000×9/12) 7,012,500
11,825,000

W-2: Tuck shop sales & cost of sales


Opening stock 2,300,000
Add: Purchases of stock (W-2.1) 18,985,000
Less: Loss due to fire, charged to expenditures (500,000)
Less: Closing stock (2,500,000)
Cost of goods sold 18,285,000

Sales (18,285,000/0.80) 22,856,250

W-2.1: Purchases of tuck shop


Closing creditors 3,330,000
Add: Payments to creditors 18,155,000
Less: Opening creditors (2,500,000)
Purchases 18,985,000

W-3: Fixed assets and depreciation


Opening WDV before disposal 28,000,000
Add: Addition (7,350,000 + 17,000,000) 24,350,000
Less: WDV of assets disposed off (800,000 – 40,000) (760,000)
Less: Depreciation for the year (W-3.1) (5,847,500)
Closing WDV 45,742,500

W-3.1: Depreciation
Depreciation on opening WDV [(28,000,000–800,000)×20%] 5,440,000
Depreciation on disposed asset (800,000×20%×3/12) 40,000
Depreciation on addition (7,350,000×20%×3/12) 367,500
Depreciation for the year 5,847,500

A.2 (a) Performance obligation


Performance obligation is a promise in a contract with a customer to transfer to the
customer either:
 a good or service (or a bundle of goods or services) that is distinct; or
 a series of distinct goods or services that are substantially the same and that
have the same pattern of transfer to the customer.

A good or service is distinct if both of the following criteria are met:


 the customer can benefit from the good or service either on its own or together
with other resources that are readily available to the customer; and
 the entity’s promise to transfer the good or service is separately identifiable
from other promises in the contract.

(b) (i) The different services being performed under the contract are separately
identifiable but the customer cannot benefit from a services separately from the
other.
Based on this, ECL should account for services in the contract as a single
performance obligation.
Page 2 of 8
Financial Accounting and Reporting-I
Suggested Answer
Certificate in Accounting and Finance – Spring 2017

(ii) Transfer of software license, software updates and support services are distinct.
However, the software license is delivered before the other services and
remains functional without updates and technical support. Further, the
customer can benefit from each of the services either on their own or together
with other services that are readily available. Thus, the entity’s promise to
transfer the good or service is separately identifiable from other promises in the
contract.

Based on the above, the contract should not be accounted for as a single
performance obligation.

(c) When items of property, plant and equipment are stated at revalued amounts the
following must be disclosed:

 The effective date of the revaluation;


 Whether an independent valuer was involved;
 For each revalued class of property, plant and equipment, the carrying amount
that would have been recognised had the assets been carried under the cost
model; and
 The revaluation surplus, indicating the change for the period and any
restrictions on the distribution of the balance to shareholders.

A.3 (a) Perpetual Inventory


Material A1
Quantity Per unit
Date Description Amount
(kg) cost
------------- Rupees -------------
31-Dec-16 Opening stock 200 3,000.00 600,000
1-Jan-17 Purchase 250 2,800.00 700,000
Balance 450 2,888.89 1,300,000
2-Jan-17 Issuance (100×5/4) (125) 2,888.89 (361,111)
Balance 325 2,888.89 938,889
15-Jan-17 Purchase 250 2,900.00 725,000
Balance 575 2,893.72 1,663,889
16-Jan-17 Purchase return (50) 2,900.00 (145,000)
Balance 525 2,893.12 1,518,889
18-Jan-17 Abnormal loss (100) 2,893.12 (289,312)
Balance 425 2,893.12 1,229,577
26-Jan-17 Issuance (160×5/4) (200) 2,893.12 (578,624)
Balance 225 2,893.12 650,952

Cost of goods sold


Labour @ Rs. FOH @ 120% Cost of goods
Date Description
Material 700 per litre of labour sold
------------------------------- Rupees -------------------------------
4-Jan-17 Sale (100 litres) 361,111 70,000 84,000 515,111
28-Jan-17 Sale (160 litres) 578,264 112,000 134,400 824,664
939,375 182,000 218,400 1,339,775

Under-absorption of overheads (275,000–218,400) 56,600


Total cost of goods sold 1,396,375

Page 3 of 8
Financial Accounting and Reporting-I
Suggested Answer
Certificate in Accounting and Finance – Spring 2017

(b) Periodic Inventory Quantity Rate Rupees


Opening inventory 200 3,000.00 600,000
Purchases 250 2,800.00 700,000
Purchases 250 2,900.00 725,000
Purchase return (50) 2,900.00 (145,000)
Balance 650 2,892.31 1,880,000

Cost of goods sold statement Rupees


Cost of raw material (325×2,892.31) 940,001
Labour (700×260) 182,000
Overheads – Actual 275,000
Cost of goods sold 1,397,001

A.4 Production units Production costs


(units in ‘000) (Rs. in ‘000) (x2) (xy)
(x) (y)
280 6,056 78,400 1,695,680
232 5,080 53,824 1,178,560
305 6,552 93,025 1,998,360
320 6,840 102,400 2,188,800
230 5,064 52,900 1,164,720
200 4,800 40,000 960,000
1,567 34,392 420,549 9,186,120

𝑛(∑ 𝑥𝑦) − (∑ 𝑥)(∑ 𝑦) 6(9,186,120) − (1,567)(34,392)


𝑏(𝑉𝑎𝑟𝑖𝑎𝑏𝑙𝑒) = = = 18.06
𝑛 ∑ 𝑥 2 − (∑ 𝑥)2 6(420,549) − (1,567)2
(∑ 𝑦) − 𝑏(∑ 𝑥) (34,392) − (18.06)(1,567)
𝑎(𝐹𝑖𝑥𝑒𝑑) = = = 1,015.33
𝑛 6

Total Incremental
Units
Capacity production cost of Total cost Cost per unit
produced in
utilization cost y=a+bx repairs (Rs.)
‘000
---------------- Rs. in ‘000 ----------------
85% 408 8,383.81 - 8,383.81 20.55
90% 432 8,817.25 100 8,917.25 20.64
95% 456 9,250.69 100 9,350.69 20.51

The most beneficial option is the production at 95% capacity level where per unit cost is at
minimum.

Page 4 of 8
Financial Accounting and Reporting-I
Suggested Answer
Certificate in Accounting and Finance – Spring 2017

A.5 PARTNERS' CAPITAL ACCOUNT


A B C A B C
-------------- Rupees -------------- -------------- Rupees --------------
Error rectification (W-1) 52,440 34,960 - Opening capital 1,440,000 960,000 -
*2
Goodwill-new ratio 1,067,954 485,434 388,347 General reserve 480,000 320,000 -
Share gifted to C (W-4) 252,717 *1
Goodwill-old ratio 1,165,040 776,694 -
15% share sold to C(W-4) 758,150 Share gifted by A(W-4) 252,717
Balance c/f 1,711,930 778,150 622,520 15% share purchased(W-4) 758,150
3,085,040 2,056,694 1,010,867 3,085,040 2,056,694 1,010,867

Balance b/f 1,711,930 778,150 622,520


Loan - - 600,000
*3
Closing balance 1,925,000 875,000 700,000 Bank (balancing) 213,070 96,850 (522,520)
1,925,000 875,000 700,000 1,925,000 875,000 700,000

*1
A: 1,941,734(W-2)×3/5, B: 1,941,734(W-2)×2/5
*2
A: 1,941,734(W-2)×55%(W-3), B: 1,941,734(W-2)×25%(W-3), C: 1,941,734(W-2)×20%(W-3)
*3
A:3,500,000×55%(W-3), B: 3,500,000×25%(W-3), C: 3,500,000×20%(W-3)

W-1: Rectification of errors Rupees


Decrease in sales (200,000)
Decrease in depreciation expense
2014 (200,000×20%) 40,000
2015 [(200,000–40,000)×20%] 32,000
2016 [(200,000–40,000–32,000)×20%] 25,600
97,600
Reversal of provision (300,000×5%) 15,000
(87,400)

W-2: Goodwill of the firm


Average profit of the last three years [(1,250,000+800,000+950,000–87,400)/3] 970,867
Goodwill (970,867×2) 1,941,734

W-3: Computation of revised profit’s sharing ratio of partners


A (60% – 5%) = 55%; B [40% – (20% – 5%)]=25%; C= 20%

W-4: Share purchase/transfer Rupees


Partner’s capital (1,440,000+960,000) 2,400,000
General reserve 800,000
Error rectification (87,400)
Goodwill 1,941,734
Total capital of old firm 5,054,334

Share gifted by A (5,054,334×5%) 252,717


Share purchased from B (5,054,334×15%) 758,150

Page 5 of 8
Financial Accounting and Reporting-I
Suggested Answer
Certificate in Accounting and Finance – Spring 2017

A.6 Cash flows from operating activities Rupees


Net profit before tax (1,525,948 – 50,000+ 30,000) 1,505,948
Adjustments for:
Depreciation expenses 932,500
Gain on disposal (70,000 – 30,000 – 168,960) (128,960)
Finance cost 141,872
Adjusted profit before working capital changes 2,451,360
Working capital changes:
Accounts receivable (595,452 – 1,273,272) (677,820)
Inventory (320,628 – 685,608) (364,980)
Accounts payable (417,120 – 694,320) (277,200)
Net cash from operating activities 1,131,360
Cash flows from investing activities
Proceed from sale of fixed assets (W-2) 2,020,560
Capital expenditure (1,821,600 – 1,200,000 – 1,478,400) (856,800)
Long term deposits (448,800 – 580,800) (132,000)
Net cash from investing activities 1,031,760
Cash flow from financing activities
Interest paid (105,600 – 63,360 – 141,872) (99,632)
Drawing made by the owner (150,000×12) (1,800,000)
Amount injected by the owner (W-1) 546,832
Repayment of short term loan (1,331,200 – 1,531,200) (200,000)
Net cash used in financing activities (1,552,500)

Net increase in cash and cash equivalents 610,320


Cash at the beginning of year 84,480
Cash at the end of year 694,800
W-1: Movement in capital account
Opening capital 13,665,280
Less: Drawings (150,000×12) (1,800,000)
Add: Profit (1,525,948–50,000+30,000) 1,505,948
13,371,228
Less: Closing capital (13,938,060+30,000–50,000) 13,918,060
Capital injected 546,832
W-2: Disposal proceeds from sale of fixed assets
Freehold land – Opening 6,600,000
Lees: Freehold land – Closing (4,778,400)
Disposal cost 1,821,600
Add: Gain on disposal of freehold land 168,960
Sale proceeds from disposal of freehold land 1,990,560
Insurance claim received against fixed assets 30,000
2,020,560

Page 6 of 8
Financial Accounting and Reporting-I
Suggested Answer
Certificate in Accounting and Finance – Spring 2017

A.7 (a) While analyzing liquidity positions of DFL, it is noted that current ratio has steadily
increased over the years and is better than industry average. However, the quick ratio
has steadily declined and is even lower then industry average. This is a clear evidence
that the increase in liquidity is caused by an increase in inventory.

Further, by considering the nature of highly perishable inventories kept by a dairy


food company, it is a possibility that DFL may bear high inventory losses due to
short expiry.

Based on the above, I do not agree with the claim of DFL’s directors.

(b) Profitability ratios A B


Gross profit ratio (GP ÷ sales) 16.36% 18.00%
Profit to sales (Profit after tax ÷ sales) 9.62% 10.46%
Return on capital employed (Profit before interest and tax ÷ capital
employed) 32.11% 23.81%
Return on asset employed (Profit before interest and tax ÷ assets) 16.69% 15.92%

Company B's gross profit and net profit ratio is slightly higher as compared to
Company A. The difference is not significant and may be on account of higher level
of sales resulting in lesser fixed costs per unit.

Company A’s return on capital employed ratio and return on asset employed ratio
are better than Company B, because Company B has accumulated large balances of
cash despite of availing long term loan. Had Company B had used its cash balances
to pay off the long term loan, it would have both of these ratio better than Company
A.

Liquidity Ratios A B
Current ratio (current assets ÷ current liabilities) 1.36 2.12
Quick ratio (current asset-inventory ÷ liabilities) 0.91 1.75

Company B has better current and quick ratio. However, it appears that these ratios
are better than Company A due to substantially high amount of trade debts in term
of percentage of sales as sales days. It also represents a risk that these trade debts may
prove irrecoverable. Moreover, they may be indicative of inefficient in debt collection
as well.

Working capital turnover ratios A B


Stock turnover days (Stock ÷ Cost of goods sold × 365) [A] 58.60 40.44
Debtor turnover days (Debtor ÷ Revenue × 365) [B] 55.27 73.00
Creditor turnover days (Creditor ÷ Cost of goods sold × 365) [C] 96.65 91.26
Cash operating cycle [A+B–C] (days) 17.22 22.18

Stock turnover of Company B is better than that of Company A. Company B is


turning over its stock 9 times whereas company A is doing it 6 times a year.

Company A is more effectively collecting it’s debtors than Company B. This could
also be due to the fact that Company B is following a lenient credit policy to attract
more revenue. This fact is also supported from higher stock turnover ratio of
Company B.

Company A have availed better credit facility from its creditors but it may have
forgone some settlement discounts which might have resulted in lower gross profit
ratio than that of Company B.
Page 7 of 8
Financial Accounting and Reporting-I
Suggested Answer
Certificate in Accounting and Finance – Spring 2017

Overall cash operating cycle of Company A is better than Company B. Furthermore


Company B has accumulated large balances of cash despite the fact that it has also
availed long term loan. Excess cash balance should have been used to pay off the
long term loan to reduce the finance cost.

(THE END)

Page 8 of 8
THE INSTITUTE OF CHARTERED ACCOUNTANTS OF PAKISTAN

EXAMINERS’ COMMENTS

SUBJECT SESSION
Financial Accounting and Reporting-I Certificate in Accounting and Finance
– Spring 2017

General:

Overall performance of the candidates in this attempt was below average as only 21.6%
of the candidates secured passing marks. Very poor performances were witnessed in
Questions 2 and 3 although apparently they were simple questions. Almost similar
situation existed in Question 5 also. It was generally felt that the performance was poor in
those areas or aspects which were tested after a relatively longer period of time.

Question-wise comments.

Question 1

This question required preparation of income statement and balance sheet of a club and
computation of cash defalcated by an accountant. Certain opening balances and
summarized receipts and payments along with other necessary information was provided
in the question.

An average response was observed as almost 50% of the candidates secured passing
marks. Many students scored high and even full marks also. However, most of the
students made simple mistakes of varying nature on even the easier aspects of the
calculations which are not expected at this level. The major errors were as follows:
 Only few students calculated the opening and closing balances of unearned
subscription correctly and tried various incorrect methods.
 While computing the amount of cash defalcated, instead of considering receipt of
subscription in cash only, total collected amount of Rs. 33m was taken as the cash
receipts. Many students tried to compute the amount by preparing bank account or
through income and expenditure account, instead of cash account and could not
conclude anything.
 Loss due to fraud was not shown in the income and expenditure account.
 Loss of inventory due to fire was ignored in the calculation of cost of sales.
 Loss on disposal of exercise equipment was determined without the impact of
depreciation of 3 months i.e. January to March.
 Accumulated fund was not shown in the statement of financial position.

Page 1 of 4
Examiners’ Comments on Financial Accounting and Reporting-I - Spring 2017

Question 2

This question was based on IFRS 15 ‘Revenue from Contracts with Customers’. It is a
new IFRS and keeping the same in view, a simple question was set to test the basic
understanding of this IFRS. However, the overall performance was poor and only 18% of
the candidates secured passing marks. Part wise comments are given below:

Question 2(a)

This part required the explanation of the term ‘Performance obligation’ and the criteria to
meet for goods and services to be classified as distinct. Very few students were able to
perform well in this part which was a clear indication of the fact that the students had
failed to grasp even the key concepts. Most of the students described the steps to
recognize revenue instead of criteria for distinct goods and services. They are advised
that displaying of knowledge which has not been asked for is of no use in the
examination marking.

Question 2(b)

Two scenarios were given in this part and candidates were required to discuss whether
contracts under those scenarios represent a single performance obligation. In the first
scenario, there were two contracts and the customer was unable to benefit from the
services separately unless both were completed and hence they were to be treated as a
single performance obligation. Based on the same criteria, the contracts referred to in the
second scenario should not have been identified as a single performance obligation. Most
of the students could not highlight these points. Many students simply answered in ‘Yes’
or ‘No’ without discussing the reasons thereof.

Question 2(c)

This was a very simple question and the candidates were asked to state the disclosure
requirements for assets carried at revalued amount under IAS 16 ‘Property, Plant and
Equipment’. The overall performance was average as many students were able to gain
passing marks and some got full marks also. However, some students attempted to
explain the treatment on revaluation of fixed asset instead of discussing the disclosure
requirements.

Question 3

This question required preparation of cost of goods sold statement by valuing inventory
under perpetual and periodic inventory methods. Despite the fact that students had
already studied the topic in Introduction to Accounting, the performance was very poor
and only 19% of the students could secure passing marks. A number of students were
totally unaware of the difference between perpetual and periodic inventory methods and
also made some very basic conceptual errors as mentioned below:

(i) Majority of the students showed quantity of finished product B2 sold as quantity of
material A1 without calculating the required quantity.
(ii) Issuance of inventory was taken at selling price instead of cost.
(iii) Cost of stocks destroyed was taken as zero.

Page 2 of 4
Examiners’ Comments on Financial Accounting and Reporting-I - Spring 2017

(iv) Many students determined the amount of closing inventory only and ignored cost
of goods sold.
(v) Purchase return was valued at average cost.
(vi) Those who computed cost of sales, mostly ignored the under absorption of
overheads.
(vii) Many students treated sale of B2 as issuance of A1.

Question 4

This was a simple question requiring estimation of cost at three production levels using
regression analysis and identifying the most beneficial option. The overall performance
was satisfactory as 54% of the candidates secured passing marks. More than 100 students
scored full marks also.

The major errors were as follows:

 Some students failed to apply the regression formula correctly.


 Some of the students calculated variable and fixed costs using hi-low method even
though it was clearly mentioned in the question to apply regression analysis.
 An important information that fixed costs would also increase if production is
increased to 90% or more was ignored.
 Many students determined the most beneficial option on the basis of total costs
instead of cost per unit.

Question 5

This question required preparation of partners’ capital accounts which involved


computation of goodwill on admission of a new partner and value of interest in the firm
transferred by a partner to his son. Overall performance in the question was below
average because many students made errors on the easier aspects of the question also.
The common mistakes are discussed below:

(i) While calculating average profit for determining goodwill,


 Effects of adjustments on profits were ignored.
 Adjustment for sales return was included as increase in profit instead of
decrease.
 Depreciation was calculated on straight line method rather than written down
value method.
 The entire amount of recovery of debtor was recorded as an increase in profit
instead of considering the impact on provision for doubtful debts only.
(ii) Amounts of share gifted by A and share sold by B were computed incorrectly as
most students ignored general reserve and/or impact of error rectification and/or
goodwill.
(iii) Conversion of C’s loan into capital was mostly ignored.

Page 3 of 4
Examiners’ Comments on Financial Accounting and Reporting-I - Spring 2017

Question 6

This was a routine question requiring preparation of statement of cash flows using
indirect method. The performance was reasonable as about 50% of the candidates secured
passing marks. The mistakes observed were as under:
 Failure to calculate rectified profit.
 Incorrect adjustment for gain on disposal.
 Incorrect calculation of capital expenditure.
 Capital expenditure was taken in financing activity instead of investing activity.
 Amount injected by owner was not calculated correctly.
The common errors noted are discussed below:
 Only few students could classify all the items under their correct head.
 Adjustments to net profit before tax were ignored by a number of students.
 Finance cost and interest paid were correctly reported but change in accrued interest
was also included in working capital changes.
 Decrease in WDV of fixed assets was considered as the sale proceeds.
 Many students included changes in short term loan as a cash flow item and also
included closing balance of short term loan in cash and cash equivalent.

Question 7(a)

This part of the question required the candidates to critically analyze the various ratios
pertaining to three years which were provided in the question and comment on the
management’s claim that the company’s liquidity has improved significantly.

The overall performance was below average as the candidates displayed lack of analytical
skills. Most of them declared on the basis of better current ratio that the management’s
point of view was correct and ignored the quick ratio. Some of them simply stated that
current ratio reflects an improvement whereas quick ratio indicated a declining trend.
Only few could provide an overall view based on both the ratios. The fact that decline in
quick ratio with increase in current ratio was indicative of stock build up was mentioned
by few candidates only. They are advised to seek guidance from ICAP’s suggested
answer.

Question 7(b)

This part of the question contained extracts from financial statements of two companies.
The requirement was to analyse the liquidity, profitability and working capital ratios. The
students were generally able to compute most of the ratios correctly but interpretation
skills were seriously lacking as most of the students only stated that as to which of the
two companies had a better ratio. In depth analysis was mostly missing.

Many students could not differentiate between the three types of ratios and classified
them incorrectly.

THE END

Page 4 of 4
Financial Accounting and Reporting - I
Summary of Marking Key
Certificate in Accounting and Finance – Spring 2017

Note regarding marking scheme:


The marking scheme is given as a guide. However, markers also award marks for alternative
approaches to a question and relevant/well-reasoned comments/explanations. Moreover, the
available marks in a question may exceed the total marks.

Mark(s)
A.1 (a) Statement showing amount of cash defalcation:
 Cash receipts from tuck shop sales 3.0
 Cash receipts from other than tuck shop sales 2.0
 Cash payments 3.0
 Loss due to fraud 1.0

(b) Income and expenditure account:


 Income 1.0
 Expenditures including depreciation 4.0

(c) Statement of financial position:


 Accumulated fund 0.5
 Liabilities including computation of unearned subscription income 4.5
 Assets 1.0

A.2 (a)  Definition of performance obligation 2.0


 01 mark for each criterion 2.0

(b) (i)  Discussion on whether goods and services in the contract represent a
single performance obligation 2.0
 Conclusion 1.0

(ii)  Discussion on whether goods and services in the contract represent a


single performance obligation 2.0
 Conclusion 1.0

(c)  01 mark for each disclosure requirement 4.0

A.3 (a)  Computation of material cost valued under perpetual inventory method
− 0.5 mark for each purchase and balance entry on inventory ledger card 3.0
− 01 mark for each entry of sales, purchase return and abnormal loss 4.0
 Computation of labour and factory overhead 2.0
 Computation of under-absorbed overheads 1.0

(b)  Computation of material cost under periodic inventory method


− 0.5 for each entry of purchase and balance entry on inventory card 1.5
− Purchase return 1.0
− Cost of material 0.5
 Computation of labour and factory overhead 2.0

Page 1 of 2
Financial Accounting and Reporting - I
Summary of Marking Key
Certificate in Accounting and Finance – Spring 2017

Mark(s)
A.4  Computation of ∑x and ∑y 1.0
 Computation of ∑x2 and ∑xy 2.5
 Computation of a and b 2.0
 02 mark each for determination of cost per unit at 85%, 90% and 95% 6.0
 Conclusion 0.5

A.5  Distribution of general reserve and rectification entry 3.5


 Distribution of goodwill among old and new partners 3.0
 Determination of share gifted by A and share purchased from B 3.0
 Determination of revised profit sharing ratio of A and B 1.0
 Adjustment to partners’ capital accounts relating to loan and cash settlement 1.0
 Closing capital 0.5

A.6  *Cash flow from operating activities:


− Net profit before tax 1.0
− Adjustment before working capital changes 1.5
− Working capital changes 1.5
 *Cash flow from investing activities:
− Payments and receipts from purchase and sale of fixed assets 3.0
− Long term deposits 0.5
 *Cash flow from financing activities:
− Interest paid 1.0
− Repayment of short term loan 0.5
− Drawing made by the owner 0.5
− Capital injection 2.0
 Cash at the beginning of the year 0.5
* No marks were deducted if students misclassified any of the above items or did not classify the above items in operating,
investing or financing activities

A.7 (a)  Discussion on the liquidity position of the company 2.0


 Evaluation of the claim of the director 1.0

(b)  Up to 0.5 mark for calculation of each ratio of each company 6.0
 Analysis of profitability ratios of both companies 2.0
 Analysis of liquidity ratios of both companies 2.0
 Analysis of working capital ratios of both companies 2.0

(THE END)

Page 2 of 2
Certificate in Accounting and Finance Stage Examination
The Institute of 8 September 2017
Chartered Accountants 3 hours – 100 marks
of Pakistan Additional reading time – 15 minutes

Financial Accounting and Reporting-I


Q.1 A, B and C had been in partnership for many years sharing profits and losses in the ratio of
their fixed capital i.e. 3:2:1. The assets and liabilities appearing in the firm’s statement of
financial position as at 31 December 2016 were as follows.
Particulars Rupees
Non-current assets 500,000
Stocks 250,000
Debtors 200,000
Cash 300,000
Current liabilities 150,000

On 31 December 2016, it was decided to dissolve the partnership. On the said date, the
current account balances of the partners were as follows:
A B C
Rs. 50,000 Rs. 30,000 Rs. (15,000)

Assets and liabilities were realized/settled as under:


(i) Debtors were taken by B at agreed value of Rs. 190,000.
(ii) A non-current asset having book value of Rs. 70,000 was taken by B at an agreed
value of Rs. 60,000.
(iii) Certain assets which had not been recorded in the books were taken by C for
Rs. 80,000.
(iv) Remaining non-current assets and stocks were sold for cash amounting to Rs. 550,000
and Rs. 225,000 respectively.
(v) Current liabilities were settled at Rs. 155,000.
(vi) Dissolution expenses of Rs. 30,000 were paid by A.

Required:
Prepare the following accounts to show the effect of dissolution:
(a) Realization (05)
(b) Partners’ capital (05)
(c) Cash (02)

Q.2 The following information pertains to Sherdil Limited (SL):


(i) Buildings and equipment were acquired on 1 January 2014 for Rs. 450 million and
Rs. 50 million respectively.
(ii) The relevant information relating to both assets is summarised below:
Depreciation Subsequent
Assets Life/rate
method measurement
Buildings Straight line 20 years Annual revaluation
Equipment Reducing balance 10% Cost
SL transfers the maximum possible amount from revaluation surplus to retained
earnings on an annual basis.
(iii) The revalued amount of buildings as determined by Accurate Valuers (Private)
Limited, an independent valuation company, on 1 January 2015 and 2016 was
Rs. 456 million and Rs. 378 million respectively.
(iv) Equipment costing Rs. 35 million was purchased on 1 August 2015. Half of the
equipment purchased on 1 January 2014 was disposed off on 30 June 2016.
Financial Accounting and Reporting-I Page 2 of 4

Required:
In accordance with International Financial Reporting Standards, prepare a note on ‘Property
plant & equipment’ (including comparative figures) for inclusion in SL’s financial statements
for the year ended 31 December 2016. (18)

Q.3 Following is the trial balance of Younus Limited (YL) as on 30 June 2017:

Debit Credit
Particular Particular
Rs. in ‘000 Rs. in ‘000
Property, plant and equipment 200,000 Share capital (Rs. 10 each) 35,000
Receivables and advances 13,000 Un-appropriated profit 66,820
Office rent 1,120 5% Bank loan 52,000
Opening stock 54,000 Trade payables 10,000
Taxation 6,000 Accumulated dep. – 30 June 2017 120,000
Cash and bank 40,000 Sales 240,000
Purchases 170,000
Selling expenses 20,000
Administrative expenses 17,000
Financial charges 2,700
523,820 523,820

The following additional information is available:


(i) On 1 July 2016 engine of a delivery truck seized and was replaced at a cost of
Rs. 2 million on the next day. Rs. 1.2 million was paid in cash whereas the remaining
amount was adjusted against the trade in value of the seized engine. The payment was
charged to selling expenses.

The delivery truck was purchased on 1 July 2010. The cost of the delivery truck is
Rs. 5 million of which approximately Rs. 1 million is attributable to the seized engine.
Delivery trucks are depreciated over their useful life of 10 years.
(ii) Certain goods despatched on 28 June 2017 reached YL’s warehouse on 2 July 2017.
Break-up of the amount paid against these goods is as follows:

Rs. in ‘000
20% advance to supplier 500
Insurance in transit 50
Delivery charges 100

The above amounts are appearing under the head ‘Receivables and advances’.

(iii) Cost of stock in hand as on 30 June 2017 is Rs. 50 million.


(iv) During the year, YL gave free samples to certain customers. The selling price and
gross profit on these goods was Rs. 5.4 million and 20% of cost respectively. No
adjustment has been made in the books in this regard.
(v) Office rent pertains to the period from July 2016 to December 2017 and is inclusive of
an upward revision of 10% with effect from 1 January 2017.
(vi) Bank loan was obtained on 1 July 2015. The principal is repayable in 20 equal
quarterly instalments. The principal along with interest is paid on the first day of the
next quarter.
(vii) Tax expense for the year is Rs. 7.7 million.

Required:
Prepare statement of financial position as at 30 June 2017 and statement of profit or loss for
the year ended 30 June 2017 in accordance with International Financial Reporting
Standards. (20)
Financial Accounting and Reporting-I Page 3 of 4

Q.4 The following cost data pertains to Al-Khair Limited (AKL):

Production (Units) Cost (Rs.)


90000 712,500
70000 590,000
40000 440,000
20000 300,000

AKL has identified that total fixed costs increase by 20% when production exceeds
35000 units and the average variable costs for all units increase by 5% if production exceeds
75000 units.

Required:
(a) Construct total cost functions for different production ranges using high/low method.
(b) Determine the most feasible option if AKL can sell 25000, 55000 and 80000 units at
Rs. 20, Rs. 17 and Rs. 13 respectively. (09)

Q.5 Progressive Steel Limited (PSL) commenced business in 2015. The following comparative
data pertains to the year ended 30 June 2017:

PSL Industry
Description
2017 2016 2017
Gross profit margin 13% 13% 16%
Net profit margin 8% 7% 10%
Return on shareholders’ equity 22% 18% 25%
Current ratio 1.2 1.6 1.5
Debt to equity ratio 40:60 30:70 50:50
Cash operating cycle in days 119 135 118

Required:
For each ratio/data give possible reasons for variation from comparative and industry data. (12)

Q.6 (a) Jupiter Limited (JL) entered into a two year contract on 1 January 2017, with a
customer for the maintenance of computer network. JL has offered the following
payment options:
Option 1: Immediate payment of Rs. 200,000.
Option 2: Payment of Rs. 110,000 at the end of each year.

The applicable discount rate is 6.596%.

Required:
Prepare journal entries to be recorded in the books of JL under each option over the (05)
period of contract.

(b) Pluto Limited (PL) sells industrial chemicals at following standalone prices:

Rupees
Products
(per carton)
C-1 100,000
C-2 90,000
C-3 110,000

PL regularly sells a carton each of C-2 and C-3 together for Rs. 170,000.

Required:
Calculate the selling price to be allocated to each product, in case PL offers to sell one
carton of each product for a total price of Rs. 260,000. (05)
Financial Accounting and Reporting-I Page 4 of 4

(c) An entity shall recognise revenue when (or as) the entity satisfies a performance
obligation by transferring a promised good or service to a customer. An asset is
transferred when (or as) the customer obtains control of that asset.

Required:
List the different indicators of transfer of control. (04)

Q.7 Saleem is the owner of S-Mart, a grocery store. His accountant resigned and left on
1 January 2017. Saleem suspects that the previous accountant was involved in some sort of
misappropriation. The information available with him is as follows:

(i) Summary of bank statement:


Receipts Rupees Payments Rupees
Balance as at 1 Jan 2016 250,000 Suppliers 1,807,500
Cheques from debtors 824,000 Salaries 48,000
Cash sales 1,450,000 Rent 72,000
Sale of old vehicle on 1 Jan 2016 15,000 Utilities 36,000
Other expenses 24,750
New vehicle on 1 Mar 2016 230,000
Balance as at 31 Dec 2016 320,750
2,539,000 2,539,000

(ii) Other balances extracted from the records maintained by the previous accountant:
31-Dec-2016 31-Dec-2015
Particulars
---------- Rupees ----------
Furniture and fixtures – WDV 555,000 550,000
Equipment – WDV 64,000 80,000
Vehicle – WDV 210,000 18,500
Inventory 215,000 250,000
Debtors 340,000 260,000
Advance rent - 3,000
Cash in hand 31,510 45,000
Creditors 354,500 100,000
Salaries payable 22,000 18,000

(iii) Before depositing the receipts from cash sales in the bank, Saleem took Rs. 12,000 per
month for personal use. All other payments were made through bank and the debtors
settled their accounts through cheques.
(iv) The creditors have confirmed the balances due from them. However review of the
statement provided by one of the creditors indicates that goods returned for cash
amounting to Rs. 24,000 were not recorded in the books.
(v) Unpaid invoice for furniture purchased during the year for Rs. 45,000 is included in
creditors.
(vi) The margin on cash sales and credit sales is 20% and 25% respectively. From 1 July
2016, prices to cash customers were further reduced by 6% due to which quantity sold
against cash in the 2nd half of the year increased by 25% as compared to the first half of
the year.
(vii) All the debtors confirmed their balances except an amount of Rs. 50,000. On
investigation it was found that the related goods had been issued against fake invoices.

Required:
(a) Determine the amount of suspected fraud. (04)
(b) Prepare statement of profit or loss for the year ended 31 December 2016. (11)

(THE END)
Financial Accounting and Reporting-I
Suggested Answers
Certificate in Accounting and Finance – Autumn 2017

Ans.1 (a) Realization


Rupees Rupees
Non-current assets 500,000 Current liabilities 150,000
Stock 250,000 Non-current assets (Cash) 550,000
Debtors 200,000 Stock (Cash) 225,000
Current liabilities (Cash) 155,000 Debtors Capital B 190,000
Dissolution exp. (by A) 30,000 Capital B 60,000
Capital C 80,000
Gain on realization
A 60,000
B 40,000
C 20,000
1,255,000 1,255,000

(b) Partnership capital account A B C Total


------------------------ Rupees ------------------------
Opening balance – in P&L ratio
(W-1) 517,500 345,000 172,500 1,035,000
Dissolution expenses 30,000 - - 30,000
Non-current assets taken over by B - (60,000) - (60,000)
Assets taken on by C - - (80,000) (80,000)
Debtors - (190,000) - (190,000)
Gain on realization 60,000 40,000 20,000 120,000
Current Account balances 50,000 30,000 (15,000) 65,000
Final settlement (Cash) (657,500) (165,000) (97,500) (920,000)
- - - -

W-1: Partners’ capital Rupees


Non-current assets 500,000
Stock 250,000
Debtors 200,000
Cash 300,000
1,250,000
Current liabilities (150,000)
Net assets 1,100,000
Less: Current account balances (65,000)
Partners’ capital 1,035,000

(c) Cash
Rupees Rupees
Opening balance 300,000 Settlement of liabilities 155,000
Realization of assets 775,000 Final settlement A 657,500
B 165,000
C 97,500
1,075,000 1,075,000

Page 1 of 7
Financial Accounting and Reporting-I
Suggested Answers
Certificate in Accounting and Finance – Autumn 2017

Ans.2 Property, plant and equipment 2016


Building Equipment
----------- Rs. in million -----------
Cost / Revalued amount
Opening 456.00 85.00
Cancellation (24.00)
Revaluation Surplus [28.5 – (28.5÷19)] (27.00)
Revaluation loss (27.00)
Disposal (25.00)
Closing 378.00 60.00
Accumulated depreciation
Opening 24.00 10.96
Cancellation (24.00)
Disposal [2.5 + 2.25+ 1.01] (5.76)
Depreciation
(378÷18) 21.00
[(74.04-20.25)×10% + (20.25×10%×6÷12)] 6.39
Closing 21.00 11.59

Opening net book value 432.00 74.04

Closing net book value 357.00 48.41

Property, plant and equipment 2015


Building Equipment
----------- Rs. in million -----------
Cost / Revalued amount
Opening 450.00 50.00
Cancellation (22.50)
Revaluation 28.50
Addition 35.00
Closing Cost 456.00 85.00

Accumulated depreciation
Opening 22.50 5.00
Cancellation (22.50)
Disposal
Depreciation
(456 ÷19) 24.00
[(45×10%)+(35×10%×5÷12)] 5.96
Closing 24.00 10.96

Opening net book value 427.50 45.00

Closing net book value 432.00 74.04

Measurement base Revaluation model Cost model


Useful life 20 years 10 years
Depreciation method Straight line Reducing balance

The last revaluation was performed on 1 July 2016 by Accurate Valuers (Private) Limited, an independent
firm of valuers. Revaluations are performed annually.

Carrying value had the cost model been used instead 382.50 405.00
(450×0.80) (450×0.90)

Page 2 of 7
Financial Accounting and Reporting-I
Suggested Answers
Certificate in Accounting and Finance – Autumn 2017

Ans.3 Younus Limited


Statement of financial position
As at 30 June 2017
2017
Rs. in ‘000
Non-current assets
Property, plant and equipment (W-1) 81,200

Current assets
Stock in trade [50,000+ (500÷0.2) + 50+ 100] 52,650
Receivables and advances [13,000–(500+50+100)] 12,350
Short term prepayments (1120 ÷ 1.6 × 0.55 ) 385
Cash & Bank 40,000
105,385
Total assets 186,585

Equity
Issued subscribed and paid up capital 35,000
Unappropriated profit (66,820+18,415) 85,235
120,235
Non-current liabilities
Long term loan (52000 – 16,000) 36,000

Current liabilities
Trade and other payables (10,000+(2,500–500) 12,000
Accrued markup (52,000 × 5% × 3 ÷12 ) 650
Current portion of long term financing (52,000 × 4 ÷ 13) 16,000
Taxation-net (7,700–6,000) 1,700
30,350
Total equity and liabilities 186,585

Younus Limited
Statement of profit or loss
For the year ended 30 June 2017
2017
Rs. in ‘000
Sales 240,000
Cost of sales (54,000+170,000–50,000 -4,500) (169,500)
Gross profit 70,500
Selling and distribution expenses (20,000 –1,200+500–100+ 4,500) (23,700)
Administrative expenses (17,000+ 735) (17,735)
Operating profit 29,065
Financial charges ( 2,700+ 650) (3,350)
Other operating income ( 800– 400) 400
Profit before taxation 26,115
Taxation (7,700)
Profit for the year 18,415

W-1: Property, plant and equipment Cost Accumulated Book value


Rs. in ‘000
Given 200,000 120,000 80,000
Reversal of old engine depreciation (1,000×10%) (100) 100
Disposal of old engine (1,000) (600) (400)
Cost of new engines 2,000 2,000
Depreciation of new engine (2,000÷4×25%) 500 (500)
201,000 119,800 81,200
Page 3 of 7
Financial Accounting and Reporting-I
Suggested Answers
Certificate in Accounting and Finance – Autumn 2017

Ans.4 (a) Units Cost (Rs.)


High 70,000 590,000
Low 40,000 440,000
30,000 150,000

Workings: Rupees
Variable cost 150,000/30,000 5.00

Total cost for 20,000 units 300,000


Variable cost at 20,000 units (5×20,000) 100,000
Fixed cost if production is upto 35,000 units 200,000

Fixed cost if production exceeds 35,000 units (200,000×1.2) 240,000

Increase in variable cost if production exceeds 75,000 units (5×1.05) 5.25

Cost function
Up to 35,000 units: 200,000+5x
35,001–75,000 units: 240,000+5x
More than 75,000 units: 240,000+5.25x

(b) Quantity Sales (Rs.) Cost (Rs.) Profit (Rs.) Remarks


25,000 500,000 325,000 175,000
55,000 935,000 515,000 420,000 Feasible option
80,000 1,040,000 660,000 380,000

Ans.5 Reasons for fluctuation with


Ratios Reason for fluctuation with Industry
previous year
Gross profit In line with previous year. No variation. Lower than industry
margin  The company is in initial phase and may
have kept the selling prices lower than the
industry to gain the market share.
 The company may not have been able to
purchase raw material at prices which is
available to its competitors.
 The company may not have been able to
obtain economies of scale in its
production which may have been obtained
by its competitors.
 Possibility of higher production costs.
Net profit Higher than previous year: Lower than industry however, the difference
margin  Tight control over operating costs. is mainly attributed to lower gross profit
 Increase in other income. margin.
 Decrease in fixed cost per unit due to
increase in sale.

Return on Higher than previous year: Lower than industry


shareholder's  Reduction in tax rates.  Lower gross profit and net profit margins.
equity  Reduction in interest rates.  Lower leverage.
 Decrease in equity might be due to  Higher net assets resulting in higher
buyback of shares. equity.
 Distribution of profits from previous year
which resulted in decrease in equity.

Page 4 of 7
Financial Accounting and Reporting-I
Suggested Answers
Certificate in Accounting and Finance – Autumn 2017

Current ratio Lower than previous year: Lower than industry


 The company might have obtained  Since the debt equity ratio is lower than
running finance facility to fund it's the industry, company might have
operations in the current year. obtained running finance or might have
 Long term loan payments might have availed extended credit terms from
become due in the next 12 month, which suppliers.
decreases the current ratio.
 Decrease in current assets due to better
inventory management/ reduction in
credit period of debtors.
Debt to equity Higher than previous year Lower than industry
ratio  Decrease in reserves due to dividend pay-  Being a new entrant the company may be
out. in the phase of expansion thereby raising
 Further debt obtained during the period. debt accordingly.
 Decrease in equity might be due to
buyback of shares.
Cash operating Lower than previous year In line with industry.
cycle  Increase in current liabilities might be due
to increase in credit period.
 Decrease in current assets which might be
due to greater stock turnover or better
inventory management.

 By giving lower credit days to debtors.

Page 5 of 7
Financial Accounting and Reporting-I
Suggested Answers
Certificate in Accounting and Finance – Autumn 2017

Ans.6 (a) Option 1: Lump sum payment


Debit Credit
Date Description
---------- Rupees ----------
01-01-17 Cash 200,000
Contract liability 200,000
31-12-17 Interest expense 13,193
Contract liability (200,000×6.596%) 13,193
31-12-17 Contract liability 110,000
Revenue 110,000
31-12-18 Interest expense 6,807
Contract liability 6,807
(200,000+13,192–110,000)×6.596%
31-12-18 Contract liability 110,000
Revenue 110,000
Option 2: Normal payment terms
Debit Credit
Date Description
---------- Rupees ----------
31-12-17 Cash 110,000
Revenue 110,000
31-12-18 Cash 110,000
Revenue 110,000
(b) DISCOUNT ALLOCATION
Stand alone 1st discount Price after 2nd discount Price after
Chemical price allocation 1st discount allocation 2nd discount
------------------------------------------- Rupees -------------------------------------------
C-1 100,000 - 100,000 3,704 96,296
(100,000×10,000/270,000)
C-2 90,000 13,500 76,500 2,833 73,667
(90,000×30,000/200,000) (76,500×10,000/270,000)
C-3 110,000 16,500 93,500 3,463 90,037
(110,000×30,000/200,000) (93,500×10,000/270,000)
300,000 30,000 270,000 10,000 260,000

(c) Indicators of transfer of control include the following:


 The entity has a present right to payment for the asset
 The customer has legal title
 The customer has physical possession (except in case of bill and hold, consignment
sales and repos)
 The customer has significant risks and rewards of ownership of the asset
 The customer has accepted the asset

Ans.7 (a) Loss due to defalcation Rupees


Cash embezzled through purchase returns 24,000
Stock embezzled through fake debtors (500,000×0.75) 37,500
Cash defalcated from cash sales (W-1) 50,740
112,240
W-1: Cash account Rupees
Opening balance 45,000
Add: Cash sales 1,631,250
Less: Drawings (144,000)
Payment into bank (1,450,000)
Closing balance of cash (31,510)
Cash defalcated 50,740

Page 6 of 7
Financial Accounting and Reporting-I
Suggested Answers
Certificate in Accounting and Finance – Autumn 2017

(b) Statement of profit or loss ------------ Rupees ------------


Total cash sales (1,687,500 -56,250) (W-2) 1,631,250
Total credit sales (W-4) 854,000
2,485,250
Less: Cost of goods sold
Opening inventory 250,000
Purchases (W-3) 2,017,000
Less: Return outward (Alternatives are available) (24,000)
Stock misappropriated (37,500)
Less: Closing stock (215,000) (1,990,500)
Gross profit 494,750
Less: Operating expenses
Rent expenses (72,000+3,000) 75,000
Utilities 36,000
Other expenses 24,750
Loss on sale of vehicle (15,000–18,500) 3,500
Salaries expense (48,000–18,000+22,000) 52,000
Loss due to defalcation 112,240
Depreciation: Furniture (550,000+45,000–555,000) 40,000
Equipment (80,000–64,000) 16,000
Vehicle (230,000–210,000) 20,000 (379,490)
Net profit 115,260

W-2: Total cash sales


Cost Gross sales Discount (6%)
Period Ratio
----------------- Rupees -----------------
First six month 1 600,000 750,000 -
Second six months 1.25 750,000 937,500 56,250
Total (W-5)1,350,000 1,687,500 56,250

W-3: Creditors Rupees


Closing balance (354,500–45,000) 309,500
Add: Payment during the year 1,807,500
2,117,000
Less: Opening balance (100,000)
Purchases during the year 2,017,000

W-4: Debtors Rupees


Closing balance (340,000–50,000) 290,000
Add: Receipts during the year 824,000
1,114,000
Less: Opening balance (260,000)
Credit sales 854,000
Cost of goods sold credit sales (854,000×75%) 640,500

W-5: Cost of goods sold Rupees


Total Cost of goods sold (P&L) 1,990,500
Less: Cost of credit sales (W-3) (640,500)
Cost of cash sales 1,350,000

(THE END)

Page 7 of 7
THE INSTITUTE OF CHARTERED ACCOUNTANTS OF PAKISTAN

Examiners’ comments
Financial Accounting and Reporting-I
Certificate in Accounting and Finance
Autumn 2017 Examinations

General Comments:

Overall passing ratio of 13.4% was far below the previous two results. 9.8% students were just
short of 5 or fewer marks and could have easily obtained them had they covered all areas of
the syllabus. 17% of the students could not even secure 20 marks in the paper.

In this paper, though Q2, Q4 and Q5 were easier questions but a significant number of students
secured less than 20% marks even in these questions due to ‘cherry-picking’ topics from the
syllabus. Moreover, students were found struggling to apply their knowledge when questions
were presented slightly differently.

Students are using past papers as a key element of their examination preparation but they
should remember that topics/sub-topics/variations not covered in past papers are also
examinable. Majority of the students lost achievable marks in Question 2 & 4 just because
such variation was not previously examined.

Some examination technique issues also need to be improved which would have lifted many
marginal fails into the pass category. Many students are failing because of technique rather
than knowledge or ability.

Question-wise Comments:

Question 1

The question required preparation of relevant accounts on Partnership Dissolution. It was an


easy question with high performance i.e. 80.4% of the students secured passing marks.
However, the following errors were observed:

 Calculation of opening capital account balances proved difficult. For arriving at the
opening balance of partners’ capital account, current account balance of Rs. 65,000 was
not adjusted against total net assets.
 Many students accounted for unrecorded assets and allocated it to partners’ capital account
instead of simply crediting the proceeds to realization account.
 Closing cash was allocated to partners in their profit sharing ratio instead of paying to each
partner his capital account’s balance.

Page 1 of 3
Examiners’ comments on Financial Accounting and Reporting-I,
CAF Examination Autumn 2017

Question 2

The question required preparation of note on property, plant and equipment. Only 20.4% of the
students secured passing marks in the question. Note in accordance with IAS 16 was examined
for the first time at FAR I level. Though it was an easier question, most of the students found it
difficult to prepare the required note. 39% of the students could not even score 20% marks in
the question which shows that they had not studied this area of the syllabus. The common
errors were as follows:

 Rather than preparing statement showing opening and closing carrying amounts (or
alternatively cost and accumulated depreciation) and movement during the years, many
students prepared T-accounts of cost and accumulated depreciation.
 Cancellation of opening accumulated depreciation for revaluation was ignored.
 Effect of disposal was taken incorrectly in calculating depreciation of remaining assets.
 Important disclosure requirements like measurement bases, useful lives, depreciation
method, revaluation details and carrying value of revalued assets at cost model for the both
the years were missed.

Question 3

It was a traditional question requiring preparation of financial statements from a trial balance
combined with several adjustments. A general weakness amongst most students was that they
failed to make a reasonable attempt at the question due to presence of one or two difficult
adjustments. Only 5.3% students secured passing marks. However, only 13.2% secured less
than 20% marks so majority of the students made just below passing performance in this
question. There were atleast 7 easy marks available in the question which could have been
obtained even if the students had just used the unadjusted amounts in the financial statements.
But it was observed that students left the question halfway if they could not incorporate the
effect of one or two adjustments. Some of the errors noted were as follows:

 In adjustment (i) the disposal of the old engine was not recorded and depreciation on the
new engine was calculated on original useful life of the delivery truck rather than the
remaining useful life.
 In adjustment (ii), goods in transit were not recorded. Those who recorded them did not
adjust the advance with the amount payable in respect of these goods.
 Amount of prepaid rent was incorrectly calculated.
 Mark-up on bank was either calculated for the full year or not calculated at all.
 For calculating current portion of loan, amount of one instalment was calculated by
dividing the outstanding loan amount with total instalments rather than outstanding
instalments.

Question 4

The question required construction of cost function for different production ranges. Only
24.2% of the students could secure passing marks whereas 60.4% of the students secured less
than 20% marks which showed that they had not studied the sub-topic. Those who had studied
the topic scored well thus answers tended to be very polarised, either very good or very poor.
Some common errors were as follows:

Page 2 of 3
Examiners’ comments on Financial Accounting and Reporting-I,
CAF Examination Autumn 2017

 Regression method and simultaneous equation methods were used though the requirement
of the question specifically mentioned the use of high low method.
 Instead of formulating cost functions, computations were prepared for fixed and variable
cost.
 In part (b), many students did not calculate the total profit for the different selling levels
and gave conclusion on the basis of cost per unit or profit per unit.

Question 5

The question required comparison of given ratios with previous year and industry data. Only
9.9% of the students could secure passing marks. Since the question was based on the
relatively infrequently examined topic of interpretation of ratios, many students were
unprepared. 62.7% of the students secured less than 20% marks.

Generally, students only focus on the formulas of the ratios. However, ratios were already
given and the students were required to identify reasons for variation from the given
information. The poor performance of the student was due to lack of their ability to interpret
the underlying ratios. Many students thought that just saying a ratio had gone up or down
amounted to interpretation. Also, merely stating increase or decrease in nominator and
denominator did not carry any marks.

Question 6

The question was based on IFRS 15 which is a new area of the syllabus. Only 4% students
could secure passing marks in this question. 55% students could not even answer the basic
concept examined in part (c). If students could not perform well in basic question then how
could they be expected to solve advance questions on this area?

 In part (a) the students had no idea of adjusting the promised amount of consideration for
the effects of the time value of money.
 In part (b), the transaction price needed to be allocated to the three products in two steps /
stages in their relative selling prices. Mostly the transaction price was allocated in a single
step.

Question 7

The question required preparation of statement of profit or loss and determination of the
suspected amount of fraud. 11% students obtained passing marks. The calculation of sales was
a difficult part of the question. Other than that, some easy marks were available to the students.
It was observed that students consumed lot of time in calculating sales and did not attempt the
remaining easier areas of the question. Some of the common errors were as under:

 Most of the students could not compute loss due to defalcation which consisted of cash
embezzled through purchase return, stock embezzled through fake debtors and cash
defalcated through cash sales.
 Cost of stock embezzled through fake debtors was not adjusted in calculation of cost of
goods sold.
 Loss on disposal of vehicle was incorrectly calculated.

(THE END)
Page 3 of 3
Financial Accounting and Reporting - I
Summary of Marking Key
Certificate in Accounting and Finance – Autumn 2017

Note regarding marking scheme:


The marking scheme is given as a guide. However, markers also award marks for alternative
approaches to a question and relevant/well-reasoned comments/explanations. Moreover, the
available marks in answer may exceed the total marks of a question.

Mark(s)
A.1 (a)  Recording of assets and liabilities taken from abstracts of statement of financial
position 1.0
 Recording of assets and liabilities determined from adjustments / further
information 3.5
 Allocation of gain/loss on realization to partners as per given ratio 0.5

(b)  Computation and allocation of opening capital among partners 1.5


 Recording of partners’ current account balances 0.5
 Recording of gain/loss on realization from part (a) 0.5
 Recording of assets taken and expenses borne by partners 2.0
 Determination of final settlement amount 0.5

(c)  Recording of opening balance 0.5


 Allocation of assets realized and liabilities settled for cash 1.0
 Allocation of final settlement amount among partners 0.5

A.2  Presentation of note in accordance with IFRS (including correct mentioning of


opening and closing balances) 4.0
 Disclosure of addition and disposal of assets 1.0
 Determination of depreciation for the year 5.0
 Treatment of revaluation surplus / loss 4.0
 Determination and disclosure of carrying value of revalued asset under cost model 2.0
 Disclosures pertaining to revaluation, measurement base, useful life and depreciation
method 2.0

A.3  Determination of:


− property, plant and equipment 2.75
− stock in trade 1.25
− receivables and advances 0.75
− short term prepayments 1.0
− cash and bank 0.25
 Determination of equity 0.75
 Determination of long-term loan 0.25
 Determination of:
− trade and other payables 0.75
− accrued markup 1.0
− current portion of long term loan 1.0
− tax liability 0.5
 Computation of:
− Sales 0.25
− Cost of sales 1.25
− Selling and distribution expenses 2.75
− Administrative expenses 1.25
− Financial charges 0.75
− Other operating income 1.0
− Taxation 0.25
 Presentation and disclosure (including correct classification) 2.25

Page 1 of 2
Financial Accounting and Reporting - I
Summary of Marking Key
Certificate in Accounting and Finance – Autumn 2017

Mark(s)
A.4 (a)  Determination of variable and fixed costs by using high/low method 3.0
 Construction of total cost functions at different production levels 3.0

(b) Analysis of given activity levels and determination of the most feasible option 3.0

A.5  Discussion on each relevant reason for variation from comparative period 6.0
 Discussion on each relevant reason for variation from industry data 6.0

A.6 (a)  Preparation of journal entries under option 1 4.0


 Preparation of journal entries under option 2 1.0

(b)  Allocation of discount when C-2 and C-3 are sold together 2.0
 Allocation of discount when C-1, C-2 and C-3 are sold together 3.0

(c) 01 mark for listing each indicator of transfer of control 4.0

A.7 (a)  Computation of cash embezzled through purchase returns 1.0


 Computation of stock embezzled through fake debtors 1.0
 Computation of cash defalcated from cash sales 2.0

(b) Statement of profit or loss


 Cash and credit sales 3.0
 Cost of goods sold 4.0
 Operating expenses other than depreciation 2.0
 Depreciation 2.0

(THE END)

Page 2 of 2
Certificate in Accounting and Finance Stage Examination
The Institute of 7 March 2018
Chartered Accountants 3 hours – 100 marks
of Pakistan Additional reading time – 15 minutes

Financial Accounting and Reporting-I


Q.1 Kidz Party & Co. (KPC) manufactures and sells toys. Following information is available
regarding four of its inventory items as on 31 December 2017:

Normal
Cost per unit
Items Units selling price
(Rs.)
per unit (Rs.)
Toy cars 10,000 1,250 1,200
Doll houses 5,000 1,800 2,700
Stuffed toys 1,850 1,200 1,900
Minion costumes 870 1,500 2,500

Following information is also available:


(i) A sales order for 3,000 toy cars @ Rs. 1,100 per unit is in hand. The remaining units
can be sold at normal selling price after incurring selling cost of Rs. 150 per unit.
(ii) Doll houses include 1,000 defective units with no scrap value. 20% of the remaining
doll houses are damaged and can be sold at 50% of cost.
(iii) Stuffed toys costing Rs. 420,000 were accidentally damaged and are beyond repair.
KPC plans to sell these toys as scrap. Proceeds from such sale are estimated at
Rs. 175,000 and the sale would require transportation cost of Rs. 6,300.
(iv) All minion costumes have manufacturing faults and can be sold in present condition at
Rs. 1,350 per unit. However, 60% of the units can be rectified at a cost of
Rs. 200 per unit after which they can be sold at Rs. 1,600 per unit.

Required:
Calculate the amount at which above inventory items should be carried as on
31 December 2017 in accordance with IAS 2 ‘Inventories’. (08)

Q.2 (a) Define ‘performance obligation’. List any six examples of promised goods and
services as per IFRS 15 ‘Revenue from Contracts with Customers’. (05)

(b) On 1 October 2017, Galaxy Telecommunications (GT) entered into a contract with a
bank for supplying 20 smart phones to the bank staff with unlimited use of mobile
network for one year. The contract price per smart phone is Rs. 34,650 and the price is
payable in full within 10 days from the date of contract. At the end of the contract, the
phones will not be returned to GT.

The entire amount received as per contract was credited by GT to advance from
customers account. The smart phones were delivered on 1 November 2017.

If sold separately, GT charges Rs. 18,000 for a smart phone and a monthly fee of
Rs. 1,800 for unlimited use of mobile network.

Required:
Prepare adjusting entry for the year ended 31 December 2017 in accordance with
IFRS 15 ‘Revenue from Contracts with Customers’. (04)
Financial Accounting and Reporting-I Page 2 of 5

Q.3 Following information pertains to Nadir Limited:

Extract from statement of profit or loss for the year ended 31 December 2017
Rs. in ‘000
Profit before taxation 8,955
Taxation (2,945)
Profit after taxation 6,010

Extract from statement of financial position as on 31 December 2017


2017 2016 2017 2016
Equity and liabilities Assets
---- Rs. in ‘000 ---- ---- Rs. in ‘000 ----
Share capital 12,400 10,000 Property plant &
Share premium 1,400 - equipment – net 21,400 15,800
Retained earnings 13,450 12,440 Current assets:
Surplus on revaluation 4,000 - Stock-in-trade 5,600 5,750
Non-current liabilities: Trade receivables – net 6,840 4,446
Long-term loans 4,100 5,000 Other receivables 2,385 800
Current liabilities: Cash & bank 2,355 3,204
Trade payables 1,900 1,400
Accruals & other payables 680 660
Tax liability 650 500
38,580 30,000 38,580 30,000

Other information:
(i) Shares issued during the year were as follows:
 10% bonus shares in March 2017.
 Right shares in July 2017.
(ii) During the year, a plant costing Rs. 9,500,000 and having a book value of
Rs. 5,200,000 was disposed of for Rs. 4,800,000 of which Rs. 1,800,000 are still
outstanding.
(iii) Depreciation for the year amounted to Rs. 7,350,000.
(iv) Financial charges for the year amounted to Rs. 1,100,000. Accrued financial charges
as on 31 December 2017 amounted to Rs. 112,000 (2016: Rs. 48,000).
(v) Provision for doubtful trade receivables is maintained at 5%.

Required:
Prepare statement of cash flows for the year ended 31 December 2017, in accordance with
IAS 7 ‘Statement of Cash Flows’ using indirect method. (15)

Q.4 Following information pertains to Dhaka Enterprises (DE).

Jul-2017 Aug-2017 Sep-2017 Oct-2017 Nov-2017 Dec-2017


Production costs
------------------------------------ Rs. in '000 -----------------------------------
Direct material 1,375 1,500 1,750 1,250 1,125 1,000
Direct labour 990 1,080 1,260 900 810 720
Overheads 3,240 3,400 3,800 3,200 2,700 2,600
5,605 5,980 6,810 5,350 4,635 4,320

----------------------------------- No. of units -----------------------------------


Production 550 600 700 500 450 400

DE is preparing its budget for the next year, therefore, it would like to determine the
relationship between production units and cost.

Required:
(a) Using regression analysis, determine the line of best fit for production units and
overheads. (Show all necessary workings) (06)
(b) Compute total prime cost and overheads for production of 650 units. (02)
Financial Accounting and Reporting-I Page 3 of 5

Q.5 A and B were partners sharing profits and losses in the ratio of 3:2. The balance sheet as on
31 December 2017 is given below:

Equity and liabilities Rupees Assets Rupees


Capital – A 35,810 Goodwill 2,000
Capital – B 26,540 Fixed assets – net 42,000
Profit and loss account 10,000 Investments 8,000
Trade creditors 32,650 Trade debtors 27,000
Provision for doubtful debts (2,000)
Stock-in-trade 20,000
Cash and bank balances 8,000
105,000 105,000

On 1 January 2018, they agreed to admit C for 1/4th share in the partnership. On admission
of C, it has been agreed that:
 value of goodwill of the firm is Rs. 32,000. Goodwill is to be written-off from the books.
 assets would be revalued as follows:
Revalued amount
Assets
(Rs.)
Fixed assets 60,000
Investments 9,000
Stock-in-trade 18,000

 provision for doubtful debts is to be made equal to 5% of the debtors.

C has contributed Rs. 38,000 in cash. Capital accounts of the old partners in the new
partnership would be adjusted in their new profit sharing ratio on the basis of C’s capital.
Any excess or deficiency would be adjusted through cash.

Required:
Prepare partners’ capital accounts on admission of C. (12)

Q.6 (a) Following information pertains to a building acquired by SK Limited (SKL) on


1 July 2012 for Rs. 360 million:

(i) The building is being depreciated on straight-line basis over 10 years.


(ii) SKL uses revaluation model for subsequent measurement of buildings. It
accounts for revaluation on net replacement value method. The details of
revaluations as carried out by independent valuer are as follows:

Fair value
Revaluation date
(Rs. in million)
31 December 2013 323
31 December 2015 208
31 December 2017 167

(iii) There is no change in useful life of the building.


(iv) SKL transfers the maximum possible amount from the revaluation surplus to
retained earnings on an annual basis.
(v) SKL’s financial year ends on 31 December.

Required:
Prepare entries to record revaluation surplus/loss on each of the above revaluation
date. (Entries to record depreciation expense, incremental depreciation and elimination of
accumulated depreciation are not required) (11)
Financial Accounting and Reporting-I Page 4 of 5

(b) Following information pertains to three exchange transactions relating to fixed assets:
(i) (ii) (iii)
--------- Rs. in million ---------
Cash received/(paid) 1.1 (2.1) -
Assets given-up:
Original cost 10.3 12.4 14.5
Book value 6.4 7.3 3.4
Estimated fair value 8.5 6.6 4.6
Assets received:
Estimated fair value 7.1 9.0 4.1

Additional information:
 In case of transaction (i), fair values of both assets are reliably measurable.
 In case of transaction (ii), fair value of the asset received is clearly more evident.
 In case of transaction (iii), fair value of neither asset is reliably measurable.

Required:
Compute gain or loss on disposal of fixed assets in each of the above transactions. (06)

Q.7 Boom Limited (BL) is a manufacturer of sports goods. Following financial statements for
the year ended 31 December 2017 have been submitted to the Chief Executive Officer
(CEO).
Statement of profit or loss
Rs. in ‘000
Revenues 21,000
Cost of sales (17,500)
Gross profit 3,500
Operating expenses (1,900)
Finance cost (450)
Profit before tax 1,150
Taxation (345)
Profit after tax 805

Statement of financial position


Rs. in ‘000
Property, plant and equipment 7,500
Current assets 1,500
9,000

Share capital 4,000


Reserves 1,000
Non-current liabilities 3,000
Current liabilities 1,000
9,000

Although performance of BL has improved from the last year, CEO wants to compare the
results with other companies operating in sports manufacturing industry. In this respect,
following industry data has been gathered:
Gross profit margin 23.5%
Net profit margin 7.7%
Current ratio 2.75
Gearing ratio 50:50
Return on non-current asset 32.9%
Return on capital employed 27.4%
Return on equity 31.3%

Required:
(a) Compute BL’s ratios for comparison with the industry. (04)
(b) For each ratio, give one possible reason for variation from the industry. (07)
Financial Accounting and Reporting-I Page 5 of 5

Q.8 Following information pertains to Alpha Traders (AT) for the year ended
31 December 2017:
(i) 60% goods are sold for cash to walk-in customers at list price. Remaining goods are
sold to corporate customers on credit at a trade discount of 2% on list price. They only
pay through cheques.
(ii) Balances extracted from AT’s records:
31-Dec-2017 31-Dec-2016
--------- Rs. in ‘000 ---------
Furniture and fittings – net ? 10,175
Stock-in-trade 14,500 12,300
Trade debtors – gross 5,900 4,400
Prepaid rent 180 145
Cash in hand 430 750
Trade creditors 9,700 8,500
Accrued salaries 310 460
(iii) All furniture and fittings were purchased on 1 July 2015 and are depreciated using
straight-line method at 5% per annum.
(iv) Provision for doubtful debts is maintained at 4%. During the year, balances totalling
Rs. 260,000 were written-off.
(v) Summarised bank statement:
Deposits Rs. in ‘000 Withdrawals Rs. in ‘000
Opening balance 9,800 Utilities 1,400
Corporate customers 34,240 Rent, rates and taxes 2,100
Cash 56,380 Repairs & maintenance 2,800
Insurance claim 5,500 Cash 6,320
Return outward 2,170 Creditors 87,200
Delivery charges recovered 330 Delivery truck (second hand) 2,300
Miscellaneous expenses 1,300
Closing balance 5,000
108,420 108,420

(vi) Cash payments for the year:


Rs. in ‘000
Salaries 6,500
Repairs & maintenance 500
Drawings ?

(vii) Insurance claim represents cost of goods lost in transit during the year.
(viii) A cheque of Rs. 300,000 issued on 15 December 2017 against rent, has not yet been
presented whereas cheque from a debtor, deposited on 31 December 2017 amounting
to Rs. 3,200,000 is not appearing in the bank statement.
(ix) Creditors are paid through cheques only. Payments made to creditors include:
 Rs. 48,000,000 after availing discount of 4%.
 A cheque of Rs. 1,900,000 issued to a supplier in December 2016. No discount
was allowed by the supplier on this payment.
(x) The delivery truck was purchased on 1 March 2017. Prior to use, the truck was
repaired at a cost of Rs. 260,000. The repair work was completed on 31 March 2017.
The amount is included in payment for repairs and maintenance above. Depreciation
on delivery truck is charged on a straight-line basis at 12.5% per annum.
Required:
Prepare the following:
(a) Statement of profit or loss for the year ended 31 December 2017. (12)
(b) Statement of financial position as on 31 December 2017. (08)
(THE END)
Financial Accounting and Reporting-I
Suggested Answers
Certificate in Accounting and Finance – Spring 2018

Ans.1 Kidz Party & Co.


Inventory valuation as on 31 December 2017
Normal Cost to Inventory
Cost NRV
Units selling price sell valuation at lower
per unit per unit
per unit per unit of cost and NRV
1 2 3 4 5=(3–4) 6
----------------------------------------- Rupees -----------------------------------------
Toy cars 7,000 1,250 1,200 150 1,050 NRV 7,350,000
3,000 1,250 1,100 - 1,100 NRV 3,300,000
10,000 10,650,000

Doll houses 1,000 1,800 - - NRV -


800 1,800 900 - 900 NRV 720,000
(5,000-1,000)×20% (1,800×50%)
3,200 1,800 2,700 - 2,700 Cost 5,760,000
5,000 6,480,000

Stuffed toys 350 1,200 500 18 482 NRV 168,700


(420,000÷1,200) (175,000÷350) (6,300÷350)
1,500 1,200 1,900 - 1,900 Cost 1,800,000
1,850 1,968,700

Minion costumes 522 1,500 1,600 200 1,400 NRV 730,800


(870×60%)
348 1,500 1,350 - 1,350 NRV 469,800
870 1,200,600
20,299,300

Ans.2 SK Limited
(a) Performance obligation:
A performance obligation is a promise in a contract with a customer to transfer to the
customer either:
 a good or service (or a bundle of goods or services) that is distinct; or
 a series of distinct goods or services that are substantially the same and that have the
same pattern of transfer to the customer.

Examples of promised goods and services


(i) Goods produced by an entity for sale
(ii) Resale of goods purchased by an entity
(iii) Resale of rights to goods or services purchased by an entity
(iv) Performing a contractually agreed-upon task for a customer
(v) Standing ready to provide goods or services
(vi) Providing a service of arranging for another party to transfer goods or services to
the customer
(vii) Granting rights to goods or services to be provided in the future that a customer
can resell
(viii) Constructing, manufacturing or developing an asset on behalf of a customer
(ix) Granting licences
(x) Granting options to purchase additional goods/services

(b) Adjusting entry Debit Credit


------ Rupees ------
Advance from customers 378,000
Revenue (Smart phones) (18,000÷*39,600×34,650) ×20 315,000
(Network-usage) *
(21,600÷ 39,600×34,650)×{20×(2÷12)} 63,000

Full price 18,000+1,800×12=39,600


Page 1 of 7
Financial Accounting and Reporting-I
Suggested Answers
Certificate in Accounting and Finance – Spring 2018

Ans.3 Nadir Limited


Statement of cash flows for the year ended 31 December 2017
Rs. in '000
Cash flows from operating activities:
Profit before tax 8,955
Adjustment for:
Depreciation 7,350
Loss on disposal 5,200–4,800 400
Financial charges 1,100
Increase in provision for doubtful receivables (6,840×5÷95) – (4,446×5÷95) 126
17,931
Working capital changes:
31-12-2017 31-12-2016
Stock-in-trade 5,600 5,750 150
Other receivables 2,385–1,800 585 800 215
Trade receivables-gross (6,840÷0.95);(4,446÷0.95) 7,200 4,680 (2,520)
Trade payables (1,900) (1,400) 500
Accrued exp./Other payables (680–112);(660–48) (568) (612) (44)
Increase in working capital (1,699)
Cash generated from operations 16,232

Payment of interest 1,100 –112+48 (1,036)


Payment of taxes 2,945–650+500 (2,795)
12,401
Cash flows from investing activities
Additions to PP&E 15,800–7,350–5,200+4,000–21,400 (14,150)
Disposals of PP&E 4,800–1,800 3,000
(11,150)
Cash flows from financing activities
Issue of right shares (12,400–10,000)–(10,000×10%)+1,400 2,800
Loan repaid 5,000–4,100 (900)
Cash dividend paid [(12,440+6,010)–(10,000×10%)]–13,450 (4,000)
(2,100)
Net cash outflows (849)
Cash and cash equivalent at beginning of the year 3,204
Cash and cash equivalent at year-end 2,355

Ans.4 Dhaka Enterprises


(a) Computation of variable and fixed cost using linear regression analysis
Factory
Units
Overheads x2 xy
Month
x y
---------------------------------Rs. in '000 ---------------------------------
Jul-17 550 3,240 302,500 1,782,000
Aug-17 600 3,400 360,000 2,040,000
Sep-17 700 3,800 490,000 2,660,000
Oct-17 500 3,200 250,000 1,600,000
Nov-17 450 2,700 202,500 1,215,000
Dec-17 400 2,600 160,000 1,040,000
n=6 3,200 18,940 1,765,000 10,337,000

Page 2 of 7
Financial Accounting and Reporting-I
Suggested Answers
Certificate in Accounting and Finance – Spring 2018

(∑ ) (∑ )(∑ )
(∑ ) (∑ )(∑ )

( ) ( )
4.04
( ) ( )

(∑ )(∑ ) (∑ )(∑ )
(∑ ) (∑ )(∑ )

( ) ( )
1,002
( ) ( )

Regression line:

(b) Total prime cost and overhead costs


Rs. in ‘000
Prime cost:
Direct material 650×(1,375÷550)* 1,625
Direct labour 650×(990÷550)* 1,170
2,795
Overheads:
Variable 650×4.04 (a) 2,626
Fixed (a) 1,002
3,628
6,423
*These can be calculated using any production level

Ans.5 Partners' capital accounts


A B C A B C
------------ Rs. ------------ ------------ Rs. ------------
Goodwill (written-off in the new ratio) 14,400 9,600 8,000 Opening balance 35,810 26,540 -
Bank (Balancing) 2,000 4,000 - Profit and loss account 6,000 4,000 -
Revaluation gain (W-2) 10,590 7,060 -
Closing balance Goodwill (in the old ratio) 18,000 12,000 -
(30,000÷0.25)×45%; 30% 54,000 36,000 30,000 Cash and bank - - 38,000
70,400 49,600 38,000 70,400 49,600 38,000

W-1: Profits and losses sharing ratio on admission of C A B C


Profit sharing ratio before admission of C 60% 40% -
C admitted to 1/4th share in the firm (0.6×0.25); (0.4×0.25) -15% -10% 25%
45% 30% 25%
OR
9 6 5

W-2: Revaluation gain/loss on admission of C Gain/(loss)


(Rs.)
Investments 9,000–8,000 1,000
Fixed assets 60,000–42,000 18,000
Provision for doubtful debts: 2,000–(27,000×5%) 650
Stock in trade 18,000–20,000 (2,000)
Net revaluation gain 17,650

Page 3 of 7
Financial Accounting and Reporting-I
Suggested Answers
Certificate in Accounting and Finance – Spring 2018

Ans.6 SK Limited
(a) Accounting entries for revaluation of building

Debit Credit
Date Description
Rs. in million
31-Dec-2013 Building 17.00
Revaluation surplus 17.00

31-Dec-2015 Revaluation surplus 13.00


Revaluation loss (P&L account) 26.00
Building 39.00

31-Dec-2017 Building 23.00


Revaluation gain (P&L account) 18.00
Revaluation surplus 5.00

W-1: Revaluation surplus/(loss) of building: 31-Dec-2013 31-Dec-2015 31-Dec-2017


--------------- Rs. in million ---------------
Fair value 323 208 167
Book value (306) (247) (144)
360-(360÷10×1.5) 323-(323÷8.5×2) 208-(208÷6.5×2)
Revaluation surplus/(loss) 17 (39) 23
Adjustment:
Revaluation surplus 17–(17÷8.5 years×2years) - 13 -
Revaluation loss 26–(26÷6.5years×2years) - - (18)
Revaluation surplus/(loss) 17 (26) 5

(b) Gain/(loss) on exchange of fixed assets:


Trade-in
Book value Gain/(Loss)
value
1 2 3=(1-2)
---------------- Rs. in million ----------------
(I) Where fair value of both the assets is given:
FV of asset given-up is used as trade in value. 8.5 6.4 2.1

(II) Where fair value of receiving asset is clearly more


evident:
FV of asset received is more evident so 'FV of asset
received minus cash paid' is used as trade in value. (9.0-2.1) 6.9 7.3 (0.4)

(III) Where fair value of neither asset is reliably


measurable:
Asset received would be recorded at book value of
asset given-up. 3.4 3.4 -

Page 4 of 7
Financial Accounting and Reporting-I
Suggested Answers
Certificate in Accounting and Finance – Spring 2018

Ans.7 Boom Limited


Comparison of BL's ratios with industry average and possible reasons for variation

Ratios Industry's
(a) BL's ratios (b) Reasons for variation from industry
ratios
Gross profit 16.67% 23.50% Lower than industry
margin  Purchase of raw material at higher prices
as compared to its competitors
 Inability to obtain economies of scale in
production as compared to its
competitors
 Higher production costs due to
inefficiencies
 Deliberately keeping selling prices lower
to gain the market share
Net profit 3.83% 7.70% Lower than industry
margin  BL’s gross profit margin is 6.8% lower
than industry (16.6% Vs 23.5%) whereas
net profit margin is only 3.9% lower
which indicates that BL’s operating
expenses as a percentage of sales are
approximately 2.9% lower than the
industry
Current ratio 1.50 2.75 Lower than industry
 Since gearing ratio is lower than the
industry so BL might have:
 obtained running finances as
compared to long-term financing
by the industry
 availed extended credit terms from
suppliers
 Low inventory levels are maintained by
BL
 Shorter credit terms are given to debtors
Gearing ratio 37.5: 62.5 50 : 50 Lower than industry
 Difficulty in raising long-term finance
from banks due to low profits
 Running finance or extended credit terms
from suppliers are available for BL
Return on non- 21.33% 32.90% Lower than industry
current assets  Lower profit margins
 Relatively newer non-current assets have
higher carrying value
Return on 20.00% 27.40% Lower than industry
capital employed  Lower profit margins
 High shareholder’s equity

Return on 16.10% 31.30% Lower than industry


equity  Lower profit margins
 Higher shareholder’s equity/low gearing
ratio

Page 5 of 7
Financial Accounting and Reporting-I
Suggested Answers
Certificate in Accounting and Finance – Spring 2018

Ans.8 Alpha Traders


(a) Statement of profit or loss for the year ended 31 December 2017
Rs. in '000
Sales
Credit sales (W-1) 39,200
Cash sales 39,200÷(100÷98)×(60÷40) 60,000
99,200
Cost of goods sold:
Opening stock (12,300)
Purchases (W-2) (88,500)
Return outward 2,170
Goods lost 5,500
Closing stock 14,500
(78,630)
Gross profit 20,570
Operating expenses:
Utilities (1,400)
Rent, rates and taxes 2,100+(145–180)+300 (2,365)
Repair and maintenance 2,800+(500-260) (3,040)
Salaries 6,500+(310–460) (6,350)
Depreciation - Furniture and fittings 10,175×(5÷92.5) (550)
Depreciation - Delivery truck (2,300+260)×(1÷8)×(9÷12) (240)
Miscellaneous expenses (1,300)
Bad debts 260+(5,900×4%)–(4,400×4%) (320)
Stock lost (5,500-5,500) -
(15,565)
5,005
Miscellaneous income:
Delivery charges 330
Discount income (W-2) 2,000
2,330
Net profit 7,335

W-1: Trade receivables - gross Rs. in '000


Opening balance 4,400 Bank 34,240+3,200 37,440
Credit sales (balancing) 39,200 Bad debts 260
Closing balance 5,900
43,600 43,600

W-2: Trade payables Rs. in '000


Bank 87,200–1,900 85,300 Opening balance 8,500
Discount rec. 48,000×4÷96 2,000 Credit purchases (balancing) 88,500
Closing balance 9,700
97,000 97,000

Page 6 of 7
Financial Accounting and Reporting-I
Suggested Answers
Certificate in Accounting and Finance – Spring 2018

(b) Alpha Traders


Statement of financial position as on 31 December 2017 Rs. in '000
Assets
Non-current assets
Furniture and fittings - net (10,175–550) (PL) 9,625
Delivery truck - net (2,300+260)–PL 240 2,320
11,945
Current assets:
Trade debtors - gross 5,900
Provision for bad and doubtful debts 5,900×4% (236)
5,664
Stock-in-trade 14,500
Prepaid rent 180
Cash 430
Bank (5,000-300+3,200) 7,900
28,674
40,619
Equity and liabilities
Equity
Opening capital
(9,800-1,900) +(4,400×0.96)+(10,175+12,300+145+750)–(8,500+460) 26,534
Net profit 7,335
Drawings (W-3) (3,260)
30,609
Current liabilities
Trade creditors 9,700
Accrued salaries 310
10,010
40,619

W-3: Cash account Rs. in '000


Opening balance 750 Bank 56,380
Withdrawals from the bank 6,320 Salaries 6,500
Cash sales PL 60,000 Repair and maintenance 500
Closing balance 430
Drawings (balancing) 3,260
67,070 67,070

(The End)

Page 7 of 7
THE INSTITUTE OF CHARTERED ACCOUNTANTS OF PAKISTAN

EXAMINERS’ COMMENTS

SUBJECT SESSION
Financial Accounting and Reporting-I Certificate in Accounting and Finance
– Spring 2018

General:

The overall performance was significantly better this time. The passing ratio was 43.2%
which was far above the last two results of 13.4% and 22%. Moreover 19% students were
just short of 9 or fewer marks and could have easily obtained them had they covered all
areas of the syllabus. The highest score in the paper was 96 marks which indicates that
the paper was very doable.

Performance in four questions (Q.1: Inventory, Q.3: Cash Flow, Q.4: Cost of production
and Q.5: Partnership) comprising of 43 marks was well above average.

Performance in Q.2: Revenues was below average. Students were found struggling to
even answer the theory part (5 marks) which was straight from the study text.

Some examination technique issues also need to be improved which would have lifted
many marginal fails into the pass category. Many students are failing because of
technique rather than knowledge or ability.

Although many students performed well but some common and illogical mistakes were
noted. The persisting issue appears to be lack of practice and poor presentation in many
cases.

Question-wise comments:

Question 1

The question required calculation of the amount of inventory in accordance with IAS 2. It
was an easy question with high performance i.e. 60.1% of the students secured passing
marks.

However, the following errors were observed:

 The correct approach was to calculate cost and NRV of each portion of individual
product and apply the lesser of cost or NRV rule and then calculate the total inventory
value for reporting purposes. However, students often aggregated the cost and NRV
of all items and then picked the lower, which was incorrect.

Page 1 of 4
Examiners’ Comments on Financial Accounting and Reporting-I - Spring 2018

 In the case of those damaged goods (minion costumes) which were repairable by
incurring cost of Rs. 200 per unit, the students failed to compute the correct NRV by
picking the higher NRV of those items from the available options.

Question 2

The question was based on IFRS 15 which is a new area of the syllabus. Only 16.2% of
the students secured passing marks. 20.9% of the students could not score any mark
which shows that they had not studied this area of the syllabus. The common errors were
as follows:

 In part (a), majority of the students were successful in reproducing the definition of
‘performance obligation’ but failed to list down examples of promised goods and
services as per IFRS 15.

 In part (b), only a handful of students were able to grab full or nearly full marks. The
entry was correct in many cases but various types of mistakes were made in
computing the revenues to be recognized.

Question 3

It was a traditional question requiring preparation of statement of cash flows. 65.3%


students secured passing marks. Some of the errors noted were as follows:

 Change in net trade receivables was shown in working capital changes but increase in
provision for doubtful debts was also shown which was incorrect.

 Although adjustment for interest expense and actual interest paid were reported
correctly, the effect of change in accrued interest was not excluded from changes in
accruals and other payables.

 While reporting sale proceeds of plant under investing activity, the amount which
remained outstanding at the year-end was not excluded.

 Impact of revaluation of fixed assets was not considered in calculating additions to


property, plant and equipment.

 In reporting proceeds from issue of share capital, premium on right shares was shown
separately. Further, many students ignored bonus shares in the calculation of net
proceeds.

Question 4

The question required determination of line of best fit for production units and overheads
and computation of prime cost and overheads at a given production level. 71.3% of the
students secured passing marks in this question. Some common errors were as follows:

 In part (a) students used total production costs in place of overheads.

Page 2 of 4
Examiners’ Comments on Financial Accounting and Reporting-I - Spring 2018

 In part (b) students only calculated overhead cost, though the prime cost was also
required to be calculated.

Question 5

The question required preparation of partners’ capital accounts on admission of a new


partner. 70.8% of the students could secure passing marks. Some common errors were as
follows:

 Full amount of goodwill was credited in old partner’s capital accounts instead of the
balance amount only, as goodwill of Rs. 2,000 was already appearing in the books.

 Allocation of balance of profit and loss account to the old partners was omitted.

 Full 5% provision for doubtful debts was adjusted in revaluation account though only
the difference should have been adjusted.

Question 6

The question was based on IAS 16. Part (a) required entries to record revaluation of a
building while part (b) required determination of gain or loss on the given exchange
transactions. Only few students were cognizant with the concept examined in part (b).
Only 34.5% students could secure passing marks in this question while 10.6% students
could not obtain any mark. Some common errors were as follows:

Question 6(a)

 Students did not read the requirement carefully and also prepared entries for
depreciation expense, incremental depreciation, etc. This resulted in loss of valuable
time and affected the performance in other questions.

 First year’s depreciation was charged for the full year instead of six months

 Revaluation adjustment was not bifurcated between P&L and revaluation surplus, on
2nd and 3rd revaluation.

Question 6(b)

 in transaction (i) since fair values of both assets were reliably measurable, the fair
value of asset given up should have been used for calculating gain or loss on disposal
but fair value of asset received was used.

 in transaction (iii) since fair values of both assets were not reliably measurable, no
gain or loss should have been recognized. However this was ignored.

Page 3 of 4
Examiners’ Comments on Financial Accounting and Reporting-I - Spring 2018

Question 7

The question required calculation of ratios for comparison with the industry data and
explanation of one possible reason for variation with the industry. 23.3% students
obtained passing marks. Some of the common errors were as under:

 While computing return on capital employed, non-current liabilities were not


included in the denominator.

 An in-depth analysis was missing as many students seemed to think that just stating
that a ratio is higher or lower amounts to interpretation. Moreover, instead of
explaining the possible reasons for variation from the industry, students tried to
explain the objectives and purposes of such ratios and whether a particular ratio is
better or worse as compared to the industry. They are advised to seek guidance from
ICAP’s suggested answer.

Question 8

The question required preparation of statement of profit or loss and statement of financial
position from the given data. 27% students obtained passing marks. Some of the common
errors were as under:

 Balance as per bank statement was not adjusted for the unpresented cheque and
deposit in transit.

 Return outward and goods lost were mostly ignored while computing cost of goods
sold.

 2% trade discount was ignored while computing cash sales.

 Many students prepared the cash account but were unaware that its balance
represented the drawings.

 Depreciation rate on furniture and fixtures was applied on the WDV rather than the
original cost which was required to be worked back.

THE END

Page 4 of 4
Financial Accounting and Reporting - I
Summary of Marking Key
Certificate in Accounting and Finance – Spring 2018

Note regarding marking scheme:


The marking scheme is given as a guide. Markers also award marks for alternative approaches to a
question and relevant/well-reasoned comments/explanations. Moreover, the available marks in
answer may exceed the total marks of a question.

Mark(s)
A.1 02 marks for valuation of each of the given inventory items 8.0

A.2 (a)  Definition of ‘performance obligation’ 2.0


 0.5 mark for listing each example of promised goods and services 3.0

(b)  1.5 marks each for computation of revenue to be recorded for smart phones and
network usage 3.0
 Preparation of adjusting entry at the year-end 1.0

A.3  Cash flows from operating activities:


− adjustments of non-cash items 2.0
− computation of working capital changes 4.0
− other adjustments 3.0
 Cash flows from investing activities 2.0
 Cash flows from financing activities 4.0

A.4 (a)  Determination of Σx, Σy, Σx2 and Σxy 3.0


 01 mark each for calculating variable overheads per unit, fixed overheads and
determining the line of best fit for production units and overheads 3.0

(b) 01 mark each for computing prime cost and overheads for production of 650 units 2.0

A.5  Distribution of:


− profit and loss 1.0
− revaluation gain 3.5
 Determination of new profit sharing ratio and goodwill adjustments 4.0
 Determination of investment by C & capital adjustments in A & B 3.5

A.6 (a)  Computation of revaluation surplus/loss on each revaluation date 4.0


 Bifurcation of revaluation increase/decrease in P&L and revaluation surplus 3.0
 Preparation of entries to record revaluation surplus/loss on each revaluation date 4.0

(b) 02 marks for computing gain/loss on each fixed asset exchange transaction 6.0

A.7 (a) Computation of BL’s ratios for comparison with the industry 4.0

(b) 01 mark for giving one possible reason against each ratio computed in (a) above 7.0

Page 1 of 2
Financial Accounting and Reporting - I
Summary of Marking Key
Certificate in Accounting and Finance – Spring 2018

Mark(s)
A.8 (a) Statement of profit or loss
 Cash and credit sales 3.0
 Cost of sales 3.0
 Operating expenses 5.0
 Miscellaneous income 1.0

(b) Statement of financial position


 Non-current assets 1.5
 Current assets 2.5
 Equity and liabilities 4.0

(THE END)

Page 2 of 2
Certificate in Accounting and Finance Stage Examination
The Institute of 5 September 2018
Chartered Accountants 3 hours – 100 marks
of Pakistan Additional reading time – 15 minutes

Financial Accounting and Reporting-I


Q.1 On 1 July 2017, Nezam took over a running business namely FC Traders (FCT). Proper
books of account are not maintained for FCT. Following information has been gathered for
preparation of statement of profit or loss for the year ended 30 June 2018:
(i) Balances of certain assets and liabilities:
30-Jun-2018 1-Jul-2017
Assets and liabilities
------ Rs. in '000 ------
Equipment 4,000 4,000
Furniture and fixtures 2,500 2,500
Trade debtors 1,600 -
Inventory 2,400 2,800
Unused miscellaneous supplies 400 300
Unpaid suppliers’ bills 2,800 1,850
Shop rent payable 400 200

(ii) Summary of bank payments for the year ended 30 June 2018:
Rs. in '000
Suppliers 13,600
Repair and maintenance 950
Shop rent 2,000
Miscellaneous supplies 800
Utilities 1,200
(iii) Payments made out of cash sales before being deposited into the bank:
Rs. in '000
Salaries and wages 1,800
Purchase of inventory 3,000
Part payment of sales commission to riders 90

(iv) Unpaid suppliers’ bills as at 30 June 2018 include a bill of Rs. 320,000 which was
mistakenly taken at Rs. 230,000.
(v) During the year, goods costing Rs. 540,000 were withdrawn by Nezam for personal
use.
(vi) Inventory as at 30 June 2018 includes goods costing Rs. 250,000 which were badly
damaged in an accident and have no sales value.
(vii) Mark-up on goods sold are as follows:
Mark-up on cost
50% of goods – sold on cash counter 35%
20% of goods – sold for cash through riders 40%
30% of goods – sold for credit 45%

(viii) The riders are entitled to 3% commission.


(ix) Fixed asset at 30 June 2018 are to be depreciated at 10% per annum.
(x) Salaries and wages for June 2018 amounting to Rs. 165,000 were paid on 5 July 2018.

Required:
Prepare a statement of profit or loss for the year ended 30 June 2018. (13)
Financial Accounting and Reporting-I Page 2 of 5

Q.2 Digital World (DW) closes its accounts on 30 June each year. This year physical stock
taking was delayed and carried out on 10 July 2018. The cost of physical stock on that date
was determined at Rs. 1,126,000. Following further information is available:

(i) Purchase invoices received from suppliers during 1 July to 10 July 2018 amounted to
Rs. 366,000. These include invoices amounting to:
 Rs. 28,000 for goods dispatched by a supplier but not received by DW till
10 July 2018.
 Rs. 20,000 for goods received on 28 June 2018.

(ii) Goods costing Rs. 44,000 were received on 8 July 2018 but the corresponding invoice
was not received till 10 July 2018.
(iii) Details of credit notes from suppliers are as follows:

Credit notes date Goods returned date Rupees


4 July 2018 27 June 2018 23,000
9 July 2018 7 July 2018 9,000
13 July 2018 9 July 2018 14,000

(iv) Selling price of goods dispatched to customers from 1 July 2018 to 10 July 2018
amounted to Rs. 375,000. This included:
 Rs. 62,500 relating to goods invoiced but not received by customers till
10 July 2018.
 Rs. 34,000 relating to goods not invoiced till 10 July 2018.

(v) DW’s stocks-in-transit from suppliers as on 30 June 2018 were amounted to


Rs. 36,000. Of these, goods costing Rs. 13,100 were received on 9 July 2018 and
remaining goods have not yet been received.
(vi) Goods costing Rs. 150,000 were found to be damaged and are expected to realize
Rs. 110,000 after repairing at a cost of Rs. 26,000. It was ascertained that 40% of the
goods were damaged in July 2018.
(vii) It was discovered that goods included in the stock valuation at Rs. 16,600 were
mistakenly valued at their selling price.
(viii) DW sells goods at a mark-up of 25% on cost.

Required:
Compute the value of stock as at 30 June 2018. (10)

Q.3 (a) List the five steps involved in recognizing revenue under IFRS 15 ‘Revenue from
Contracts with Customers’. (03)

(b) On 1 June 2018 Ravi Limited (RL) delivered 500 units of one of its products to Bravo
Limited (BL) at Rs. 200 per unit. BL immediately paid the amount and obtained
control upon delivery. BL is allowed to return unused units within 30 days and receive
a full refund. RL’s cost of the product is Rs. 150 per unit and it uses perpetual system
for recording inventory transactions.

On 30 June 2018, BL returned 20 units.

Required:
Prepare necessary journal entries in the books of RL on 1 June 2018 and 30 June 2018
under each of the following independent situations:
(i) Based upon historical data, RL estimates that 5% units will be returned on
expiry of 30 days. (05)
(ii) The product is new and RL has no relevant historical evidence of product
returns or other available market evidence. (04)
Financial Accounting and Reporting-I Page 3 of 5

Q.4 Royal Fashions (RF) and Imperial Garments (IG) are two partnership businesses. Partners
of both firms were sharing profits in their capital ratios. It has now been decided to merge
the two businesses with effect from 1 July 2018 under the name of Quality Apparels (QA).
The respective statements of financial position as at 30 June 2018 were as under:

RF IG RF IG
Capital and liabilities Assets
Rs. in million Rs. in million
Capital accounts: Land 14 18
A 24 - Buildings 8 13
Z 16 - Machines 17 20
B - 27 Inventory 9 12
G - 33 Trade debtors - net of 2%
Trade payables 18 14 provision 12 15
Accruals and other payables 5 6 Cash 3 2
63 80 63 80

Following terms and conditions have been agreed for the merger:
(i) Total capital of QA will be Rs. 100 million. Capital as well as profits will be shared
equally by all the partners in the new firm. Any excess or deficiency in partners’
capital accounts will be adjusted through cash.

(ii) Agreed values of some of the assets will be as under:

RF IG
--- Rs. in million ---
Land 16.0 17.0
Machines 15.0 15.0
Inventory 10.0 12.6

(iii) Provision for doubtful debts will be increased from 2% to 5%.


(iv) Accruals and other payables of IG will be increased by Rs. 2 million.
(v) Remaining assets (including cash) and liabilities will be taken over by QA at their
book values.
(vi) Goodwill will be determined at 20% and 25% of the agreed values of the net assets
(excluding cash) of RF and IG respectively. No goodwill will be maintained in the
books of QA.

Required:
Prepare capital accounts in the old and new firms. (12)

Q.5 The following information is available in respect of machines of Akmal Brothers:

(i) The balances of cost and accumulated depreciation of machines as on 1 January 2017
were Rs. 800,000 and Rs. 333,000 respectively.
(ii) A machine acquired on 1 January 2014 having net book value of Rs. 31,935 on
1 January 2017 was sold for Rs. 34,000 on 30 April 2017. Cost of disposal incurred
was Rs. 5,000.
(iii) On 1 July 2017, a machine having fair value of Rs. 40,000 on that date was exchanged
for a new machine. The balance of the purchase price was paid through a cheque of
Rs. 80,000. The list price of the new machine was Rs. 130,000. The old machine had
been acquired at a cost of Rs. 65,000 on 1 October 2015.
(iv) Machines are depreciated at 15% per annum using the reducing balance method.

Required:
Prepare the following ledger accounts pertaining to the machines for the year ended
31 December 2017:
(a) Cost (03)
(b) Accumulated depreciation (05)
(c) Gain/loss on disposal (04)
Financial Accounting and Reporting-I Page 4 of 5

Q.6 Following is a summarised trial balance of Omega Limited (OL) for the year ended
30 June 2018:
Rs. in million
Particulars Debit Particulars Credit
Land at revalued amount 30 Share capital 40
Buildings at revalued amount 60 Retained earnings 18
Equipment and other assets - at cost 47 Revaluation surplus 43
Trade receivables 21 Trade payables 29
Opening stock-in-trade 16 Accruals and other payables 9
Advances and other receivables 6 Accumulated depreciation:
Cash and bank balances 3 - Buildings 9
Purchases 180 - Equipment and other assets 18
Freight-in 4 Provision for doubtful receivables 4
Selling & administrative expenses Sales 235
(including depreciation expense) 39 Suspense account 1
406 406

Additional information:
(i) Cost of closing stock-in-trade was Rs. 19 million. However, following matters were
noted during physical inventory count:
 Stock held under ‘Bill-and-hold arrangement’ was accepted and paid by the
customer on 25 June 2018. Proceeds amounting to Rs. 2.7 million were credited to
other payables. These goods were included in OL’s stock. Such stock items are
sold at cost plus 35%.
 Stock items costing Rs. 3 million were damaged badly and could not be sold. A
claim was lodged with the insurance company which accepted the claim on
30 June 2018 to the extent of 80%.
 Stock items costing Rs. 4.5 million lying with a third party were not included in
stock-in-trade.
(ii) OL’s policy for provision for doubtful trade receivables is as under:
 Full provision is made for balances due for more than one year.
 Provision at 5% is made on all other balances.
As on 30 June 2018, balances due for more than one year aggregated Rs. 3.4 million.
This includes a balance of Rs. 2 million which is no more recoverable and is required
to be written-off.
Suspense account represents an amount recovered from a customer whose balance
was written-off in 2016.
(iii) On 30 June 2018, a portion of land and building was sold for Rs. 30 million which had
been revalued once only on 30 June 2015. Relevant details are as follows:
As on 30 June 2015
Written down
Fair value Remaining
value
useful life
------ Rs. in million ------
Land 3 10 Indefinite
Building 13 18 20 years
Proceeds from the disposal are due to be received in September 2018. The disposal has
not yet been accounted for.
OL transfers maximum possible amount from revaluation surplus to retained earnings
on an annual basis.
(iv) Unrecorded freight-in invoices amounting to Rs. 0.5 million are pending with cashier
for payment.
(v) A cheque for Rs. 1.8 million received as advance from a customer has not been
recorded.
(vi) Income tax liability is estimated at 30% of profit before tax.
Financial Accounting and Reporting-I Page 5 of 5

Required:
Prepare a statement of financial position as on 30 June 2018 and a statement of profit or loss
for the year ended 30 June 2018 in accordance with IFRSs. (21)

Q.7 SK Limited (SKL) deals in a single product. Following are the summarized financial
statements of SKL for the year ended 31 December 2017:

Statement of financial position Statement of profit or loss


2017 2016 2017 2016
Rs. in million Units sold in million 39 30
Fixed assets 410 240
Current assets 90 200 Rs. in million
500 440 Sales 371 300
Cost of goods sold (273) (210)
Capital 280 260 Gross profit 98 90
Long-term loan 170 100 Selling and administrative (55) (60)
Current liabilities 50 80 Finance cost (13) (8)
500 440 Net profit 30 22

Additional information:
(i) With effect from 1 January 2017, selling price was decreased by 5% to boost sales
volume.
(ii) During the year 2017, suppliers demanded price increase of 4%. SKL resisted the price
increase. However, both parties agreed to reduce the credit period.
(iii) SKL had been running its business in a rented building whose annual rent was
Rs. 15 million. During the year, SKL purchased this building for Rs. 200 million.
Funds were arranged partially through a long-term loan. Useful life of the building is
estimated at 40 years.
(iv) 75% of the selling and administration cost incurred in 2016 was fixed cost.

Required:
(a) Compute the following ratios for 2016 and 2017:

 Gross profit margin  Net profit margin


 Return on assets  Return on capital employed
 Debt equity ratio  Current ratio (08)
(b) Keeping in view the above information, comment on profitability and liquidity
position of SKL for 2017. (04)

Q.8 (a) Describe any four differences between financial accounting system and cost
accounting system. (04)

(b) Describe behaviour of each of the following costs graphically by denoting total cost on
vertical axis and level of activity on horizontal axis:

(i) Factory building rent - Fixed amount per month


(ii) Direct labour cost - Fixed per unit
(iii) Supervision cost - One supervisor is required for every 20 direct workers
(iv) Machine rental cost - Fixed monthly rent and an additional cost of
Rs. 100 per unit for the production exceeding certain limit (04)

(THE END)
Financial Accounting and Reporting-I
Suggested Answers
Certificate in Accounting and Finance – Autumn 2018

Note:
The suggested answers are provided for the guidance of the students. However, there are alternative
solution(s) to the questions which are also considered by the Examination Department while marking
the answer scripts.

Ans.1 FC Traders
Statement of profit or loss account for the year ended 30 June 2018
Rs. in '000
Sales:
50% cash sales at counter A×0.5×1.35 11,813
30% credit sales A×0.3×1.45 7,612
20% cash sales through riders A×0.2×1.40 4,900
24,325

Cost of sales:
Opening inventory 2,800
Purchases (14,640(W-1)+3,000) 17,640
Damaged stock (250)
Goods withdrawn (540)
Closing inventory 2,400–250 (2,150)
(A) (17,500)
Gross profit 6,825

Expenses:
Repair and maintenance 950
Shop rent 2,000+(400–200) 2,200
Misc. supplies used 800+(300–400) 700
Utilities 1,200
Staff salaries 1,800+165 1,965
Riders commission 4,900×3% 147
Depreciation – Equipment 4,000×10% 400
– Furniture & fixtures 2,500×10% 250
Damaged stock 250
(8,062)
Net loss (1,237)

W-1: Creditors Rs. in '000


Payment 13,600 Opening balance 1,850
Closing balance
[2,800+(320-230)] 2,890 Credit purchases (balancing) 14,640
16,490 16,490

Page 1 of 8
Financial Accounting and Reporting-I
Suggested Answers
Certificate in Accounting and Finance – Autumn 2018

Ans.2 Digital World


Stock as on 30 June 2018
Rs.
Physical inventory as on 10 July 2018 1,126,000

(i) Purchase invoices received during 1 July - 10 July 2018 (366,000)


Goods invoiced but not received 28,000
Goods received prior to year-end 20,000
(318,000)

(ii) Goods received on 8 July 2018 but invoice was not yet received (44,000)

(iii) Credit notes received after year-end for the goods returned on:
7-Jul-2018 9,000
9-Jul-2018 14,000
23,000

(iv) Cost of goods dispatched during 1 July - 10 July 2018 (375,000/1.25) 300,000

(v) Goods in transit as on 30-6-2018 received on 9 July 2018 (13,100)

(vi) NRV adjustment of goods damaged: Cost 150,000


NRV (110,000-26,000) (84,000)
Total loss 66,000
Loss pertains to 60% of the goods damaged prior to year-end (66,000×60%) (39,600)

(vii) Closing inventory items mistakenly valued at selling price (16,600×25/125) (3,320)

Physical inventory as on 30 June 2018 1,030,980


Stock-in-transit as on 30 June 2018 36,000
1,066,980

Page 2 of 8
Financial Accounting and Reporting-I
Suggested Answers
Certificate in Accounting and Finance – Autumn 2018

Ans. 3 (a) Five steps involved in recognising revenue under IFRS 15:
(i) Identify the contract(s) with a customer
(ii) Identify the separate performance obligations
(iii) Determine the transaction price
(iv) Allocate the transaction price
(v) Recognize revenue when or as an entity satisfies performance obligations

(b) Ravi Limited


(i) General Journal
Debit Credit
Date Description
---- Rs. in million ----
1 Jun 18 Cash/Bank (500×200) 100,000
Refund liability [25(500×5%)×200] 5,000
Sales (475×200) 95,000

Cost of goods sold (475×150) 71,250


Right to recover product (25×150) 3,750
Inventory (500×150) 75,000

30 Jun 18 Refund liability 5,000


Sales (5×200) 1,000
Cash/Bank (20×200) 4,000

Cost of goods sold (5×150) 750


Inventory (20×150) 3,000
Right to recover product (25×150) 3,750

(ii) General Journal


Debit Credit
Date Description
---- Rs. in million ----
1 Jun 18 Cash/Bank (500×200) 100,000
Contract liability (500×200) 100,000

Right to recover product (500×150) 75,000


Inventory (500×150) 75,000

30 Jun 18 Contract liability (500×200) 100,000


Sales (480×200) 96,000
Cash/Bank (20×200) 4,000

Cost of goods sold (480×150) 72,000


Inventory (20×150) 3,000
Right to recover product (500×150) 75,000

Page 3 of 8
Financial Accounting and Reporting-I
Suggested Answers
Certificate in Accounting and Finance – Autumn 2018

Ans.4 Capital accounts - Royal Fashions


A Z A Z
Rs. in million Rs. in million
Quality Apparels (balancing) 28.90 19.26 Balance b/f 24.00 16.00
Realisation in 60:40 W.1 0.38 0.25
Goodwill in 60:40 W.2 4.52 3.01
28.90 19.26 28.90 19.26

Capital accounts - Imperial Garments


B G B G
Realisation loss in 45:55 W.1 3.54 4.32 Balance b/f 27.00 33.00
Quality Apparels (balancing) 29.10 35.58 Goodwill in 45:55 W.2 5.64 6.90
32.64 39.90 32.64 39.90

Capital accounts - Quality Apparels


A Z B G A Z B G
Goodwill
(4.52+3.01+5.64+6.9)/4 5.02 5.02 5.02 5.02 Transferred 28.90 19.26 29.10 35.58
Cash and Bank - 5.56 Cash & Bank 1.12 10.76 0.92 -
Balance c/f 25.00 25.00 25.00 25.00
30.02 30.02 30.02 35.58 30.02 30.02 30.02 35.58

W-1: Gain/(loss) on valuation of assets and liabilities on merger of the firms


RF IG
--- Rs. in million ---
Land (16–14); (17–18) 2.00 (1.00)
Machines (15–17); (15–20) (2.00) (5.00)
Inventory (10–9)(12.6–12) 1.00 0.60
Provision for doubtful debts (12/0.98×0.03) (0.37) -
Provision for doubtful debts (15/0.98×0.03) - (0.46)
Accruals and other payables (6–8) - (2.00)
0.63 (7.86)

W-2: Goodwill of the old firms RF IG


--- Rs. in million ---
Net assets prior to merger (24+16); (27+33) 40.00 60.00
Net increase/(decrease) in assets & liabilities on revaluation (W-1) 0.63 (7.86)
40.63 52.14
Cash and bank balances (3.00) (2.00)
Net assets (excluding cash) 37.63 50.14
Goodwill at 20% and 25% C 7.53 12.54

Page 4 of 8
Financial Accounting and Reporting-I
Suggested Answers
Certificate in Accounting and Finance – Autumn 2018

Ans.5 Cost - Machines


Date Description Rs. Date Description Rs.
1 Jan 17 Opening balance 800,000 30 Apr 17 Gain/loss on disposal 52,000
[31,935÷(0.85)3]
1 Jul 17 Gain/loss on disposal 40,000 1 Jul 17 Gain/loss on disposal 65,000
1 Jul 17 Bank 80,000 31 Dec 17 Closing balance 803,000
920,000 920,000

Accumulated depreciation - Machines


Date Description Rs. Date Description Rs.
30 Apr 17 Gain/loss on disposal(W-1) 21,662 1 Jan 17 Opening balance 333,000
1 Jul 17 Gain/loss on disposal(W-2) 15,810 Depreciation exp. (W-3) 71,868
31 Dec 17 Closing balance 367,396
404,868 404,868

Gain/Loss on disposals
Date Description Rs. Date Description Rs.
30 Apr 17 Cost 52,000 30 Apr 17 Accumulated dep. (W-1) 21,662
30 Apr 17 Bank (Cost of disposal) 5,000 30 Apr 17 Bank (Sale proceeds) 34,000
1 Jul 17 Cost 65,000 1 Jul 17 Accumulated dep. (W-2) 15,810
1 Jul 17 Cost (Trade in at fair value) 40,000
31 Dec 17 Loss on disposal (P&L) 10,528
122,000 122,000

W-1: Accumulated depreciation - machine sold Rs.


Till 31-12-2016 (52,000–31,935) 20,065
For 1-1-2017 to 30-4-2017 (52,000–20,065)×15%×4÷12 1,597
21,662

W-2: Accumulated depreciation - machine exchanged Rs.


For 1-10-2015 to 31-12-2015 (65,000 – 0)×15%×3÷12 2,438
For the year ended 31-12-2016 (65,000 – 2,438)×15% 9,384
11,822
For 1-1-2017 to 30-6-2017 [65,000 – 11,822]×15%×6÷12 3,988
15,810

W-3: Depreciation for the year Rs.


On opening balance of:
- Machine sold (52,000 – (W-1)20,065) × 15%×4÷12 1,597
- Machine exchanged (65,000 – (W-2)11,822) × 15%×6÷12 3,988
- Other assets (Balancing) (683,000 – 301,113) × 15% 57,283
Balance 1-1-2017 800,000 333,000
On additions during the year (40,000+80,000)×15%×6÷12 9,000
71,868

Page 5 of 8
Financial Accounting and Reporting-I
Suggested Answers
Certificate in Accounting and Finance – Autumn 2018

Ans.6 Omega Limited


Statement of profit or loss for the year ended 30 June 2018
Rs. in million
Sales revenue 235+2.7 237.70
Cost of sales (W-1) (179.00)
Gross profit 58.70
Selling and administrative expenses (W-2) (38.88)
Operating profit 19.82
Gain on disposal of office block 30–10– 15.3(18×17÷20) 4.70
Profit before tax 24.52
Taxation 24.52×30% (7.36)
Profit after tax 17.16

W-1: Cost of sales Rs. in million


Opening stock 16.00
Purchases 180.00
Freight-in 4+0.5 4.50
Damaged inventory (3.00)
Closing stock 19–(2.7÷1.35) –3+4.5 (18.50)
179.00

W-2: Selling and administrative expenses Rs. in million


Selling and administrative expenses 39.00
Inventory claim short received 3–(3×80%) 0.60
Bad debt - Balance written-off during the year 2.00
- Excess provision written back (2.28(BS)-4) (1.72)
- Recovery of balance written-off in 2016 (1.00)
38.88

Omega Limited
Statement of financial position as on 30 June 2018
Rs. in million
Non-current assets:
Property, plant and equipment (W-3) 84.70

Current assets
Stock-in-trade PL 18.50
Trade receivables (W-4) 16.72
Advances and other receivables 6+30+(3×0.8) 38.40
Cash and bank balances 3+1.8 4.80
78.42
Total assets 163.12
Equity
Share capital 40.00
Retained earnings 18+7+4.25+17.16(PL) 46.41
Revaluation surplus 43– 7(10–3)–4.25[(18–13)×(17÷20)] 31.75
118.16
Current liabilities
Trade and other payables 29+9+0.5+1.8–2.7 37.60
Tax liability PL 7.36
44.96

Total equity and liabilities 163.12


W-3: Property, plant and equipment Rs. in million
Land 30–10 20.00
Buildings 60–9–15.30(PL)[18–2.7(18×3÷20)] 35.70
Equipment and other assets 47–18 29.00
84.70

Page 6 of 8
Financial Accounting and Reporting-I
Suggested Answers
Certificate in Accounting and Finance – Autumn 2018

W-4: Trade receivables Rs. in million


Trade receivables 21-2 19.00
Provision for doubtful receivables (3.4–2)+[(19–1.4)×0.05] (2.28)
16.72

Ans.7 (a) SK Traders


Computation of ratios
2017 2016
(i) Gross profit margin 26.42% 30.00%
(98÷371)×100 (90÷300)×100

(ii) Net profit margin 8.09% 7.33%


(30÷371)×100 (22÷300)×100

(iii) Return on assets 8.60% 6.82%


(30+13)÷500×100 (22+8)÷440×100

(iv) Return on capital employed 9.56% 8.33%


(30+13)/(280+170)×100 (22+8)÷(260+100)×100

(v) Debt equity ratio 0.38 0.28


170÷(280+170) 100÷(260+100)

(vi) Current ratio 1.80 2.50


90÷50 200÷80

(b) (i) Profitability:


In 2017, gross profit margin of SKL has reduce from 30% to 26.42%. however,
gross and net profits amounts have been increased by Rs. 8 million mainly due to:
 Increase in sales volume as a result of 5% decrease in selling price. This
resulted in increase in gross profit by 8.89%[(98-90)÷90×100].
 Acquisition of building has resulted in savings in expenses as rent saved
(Rs. 15 million) is higher than the depreciation (Rs. 5 million) and increased in
finance cost (Rs. 5 million).
 Since 75% of selling and administrative cost was fixed, expenses did not
increase due to increase in sales volume (economies of scale).

(ii) Liquidity:
The decrease in current ratio from 2.5 to 1.8 is net effect of the following:
 Cash payment for purchase of building which significantly decreased current
assets.
 Prompt payment to suppliers which decreased the current liabilities.

Page 7 of 8
Financial Accounting and Reporting-I
Suggested Answers
Certificate in Accounting and Finance – Autumn 2018

Ans.8 (a) Financial accounting system Cost accounting system


(i) Requirement
Prepared to meet a legal or regulatory Prepared to meet the needs of management.
requirement.

(ii) Utilisation
Used to prepare financial statements for Used to prepare information for management
shareholders and other external users. (Might (internal use only).
also provide some information for
management but this is not their primary
purpose).

(iii) Time frame


Prepared within a time frame specified by a Prepared within a time frame specified by
legal or regulatory framework. management.

(iv) Activities
Records revenues, expenditure, assets and Records costs of activities and used to
liabilities. provide detailed information about costs,
revenues and profits for specific products,
operations and activities.

(v) Convention
Used mainly to provide a historical record of Provides historical information, but also used
performance and financial position. extensively for forecasting (forward-looking).

(b) Describing cost behaviour graphically:

(i) Factory building rent - Fixed amount (ii) Direct labour cost - Fixed per unit
per month

(iii) Supervision cost - One supervisor is (iv) Machine rental cost - Fixed monthly
required for every 20 direct workers rent and an additional cost of Rs. 100
per unit for the production exceeding
certain limit.

(THE END)

Page 8 of 8
THE INSTITUTE OF CHARTERED ACCOUNTANTS OF PAKISTAN

EXAMINERS’ COMMENTS

SUBJECT SESSION
Financial Accounting and Reporting-I Certificate in Accounting and Finance
– Autumn 2018

General

The overall performance was significantly better this time. The passing ratio was 56.6%
which was much better than the last result of 43.2%. In fact it is the highest for this paper
since introduction of new education scheme. The highest score in the paper was 93 marks
which indicated that the paper was very doable. A significant number of students secured
80 or more marks in the paper.

Performances in all questions were quite good except for Q3 and Q8. Poor performances
in both the questions were mainly because such variations had not been examined
previously.

Although many students performed well but too often illogical mistakes were noted in
easy areas as well. The persisting issue appears to be lack of practice and poor
presentation in many cases.

It was observed that students spend too much time on completing the question even
though they have no idea of the difficult part of the questions. Students are strongly
advised to switch to the next question after they have spent reasonable time on a question.
This will ensure that they attempt all questions. 17.5% students were just short of 9 or
less marks and could have easily obtained them had they covered all areas on the syllabus
and/or attempted all questions in the paper.

Question-wise comments:

Question 1

The question required statement of profit or loss from the given information. The overall
performance in this question was average and 54.3% of the students secured passing
marks. Many students made errors on the easier aspects of the question which deprived
them of some precious marks. Some common errors were as follows:

 Students failed to notice that sales should have been calculated by applying profit
margin on cost of sales. Students often prepared cash account and assumed the
balancing figure to be sales.
 All effects of adjustment for damaged stock were not incorporated.

Page 1 of 3
Examiners’ Comments on Financial Accounting and Reporting-I - Autumn 2018

Question 2

The question required computation of value of stock as at year end. The crux of the
question was that all physical movements after the year end needed to be reversed. The
performance in this question was above average as 77.1% of the students secured passing
marks. However, various errors were also observed. Most students ignored the fact that
40% of the goods were damaged after the year end in July 2018 so NRV adjustment was
required to be made in respect of 60% of the damaged goods only.

Question 3

The question was based on IFRS 15 which is a new area of the syllabus. Only 16.1% of
the students secured passing marks in this question. 18.2% of the students could not score
any mark which shows that they had not studied this area of the syllabus.

In theory based part (a), majority of the students were successful in listing down the five
steps involved in recognizing revenue as per IFRS 15. However, a significant minority
could not even list down these steps.

In part (b), only few students were able to secure good or even passing marks. The crux
of the question was the timing of revenue recognition in each case. In case (i) 95% of the
revenue should have been recognized on 1 June and remaining on 30 June while in case
(ii) whole revenue should have been recognized on 30 June. Answers to this part of the
question were generally poor as most of the students failed to pick the underlying concept
which was being tested. The performance in case (i) was generally better than case (ii).

Question 4

The question required effects of amalgamation on partners’ capital accounts in old and
new firm. The performance in this question was good and 68.6% of the students secured
passing marks. Many students unnecessarily prepared realization account and subjected
themselves to time pressure. Some common errors were as follows:

 Provision for doubtful debts was calculated as 5% or 3% of the given balance without
realizing that the given balance was already net of 2% provision.
 Cash was not excluded for calculating goodwill.

Question 5

The question required preparation of ledger accounts related to fixed assets (Machines).
70.7% of the students secured passing marks. Some common errors were as follows:

 The new machine was recorded at the list price, instead of the sum of the fair value
of old machine (which was exchanged) and the amount paid.
 Various types of errors were made in calculating deprecation for the year.
 Cost of disposal was altogether ignored.

Page 2 of 3
Examiners’ Comments on Financial Accounting and Reporting-I - Autumn 2018

Question 6

The question required statement of financial position and statement of profit or loss in
accordance with IFRSs. The overall performance in this question was average as 51.2%
of the students secured passing marks. A general weakness amongst most students was
that they failed to incorporate the double effect of each adjustment / correction. Many
students seemed to waste time in tallying the statement of financial position which was
totally unnecessary. Some common errors were as follows:

 Stock held under ‘Bill and Hold arrangement’ was correctly excluded from the
closing stock; however, the corresponding adjustment for revenue was not made.
 In respect of disposal of land and building, incorrect amounts were used to record
disposal. Moreover, the adjustment to transfer the remaining revaluation surplus of
the disposed of item was hardly seen.
 Amount written off was not excluded from balances due for more than one year for
calculating specific provision.

Question 7

The question required computation of ratios and comments on the profitability and
liquidity, based on the given financial statements. The performance in this question was
below average and only 43.7% of the students could secure passing marks. The
computational part was easy but still some students missed those easy marks. Majority of
the students performed badly while commenting on the profitability and liquidity. The
main reason for such a performance was that the students did not understand the
requirement and did not relate the reasoning to the additional information given in the
question.

Question 8

The question comprised of two parts. The performance in this question was well below
average and only 29.1% of the students could secure passing marks.

Part (a) asked for four differences between financial accounting and cost accounting
system. This was answered in an average manner, with students at times repeating the
same point in different words. The students who were able to clearly identify the
differences secured full marks.

Part (b) required graphical presentation of different types of costs. Most of the students
were unable to present the correct graph of machine rental cost.

THE END

Page 3 of 3
Financial Accounting and Reporting - I
Summary of Marking Key
Certificate in Accounting and Finance – Autumn 2018

Note regarding marking scheme:


The marking scheme is given as a guide. Markers also award marks for alternative approaches to a
question and relevant/well-reasoned comments/explanations. Moreover, the available marks in
answer may exceed the total marks of a question.

Mark(s)
A.1  Determination of sales 2.5
 Computation of credit and cash purchases 2.5
 Treatment of damaged stock 1.5
 Withdrawal of goods for personal use 1.0
 Determination of closing inventory 0.5
 0.75 mark each for computation of shop rent, misc. supplies used and staff salaries 2.25
 01 mark each for calculation of riders’ commission and depreciation 2.0
 0.25 mark each for the correct treatment of other amounts 0.75

A.2  Adjustment pertaining to purchases for the intervening period i.e. from the
year-end date to the inventory count date 3.5
 Adjustment for credit notes and sales for the intervening period i.e. from the
year-end date to the inventory count date 4.0
 Adjustments for goods-in-transit 1.0
 Adjustment for NRV 1.5

A.3 (a) Up to 0.75 mark for each step involved in recognizing revenue under IFRS 15 3.0

(b) For accounting entries under the situation where:


(i) it is estimated that 5% units from the sold units would be returned
 entry on 1 June 2018 3.0
 entry on 30 June 2018 2.0

(ii) no estimate can be made for return of units sold


 entry on 1 June 2018 2.0
 entry on 30 June 2018 2.0

A.4  Calculation of profit sharing % in the partners’ capital ratio 1.0


 Computation of realisation gain/(loss) on merger of the two firms 4.0
 Computation of goodwill for RF and IG 3.0
 Posting of realization gain and goodwill to the partners’ capital accounts of RF
and IG 2.0
 Posting of goodwill written-off in the books of QA 1.0
 Computing and posting of cash to be contributed by/payable to the partners 1.0

Page 1 of 2
Financial Accounting and Reporting - I
Summary of Marking Key
Certificate in Accounting and Finance – Autumn 2018

Mark(s)
A.5 (a)  Posting of amounts to cost account 1.5
 Calculation of cost from the given WDV amount 1.0
 Computation of correct amount of addition 0.5

(b)  Posting of amounts to accumulated depreciation account 2.0


 Calculation of accumulated depreciation 3.0

(c)  Posting of amounts to gain/loss on disposal account 3.5


 Correct treatment of trade-in at fair value 0.5

A.6  Presentation and disclosures 1.0


 Computation of items pertaining to the statement of profit or loss:
− sales revenue and cost of sales 3.5
− selling and administrative expenses 2.0
− gain on disposal of office block 1.0
− tax 0.5
 Computation of items pertaining to the statement of financial position:
− property, plant and equipment 3.0
− trade receivables and provision for doubtful debts 2.0
− other current assets 2.0
− share capital and retained earnings 2.0
− revaluation surplus 1.5
− current liabilities 2.5

A.7 (a)  0.5 mark each for computation of gross and net profit margins, return on
assets and current ratio for 2016 and 2017 4.0
 01 mark each for computation of debt equity ratio and return on capital
employed for 2016 and 2017 4.0

(b) 02 marks each for giving comments on profitability and liquidity position of SKL
for 2017 based on the given information 4.0

A.8 (a) 01 mark for stating each difference between financial and cost accounting systems 4.0

(b) 01 mark each for describing cost behavior graphically 4.0

(THE END)

Page 2 of 2
Certificate in Accounting and Finance Stage Examination
The Institute of 6 March 2019
Chartered Accountants 3 hours – 100 marks
of Pakistan Additional reading time – 15 minutes

Financial Accounting and Reporting-I


Q.1 X, Y and Z were partners sharing profits and losses in the ratio of 4:3:3 respectively. The
balance sheet as on 31 December 2018 is given below:

Rs. in '000 Rs. in '000


Creditors 660 Cash and bank balances 250
Accrued expenses 150 Stock-in-trade 460
General reserve 250 Trade debtors 800
Capital: Provision for doubtful debts (80)
X 502 Machines 400
Y 358 Vehicles 310
Z 400 Equipment 180
2,320 2,320

On 31 December 2018, Z retired from the partnership. The following has been agreed in this
respect:
(i) Goodwill of the firm has been determined at Rs. 380,000. It has been estimated that
the value of goodwill after Z’s retirement would be Rs. 300,000. Goodwill is to be
written off from the books.
(ii) Machines would be adjusted to 85% of the book value whereas equipment would be
appreciated by 20%.
(iii) Trade debtors amounting to Rs. 100,000 would be written off. Existing percentage of
provision for doubtful debts would be maintained.
(iv) An accrual for repairs and maintenance amounting to Rs. 41,000 would be recorded
in the books.
(v) Z’s balance would be settled as follows:
 Immediate cash payment of Rs. 150,000.
 A vehicle would be given at an agreed value of Rs. 120,000 (book value
Rs. 70,000).
 Fully depreciated items of furniture would be given at an agreed value of
Rs. 35,000.
 Remaining balance would be paid after 6 months along with interest at 10%.
(vi) To determine new profit sharing ratio, Z’s share would be divided equally between X
and Y.
(vii) Y’s capital would be adjusted in new profit sharing ratio on the basis of X’s capital.
Any excess or deficiency would be adjusted through cash.

Required:
Prepare partners’ capital and revaluation accounts on the retirement of Z. (12)
Financial Accounting and Reporting-I Page 2 of 5

Q.2 (a) Compare ‘Regression analysis’ with ‘High-low analysis’ for cost estimation. (03)

(b) Describe the behaviour of each of the following costs graphically by denoting
‘Per unit cost’ on vertical axis and ‘Level of activity’ on horizontal axis:
(i) Factory building rent – Fixed amount per month.
(ii) Direct labour cost – Increases proportionately with production.
(iii) Supervision cost – One supervisor is required for every 20 direct workers.
(iv) Direct material cost – Bulk discount is available on all purchases once the total
purchases exceed a certain level. (05)

Q.3 Following are the summarised financial statements of Keyboard Limited (KL):

Statement of financial position


2018 2017 2016
----------- Rs. in '000 -----------
Fixed assets 12,500 10,800 11,800

Current assets:
Inventory 4,000 4,500 3,000
Debtors 4,200 3,200 1,800
Cash - 800 2,100
8,200 8,500 6,900
20,700 19,300 18,700

Equity and reserves 10,400 9,000 8,600


Long term loan 4,400 5,000 5,600

Current liabilities:
Creditors 3,500 4,400 4,200
Bank overdraft 1,500 - -
Accrued expense 900 900 300
5,900 5,300 4,500
20,700 19,300 18,700

Statement of profit or loss


2018 2017 2016
----------- Rs. in '000 -----------
Sales 27,000 24,400 21,000
Cost of goods sold (21,300) (19,400) (17,200)
Gross profit 5,700 5,000 3,800
Operating expenses (3,400) (3,000) (2,400)
Finance cost (300) (350) (400)
Net profit 2,000 1,650 1,000

Required:
(a) Compute working capital cycle in days and liquidity ratios for 2018 and 2017. (11)
(b) Suggest three possible measures that can be taken by KL to improve working capital
cycle days. (03)
Financial Accounting and Reporting-I Page 3 of 5

Q.4 (a) List the criteria that must be met to account for a contract with customer under
IFRS 15 ‘Revenue from Contracts with Customers’. (04)

(b) Guitar World (GW) normally sells Machine A13 for Rs. 1.7 million. Maintenance
services for such type of machines are provided separately at Rs. 25,000 per month.
Details of two contracts for sale of Machine A13 are as follows:
(i) On 1 July 2018, GW signed a contract with Energene Limited to sell Machine
A13 with one year free maintenance services at a lumpsum payment of
Rs. 1.8 million. The amount was received upon delivery of machine on
1 August 2018.
(ii) On 1 October 2018, GW sold Machine A13 to Vitalene Limited for
Rs. 1.95 million. As per the contract, payment would be made after 2 years.
Maintenance services would also be provided for Rs. 25,000 per month for
two years which would be paid at the end of each month.

Required:
With reference to IFRS-15 ‘Revenue from Contracts with Customers’, explain how
the above contracts should be recorded in GW’s books for year ended
31 December 2018. (Show supporting calculations but entries are not required) (11)

Q.5 The following information pertains to Piano Limited (PL):

Plant Equipment
Acquisition
 Date of acquisition 1 January 2015 1 July 2015
 Cost Rs. 500 million Rs. 360 million
 Estimated useful life 10 years 12 years
 Residual value Rs. 60 million Nil
 Depreciation method Straight line method Straight line method

Revaluation on 31 December 2016


 Fair value Rs. 526 million Rs. 280 million
 Residual value Rs. 78 million Nil

Revaluation on 31 December 2018


 Fair value Rs. 310 million Rs. 275 million
 Residual value Rs. 64 million Nil

Additional information:
(i) PL uses revaluation model for subsequent measurement and accounts for revaluation
on net replacement value method.
(ii) There is no change in useful life of plant. The remaining useful life of equipment was
estimated as 15 years and 10 years in 2016 and 2018 respectively.
(iii) PL transfers maximum possible amount from the revaluation surplus to retained
earnings on an annual basis.
(iv) PL’s financial year ends on 31 December.

Required:
(a) Calculate depreciation on each asset for 2015 to 2018. (08)
(b) Prepare entries to record revaluation in 2018. (Entries to record depreciation expense,
incremental depreciation and elimination of accumulated depreciation are not required.
Further, entries prior to 2018 are also not required.) (08)
Financial Accounting and Reporting-I Page 4 of 5

Q.6 Violin Family Club was formed in 2016. Following are the details of assets and liabilities of
the club as on 31 December 2017:

Assets Rs. in '000 Liabilities Rs. in '000


Subscription in arrears: Bank overdraft 181
2016 15 Subscription in advance for 2018 45
2017 90 Accrued electricity 23
Advance rent 24 Canteen wages 11
Canteen stock 215 Canteen creditors 118
Snooker tables 960
Furniture & equipment 720
2,024 378

Additional information:
(i) Some of the balances as on 31 December 2018 are as follows:
Assets Rs. in '000 Liabilities Rs. in '000
Subscription in arrears for 2018 30 Accrued electricity 35
Canteen stock 247 Canteen creditors 142
(ii) Break-up of the subscription received during 2018 is as follows:
Related to year Rs. in '000
2017 60
2018 920
2019 75
The club’s management has decided to write-off the remaining subscription in arrears
relating to the year 2016 and 2017.
(iii) A scheme was introduced in 2016 under which a person is awarded life time
membership upon payment of Rs. 120,000. Life memberships received in the years
2016, 2017 and 2018 were 5, 8 and 6 respectively. Life memberships are credited to
‘Life Membership Fund’ upon receipt and are transferred to income equally over
10 years, starting from the year of admission.
(iv) The club operates a canteen. Till last year, the canteen earned a gross profit of 20% of
sales. Effective 1 January 2018, selling prices were increased by 10%.
(v) Details of some payments during 2018 are as follows:
Rs. in '000
Canteen creditors 512
Salaries 285
Equipment 66
Electricity 263
(vi) Equipment acquired during the year is only 30% paid and the remaining amount is
payable in February 2019.
(vii) Wages of canteen staff are paid on 5th of each month.
(viii) The club operates from a rented place. The rent is paid quarterly in advance on
1 March, 1 June, 1 September and 1 December. As per agreement, annual rent was
increased by Rs. 6,000 with effect from 1 September 2018.
(ix) Balance of snooker tables as at 31 December 2017 represents the book value of
5 similar tables purchased in 2016. One of the tables was sold to a member for cash
during the year for Rs. 212,000.
(x) Snooker tables are depreciated at 12.5% on straight line method while furniture &
equipment are depreciated at 20% using reducing balance method. Full year
depreciation is charged in the year of addition whereas no depreciation is charged in
the year of disposal.
Financial Accounting and Reporting-I Page 5 of 5

Required:
(a) Prepare income and expenditure account for the year ended 31 December 2018. (12)
(b) Prepare statement of financial position as on 31 December 2018. (09)

Q.7 Junior Accountant of Drum Limited has prepared the following statement of cash flows for
the year ended 31 December 2018:

Statement of cash flows


Rs. in '000
Cash flows from operating activities
Increase in retained earnings 1,360
Increase in dividend payable 200
Increase in net trade receivables (100)
Increase in interest accrued 50
1,510
Cash flows from investing activities
Increase in land and building (2,600)
Increase in equipment (1,550)
Decrease in inventory 400
Decrease in tax payable (60)
(3,810)
Cash flows from financing activities
Increase in share capital and premium 2,350
Decrease in long term loan (1,000)
Increase in trade and other payables 600
1,950
Decrease in cash balance during the year (350)
Opening cash balance 450
Closing cash balance 100

Junior Accountant informed you that he has taken the difference of opening and closing
balances of each balance sheet item and classified each difference as either operating,
investing or financing cash flows. He further informed that the statement is tied up with the
cash balances appearing in the balance sheet. He has ignored the following information:

(i) Depreciation on building and equipment amounted to Rs. 480,000 and Rs. 810,000
respectively.
(ii) During the year, an equipment costing Rs. 560,000 and having a book value of
Rs. 310,000 was sold for Rs. 440,000.
(iii) Provision for doubtful debts was increased by Rs. 140,000.
(iv) Dividend amounting to Rs. 700,000 was paid during the year.
(v) Interest and tax expenses for the year amounted to Rs. 378,000 and Rs. 650,000
respectively.
(vi) Trade and other payables as at 31 December 2018 included Rs. 950,000 for purchase
of land and building.

Required:
Prepare statement of cash flows for the year ended 31 December 2018, in accordance with
IAS 7 ‘Statement of Cash Flows’ using indirect method. (14)

(THE END)
Financial Accounting and Reporting-I
Suggested Answers
Certificate in Accounting and Finance – Spring 2019

Ans.1 Partners' capital


X Y Z X Y Z
---- Rs. in '000 ---- ---- Rs. in '000 ----
Revaluation loss 28 21 21 Balance 502 358 400
Goodwill written off (380 in 11:9) 209 171 Goodwill (380 in 4:3:3) 152 114 114
General reserve (250 in
Settlement with Z: 4:3:3) 100 75 75
Cash 150 Investment by Y (balancing) 68
Vehicle 120
Furniture 35
Loan (balancing) 263
Closing balance X 517
Closing balance Y (517×9÷11) 423
754 615 589 754 615 589

Revaluation
Rs. in '000 Rs. in '000
Machine (400×0.15) 60 Equipment (180×20%) 36
Provision for doubtful debts 10
Trade debtors 100 (100×10%)
Accruals 41 Vehicle (120–70) 50
Furniture 35
Revaluation loss of 70
X 28
Y 21
Z 21
201 201

W-1: Ratios X Y Z
Old profit share 4/10 3/10 3/10
Share of Z divided equally between X and Y 3/20 3/20 –3/10
New profit share 11/20 9/20 -

Ans.2 (a) Regression analysis and high-low analysis compared


Following are the important differences between linear regression analysis and the high-
low method.

Regression analysis High low method


Regression analysis uses as many sets of data High-low analysis uses just two sets of
for x and y as are available. data for x and y, the highest and the
lowest values for x and the corresponding
values of y.
Regression analysis can be used to assess the High low method cannot be used to
extent to which values of y depend on values assess the extent to which values of y
of x. depend on values of x.
Regression analysis uses complex arithmetic High low method uses simple
and a calculator or small spreadsheet model mathematics.
is normally needed.
Effect of extreme values is minimal on High low method is highly affected by
regression analysis. extreme values in the data.

Page 1 of 7
Financial Accounting and Reporting-I
Suggested Answers
Certificate in Accounting and Finance – Spring 2019

(b)

Ans.3 (a) Working capital cycle 2018 2017


------------- Number of days -------------
Average days to collect debtors W-1 50.0 37.4
Average inventory holding period W-2 72.8 70.5
Less: Average time to pay creditors W-3 (69.3) (75.1)
53.5 32.8

Liquidity ratios: 2018 2017


8,200 8,500
Current assets
Current Ratio = 5,900 5,300
Current liabilities
𝟏. 𝟑𝟗 ∶ 𝟏 𝟏. 𝟔𝟎 ∶ 𝟏

8,200 − 4,000 8,500 − 4,500


Current assets − inventory
Quick Ratio = 5,900 5,300
Current liabilities
𝟎. 𝟕𝟏 ∶ 𝟏 𝟎. 𝟕𝟓 ∶ 𝟏

[(4,200 + 3,200) ÷ 2] [(3,200 + 1,800) ÷ 2]


Debtors collection period Average debtors × 365 × 365
W-1: = × 365 27,000 24,400
(in days) Credit sales
50 days 37.4 days

Average inventory [(4,000 + 4,500) ÷ 2] [(4,500 + 3,000) ÷ 2]


Inventory holding period × 365 × 365
W-2: (in days)
= × 365 21,300 19,400
Cost of sales
72.8 days 70.5 days

Average Creditors [(3,500 + 4,400) ÷ 2] [(4,400 + 4,200) ÷ 2]


Creditors payment period × 365 × 365
W-3: = × 365 [21,300 + 4,000 − 4,500] [19,400 + 4,500 − 3,000]
(in days) Credit purchases
69.3 days 75.1 days

Page 2 of 7
Financial Accounting and Reporting-I
Suggested Answers
Certificate in Accounting and Finance – Spring 2019

(b) Measures to improve working capital cycle days:


 Give incentives to customers to pay on time
 Do not transact with customers who have a history of defaulting/late payments
 Automate the monitoring of accounts receivables

Ans.4 (a) The general IFRS 15 model applies only when all of the following conditions are met:

 the parties to the contract have approved the contract.


 the entity can identify each party’s rights.
 the entity can identify the payment terms for the goods and services to be
transferred.
 the contract has commercial substance.
 it is probable that the entity will collect the consideration.

(b) (i) The contract contains two distinct performance obligations i.e. selling the machine
and providing the maintenance services as:

 the customer can separately benefit from the machine without the
maintenance services from GW (or GW sells maintenance services separately)
and
 the machine and maintenance services are separately identifiable in the
contract.

Thus GW will allocate the transaction price between the two performance
obligations as follows:

Standalone price Proportion Transaction price


Machine 1,700,000 85 1,530,000
Maintenance 300,000 15 270,000
(Rs. 25,000×12) (Rs. 22,500×12)
2,000,000 100 1,800,000

Revenue related to sale of machine would be recognized at a point in time i.e. upon
delivery on 1 August 2018.

While revenue related to maintenance service would be recognized over time i.e. as
the services are rendered.

Till 31 December 2018, revenue would be recognized in respect of:

 Sale of machine Rs. 1,530,000


 Maintenance service Rs. 112,500 i.e Rs. 22,500 for 5 months

Remaining amount of Rs. 157,500 would appear in liabilities as deferred revenue.

(ii) The contract contains two distinct performance obligations i.e. selling the machine
and providing the maintenance services.

The contract includes a significant financing component in respect of sale of


machine which is evident from the difference between the amount of promised
consideration of Rs. 1.95 million and the cash selling price of Rs. 1.7 million.

Revenue related to machine would be recognized upon delivery on 1 October 2018.

Page 3 of 7
Financial Accounting and Reporting-I
Suggested Answers
Certificate in Accounting and Finance – Spring 2019

Revenue related to maintenance service would be recognized as the services are


rendered each month.

The difference between promised consideration and cash selling price of Rs.
250,000 would be recognized as interest revenue over two years using the implicit
rate of 7.1% [(1.95÷1.7)1/2–1].

Till 31 December 2018, revenue would be recognized in respect of:

 Sale of machine Rs. 1,700,000


 Maintenance service Rs. 75,000 i.e Rs. 25,000 for 3 months
 Interest revenue Rs. 30,175 (Rs. 1.7 million × 7.1% × 3/12)

Ans.5 (a) Plant Equipment


----- Rs. in million -----
Cost 500 360
[(500–60)/10],
Depreciation 2015 [(360/12)×(6/12)] (44) (15)
456 345
Depreciation 2016 [(456–78)/9], [345/15] (42) (23)
414 322
Revaluation–surplus/(loss) 2016 (balancing) 112 (42)
Fair value 526 280
Depreciation 2017 [(526–78)/8], [280/14] (56) (20)
470 260
Depreciation 2018 [(470–64)/7], [260/10] (58) (26)
412 234

(b) Piano Limited


Accounting entries for revaluation

Debit Credit
Date Description
------ Rs. in million ------
31-Dec-2018 Revaluation loss (P&L account) 18.00
Revaluation surplus 84.00
Plant 102.00

31-Dec-2018 Equipment 41.00


Revaluation gain (P&L account) 35.10
Revaluation surplus 5.90

W-1: Revaluation surplus/(impairment): Plant Equipment


----- Rs. in million -----
Fair value 310 275
Book value [From part(a)] (412) (234)
Increase / (Decrease) in asset (102) 41
Adjustment for previous:
Revaluation surplus 112–(112÷8)– (112÷8) 84
Revaluation loss 42–(42÷14)–(39÷10) (35.1)
Revaluation surplus/(Revaluation loss) (18) 5.9

Page 4 of 7
Financial Accounting and Reporting-I
Suggested Answers
Certificate in Accounting and Finance – Spring 2019

Ans.6 Violin Family Club


(a) Income and expenditure account for the year ended 31 December 2018
Rs. in '000
Income
Subscription (W-1) 995
Gain on disposal of table 212–960÷5 20
Profit from canteen 57
Life membership (W-2) 228
1,300
Expenditures
Rent [36(24×3÷2)+36+37.5+37.5]+[24–25(37.5×2÷3)] 146
Salaries 285
Electricity 263+(35–23) 275
Depreciation – snooker tables (960–192)×(12.5÷75) 128
Depreciation – furniture & equipment (720+220)×20% 188
Subscription written off 45
(1,067)
Excess of income over expenditure 233

Canteen trading account for the year ended 31 December 2018


Rs. in '000
Sales 504×110÷80 693
Cost of goods sold
Opening stock 215
Purchases 512+142–118 536
Closing stock (247)
504
Gross profit 189
Expenses
Wages 11×12 (132)
Profit from canteen 57

W-1: Subscription Rs. in '000


Opening arrears: Opening advance 2018 45
2016 15 Receipts (60+920+75) 1,055
2017 90 Write off (15+30) 45
Income balance 995 Closing arrears 30
Closing advance 75
1,175 1,175

W-2: Life membership Rs. in '000


Income
[(5+8+6)×120÷10] 228 Opening balance
(5×120×8÷10)+(
Closing balance 1,836 8×120×9÷10) 1,344
Receipt (6×120) 720
2,064 2,064
(b) Violin Family Club
Statement of financial position as on 31 December 2018 Rs. in '000
Assets
Non-current assets
Snooker table (960–192–128) 640
Furniture & equipment (720+220–188) 752
1,392

Page 5 of 7
Financial Accounting and Reporting-I
Suggested Answers
Certificate in Accounting and Finance – Spring 2019

Current assets:
Canteen stock 247
Prepaid rent 25
Subscriptions in arrears 30
Bank (W-3) 1,094
1,396
2,788

General funds
Opening balance (2,024–378)–1,344(W-2) 302
Excess of income over expenditure 233
535

Life membership fund (W-2) 1,836

Liabilities
Canteen creditors 142
Accrued electricity 35
Subscription in advance (W-1) 75
Creditors for equipment (220–66) 154
Canteen wages payable 11
417
2,788

W-3: Bank/cash Rs. in '000


Subscriptions 1,055 Opening balance 181
Life membership (W-2) 720 Rent 147
Sale proceeds from table 212 Salaries 285
Canteen receipts 693 Electricity 263
Canteen creditors 512
Canteen salaries 132
Equipment 66
Closing balance 1,094
2,680 2,680

Ans.7 Drum Limited


Statement of cash flows for the year ended 31 December 2018
Rs. in ‘000
Cash flows from operating activities
Profit before tax 1,360+700+200+650 2,910
Adjustment for:
Depreciation 480+810 1,290
Gain on disposal (130)
Increase in provision of doubtful debts 140
Interest expense 378
4,588
Working capital change
Decrease in inventory 400
Increase in trade receivables –100–140 (240)
Decrease in trade payable 600–950 (350)
(190)
Cash generated from operations 4,398
Interest paid –378+50 (328)
Tax paid –650–60 (710)
3,360
Page 6 of 7
Financial Accounting and Reporting-I
Suggested Answers
Certificate in Accounting and Finance – Spring 2019

Cash flows from investing activities


Purchase of land and building –2,600–480+950 (2,130)
Purchase of equipment –1,550–810–310 (2,670)
Disposal of equipment 440
(4,360)
Cash flows from financing activities
Issuance of shares 2,350
Loan repaid (1,000)
Dividend paid (700)
650
Decrease in cash during the year (350)
Opening cash 450
Closing cash 100

(THE END)

Page 7 of 7
INSTITUTE OF CHARTERED ACCOUNTANTS OF PAKISTAN

CERTIFICATE IN ACCOUNTING AND FINANCE (CAF) EXAMINATIONS

EXAMINERS’ COMMENTS

SUBJECT SESSION
Financial Accounting and Reporting-I (FAR-I) Spring 2019

Passing %

Question-wise
Overall
1 2 3 4 5 6 7
55% 07% 60% 22% 28% 21% 53% 29%

General comments

Performance in Q.2 was poor while it was average in Q.4, Q.5 and Q.6.

Although many students performed well, some shortcomings such as lack of practice, poor
presentation, etc. were commonly noted.

A common weakness amongst most students was that they failed to make a reasonable
attempt of all questions. Students are advised to move to the next question after spending a
reasonable time on a particular question. This would help them to attempt all questions of
the paper. In this paper, 20% students were just short of 9 or less marks and could have
crossed the line had they attempted all questions in the paper.

Question-wise common mistakes observed

Question 1

 General reserve was not distributed among partners.


 Goodwill were written off with the amount that would be after the retirement of Z.
 New profit sharing ratio was incorrectly calculated.

Question 2(a)

Students failed to identify any correct difference other than one based on number of data
set.

Question 2(b)

Graphs were made on the basis of ‘total cost’ instead of ‘per unit cost’ on y-axis.

Question 3

 Closing balances were used instead of average balances for calculating ratios for
working capital cycle.

Page 1 of 2
Examiners’ comments on Financial Accounting and Reporting-I Spring 2019

 Cost of sales was used instead of purchases for calculating creditor payment period.
 Part (b) was either not attempted or attempted half-heartedly which showed lack of
knowledge.

Question 4(a)

Students could not understand the actual requirement of the question and often gave
altogether incorrect answer.

Question 4(b)

 Student did not read the requirement carefully and restricted their answers to journal
entries (which were not even required) or/and calculations only.
 In situation (i), explanations were often correct but incomplete.
 In situation (ii), students could not identify existence of significant financing component
in the contract.

Question 5

 Depreciation on equipment was required for 6 months whereas full year depreciation
was taken in first year.
 Change in residual value of plant was taken from 2017 instead of 2016.
 Students incorporated effects of revaluation even before calculating depreciation for
2016 and 2018.
 Several mistakes were made in bifurcating the effect of revaluation into profit & loss
account and revaluation surplus.

Question 6

 Annual increase in rent by Rs. 6,000 was considered as monthly or quarterly increase.
 Students could not pick the fact that balance of snooker table as at 31 December 2007
represented 75% of the cost as those was purchased in 2016.
 Opening balance of life membership fund was not correctly calculated.
 Opening balance of life membership fund was not deducted while calculating opening
general fund.
 Students left the question incomplete after getting stuck in some areas of the question.

Question 7

 Students either did not calculate profit before tax or atleast missed one amount in the
calculation.
 Effects of depreciation and disposal were not incorporated in calculating additions to
PPE.
 In cash flow from operating activities, deductions had been shown as additions and vice
versa.

The End

Page 2 of 2
Financial Accounting and Reporting - I
Summary of Marking Key
Certificate in Accounting and Finance – Spring 2019

Note regarding marking scheme:


The marking scheme is given as a guide. Markers also award marks for alternative approaches to a
question and relevant/well-reasoned comments/explanations. Moreover, the available marks in
answer may exceed the total marks of a question.

Mark(s)
A.1  Preparation of revaluation account 3.0
 Preparation of partners’ capital accounts
– Goodwill 2.0
– General reserve and revaluation loss 2.0
– Settlement of Z 2.0
– Revised capital 2.0
 New profit sharing ratio 1.0

A.2 (a) 01 mark for each valid difference 3.0

(b)  01 mark each for factory building rent and direct labour cost 2.0
 1.5 marks each for supervision cost and direct material cost 3.0

A.3 (a)  Computation of working capital cycles in days for 2018 and 2017:
– Working capital cycle in days 1.0
– Debtors collection period 2.0
– Inventory holding period 2.0
– Creditors payment period 3.0
 Current and quick ratios 3.0

(b) 01 mark for each valid possible measure 3.0

A.4 (a) 01 mark for each criteria 4.0

(b) (i) Contract signed with Energene Limited:


 Distinct performance obligations 1.0
 Transaction price and its allocation 2.0
 Timings of recognition of revenue and amount of revenue 2.0
 Deferred revenue 1.0

(b) (ii) Contract signed with Vitalene Limited:


 Identification and explanation of significant financing component 1.5
 Timings of recognition of revenue and amount of revenue 3.5

A.5 (a) 02 marks for each year’s depreciation 8.0

(b)  1.5 marks for each entry to record revaluation 3.0


 Computation of revaluation surplus/loss for 2018 1.0
 Adjustments of revaluation surplus/loss recorded in 2016 4.0

Page 1 of 2
Financial Accounting and Reporting - I
Summary of Marking Key
Certificate in Accounting and Finance – Spring 2019

Mark(s)
A.6 (a) Computation of items pertaining to income and expenditure account:
 Subscription revenue 3.0
 Profit from canteen 3.0
 Life membership revenue 2.0
 Rent expense 2.0
 Depreciation expense 2.0

(b)  Computation of items pertaining to financial position:


– Non-current assets 1.0
– Bank 2.0
– Current assets (other than bank) 1.0
– General funds 2.0
– Liabilities 2.0
 Presentation and disclosure 1.0

A.7  Profit before tax 2.0


 Adjustment in profit 2.0
 Working capital changes 2.5
 01 mark each for interest, tax and dividend paid 3.0
 Investing activities 3.5
 Financing activities 1.0

(THE END)

Page 2 of 2
Certificate in Accounting and Finance Stage Examination
The Institute of 4 September 2019
Chartered Accountants 3 hours – 100 marks
of Pakistan Additional reading time – 15 minutes

Financial Accounting and Reporting-I


Section A

Q.1 The following information pertains to Wednesday Limited (WL) for the year ended
30 June 2019:
(i) Shareholders' equity as at 1 July 2018:
Rs. in million
Share capital (Rs. 100 each) 200
Share premium 85
Retained earnings 124
Revaluation surplus 65
(ii) On 30 November 2018, WL issued 30% right shares at a premium of
Rs. 120 per share.
(iii) Cash dividend and bonus shares for the last two years:
Final dividend *Interim dividend
For the year ended
Cash Bonus Cash Bonus
30 June 2018 18% - 20% -
30 June 2019 - 25% - 10%
*Declared with half yearly accounts
(iv) Profit for the year amounted to Rs. 95 million.
(v) Revaluation surplus arising during the year amounted to Rs. 35 million whereas
transfer of incremental depreciation for the year was Rs. 9 million.

Required:
Prepare WL’s Statement of Changes in Equity for the year ended 30 June 2019. (07)
(Column for total and comparative figures are not required)

Q.2 Discuss how the following should be dealt with in the financial statements of relevant
entities according to IAS 20 Accounting for Government Grants and Disclosure of
Government Assistance:
(a) The government makes a grant to an entity which is planning to develop teaching
software for children with learning difficulties. The purpose of the grant is to help the
entity to meet its general financing requirement in the initial phase. There are no
further conditions attached to the grant. (01)

(b) A manufacturing entity sets up a plant in an area of high unemployment. A


government grant of Rs. 4 million is received with a condition that the grant is
repayable in full if the number of its employees fell below 100 at any time during the
next four years. It is highly probable that the entity will comply with the condition
attached to the grant. (03)

(c) Free technical advice has been provided by the government’s export promotion
department to help an exporter to market his new technology in North America. (01)
Financial Accounting and Reporting-I Page 2 of 7

Q.3 Tuesday Manufacturers Limited produces a single product. The following costs were
incurred in the month of June 2019:
Rs. in '000
Direct labour 2,075
Depreciation on plant and machinery 380
Distribution costs 589
Factory manager’s salary 247
Indirect labour 848
Indirect material consumed 345
Raw material purchases 3,845
Selling costs 1,248
Other production overheads 580
Other administration overheads 388

Following other information is available:


(i) On 1 June 2019, stock of finished goods consisted of 1,350 units valued at
Rs. 1,640 per unit while stock of raw materials was valued at Rs. 1,490,000.
(ii) 5,200 units of finished goods were produced during June 2019.
(iii) There was no work-in-progress at the end of the month whereas work in progress at
1 June 2019 was valued at Rs. 208,000.
(iv) Stock of raw materials on 30 June 2019 was valued at Rs. 970,000.
(v) 1,500 units of finished goods were available in stock as on 30 June 2019.
(vi) Cost of finished goods is determined using FIFO method.

Required:
Compute cost of goods sold for the month of June 2019. (07)

Q.4 Select the most appropriate answer(s) from the options available for each of the following
Multiple Choice Questions (MCQs).

(i) An entity made a profit of Rs. 480,000 for the year 2018 based on historical cost
accounting principles. It had opening capital of Rs. 1,100,000. During 2018, specific
price indices increased by 15% while general price indices increased by 12%. How
much profit should be recorded for 2018 under real financial capital maintenance
concept?
(a) Rs. 480,000 (b) Rs. 315,000
(c) Rs. 348,000 (d) Rs. 645,000 (01)

(ii) Morning Football Club has a monthly subscription fee of Rs. 800 per member. The
club has 240 members on 31 December 2018. No fresh members were admitted
during 2018 but 30 members left the club on 1 July 2018. As at 31 December 2018,
the club has received subscription in advance amounting to Rs. 60,000. The club’s
subscription income for 2018 would be:
(a) Rs. 2,448,000 (b) Rs. 2,388,000
(c) Rs. 2,160,000 (d) Rs. 2,100,000 (02)

(iii) Which of the following can NOT be a ‘qualifying asset’ under IAS 23 Borrowing
Costs?
(a) Inventories
(b) Manufacturing plants
(c) Assets that are ready for their intended use when acquired
(d) Investment property (01)
Financial Accounting and Reporting-I Page 3 of 7

(iv) Afternoon Limited (AL) uses cost model for its property, plant and equipment and
fair value model for its investment property. AL has an office building which was
being used for administrative purposes. At 1 July 2018, the building had a carrying
amount of Rs. 20 million. On that date, the building was let out to a third party and
therefore reclassified as an investment property. The building had a fair value of
Rs. 23 million on 1 July 2018 and Rs. 23.4 million on 30 June 2019.
What would be the increase in the profit or loss and other comprehensive income for
the year ended 30 June 2019?
Profit or loss Other comprehensive income
(a) Nil Rs. 3.4 million
(b) Rs. 0.4 million Rs. 3 million
(c) Rs. 3.4 million Nil
(d) Rs. 3 million Rs. 0.4 million (02)

(v) Which TWO of the following fall under the definition of investment property?
(a) Property occupied by an employee
(b) A building owned by an entity and leased out under an operating lease
(c) Property being constructed on behalf of third party
(d) Land held for long term appreciation (01)

(vi) Under IAS 36 Impairment of Assets, if the fair value less costs to sell of an asset
cannot be determined then:
(a) the asset is not impaired
(b) the recoverable amount is the value in use
(c) the net realizable value is used
(d) the carrying value of the asset remains the same (01)

(vii) Which TWO of the following would be external indicators that one or more of an
entity's assets may be impaired?
(a) An unusually significant fall in the market value of one or more assets
(b) Evidence of obsolescence of one or more assets
(c) A decline in the economic performance of one or more assets
(d) An increase in market interest rates used to calculate value in use of the assets (01)

(viii) Which of the following future cash flows should NOT be included in the calculation
of value in use of an asset?
(a) Cash flows from disposal
(b) Income tax payments
(c) Cash flows from the sale of inventory produced by the asset
(d) Cash outflows on the maintenance of the asset (01)

(ix) Night Limited has a current ratio of 1.8. This ratio will increase if Night Limited:
(a) receives cash in respect of a short term loan
(b) receives cash from an existing receivable
(c) pays an existing trade payable
(d) purchases inventory on credit (01)

(x) A debtor turnover of 6 times means that:


(a) one-sixth of the debtors are collected in one month
(b) average debtors are collected in 6 months
(c) average debtors are collected in 2 months
(d) average debtors are collected 2 times in a year (01)
Financial Accounting and Reporting-I Page 4 of 7

Section B

Q.5 Following are the extracts from the financial statements of Sunday Traders Limited (STL)
for the year ended 30 June 2019:

Statement of financial position as on 30 June 2019


2019 2018 2019 2018
Assets Equity & liabilities
Rs. in million Rs. in million
Property, plant and equipment 8,555 7,240 Share capital (Rs. 100 each) 4,650 3,450
Investment property 1,800 1,120 Share premium 1,600 1,240
Stock in trade 4,800 4,500 Retained earnings 1,652 (655)
Prepayments 184 268 Long term loans 6,024 6,523
Trade receivables 3,800 3,600 Trade payables 3,422 5,390
Cash 194 480 Contract liability 250 40
Accrued liabilities 310 180
Interest payable 135 110
Current maturity of long
term loans 850 700
Provision for taxation 440 230
19,333 17,208 19,333 17,208

Statement of profit or loss for the year ended 30 June 2019


Rs. in million
Sales 29,700
Cost of sales (15,750)
Gross profit 13,950
Distribution cost (6,185)
Administrative cost (2,302)
Other income 404
Profit before interest and tax 5,867
Interest expense (1,210)
Profit before tax 4,657
Tax expense (1,150)
Profit after tax 3,507

Additional information:
(i) 72% of sales were made on credit.
(ii) Depreciation expense for the year amounted to Rs. 750 million which was charged to
distribution and administrative cost in the ratio of 3:1.
(iii) Distribution cost includes:
 Rs. 40 million in respect of loss on disposal of equipment. The written down
value at the time of disposal was Rs. 152 million.
 impairment loss on vehicles amounting to Rs. 24 million.

(iv) Loan instalments (including interest) of Rs. 1,984 million were paid during the year.
(v) Other income comprises of:
 increase in fair value of investment property amounting to Rs. 220 million.
 rent received from investment property amounting to Rs. 184 million.

(vi) During the year, STL issued right shares at premium.

Required:
Prepare STL’s statement of cash flows for the year ended 30 June 2019 using direct method. (19)
Financial Accounting and Reporting-I Page 5 of 7

Q.6 The following information pertains to Monday Limited (ML):

(i) The balances of property, plant and equipment as on 1 January 2018:


Cost/Revalued Accumulated
Assets amount depreciation
----------- Rs. in million -----------
Office building 240 36
Equipment 190 60

Revaluation surplus related to the office building as at 1 January 2018 amounted to


Rs. 8.5 million.

(ii) On 1 September 2018, a new equipment was acquired by making payment of


Rs. 70 million to the supplier. An old equipment was also given in exchange to the
supplier. The fair values of the old and new equipment were assessed at
Rs. 21 million and Rs. 93 million respectively. The old equipment had been acquired
at a cost of Rs. 40 million on 1 July 2016. Cost incurred on installing the new
equipment amounted to Rs. 5 million.

(iii) On 1 January 2018, ML commenced construction of a manufacturing plant. The


whole process of assembling and installation was completed on 31 October 2018.
However, the work was stopped from 16 to 31 August 2018 due to unexpected rains.

The total cost of Rs. 660 million incurred on the plant was paid as under:
Description Payment date Rs. in million
1st payment 1 February 2018 140
2nd payment 1 April 2018 214
3rd payment 1 September 2018 146
4th payment 1 December 2018 160

The plant was financed through a bank loan of Rs. 500 million obtained on
1 March 2018. The loan carries a mark-up of 18% payable annually. The surplus
funds available from the loan were invested in a saving account and earned
Rs. 17 million during capitalization period.

(iv) On 31 December 2018, the revalued amount of office building was assessed at
Rs. 178 million by Precise Valuers, an independent valuation firm. Value in use of
the office building as at 31 December 2018 was estimated at Rs. 186 million.

(v) Other relevant details are as follows:


Depreciation Subsequent
Assets Life/rate
method measurement
Office building Straight line 20 years* Revaluation
Equipment Reducing balance 20% Cost
Manufacturing plant Straight line 15 years Cost
* Remaining life at the date of last revaluation

ML accounts for revaluation on net replacement value method and transfers the
maximum possible amount from revaluation surplus to retained earnings on an
annual basis.

Required:
In accordance with IFRSs, prepare a note on ‘Property plant and equipment’ for inclusion
in ML’s financial statements for the year ended 31 December 2018.
(Comparatives figures and column for total are not required) (17)
Financial Accounting and Reporting-I Page 6 of 7

Q.7 Friday Traders (FT) is engaged in the business of supplying Blenders and Juicers. FT
purchases its products from Sigma Electronics. FT is presently negotiating with a bank for a
long term loan and has been asked to provide the latest financial statements. Since FT does
not maintain proper accounting records, you are requested to prepare the financial
statements from the following information:

(i) Assets and liabilities as on 1 January 2018:


Rs. in '000
Equipment (40% depreciated) 2,490
Stock (stock value of Blenders was double of the Juicers) 3,705
Prepaid rent up to 30 April 2018 280
Trade debtors (only for Blenders) 1,410
Payable to Sigma Electronics 3,600
Salaries payable 98
Bank overdraft 740

(ii) Sales of Blenders are made on credit while Juicers are sold on cash basis.
(iii) Upto last year, FT was earning a gross profit of 30% on cost of Blenders and 35% on
sale value of Juicers. With effect from 1 January 2018:
 FT increased sales prices of both the products by 20%; and
 Sigma Electronics increased the prices of Juicers only by 40%.

(iv) 60% of the amount of purchases made during the year represents blenders.
(v) Summary of bank transactions during the year:
Rs. in '000
Receipts from credit customers 6,570
Payments:
Sigma Electronics 8,850
Insurance for one year starting 1 February 2018 204
Rent 826
Equipment 550
Salaries and wages 685
11,115

(vi) Debtors amounting to Rs. 138,000 are considered as irrecoverable.


(vii) Rent of the premises was increased by 30% with effect from 1 September 2018.
(viii) Following payments were made from cash sales and remaining amounts were
deposited into the bank:
Rs. in '000
Repairs and maintenance 186
Salaries and wages 124
Drawings 477
787

(ix) Equipment is depreciated at 8% on cost.


(x) Some balances ascertained as at 31 December 2018:
Rs. in '000
Stock* – Juicers 975
– Blenders 2,597
Payable to Sigma Electronics 2,420
Salaries payable 134
*Comprises of stock purchased in 2018

Required:
(a) Prepare statement of profit or loss account for the year ended 31 December 2018. (10)
(b) Prepare statement of financial position as at 31 December 2018. (08)
Financial Accounting and Reporting-I Page 7 of 7

Q.8 Thursday Enterprise (TE) is a supplier of product Zee and has provided you the following
information:

(a) On 1 August 2018, TE entered into a six months contract with customer Alpha for
sale of Zee for Rs. 250 per unit, under the following terms and conditions:
 if Alpha purchases more than 5,000 units during the contract period, the price
per unit would be retrospectively reduced to Rs. 215 per unit.
 TE’s unconditional right to receive consideration would be established upon:
− completion of quality control procedures by Alpha for the first order. The
procedure would take a week after receiving the goods.
− placement of order by Alpha for subsequent orders.

At the inception of the contract, TE concludes that Alpha’s purchases will not exceed
the 5,000 units threshold for the discount.

Alpha placed the following orders:

Delivery date
Order date Units Payment date
(Transfer of control)
10 August 2018 3,000 28 August 2018 12 September 2018
25 December 2018 4,000 15 January 2019 10 January 2019 (10)

(b) On 1 February 2019, TE entered into a six months contract with another customer
Beta for sale of Zee for Rs. 250 per unit, under the following terms and conditions:
 if the Beta purchases more than 15,000 units during the contract period, the price
per unit would be retrospectively reduced to Rs. 215 per unit.
 TE’s unconditional right to receive consideration would be established upon
delivery of goods to Beta.

At the inception of the contract, TE concludes that Beta will meet 15,000 units
threshold for the discount.
Beta placed the following orders:

Delivery date
Order date Units Payment date
(Transfer of control)
14 February 2019 10,000 28 February 2019 20 March 2019
1 June 2019 8,000 15 July 2019 18 July 2019 (05)

Required:
In respect of the above contracts, prepare journal entries to be recorded in the books of TE
for the years ended 31 December 2018 and 2019.
(Entries without date will not be awarded any marks)

(THE END)
Financial Accounting and Reporting-I
Suggested Answers
Certificate in Accounting and Finance – Autumn 2019

A.1 Wednesday Limited


Statement of changes in equity
For the year ended 30 June 2019
Share Share Retained Revaluation
capital premium earnings surplus
--------------- Rs. in million ---------------
Balance as at 1 July 2018 (As given) 200 85 124 65
Final cash dividend @ 18% for 2018
(200×18%) (36)
Right issue @ 30% 60 72
(200×30%) (0.6×120)
Interim bonus dividend @ 10% for 2019
(260×10%) 26 (26)
Total comprehensive income for the year:
Profit for the year 95
Other comprehensive income 35
Transfer of incremental depreciation 9 (9)
Balance as at 30 June 2019 286 157 166 91

A.2 (i) The grant has been provided for the purpose of giving immediate financial support to
the entity with no further conditions, so this grant should be immediately recognised
in profit or loss in full in the period in which the entity qualifies to receive it (when it is
receivable) with disclosure to ensure that its effect is clearly understood.

(ii) Since there is reasonable assurance that conditions attaching to the grant will be met,
the grant is recognised in statement of profit or loss over the four year period in which
the entity incurs the costs of employing 100 people. Amount taken to statement of
profit or loss may be either be presented as other income or shown as deduction from
the related expense. The remaining amount of grant will be presented as deferred
income under liabilities in the balance sheet.

(iii) Free technical advice is government assistance that cannot reasonably have a value
placed upon it and therefore should not be recognised. However, an indication of such
assistance should be disclosed in financial statements.

Page 1 of 7
Financial Accounting and Reporting-I
Suggested Answers
Certificate in Accounting and Finance – Autumn 2019

A.3 Tuesday Manufacturers Limited


Cost of goods sold
For the month of June 2019
Rs. in ‘000
Opening finished goods (1,350×1,640) 2,214
Cost of goods manufactured (W-1) 9,048
Closing finished goods [1,500×(9,048,000÷5,200)] (2,610)
Cost of goods sold 8,652

W-1: Cost of goods manufactured Rs. in ‘000


Raw Material:
Opening raw material 1,490
Raw material purchases 3,845
Closing raw material (970)
Raw material consumed 4,365
Direct labour 2,075
Prime cost 6,440
Overheads:
Depreciation on plant and machinery 380
Factory manager’s salary 247
Indirect labour 848
Indirect material consumed 345
Other production overheads 580
2,400
Total manufacturing cost 8,840
Opening work in progress 208
Closing work in progress -
Cost of goods manufactured 9,048

A.4 (i) (c) Rs. 348,000


(ii) (a) Rs. 2,448,000
(iii) (c) Assets that are ready for their intended use when acquired
(iv) (b) Profit or loss Other comprehensive income
Rs. 0.4 million Rs. 3 million
(v) (b) & (d) [A building owned by an entity and leased out under an operating lease]
& [Land held for long term appreciation]
(vi) (b) the recoverable amount is the value in use
(vii) (a) & (d) [An unusually significant fall in the market value of one or more assets]
& [An increase in market interest rates used to calculate value in use of
the assets]
(viii) (b) Income tax payments
(ix) (c) pays an existing trade payable
(x) (c) average debtors are collected in 2 months

Page 2 of 7
Financial Accounting and Reporting-I
Suggested Answers
Certificate in Accounting and Finance – Autumn 2019

A.5 Sunday Traders Limited


Statement of Cash Flows
For the year ended 30 June 2019
Cash flows from operating activities Rs. in million
Cash receipts from customers (Cash sales: 8,316 ; Credit sales: 21,394) (W-1) 29,710
Cash receipts from tenants 184
Cash paid to suppliers (W-2) (18,018)
Cash paid to other vendors (W-3) (7,459)
Cash generated from operations 4,417
Interest paid 110+1,210–135 (1,185)
Income taxes paid 230+1,150–440 (940)
Net cash inflow from operating activities 2,292

Cash flows from investing activities


Purchase of property, plant and equipment (8,555–7,240)+24+152+750 (2,241)
Proceeds from disposal of property, plant and equipment 152–40 112
Purchase of investment property (1,800–1,120)–220 (460)
Net cash outflow from investing activities (2,589)

Cash flows from financing activities


Proceeds from issue of shares (4,650–3,450)+(1,600–1,240) 1,560
Dividend paid 1,652+655–3,507 (1,200)
Repayment of loans 1,984–1,185 (799)
New loans acquired (6,024–6,523)+799+(850–700) 450
Net cash inflow from financing activities 11
Net decrease in cash and cash equivalents (286)
Cash and cash equivalent at the beginning of the year 480
Cash and cash equivalent at the end of the year 194

Workings:
W-1: Cash receipts from customers – sales Rs. in million
Sales for the year 29,700
Increase in trade receivables balances 3,600–3,800 (200)
Increase in contract liability balances 250–40 210
Cash received from customers 29,710

W-2: Cash paid to suppliers Rs. in million


Cost of sales 15,750
Increase in stock balances 4,800–4,500 300
Decrease in trade payable balances 5,390–3,422 1,968
Cash paid to suppliers 18,018

W-3: Cash paid to other vendors Rs. in million


Distribution cost 6,185
Administrative cost 2,302
Depreciation (750)
Loss on disposal (40)
Impairment (24)
Increase in accrued liabilities balances 180–310 (130)
Decrease in prepayment balances 184–268 (84)
7,459

Page 3 of 7
Financial Accounting and Reporting-I
Suggested Answers
Certificate in Accounting and Finance – Autumn 2019

A.6 Monday Limited


Notes to the financial statement
For the year ended 31 December 2018
Property, plant and equipment:
Office building Equipment Plant
------------------ Rs. in million ------------------
Gross carrying amount – opening 240.00 190.00 -
Accumulated depreciation (36.00) (60.00) -
Opening carrying amount 204.00 130.00 -

Additions 96.00 699.25


(70+21+5) (W-3)
Depreciation for the year (12.00) (30.48) (7.77)
(W-2) (9699.25÷15×2÷
(240÷20) 12)
Disposal (24.96)
(W-1)
Revaluation
- Surplus (8.00)
8.5(8.5÷17)
- P&L (6.00)
Closing carrying amount 178.00 170.56 691.48

Gross carrying amount – closing 178.00 246.00 699.25


Accumulated depreciation - (75.44) (7.77)
Closing carrying amount 178.00 170.56 691.48

Office building Equipment Plant


Measurement base Revaluation model Cost model Cost model
Useful life (years)/depreciation rate
20 years 20% 15 years
%
Depreciation method Straight line Reducing balance Straight line
The last revaluation was performed on 31 December 2018 by Precise Valuers, an independent firm of
valuers.
Carrying value of building, had the cost model been used instead (178+6) 184
W-1: Disposal Rs. in million
2016 40×20%×6÷12 4.00
2017 36×20% 7.20
2018 28.8×20%×8÷12 3.84
Accumulated depreciation 15.04

Book value (4015.04) 24.96

W-2: Depreciation for the year – equipment Rs. in million


Disposal 28.8×20%×8÷12 3.84
Others (13028.8)×20% 20.24
Addition 96×20%×4÷12 6.40
30.48

W-3: Cost of plant Rs. in million


Payments 660.00
Borrowing cost:
Interest cost 500×18%×7.5÷12 56.25

Page 4 of 7
Financial Accounting and Reporting-I
Suggested Answers
Certificate in Accounting and Finance – Autumn 2019

Investment income (17.00)


699.25

A.7 Friday Traders


(a) Statement of profit or loss for the year ended 31 December 2018
Juicers Blenders
-------- Rs. in '000 --------
Sales
Credit sales - Blenders 4,475×156÷100(W-1) 6,981
Cash sales - Juicers Opening stock: 1,235×120÷65(W-1) 2,280
Rem.: 2,093(3,328-1,235)×120÷91(W-1) 2,760
5,040 6,981
Cost of goods sold:
Opening stock 1,235 2,470
Purchases 7,670(W-3) in 40:60 3,068 4,602
Closing stock (975) (2,597)
(3,328) (4,475)
Gross profit 1,712 2,506
Total gross profit 4,218

Operating expenses:
Insurance 204×11÷12 187
Rent (70×8)+(91×4) 924
Repair 186
Bad debts written off 138
Salary (124+685)+(134–98) 845
Depreciation - equipment (2,490÷0.6×8%)+(550×8%) 376
(2,656)
Net profit 1,562
W-1: POLICIES
Blenders Juicers
Updated Updated with
Previous Updated Previous
with sales sales & cost
(130×1.2) (100×1.2)1
Sales
130 156 100 20 120
Cost 100 100 65 65 (65×1.4) 91
Profit 30 56 35 55 29

W-2: Trade debtors – gross Rs. in '000


Opening balance 1,410 Receipts 6,570
Sales 6,981 Write off 138
Closing (balancing) 1,683
8,391 8,391

W-3: Trade payables (Sigma Electronics) Rs. in '000


Bank 8,850 Opening 3,600
Closing 2,420 Purchases (Bal) 7,670
11,270 11,270

Friday Traders

Page 5 of 7
Financial Accounting and Reporting-I
Suggested Answers
Certificate in Accounting and Finance – Autumn 2019

(b)
Statement of financial position as on 31 December 2018
Rs. in '000
Assets
Non-current assets
Equipment 2,490+550–376 2,664
Current assets:
Stock 975+2,597 3,572
Trade debtors (W-2) 1,683
Prepaid rent 280+826–924 182
Prepaid insurance 204–187 17
5,454
8,118
Equity and liabilities:
Opening capital 2,490+3,705+280+1,410–3,600–98–740 3,447
Net profit 1,562
Drawings (477)
4,532
Current liabilities
Trade payables (W-3) 2,420
Bank overdraft (W-4) 1,032
Salary payable 134
3,586
8,118

W-4 Bank Rs. in '000


Receipts from trade Opening balance
debtors 6,570 740
Amount banked 4,253 Trade payables 8,850
5,040-186-124-477 Insurance 204
Rent 826
Equipment 550
Closing balance 1,032 Salaries & wages 685
11,855 11,855

A.8 Thursday Enterprise

(a) General Journal


Debit Credit
Date Description
---------- Rupees ----------
28-08-18 Contract asset – Alpha (3,000×250) 750,000
Revenues 750,000

05-09-18 Receivable - Alpha 750,000


Contract asset - Alpha 750,000

12-09-18 Cash/Bank 750,000


Receivable - Alpha 750,000

25-12-18 Revenue (3,000×35) 105,000


Contract liability / Refund liability – Alpha 105,000

Page 6 of 7
Financial Accounting and Reporting-I
Suggested Answers
Certificate in Accounting and Finance – Autumn 2019

25-12-18 Receivable (4,000×215)–105,000 755,000


Contract liability / Refund liability – Alpha 755,000

10-01-19 Cash/Bank 755,000


Receivable - Alpha 755,000

15-01-19 Contract liability / Refund liability – Alpha 860,000


Revenue (755,000+105,000) 860,000

(b) General Journal

Debit Credit
Date Description
---------- Rupees ----------
28-02-19 Receivable – Beta (10,000×250) 2,500,000
Revenue (10,000×215) 2,150,000
Contract liability / Refund liability – Beta 350,000

20-03-19 Cash/Bank 2,500,000


Receivable – Beta 2,500,000

15-07-19 Receivable – Beta 1,370,000


Contract liability / Refund liability – Beta 350,000
Revenue 1,720,000

18-07-19 Cash/Bank 1,370,000


Receivable – Beta 1,370,000

(THE END)

Page 7 of 7
THE INSTITUTE OF CHARTERED ACCOUNTANTS OF PAKISTAN

EXAMINERS’ COMMENTS

SUBJECT SESSION
Financial Accounting & Certificate in Accounting and Finance (CAF)
Reporting I Examination - Autumn 2019

Passing %

Question-wise
Overall
1 2 3 4 5 6 7 8
76% 35% 74% 53% 50% 20% 42% 13% 39%

General comments

An overall passing ratio of 39% stands between the last two results of 29% and 57%
respectively. 19% examinees were just short of 9 or fewer marks and could have easily
obtained them if they have covered all areas of the syllabus. The highest score in the
paper was 94 marks.

Performance in Q6 and Q8 was poor. Q6 (IAS 16) has a higher difficulty level as
compared to previously examined questions on this topic at FAR 1 due to recent changes
in syllabus. Examinees who only focused on previously examined questions faced
difficulty in attempting the question. Poor performance in Q8 (IFRS 15) was mainly due
to the inherent difficulty (newness) of the topic.

Although many examinees performed well, some shortcomings such as lack of practice,
poor presentation, etc. were commonly noted in most scripts. Many examinees secured
good marks in two to three questions but failed to obtain reasonable marks in the
remaining questions.

It has been observed that examinees often spend extra time on completing a question
which affect their performance in the other questions. Examinees are therefore strongly
advised to move to the next question after they have spent reasonable time on a particular
question. This would help them to attempt all questions of the paper.

Question-wise common mistakes observed

Question 1

 Amount of premium in right issue was taken at Rs. 12 million instead of Rs. 72
million.

Page 1 of 3
Examiners’ Comments on Financial Accounting & Reporting I – CAF Examination
Autumn 2019

 Several mistakes were made in presenting relevant dividends in the statement for the
year ended 30 June 2019. Please refer ICAP’s suggested solution for correct
presentation of relevant dividends.

Question 2

 In (i), grant was considered as related to asset.


 In (ii), answers were correct to the extent discussed but lacked completeness.

Question 3

Average cost of closing stock was calculated incorrectly and was often stated at Rs. 1,640
which was the cost of opening finished goods.

Question 4

MCQ numbers (i), (ii), and (x) were least well answered.

Question 5

 Few examinees prepared the statement of cash flow by using indirect method.
 Effects of contract liability and/or cash sales were not considered while calculating
receipts from customers.
 Receipt of rent was not presented anywhere in the statement.
 While calculating amount paid for additions to property, plant and equipment,
amount of impairment was ignored.
 While calculating amount paid for additions to investment property, increase in fair
value was ignored.
 Dividend paid was not computed and/or presented.
 Repayment of loan did not include effect of current maturity.

Question 6

 Amount of additions to equipment was calculated on the basis of fair value of the
new equipment instead of fair value of the old equipment.
 Adjustment of revaluation of building was not properly presented and/or incorrectly
bifurcated into revaluation surplus and profit or loss.
 Schedule of property, plant and equipment was often incomplete in terms of
presentation.
 Carrying value of building if the cost model had been used was incorrectly
calculated.
 Correct capitalization period of 7.5 months was not taken into account for borrowing
cost on manufacturing plant. Further, lengthy calculations were made for calculating
borrowing cost to be capitalized which were not required as it was the case of
‘Specific borrowings’.

Page 2 of 3
Examiners’ Comments on Financial Accounting & Reporting I – CAF Examination
Autumn 2019

Question 7

 Several types of mistakes were made in calculating sales and often were not
computed at all. Please refer ICAP’s suggested solution for correct calculation.
 Rent expense and related prepaid were incorrectly calculated.
 Depreciation of old equipment was calculated on book value.

Question 8

 Examinees could not differentiate between receivable and contract asset.


 In part (a), reduction in revenue due to placement of second order by Alpha was not
made.
 In part (b), revenue was recognized @ Rs. 250 instead of Rs. 215 upon first order by
Beta.

(THE END)

Page 3 of 3
Financial Accounting and Reporting - I
Summary of Marking Key
Certificate in Accounting and Finance – Autumn 2019

Note regarding marking scheme:


The marking scheme is given as a guide. Markers also award marks for alternative approaches to a
question and relevant/well-reasoned comments/explanations. Moreover, the available marks in
answer may exceed the total marks of a question.

Mark(s)
A.1 Statement of changes in equity
 Opening and closing balances 1.0
 Dividends 2.0
 Right issue 1.0
 Total comprehensive income 2.0
 Incremental depreciation 1.0

A.2 Discussion on:


(a) Grant for general financing requirement 1.0
(b) Grant for employing 100 persons:
 Accounting treatment 1.5
 Presentation and disclosure 1.5
(c) Free technical advice 1.0

A.3  Prime cost 1.5


 Production overheads 3.0
 Closing finished goods 1.5
 Other items 1.0

A.4 Marks as mentioned on the question paper against each MCQ 12.0

A.5  Operating activities


− Receipts from customers 3.0
− Payments to suppliers 2.0
− Payment to other vendors 4.0
− Interest and taxes 2.0
 Financing activities
− Property, plant and equipment 2.0
− Investment property 2.0
 Investing activities
− Dividend and issue of shares 2.0
− New loans and repayment of loans 2.0

A.6  Appropriate format of the schedule 2.0


 Additions during the year 5.0
 Disposals during the year 1.0
 Depreciation for the year 4.0
 Revaluation adjustments 2.0
 Other disclosures 3.0

Page 1 of 2
Financial Accounting and Reporting - I
Summary of Marking Key
Certificate in Accounting and Finance – Autumn 2019

Mark(s)
A.7 (a) Statement of profit or loss
 Sales 4.0
 Cost of goods sold 2.0
 Operating expenses 4.0

(b) Statement of financial position


 Non-current assets (including depreciation) 2.0
 Current assets 2.0
 Equity 1.5
 Current liabilities 2.5

A.8 (a)  Up to 01 mark for each journal entry 6.0


 Correct revenue recognition date 1.0
 Computation of correct relevant amounts 3.0

(b)  Up to 01 mark for each journal entry 2.5


 Correct revenue recognition date 1.0
 Computation of correct relevant amounts 1.5

(THE END)

Page 2 of 2
Certificate in Accounting and Finance Stage Examination

The Institute of 4 March 2020


Chartered Accountants 3 hours – 100 marks
of Pakistan Additional reading time – 15 minutes

Financial Accounting and Reporting-I


Instructions to examinees:
(i) Answer all EIGHT questions.
(ii) Answer in black pen only.

Section A

Q.1 Rakaposhi Traders (RT) was unable to retrieve complete information required to prepare its
statement of profit or loss due to a computer virus attack. In order to compute profit for the
year ended 31 December 2019, RT has gathered the following information:

(i) List of all assets and liabilities as on 1 January 2019:

Liabilities Rs. in '000 Assets Rs. in '000


Creditors 310 Furniture - net 460
Accrued rent 33 Inventories 200
Debtors 170
Cash in hand 37
Cash at bank 85
343 952

(ii) Inventories increased by 30% during the year.


(iii) Credit sales during the year amounted to Rs. 2,500,000. Collections from debtors
amounted to Rs. 2,400,000 out of which Rs. 300,000 were received in cash. A debtor’s
balance of Rs. 15,000 is irrecoverable.
(iv) Balance as per bank statement as on 31 December 2019 amounted to Rs. 90,000.
However, it does not include a cheque of Rs. 40,000 deposited on 31 December 2019.
(v) Following information has been collected from the counterfoils of cheque books:

Rs. in '000
Payment to creditors 1,375
Drawings 275
Salaries 600
Cash withdrawn for office use 120

(vi) Cash in hand as at 31 December 2019 amounted to Rs. 50,000. Details of cash sales
and cash payments (expenses, payment to creditors and cash purchases) are not
available.
(vii) On 1 April 2019, the owner brought into the business a vehicle having a market value
of Rs. 360,000.
(viii) Creditors’ closing balance of Rs. 425,000 was determined from account statements
obtained from the creditors.
(ix) Rent amounting to Rs. 23,000 was outstanding as on 31 December 2019.
(x) Depreciation is charged at 10% on fixed assets.

Required:
Compute the net profit or net loss for the year ended 31 December 2019. (08)
Financial Accounting and Reporting-I Page 2 of 6

Q.2 You are working as Finance Manager in Broad Peak Limited (BPL). Faraz has recently
joined BPL as an internee for three months. You have asked him to develop an
understanding of the statement of cash flows. After going through few statements, he has
raised the following queries:

(i) Depreciation is not a cash flow but was still appearing as an addition in the statement
of cash flows.
(ii) In the statement of cash flows of a competitor, interest paid was shown as a financing
activity but BPL showed it in operating activities.
(iii) BPL purchased inventories throughout the year but total purchases of inventory were
not shown in the statement. However, only decrease in inventory was added.
(iv) Cash and bank balance in the statement of financial position was not in agreement
with the opening and closing balances at the end of statement of cash flows.

Required:
Briefly answer the queries raised by Faraz. (08)

Q.3 Briefly describe the measurement bases that may be used to measure the value of assets in
the financial statements. (06)

Q.4 Select the most appropriate answer(s) from the options available for each of the following
Multiple Choice Questions (MCQs).

(i) Which of the following companies is most likely to face cash flow problems?
(a) A loss making government organisation
(b) A company which has recently sold part of its operations so as to concentrate on
its core areas
(c) A reasonably profitable and long established company with no expansion plans
(d) A profitable retailer about to embark on ambitious expansion plans (01)

(ii) A plant has a carrying amount of Rs. 1,500,000 as at 31 December 2019. Its fair value
is Rs. 900,000 and costs of disposal are estimated at Rs. 50,000. A new plant would
cost Rs. 2,500,000. Cash flows from the plant for the next four years are estimated at
Rs. 350,000 per annum. Applicable discount rate is 10%.

What is the approximate impairment loss on the plant to be recognised in the financial
statements as at 31 December 2019?
(a) Rs. 650,000 (b) Rs. 390,000
(c) Rs. 1,000,000 (d) Nil (02)

(iii) A debit balance on the retained earnings account indicates that the company has:
(a) made more dividend payments than the profit earned
(b) redeemed some of its share capital
(c) accumulated losses
(d) issued bonus shares (01)

(iv) The correct accounting treatment of initial operating losses incurred during the
commercial production due to under-utilization of the plant would be to:
(a) capitalise as a directly attributable cost
(b) defer and charge to profit or loss account when profit is earned from the plant
(c) charge directly to retained earnings since these are not considered to be normal
operating losses
(d) charge to profit or loss account (01)
Financial Accounting and Reporting-I Page 3 of 6

(v) A manufacturing company has four types of cost (identified as A, B, C and D). The
total cost of each type at two different production levels is:

Total cost for 100 units Total cost for 150 units
Cost type
----------------------- Rupees -----------------------
A 1,500 2,250
B 1,800 2,400
C 2,000 3,000
D 3,000 4,200

Which two types of cost would be classified as semi-variable?


(a) A and C (b) A and D
(c) B and C (d) B and D (02)

(vi) In measuring value in use, the discount rate used for discounting the cash flows
should be the:
(a) pre-tax rate that reflects the market assessment of time value of money and risks
specific to the asset
(b) pre-tax rate that reflects the market assessment of time value of money and risks
specific to the entity
(c) post-tax rate that reflects the entity’s assessment of time value of money and
risks specific to the asset
(d) pre-tax rate that reflects the entity’s assessment of time value of money and risks
specific to the asset (01)

(vii) Which of the following is NOT considered as an item of property, plant and
equipment?
(a) A standby generator expected to be used for seven years
(b) A plot of land held for resale
(c) A bus for pick and drop of staff members
(d) A generator for rental to others (01)

(viii) Capitalisation of borrowing costs should be suspended:


(a) when substantially all the activities necessary to prepare a qualifying asset for its
intended use or sale are complete
(b) during a temporary delay which is a necessary part of the process of getting an
asset ready for its intended use or sale
(c) during extended periods in which active development of a qualifying asset is
interrupted
(d) all of the above (01)
Financial Accounting and Reporting-I Page 4 of 6

Section B

Q.5 Following is the trial balance of Chongtar International Hospital as on 31 December 2019:

Debit Credit
---- Rs. in million ----
Burns ward - capital work in progress 55.3
Cafeteria sales 24.4
Cash and bank balances 8.4
Donations for burns ward 75.1
Expenses and gifts for ‘walk on diabetes day’ 2.6
Fees from patients 125.0
General donations 82.6
General fund 195.6
Inventory - cafeteria 4.7
Inventory - medicines 19.4
Inventory - hospital supplies 8.5
Medical equipment 185.4 64.2
Miscellaneous expenses 8.5
Other fixed assets 110.7 54.7
Payables 38.9
Purchases - cafeteria 16.4
Purchases - medicines 60.5
Purchases - hospital supplies 18.7
Receivables - panel corporates 31.4
Rent 19.6
Sponsorship for ‘walk on diabetes day’ 2.2
Salaries - administrative staff 24.0
Salaries - doctors and nursing staff 38.2
Short term investments 38.0
Utilities 12.4
662.7 662.7

Additional information:
(i) Cost of closing physical inventory of medicines and hospital supplies was
Rs. 25.8 million and Rs. 13.8 million respectively. Medicines costing Rs. 3.1 million
were found expired. Medicines are only used to treat the admitted patients and are not
sold separately.
(ii) Year-end physical count of cafeteria inventory could not take place. Goods are sold in
cafeteria at a gross margin of 25% on sales.
(iii) Rent outstanding at year-end was Rs. 1.4 million.
(iv) 15% of salaries and 10% of rent are related to cafeteria.
(v) Hospital facilities of Rs. 48.6 million were provided free of charge to the patients.
(vi) ‘Walk on diabetes day’ was organised in December 2019. Expenses relating to the
event amounting to Rs. 1.2 million were outstanding and unrecorded at year end.
(vii) Medical equipment having fair value of Rs. 36.8 million were received as donation.
These have been brought into use but have not been recorded in the books.
(viii) Depreciation is charged on reducing balance method at 15% per annum.

Required:
(a) Prepare income and expenditure account for the year ended 31 December 2019 (12)
(b) Prepare statement of financial position as on 31 December 2019 (06)
Financial Accounting and Reporting-I Page 5 of 6

Q.6 Following are the summarised financial statements of Shispare Limited (SL) and its
competitor Trivor Limited (TL) for the year ended 31 December 2019:

Statement of financial position


SL TL SL TL
Assets Equity & liabilities
Rs. in million Rs. in million
Fixed assets 5,400 7,800 Capital and reserves 8,400 9,450
Current assets: Long-term loan 1,900 4,600
Inventory 4,800 7,100 Current liabilities:
Debtors 2,700 3,200 Creditors 2,900 4,500
Cash 1,200 800 Accrued expenses 900 350
8,700 11,100 3,800 4,850
14,100 18,900 14,100 18,900

Statement of profit or loss


SL TL
--- Rs. in million ---
Sales 16,700 35,400
Cost of goods sold (11,400) (27,800)
Gross profit 5,300 7,600
Operating expenses (3,500) (4,900)
Finance cost (250) (600)
Net profit 1,550 2,100

Required:
Compute relevant ratios for SL and TL to assess which company seems to:
(i) give more incentives to its customers to pay on time
(ii) avail extended credit terms from its suppliers
(iii) be more efficient in the use of capital
(iv) keep lower selling prices to gain the market share
(v) have better liquidity position
(vi) have higher ability to convert its assets into profit
(vii) control operating expenses more efficiently
(viii) have higher ability to raise bank loan in future (16)

Q.7 Financial statements of Trich Mir Limited (TML) for the year ended 31 December 2019 are
under preparation. While reviewing revenues from contract with customers, following
matters have been identified:
(i) On 1 October 2019, TML sold Machine C to Chan Limited for Rs. 25 million. As per
the contract, payment would be made after 2 years. The accountant recognised sales
revenue of Rs. 25 million upon delivery on 1 October 2019. Further, commission paid
to sales employees for winning the contract of Rs. 1.6 million was capitalised and is
being amortised over 2 years period. Applicable discount rate is 10% per annum.
(ii) TML entered into a contract to manufacture a specialised machine for Dhan Limited
at a price of Rs. 30 million. The contract meets the criteria of recognition of revenue
over time. At the year end, the machine was 60% complete and it was estimated that a
further cost of Rs. 10 million would be incurred. Cost of Rs. 15 million incurred till
year end has been included in closing inventory and receipts of Rs. 11 million have
been credited to revenues.
(iii) TML entered into a contract to sell one unit of Machine A and Machine B for a total
price of Rs. 16 million. Machine A was delivered in December 2019 to the customer
while Machine B was delivered in January 2020. The consideration of Rs. 16 million
is due only after TML transfers both the machines to the customer. TML sells
machines A and B at standalone prices of Rs. 12 million and Rs. 8 million
respectively. The accountant recognised receivable and revenue of Rs. 12 million upon
delivery of Machine A.
Financial Accounting and Reporting-I Page 6 of 6

Required:
Prepare correcting entries for the year ended 31 December 2019 in accordance with IFRS 15
‘Revenue from Contracts with Customers’. (14)

Q.8 Following information pertains to non-current assets of Distaghil Limited (DL):

(i) DL purchased specialised vehicles for Rs. 370 million on 1 July 2017. The vehicles
have an estimated useful life of 10 years with residual value of Rs. 30 million.

The revalued amounts of the vehicle as at 31 December 2018 and 2019 were
determined at Rs. 302 million and Rs. 290 million respectively. There was no change
in useful life or residual value.

(ii) DL setup a manufacturing plant in a remote area at a cost of Rs. 280 million. The
plant had a useful life of 8 years. The plant was purchased on 1 January 2018 and was
available for use on 1 April 2018. The commercial production started on 1 June 2018.

On 1 July 2018, DL received a government grant of Rs. 120 million towards the cost
of the plant. The sanction letter states that if DL ceases to use the plant in the remote
area before 31 December 2021, DL would be required to repay the grant in full.

(iii) A warehouse was given on rent on 1 January 2018. Previously, the warehouse was in
use of DL.

On 1 January 2018, carrying value and remaining useful life of the warehouse was
Rs. 80 million and 16 years respectively. Fair value of the warehouse on various dates
are as follows:
Rs. in million
01 January 2018 104
31 December 2018 96
31 December 2019 115

Other information:
 DL uses cost model for subsequent measurement of property, plant and equipment
except for specialised vehicles for which revaluation model is used.
 DL transfers the maximum possible amount from the revaluation surplus to retained
earnings on an annual basis.
 Government grant is recorded as deferred income and a part of it is transferred to
income each year.
 Investment property is carried at fair value model.

Required:
Prepare relevant extracts from DL’s statement of profit or loss and other comprehensive
income for the year ended 31 December 2019 and statement of financial position as on that
date. (Show comparative figures) (20)

(THE END)
Financial Accounting and Reporting-I
Suggested Answers
Certificate in Accounting and Finance – Spring 2020

A.1 Rakaposhi Traders


Net profit/(loss) for the year ended 31 December 2019
Rs. in '000
Closing net assets (W-1) 994
Opening net assets 952–343 (609)
Increase in net assets 385
Drawings for the year 275
Additional investment for the year (360)
Net profit for the year 300

W-1: Closing net assets Rs. in '000


Furniture - net 460×0.90 414
Vehicle – net 360–(360×0.1×9÷12) 333
Inventories 200×1.3 260
Debtors 170+2,500–2,400–15 255
Cash at bank 90+40 130
Cash in hand 50
1,442
Creditors 425
Rent accrued 23
(448)
994

A.2 (i) A statement of cash flows begins with net profit which is arrived after deducting
depreciation expense. So to convert the net profit into net cash flow the deduction of
depreciation is reversed (i.e. added).
(ii) As per IAS 7, interest paid can be shown as either cash flow from financing activities
or cash flow from operating activities. Both classifications are correct as long as they
are consistently applied by an entity.
(iii) A statement of cash flows begins with net profit which is arrived after deducting cost
of sales. So to convert the effect of cost of goods sold into outflow for purchases of
inventory, change in inventory is adjusted i.e. increase is deducted and decrease is
added.
(iv) Statement of financial position shows cash and bank balances while the statement of
cash flows ends with cash and cash equivalents which may differ from cash and bank
balances due to existence of bank overdraft and short term investments.

A.3 (i) Historical cost


The historical cost of an asset, when it is acquired or created is the value of the cost
incurred in acquiring or creating the asset, comprising the consideration paid to
acquire or create the asset plus transaction cost.
(ii) Current value
 Current value measures provide monetary information about assets using
information updated to reflect conditions at the measurement date.
 Current value measurement bases include:
– Fair value
– Value in use for assets
– Current cost

Page 1 of 6
Financial Accounting and Reporting-I
Suggested Answers
Certificate in Accounting and Finance – Spring 2020

Fair value
Fair value is the price that would be received to sell an asset in an orderly
transaction between market participants at the measurement date. Fair value
reflects the perspective of market participants.
Value in use
Value in use is the present value of the cash flows or other economic benefit that
an entity expects to derive from the use of an asset and from its ultimate disposal.
Value in use reflect entity specific assumptions rather than assumptions by
market participants.
Current cost
The current cost of an asset is the cost of an equivalent asset at the measurement
date comprising the consideration that would be paid at the measurement date
plus the transaction cost that would be incurred at that date.
Current cost, like historical cost is an entry value; while fair value is an exit value.
However, unlike historical cost, current cost reflects conditions at the
measurement date.

A.4 The most appropriated answer(s) for Multiple Choice Questions (MCQs)
(i) (d)A profitable retailer about to embark on ambitious expansion plans.
(ii) (b)Rs. 390,000
(iii) (c)accumulated losses
(iv) (d)charge to profit or loss account
(v) (d)B and D
(vi) (a)Pre-tax rate that reflects the market assessment of time value of money and risks
specific to the asset
(vii) (b) A plot of land held for resale
(viii) (c) during extended periods in which active development of a qualifying asset is
interrupted.

A.5 Chongtar International Hospital


(a) Income and expenditure account for the year ended 31 December 2019
Rs. in million
Income:
Fees from patients 125.0
General donations 82.6+36.8 119.4
Profit from cafeteria 0.4
244.8
Expenditures:
Salaries - Administrative staff 24×0.85 20.4
Salaries - Doctors and nursing staff 38.2
Medicines used (19.4+60.5) – (25.8–3.1) 57.2
Hospital supplies used (8.5+18.7)–13.8 13.4
Rent (19.6 +1.4)×0.9 18.9
Walk on diabetes day -net (2.6+1.2)–2.2 1.6
Depreciation - Medical equipment [(185.4–64.2)+36.8]×15% 23.7
Depreciation - Other fixed assets (110.7–54.7)×15% 8.4
Utilities 12.4
Miscellaneous expenses 8.5
(202.7)
Excess of income over expenditure 42.1
Page 2 of 6
Financial Accounting and Reporting-I
Suggested Answers
Certificate in Accounting and Finance – Spring 2020

Cafeteria trading account for the year ended 31 December 2018


Rs. in million
Sales 24.4
Cost of goods sold:
Opening stock 4.7
Purchases 16.4
Closing stock Balancing (2.8)
(18.3)
Gross profit 24.4×0.25 6.1
Expenses:
Salaries 24×0.15 (3.6)
Rent (19.6+1.4)×0.1 (2.1)
Profit from cafeteria 0.4

Chongtar International Hospital


(b) Statement of financial position as on 31 December 2019
Rs. in million
Non-current assets:
Medical Equipment (185.4–64.2)+36.8–23.7 134.3
Other fixed assets (110.7–54.7–8.4) 47.6
Burns ward - CWIP 55.3
237.2
Current assets:
Cafeteria stock 2.8
Medicines stock (25.8 -3.1) 22.7
Hospital supplies 13.8
Receivables 31.4
Short term investments 38.0
Cash in hand 8.4
117.1
354.3
General funds:
Opening balance 195.6
Excess of income over expenditure 42.1
237.7

Burns ward fund 75.1

Liabilities:
Creditors 38.9
Accrued expenses 1.4+1.2 2.6
41.5
354.3

A.6 Relevant ratios SL TL


(i) Debtors collection period 59.01 days 32.99 days
Debtors 2,700 3,200
= ×365 = ×365 = ×365
Sales 16,700 35,400
TL is giving more incentives to its
customers to pay on time.
Page 3 of 6
Financial Accounting and Reporting-I
Suggested Answers
Certificate in Accounting and Finance – Spring 2020

(ii) Creditors payment period 92.85 days 59.08 days


Creditors 2,900 4,500
= ×365 = ×365 = ×365
Purchases 11,400 27,800
SL avail extended credits terms
(iii) Return on capital 17.48% 19.22%
Profit before interest 1,550+250 2,100+600
= ×100 = ×100 = ×100
Capital employed 8,400+1,900 9,450+4,600
TL is more efficient in the use of capital
(iv) Gross profit margin 31.74% 21.47%
Gross profit 5,300 7,600
= ×100 = ×100 = ×100
Sales 16,700 35,400
TL is deliberately keeping selling prices
lower to gain the market share.
(v) Current ratio 2.29 2.29
Current assets 8,700 11,100
= = =
Current liabilities 3,800 4,850
Quick ratio 1.03 0.82
Current assets-inventory 8,700 − 4,800 11,100 − 7,100
= = =
Current liabilities 3,800 4,850
SL has better liquidity position
(vi) Return on assets 12.77% 14.29%
Profit before interest 1,550+250 2,100+600
= ×100 = ×100 = ×100
Total assets 14,100 18,900
TL has higher ability to convert its assets
into profit
(vii) Operating expenses %age 20.95% 13.84%
Operating expenses 3,500 4,900
= ×100 = ×100 = ×100
Sales 16,700 35,400
TL is efficiently controlling the operating
expenses.
(viii) Gearing ratio 0.18 0.33
Debt 1,900 4,600
= = =
Debt + Equity 8,400+1,900 9,450+4,600
SL is going to raise a bank loan relatively
easily in future

A.7 Trich Mir Limited


Correcting entries for the year ended 31 December 2019

Debit Credit
S.No. Description
---- Rs. in million ----
(i) Revenues 25–20.66{25×(1.1)–2} 4.34
Receivable 4.34
Receivable 20.66×10%×(3÷12) 0.52
Interest income 0.52
Commission expense 1.60
Amortization expense 1.6÷2×3÷12 0.20
Contract cost 1.40
(ii) Cost of goods sold 15.00
Inventories 15.00
Page 4 of 6
Financial Accounting and Reporting-I
Suggested Answers
Certificate in Accounting and Finance – Spring 2020

Receivable (30×60%)–11 7.00


Construction revenues 7.00
(iii) Revenues 12–12×16÷(12+8) 2.40
Receivable 2.40
Contract asset (12–2.4) 9.60
Receivable 9.60

A.8 Distaghil Limited


Extracts from statement of financial position as on 31 December 2019
2019 2018
---- Rs. in million ----
Non-current assets:
Property, plant and equipment:
 Vehicles (W-1) 290.00 302.00
 Plant (W-2) 218.75 253.75

Investment property 115.00 96.00

Share capital and reserves:


Revaluation surplus:
 Vehicles 5.00 -
 Warehouse [2019: 22.5–(24÷16)], [2018: 24–(24÷16)] 21.00 22.50

Non-current liabilities:
Deferred government grant (W-2) 93.75 108.75

Extracts from statement of profit or loss and other comprehensive income


For the year ended 31 December 2019
2019 2018
------ Rs. in million ------
Profit or loss:
Depreciation:
 Vehicles (W-1) (32.00) (34.00)
 Plant (W-2) (35.00) (26.25)

Impairment / Revaluation loss - vehicles (W-1) - (17.00)


Revaluation gain - vehicles (W-1) 15.00
Grant income - Plant (W-2) 15.00 11.25
Change in fair value of investment property - warehouse
(2019: 115–96), (2018: 96–104) 19.00 (8.00)

Other comprehensive income:


Revaluation surplus:
 Vehicles (W-1) 5.00
 Warehouse (104–80) - 24.00

W-1: Vehicles Rs. in million


Purchased on 1 July 2017 370
Depreciation for 2017 (370–30)÷10×(6÷12) (17)
Book value on 31 December 2017 353
Page 5 of 6
Financial Accounting and Reporting-I
Suggested Answers
Certificate in Accounting and Finance – Spring 2020

Depreciation for 2018 (370–30)÷10 (34)


Book value on 31 December 2018 before revaluation 319
Revaluation loss Balancing (17)
Revalued amount on 31 December 2018 302
Depreciation for 2019 (302–30)÷8.5 (32)
Book value on 31 December 2019 before revaluation 270
Revaluation gain to P&L 17×(7.5÷8.5) 15
Revaluation surplus Balancing 5
Revalued amount on 31 December 2019 290

W-2: Plant and government grant Plant Gov. grant


-------- Rs. in million --------
Initial balance 280.00 120.00
Depreciation for 2018 (280÷8)×(9÷12) (26.25)
Grant income for 2018 (120÷8)×(9÷12) (11.25)
Balance on 31 December 2018 253.75 108.75
Depreciation for 2019 280÷8 (35.00)
Grant income for 2019 120÷8 (15.00)
Balance on 31 December 2019 218.75 93.75

(THE END)

Page 6 of 6
THE INSTITUTE OF CHARTERED ACCOUNTANTS OF PAKISTAN

EXAMINERS’ COMMENTS

SUBJECT SESSION
Financial Accounting & Certificate in Accounting and Finance (CAF)
Reporting I Examination - Spring 2020

Passing %

Question-wise
Overall
1 2 3 4 5 6 7 8
51% 49% 10% 56% 65% 63% 25% 46% 51%

General:

An overall passing ratio of 51% is much higher than previous two sessions’ results i.e.
39% and 29% respectively. The high result was mainly due to better than expected
performance in Q8 (based on newly added areas of the syllabus) which helped many
marginal cases. The highest score in the paper was 90 marks.

Performance in Q3 and Q7 was poor. Q3 (IAS 16) was based on measurement basis
which seems to have been completely overlooked by examinees. Poor performance in Q7
(IFRS 15) could be construed to the newness of the topic.

Although many students performed well, some shortcomings such as lack of practice,
poor presentation, etc. were commonly noted in most scripts. Many students seems
failing because of technique to approach the answer rather than knowledge or
understanding of the subject.

Question-wise common mistakes observed

Question 1

 Due to incomplete information for preparing a complete statement of profit or loss,


net profit or loss could only be determined using “Net assets approach”. However,
majority of the examinees prepared statement of profit or loss and wasted their
precious time.
 Vehicle invested into the business was included in closing assets but was not
deducted from increase in net assets for determining net profit.

Page 1 of 2
Examiners’ Comments on Financial Accounting & Reporting I – CAF Examination
Spring 2020

Question 2

 Answers were correct to the extent discussed but often lacked completeness.
 Answer to query (iv) was either totally incorrect or left un-attempted.

Question 3

Majority of the examinees discussed cost and revaluation model.

Question 4

MCQs at serial (iv) and (vi) were least well answered.

Question 5

 Donation for burns ward was shown in income & expenditure account.
 Medical equipment received as donation was not included in income.
 Hospital facilities provided free of charge were treated as expenditure.
 Cafeteria revenues were presented in income while cafeteria expenses were presented
in expenditure instead of presenting profit from cafeteria as a single line item.
 Statement of financial position was often left incomplete.

Question 6

 Examinees calculated ratios without mentioning the corresponding part number. In


such cases, only those examinees were awarded marks who wrote the conclusion.
 In part (v), examinees restricted the calculations to current ratio only. Since the
current ratio of both entities was equal, quick ratio was also required to be calculated
to reach to the conclusion.
 For part (iii) and (vi), examinees often calculated irrelevant ratios.

Question 7

 Examinees prepared the correct entries that should have been prepared on the
transaction date. However, the question required correcting entries to rectify the
wrong entries already made in the books.
 In part (i), correction for contract cost was ignored.
 In part (ii), entry to record cost of goods sold was not made.
 In part (iii), examinees could not differentiate between receivable and contract asset.

Question 8

 Comparative figures for 2018 were not disclosed as required by the question.
 Revaluation surplus was not presented in at least one statement.
 Adjustment of revaluation of vehicle in 2019 was bifurcated into revaluation surplus
and profit or loss with incorrect amounts.
 Grant income for 2018 was calculated for full year.

The End

Page 2 of 2
Financial Accounting and Reporting - I
Summary of Marking Key
Certificate in Accounting and Finance – Spring 2020

Note regarding marking scheme:


The marking scheme is given as a guide. Markers also award marks for alternative approaches to a
question and relevant/well-reasoned comments/explanations. Moreover, the available marks in
answer may exceed the total marks of a question.

Mark(s)
A.1  Computation of closing net assets (Up to 01 mark for each item) 5.0
 Computation of increase in net assets 1.0
 Drawings and investment for the year (01 mark each) 2.0

A.2 02 marks for explanation of each query raised 8.0

A.3 0.5 mark for identification of each measurement base and 01 mark for its explanation 6.0

A.4 Marks as mentioned on the question paper against each MCQ 10.0

A.5 (a) Computation of items pertaining to income and expenditure account:


 Income from general donations and patients’ fees 2.0
 Profit from cafeteria 2.0
 Medicines and hospital supplies used 2.0
 Depreciation expense 2.0
 Net expenses for walk on diabetes day 1.0
 0.5 mark each for other expenses 3.0

(b) Computation of items pertaining to statement of financial position:


 Non-current assets 2.0
 Current assets 2.0
 General and specific funds 1.0
 Liabilities 1.0

A.6  Identification of an appropriate ratio (0.5 mark each) 4.0


 01 mark for computing the ratios 8.0
 Conclusion (0.5 mark each) 4.0

A.7 (i)  Calculation of amounts for correcting entries 3.0


 0.5 mark for each correct head of account in correcting entries 3.0

(ii)  Calculation of amounts for correcting entries 2.0


 0.5 mark for each correct head of account in correcting entries 2.0

(iii)  Calculation of amounts for correcting entries 2.0


 0.5 mark for each correct head of account in correcting entries 2.0

Page 1 of 2
Financial Accounting and Reporting - I
Summary of Marking Key
Certificate in Accounting and Finance – Spring 2020

Mark(s)
A.8 Extracts from statement of financial position
 Appropriate line items (0.5 mark each) 2.0
 Calculation of amounts of:
− property, plant and equipment 1.5
− investment property 0.5
− revaluation surplus 2.0
− deferred government grant 1.0
Extracts from statement of profit or loss and other comprehensive income
 Appropriate line item (0.5 mark each) 3.0
 Calculation of amounts of:
− depreciation 4.0
− revaluation gain and loss 2.0
− grant income 1.0
− change in fair value of investment property 1.0
− revaluation surplus 2.0

(THE END)

Page 2 of 2
Certificate in Accounting and Finance Stage Examination

The Institute of 23 September 2020


Chartered Accountants 3 hours – 100 marks
of Pakistan Additional reading time – 15 minutes

Financial Accounting and Reporting-I


Instructions to examinees:
(i) Answer all EIGHT questions.
(ii) Answer in black pen only.
(iii) Multiple Choice Questions must be answered in answer script only.

Section A

Q.1 Following information pertain to Katas Industries Limited for the year ended 30 June 2020:
(i) Purchase of raw material:
Rs. in '000
Purchase price 96,100
Discount on bulk purchases 3,290
Early settlement discounts 1,580

(ii) Cost incurred at various locations:

Warehouse
Factory Head office Sales office Raw Finished
Description
material goods
------------------------------ Rs. in '000 ------------------------------
Salaries & wages 9,200* 2,000 3,800 860 640
Depreciation 3,500 1,250 750 150 120
Rent 3,640 - 2,360 380 160
Utilities 2,780 940 1,230 450 235
*75% of factory salaries & wages vary with the level of production

(iii) Breakup of inventories:


1 July 2019 30 June 2020
--------- Rs. in '000 ---------
Raw material 6,800 8,500
Work in progress 1,980 1,600
Finished goods 8,960 12,000

(iv) Due to a machine break down, raw material costing Rs. 1,560,000 was lost during the
production process.

Required:
Prepare statement of cost of goods manufactured for the year ended 30 June 2020. (Also show
total prime cost) (08)

Q.2 Ratios are computed by using numerical values from financial statements to gain meaningful
information about an entity. However, due to inherent limitations of ratio analysis, it may not
reflect the correct financial situation.

Required:
Briefly explain any four limitations of ratio analysis. (06)
Financial Accounting and Reporting-I Page 2 of 6

Q.3 On 1 July 2014, Indus Pharma Limited (IPL) received a government grant of Rs. 280 million
to setup a plant in an under-developed rural area. The grant is repayable in full if the conditions
attached to the grant are not met for a period of five years from the date of commencement of
the production. At the inception, it was highly probable that IPL would comply with the
conditions for the required period.

IPL incurred total cost of Rs. 630 million on plant and it started production on
1 January 2015. Useful life of the plant was estimated at 7 years. IPL deducted government
grant in arriving at the carrying amount of the asset.

In January 2019, IPL showed its inability to comply with the conditions attached to the grant
and regulatory authority issued a notice to IPL for repayment of the grant in full. Accordingly,
the grant was repaid by IPL.

In view of repayment of the grant, IPL carried out an impairment review of the plant on
31 December 2019. Net annual cash inflows for the remaining life of the plant have been
estimated at Rs. 90 million and Rs. 80 million for 2020 and 2021 respectively. These cash
inflows are net of annual interest and maintenance cost of Rs. 10 million and Rs. 6 million
respectively for both years. Applicable discount rate is 12%.

On the date of impairment review, the existing plant can be sold in the local market for
Rs. 160 million. Estimated cost of disposal would be Rs. 5 million.

Required:
Prepare journal entries for the year ended 31 December 2019 in respect of the above
information. (Show all necessary workings. Narrations are not required) (08)

Q.4 Select the most appropriate answer from the options available for each of the following
Multiple Choice Questions.

(i) Which of the following statements is correct about financial statements based on
historical cost in times of rising prices?
(a) Profits will be overstated and assets will be understated
(b) Assets will be overstated
(c) Profits as well as assets will be understated
(d) Depreciation will be overstated (01)

(ii) Under IAS 40 ‘Investment property’, which of the following disclosures is NOT
required to be made under cost model?
(a) Fair value of the property
(b) Depreciation method
(c) Reconciliation of carrying amounts at the beginning and end of a period
(d) Residual value of the property (01)

(iii) Which of the following would cause negative net cash flow from operating activities?

(a) Decrease in depreciation expense


(b) A substantial investment in fixed assets
(c) A significant increase in credit sales
(d) Repayment of a long-term loan (01)

(iv) A company pays to its salesman a minimum salary plus commission based on sales.
Salesman’s total remuneration is the example of:

(a) fixed cost (b) semi-variable cost


(c) stepped cost (d) variable cost (01)
Financial Accounting and Reporting-I Page 3 of 6

(v) Alpha Club’s financial year ends on 31 December. Following information pertain to its
members’ subscription:
Rupees
Subscription received in 2018 for 2019 180,000
Subscription received in 2019 for 2018 90,000
Subscription received in 2019 for 2019 1,400,000
Subscription received in 2019 for 2020 200,000
Subscription for 2018 outstanding as on 31 December 2018 150,000
Subscription for 2019 outstanding as on 31 December 2019 325,000

Subscription income for the year ended 31 December 2019 is:

(a) Rs. 1,845,000 (b) Rs. 1,705,000 (c) Rs. 1,905,000 (d) Rs. 1,665,000 (02)

(vi) A company has current ratio and quick ratio of 2.0 and 0.8 respectively. If the company
uses its positive cash balance to pay a creditor, it will:

(a) increase current ratio as well as quick ratio


(b) increase current ratio and decrease quick ratio
(c) have no effect on current ratio as well as quick ratio
(d) decrease current ratio as well as quick ratio (01)

(vii) Which of the following would increase gearing ratio?

(a) Issuance of shares at premium


(b) Issuance of shares at discount
(c) Issuance of bonus shares
(d) Declaration and payment of cash dividend (01)

(viii) Which of the following statements is correct in the context of capitalisation of borrowing
costs?
(a) If funds have been arranged from various general borrowings, the amount to be
capitalised is based on the weighted average cost of borrowings
(b) Capitalisation always commences as soon as expenditure for the asset is incurred
(c) Capitalisation always continues until the asset is brought into use
(d) Capitalisation always commences as soon as borrowing costs are incurred (01)

Section B

Q.5 (a) Stupa Limited (SL) sells electrical products at following standalone prices:

Products Rupees
E-1 30,000
E-2 30,000
E-3 50,000

Required:
Calculate transaction price to be allocated to each product under each of the following
independent situations:
(i) SL offered to sell one unit of each of the above products for Rs. 90,000. SL
regularly sells one unit each of E-2 and E-3 together for Rs. 70,000. (04)
(ii) SL offered to sell one unit of E-1 and two units of E-3 for Rs. 104,000. (02)
Financial Accounting and Reporting-I Page 4 of 6

(b) On 1 October 2018, Kushan Construction Limited (KCL) entered into a contract to
construct a commercial building for a customer for Rs. 50 million and a bonus of
Rs. 10 million if the building is completed on or before 31 December 2019.

Till 30 June 2019, KCL expected that the building will be completed within time at a
total cost of Rs. 40 million. However, due to bad weather and time involved in
regulatory approvals, the building was completed on 28 February 2020 at a total cost of
Rs. 42 million of which Rs. 26 million was incurred till 30 June 2019.

Required:
Compute profit to be recognized for the years ended 30 June 2019 and 2020, if:
(i) performance obligation under the contract is satisfied over time. (04)
(ii) performance obligation under the contract is satisfied at a point in time. (01)

(c) The nature, timing and amount of consideration promised by a customer affect the
estimate of the transaction price.

Define the term ‘transaction price’ and list down the factors that may affect
determination of the transaction price. (04)

Q.6 Statement of financial position of Taxila Limited (TL) as on 30 June 2020 is as follows:

2020 2019 2020 2019


Assets Equity & liabilities
Rs. in million Rs. in million
Property, plant and equipment 1,619 1,200 Share capital (Rs. 100 each) 1,200 800
Investment property 290 120 Share premium 290 150
Inventories 205 180 Retained earnings 260 90
Trade receivables 342 291 Revaluation surplus 215 200
Prepayments and other receivables 14 20 Long-term loans 367 445
Short-term investments 60 48 Trade and other payables 144 120
Cash and bank balances 24 6 Current portion of
long-term loans 78 60
2,554 1,865 2,554 1,865

Additional information:
(i) Equipment having fair value of Rs. 240 million was acquired by issuing 2 million shares.
(ii) As a result of revaluation carried out on 30 June 2020, property, plant and equipment
was increased by Rs. 80 million out of which Rs. 35 million was credited to profit and
loss account.
(iii) During the year, fully depreciated items of property, plant and equipment costing
Rs. 36 million were sold for Rs. 8 million out of which Rs. 3 million is still outstanding.
(iv) Depreciation on property, plant and equipment for the year amounted to
Rs. 290 million.
(v) An investment property was acquired for Rs. 180 million. TL applies cost model for
subsequent measurement of its investment property.
(vi) Financial charges for the year amounted to Rs. 45 million. Trade and other payables
include accrued financial charges of Rs. 12 million (2019: Rs. 17 million).
(vii) Short-term investments amounting to Rs. 35 million are readily convertible to cash
(2019: Rs. 20 million). Investment income for the year amounted to Rs. 6 million.

Required:
Prepare TL’s statement of cash flows for the year ended 30 June 2020 in accordance with the
requirements of IFRSs. (17)
Financial Accounting and Reporting-I Page 5 of 6

Q.7 You have been appointed as accountant of Gandhara Enterprises (GE) to replace Nasim who
was terminated on suspicion of fraud. Following information has been compiled for
preparation of GE’s financial statements for the year ended 30 June 2020:

(i) Summarised bank statement:

Receipts Rs. in '000 Payments Rs. in '000


Opening balance 600 Creditors 8,300
Cheques from debtors 7,420 Salaries 900
Cash 2,400 Repair and maintenance 450
Rent 980 Utilities 500
Office furniture 150
Drawings 640
Closing balance 460
11,400 11,400

(ii) Other balances worked out from the available records:

30-Jun-2020 30-Jun-2019
Particulars
------- Rs. in '000 -------
Fixed assets – WDV 3,400 3,460
Inventories 750 715
Goods in transit 140 -
Debtors 900 730
Unearned rent 300 450
Cash in hand 48 36
Creditors 895 690
Salaries payable 86 120

(iii) All debtors settle their accounts through cheques. All payments are made through
cheques except for average monthly petty expenses of Rs. 25,000.
(iv) Cheques of Rs. 950,000 issued to creditors in the last week of June 2020 were presented
in July 2020. Cheques from debtors amounting to Rs. 860,000 deposited on
30 June 2020 were cleared in July 2020.
(v) Goods are sold on cash and credit at cost plus 25% and 30% respectively.
(vi) Apart from misappropriating amounts from cash sales, the following matters were also
noted in respect of Nasim’s fraud:
 Physical cash count revealed that cash in hand was Rs. 20,000.
 Fixed assets having written down value of Rs. 65,000 were sold for Rs. 120,000
which was not recorded in the books.
 Goods in transit represent goods purchased in May 2020. However, in actual there
were no goods in transit.
 Goods costing Rs. 130,000 appearing in the closing inventory sheets were not found
physically.
 All the debtors confirmed their balances except for an amount of Rs. 260,000. It
was found that the related goods had been issued against fake invoices.

Required:
(a) Determine the amount of suspected fraud. (06)
(b) Prepare GE’s statement of profit or loss for the year ended 30 June 2020. (11)
Financial Accounting and Reporting-I Page 6 of 6

Q.8 Following information pertain to property, plant and equipment of Harappa Industries
Limited (HIL) for the year ended 30 June 2020:

(i) Balance as on 30 June 2019


Cost/revalued Accumulated Revaluation Depreciation Useful
Assets
amount depreciation surplus method life/rate
----------- Rs. in '000 -----------
Land* 100,000 - - - Infinite
Buildings 70,000 14,000 16,000 Straight line 20 years
Plant 180,000 60,000 - Straight line 15 years
Vehicles 8,800 4,000 - Reducing balance 20%
*
An amount of Rs. 12 million had been charged to profit or loss upon previous revaluation

(ii) On 30 June 2020, the revalued amounts of the land and buildings were assessed by
Smart Consultant at Rs. 120 million and Rs. 35 million respectively.
(iii) Setting up of a new plant was commenced on 1 July 2019 and substantially completed
on 29 February 2020. The plant was available for use on 1 April 2020 and immediately
put into use. Useful life of the plant was estimated at 10 years. Details of the cost
incurred are as under:

Description Payment date Rs. in '000


1st payment 1 August 2019 12,000
2nd payment 1 October 2019 48,000
3rd payment 29 February 2020 48,000
4th payment 31 July 2020 12,000
120,000

The cost of the plant was financed through an existing running finance facility with a
limit of Rs. 200 million carrying mark-up of 12% per annum. A government grant of
Rs. 20 million related to the plant was received on 1 January 2020. The grant amount
was used for repayment of the running facility.

(iv) One of the vehicles had an engine failure on 1 January 2020 and its engine had to be
sold as scrap for Rs. 0.1 million. The vehicle had been acquired on 1 January 2018 at a
cost of Rs. 2.5 million. 40% of the cost is attributable to its engine. Though the engine
of similar capacity was available at a cost of Rs. 1.2 million, the old engine was replaced
on 1 January 2020 with a higher capacity engine at a cost of Rs. 1.8 million.
(v) HIL uses cost model for subsequent measurement of property, plant and equipment
except for land and buildings.
(vi) HIL accounts for revaluation on net replacement value method and transfers the
maximum possible amount from revaluation surplus to retained earnings on an annual
basis.
(vii) HIL deducts government grant in arriving at the carrying amount of the asset.

Required:
In accordance with IFRSs, prepare a note on ‘Property, plant and equipment’ for inclusion in
HIL’s financial statements for the year ended 30 June 2020. (20)
(Comparatives figures and column for total are not required)

(THE END)
Financial Accounting and Reporting-I
Suggested Answers
Certificate in Accounting and Finance – Autumn 2020

A.1 Katas Industries Limited


Statement of cost of goods manufactured for the year ended 30 June 2020

Rs. in '000
Raw materials consumed (W-1) 89,550
Salaries and wages 9,200×0.75 6,900
Prime cost 96,450
Manufacturing overheads:
Salaries and wages 9,200×0.25+860 3,160
Depreciation 3,500 + 150 3,650
Rent 3,640 + 380 4,020
Utilities 2,780 + 450 3,230
Manufacturing overheads 14,060
Total manufacturing costs 110,510
Work in progress - opening 1,980
Work in progress - closing (1,600)
Cost of goods manufactured 110,890

W-1: Materials consumed Rs. in '000


Opening stock 6,800
Purchases 96,100
Discount on bulk purchase (3,290)
Abnormal production losses (1,560)
Closing stock (8,500)
89,550

A.2 Limitations of ratio analysis:

(i) Historical
All information used in ratio analysis is derived from actual historical results. This
does not mean that the same results will carry forward into the future. However,
ratio analysis can be used on pro forma information and compare it to historical
results for consistency.

(ii) Historical versus current cost


The information on the income statement is stated in current costs (or close to it),
whereas many elements of the balance sheet are stated at historical cost (which
could vary substantially from current costs). This disparity can result in unusual ratio
results.

(iii) Inflationary effect


If the rate of inflation has changed in any of the periods under review, this can mean
that the numbers are not comparable across periods. For example, if the inflation rate
was 100% in one year, sales would appear to have doubled over the preceding year,
when in fact sales did not change at all.

(iv) Aggregation
The information in a financial statement line item that is used for a ratio analysis may
have been aggregated differently in the past, so that running the ratio analysis on a
trend line does not compare the same information through the entire trend period.

Page 1 of 7
Financial Accounting and Reporting-I
Suggested Answers
Certificate in Accounting and Finance – Autumn 2020

A.3 Indus Pharma Limited


General Journal
Debit Credit
Date Description
Rs. in million
Jan. 2019 Plant 280
Cash/Bank 280

Jan. 2019/ Depreciation expense/Profit or loss 280÷7×4 160


31-12-2019 Accumulated depreciation - Plant 160

31-12-2019 Depreciation expense 630÷7 90


Accumulated depreciation - Plant 90

31-12-2019 Impairment loss (W-1) 19


Accumulated impairment - Plant 19

W-1: Impairment review as on 31 December 2019 Rs. in million


Year Net inflows Discounting at 12% Present value
2020 (90+10) 100 0.8929 89
2021 (80+10) 90 0.7972 72
Value in use 161
Fair value less cost to sell 160–5 155
Recoverable amount (Higher of both) 161
WDV of the plant as on 31 December 2019 630÷7×2 180
Impairment loss 180–161 19

A.4 (i) (a) Profits will be overstated and assets will be understated
(ii) (d) Residual value of the property
(iii) (c) A significant increase in credit sales
(iv) (b) Semi-variable cost
(v) (c) Rs. 1,905,000
(vi) (b) Increase current ratio and decrease quick ratio
(vii) (d) Declaration and payment of cash dividend
(viii) (a) If funds have been arranged from various general borrowings, the amount to be
capitalised is based on the weighted average cost of borrowings

A.5 (a) Allocation of transaction price


(i) Standalone Allocation of Rs. 70,000 to Allocation of Rs. Transaction
Product price E-2 and E-3 90,000 price
-------------------------------------- Rs. --------------------------------------
E-1 30,000
30,000 (30,000×90,000÷100,000) 27,000
E-2 30,000
(30,000×70,000÷80,000
) 26,250 (26,250×90,000÷100,000) 23,625
E-3 50,000 43,750 39,375

Page 2 of 7
Financial Accounting and Reporting-I
Suggested Answers
Certificate in Accounting and Finance – Autumn 2020

(50,000×70,000÷80,000 (43,750×90,000÷100,000)
)
110,000 100,000 90,000

(ii) Standalone price Allocation of Rs. 104,000 Transaction price


Product
-------------------------------------- Rs. --------------------------------------
E-1 30,000 (30,000×104,000÷130,000) 24,000
E-3 (50,000×2)
100,000 (100,000×104,000÷130,000) (40,000×2) 80,000
130,000 104,000

(b) (i) Computation of profit – performance obligation satisfied ‘over time’:

2019 2020
Completion % 65% 100%
(26÷40×100)
----------------- Rs. -----------------
Revenue 39.0 11.0
(50+10)×65% (50–39)
Cost (26.0) (16.0)
(42–26)
Profit/(loss) 13.0 (5.0)

(ii) Computation of profit – performance obligation satisfied ‘at a point in time’:

2019 2020
----------------- Rs. -----------------
Revenue - 50.0
Cost - (42.0)
Profit - 8.0

(c) Transaction price:


The transaction price is the amount of consideration to which an entity expects to be
entitled in exchange for transferring promised goods or services to a customer,
excluding amounts to be collected on behalf of third parties.

Factors affecting determination of the transaction price:


(i) Variable consideration
(ii) Constraints on variable consideration
(iii) Existence of a significant financing components (time value of money)
(iv) Non-cash consideration
(v) Consideration payable to a customer

A.6 Taxila Limited


Statement of cash flows for the year ended 30 June 2020
Rs. in million
Cash flows from operating activities
Profit (W-1) 140
Adjustments for:
Depreciation on property, plant and equipment 290
Depreciation on investment property 120+180–290 10
Gain on disposal of property, plant and equipment (8)
Revaluation gain (35)
Interest expense 45

Page 3 of 7
Financial Accounting and Reporting-I
Suggested Answers
Certificate in Accounting and Finance – Autumn 2020

Operating profit before working capital changes 442


Changes in working capital:
Increase in inventory 205–180 (25)
Decrease in prepayments and other receivables (14–3)–20 9
Increase in trade receivables 342–291 (51)
Decrease in short-term investments (60–35)–(48–20) 3
Increase in trade and other payables (144–12)–(120–17) 29
(35)
Cash generated from operations 407
Interest paid 17+45–12 (50)
Net cash flows from operating activities 357

Cash flows from investing activities


Purchase of property, plant and equipment (W-2) (389)
Purchase of investment property (180)
Proceeds from disposal of property, plant & equipment 8–3 5
Net cash flows used in investing activities (564)

Cash flows from financing activities


Proceeds from issue of shares at premium (W-3) 300
Repayment of long term loan (367+78)– (445+60) (60)
Net cash flows from financing activities 240
Net increase in cash and cash equivalents 33
Cash and cash equivalents at beginning of the year 6+20 26
Cash and cash equivalents at the end of the year 24+35 59

W-1: Profit for the year Rs. in million


Retained earnings – closing 260
Transfer from revaluation surplus 200+(80–35)–215 (30)
Retained earnings – opening (90)
140

W-2: Purchase of property, plant and equipment Rs. in million


Opening balance 1,200
Equipment acquired against issuance of shares 240
Revaluation surplus 80
Depreciation for the year (290)
Closing balance (1,619)
389

W-3: Issuance of shares at premium Rs. in million


Share capital and share premium – closing 1,200+290 1,490
Issuance of shares for property, plant and equipment (240)
Share capital and share premium – opening 800+150 (950)
300

Page 4 of 7
Financial Accounting and Reporting-I
Suggested Answers
Certificate in Accounting and Finance – Autumn 2020

A.7 Gandhara Enterprises


Rs. in '000
(a) Amount of suspected fraud:
Difference in cash balance 48–20 28
Proceeds from sale of fixed assets 65+55 120
Fake credit sales invoices 260÷1.3 200
Fake goods in transit 140
Embezzlement through inventory 750–620 130
Cash defalcated from cash sales (W-1) 763
1,381

W-1: Cash
Rs. in '000 Rs. in '000
Petty expenses
Opening balance 36 25×12 300
Cash sales PL 3,475 Cash banked 2,400
Cash shortage 48–20 28
Closing balance 20
Cash defalcated from
cash sales (Bal.) 763
3,511 3,511

(b) Gandhara Enterprises


Statement of profit or loss for the year ended 30 June 2020
--------- Rs. in '000 ---------
Sales - Credit (W-2) 8,190
- Cash [9,080–6,300(8,190÷1.3)]×1.25 3,475
11,665
Cost of goods sold
Opening inventory 715
Purchases (W-3) 9,455
Goods in transit (140)
Good issues against fake invoices 260÷1.3 (200)
Goods physically not found (130)
Closing stock 750–130 (620)
Cost of sales (9,080)
Gross profit 2,585
Operating expenses
Salaries 86+900–120 866
Utilities 500
Repair and maintenance 450
Petty cash expenses 25×12 300
Depreciation 3,460+150–3,400 210
Loss due to defalcation (a) 1,381
Total operating expenses (3,707)
(1,122)
Rent income 980+450–300 1,130
Gain on disposal of fixed assets 120–65 55
Net profit 63

Page 5 of 7
Financial Accounting and Reporting-I
Suggested Answers
Certificate in Accounting and Finance – Autumn 2020

W-2: Debtors
Rs. in '000 Rs. in '000
Opening balance 730 Bank 7,420
Credit sales (Bal.) 8,190 Uncleared cheques 860
Closing balance (900–
260) 640
8,920 8,920

W-3: Creditors
Rs. in '000 Rs. in '000
Bank 8,300 Opening balance 690
Unpresented cheques 950 Purchases (Bal.) 9,455
Closing balance 895
10,145 10,145

A.8 Harappa Industries Limited


Notes to the financial statements for the year ended 30 June 2020
1 Property, plant and equipment:
Land Buildings Plant Vehicles
------------------ Rs. in '000 ------------------
Gross carrying amount - opening 100,000 70,000 180,000 8, 800
Accumulated depreciation - (14,000) (60,000) (4,000)
Opening carrying amount 100,000 56,000 120,000 4,800
Additions - - (W-1)102,840 1,800
Depreciation for the year - (3,500) (14,571) (1,068)
(70,000÷20
) (W-2) (W-4)
Disposals - - - (W-3) (648)
Revaluation
(W-5)
- Surplus (Bal.) 8,000 (15,000) - -
- P&L 12,000 (Bal.) (2,500) - -
Closing carrying amount 120,000 35,000 208,269 4,884

Gross carrying amount - closing 120,000 35,000 282,840 9,600


Accumulated depreciation - - (74,571) (4,716)
Closing carrying amount 120,000 35,000 208,269 4,884

1.1 Land Buildings Plant Vehicles


Measurement base Revaluation Revaluation Cost model Cost model
Useful life /depreciation rate Infinite 15 Years 15/10 years 20%
Depreciation method - Straight line Straight line Reducing bal.

1.2 The last revaluation was performed on 30 June 2020 by Smart Consultants, an independent firm of
valuers.
1.3 Had revaluations not made, the carrying value of the land and buildings as on 30 June 2020 would
have been Rs. 112 million (100+12) and Rs. 37.5 million (35,000+2,500) respectively.

W-1: Cost - Plant Rs. in '000


Cost 120,000
Government grant (20,000)
Capitalisation of borrowing cost:

Page 6 of 7
Financial Accounting and Reporting-I
Suggested Answers
Certificate in Accounting and Finance – Autumn 2020

1 August - 1 October 2019 12,000×12%×2÷12 240


1 October - 31 December 2019 60,000×12%×3÷12 1,800
1 January - 29 February 2020 (60,000–20,000)×12%×2÷12 800
2,840
102,840

W-2: Depreciation – Plant Rs. in '000


On opening balance 180,000÷15 12,000
On the new plant 102,840(W-1)÷10×3÷12 2,571
14,571

W-3: Written down value - Engine disposed off Rs. in '000


Cost 2,500×40% 1,000
Accumulated depreciation:
For the six months ended 30 June 2018 1,000×20%×6÷12 100
For 2018-2019 (1,000–100)×20% 180
For the six months ended 31 December 2019 (1,000–280)×20%×6÷12 72
352
648

W-4: Depreciation – Vehicles Rs. in '000


On disposal of old engine (W-3) 72
On remaining opening balance [(8,800–1,000)–(4,000–280)]×20% 816
On addition of new engine 1,800×20%×6÷12 180
1,068

W-5: Revaluation surplus – Buildings Rs. in '000


Opening balance 16,000
Incremental
depreciation 16,000÷16[(56,000÷70,000)×20] (1,000)
15,000

(THE END)

Page 7 of 7
INSTITUTE OF CHARTERED ACCOUNTANTS OF PAKISTAN

EXAMINERS’ COMMENTS

SUBJECT SESSION
Financial Accounting & Certificate in Accounting and Finance (CAF)
Reporting I Examination - Autumn 2020

Passing %

Question-wise
Overall
1 2 3 4 5 6 7 8
78% 27% 48% 31% 25% 43% 50% 12% 31%

General:

An overall passing ratio of 31% is much lower than previous two sessions’ results i.e. 51%
and 39% respectively. The low result was mainly due to lower than expected performance
in Q8 (based on IAS 16) and overall below average performance of the repeaters. The
highest score in the paper was 87 marks.

Performance in Q5 and Q8 was poor. Poor performance in Q7 (IFRS 15) was mainly due
to the inherent difficulty (newness) of the topic. Q8 was based on IAS 16 which seems to
have been overlooked by many examinees in this session on the assumption that it has been
tested in previous session.

16% examinees were just short of 9 or fewer marks and could have easily obtained them
if they had not missed straightforward marks and made basic mistakes. The most
commonly noted issues are lack of practice and poor presentation.

Question-wise common mistakes observed

Question 1

In manufacturing overheads, examinees either did not include raw material warehouse cost
or also included cost incurred at other locations along with factory cost.

Question 2

A majority of examinees had not studied the topic. Thus answers tended to be very
polarized, either very good or very poor.

Page 1 of 3
Examiners’ Comments on Financial Accounting & Reporting I – CAF Examination
Autumn 2020

Question 3

 Examinees did not read the requirement carefully and also prepared entries for the years
2014 to 2018. This resulted in loss of valuable time and also affected the performance
in other questions.
 Examinee presented entries as if the grant had been recorded initially by setting up
deferred income account.
 Annual interest cost was either ignored or deducted from net annual cash inflows while
calculating value in use.

Question 4

 MCQs at serial (v) and (vi) were least well answered.


 Many examinees wasted valuable time in reproducing the wordings of correct option
instead of only mentioning the serial of the option.

Question 5

 In part (b), examinees had no idea of the examined areas.


 In part (c), examinees did not mention that transaction price should exclude amounts
to be collected on behalf of third parties. Only few examinees were cognizant with the
factors and they were often left un-attempted.

Question 6

 Effect of transfer from revaluation surplus was not considered while determining profit.
 Depreciation on investment property was not shown in ‘adjustments for’.
 The adjustment for short term investments readily convertible to cash, outstanding
proceeds from disposal and/or accrued interest were not considered in ‘changes in
working capital’.
 Repayment of loan did not include effect of current maturity.
 Purchase of property, plant and equipment against issuance of shares was presented in
the statement though it was a non-cash transaction.

Question 7

 In part (a), all components (as given in point vi of the question) of loss from suspected
fraud were not included in the calculation.
 Goods in transit and goods issued against fake invoices were not deducted in
calculating cost of the goods sold.
 Unpresented cheques and deposit in transit were not correctly incorporated in the
creditor and debtor accounts.
 Examinees prepared cash account and assumed the balancing figure to be cash sales
and ignored the misappropriations from cash sales.

Page 2 of 3
Examiners’ Comments on Financial Accounting & Reporting I – CAF Examination
Autumn 2020

Question 8

 Adjustment of revaluation of building was not properly presented and/or incorrectly


bifurcated into revaluation surplus and profit or loss.
 Schedule of property, plant and equipment was often incomplete in terms of
presentation.
 Borrowing cost was capitalized till 31 March 2020.
 Carrying values of land and building if the cost model had been used were incorrectly
calculated.

(THE END)

Page 3 of 3
Financial Accounting and Reporting - I
Summary of Marking Key
Certificate in Accounting and Finance – Autumn 2020

Note regarding marking scheme:


The marking scheme is given as a guide. Markers also award marks for alternative approaches to a
question and relevant/well-reasoned comments/explanations. Moreover, the available marks in
answer may exceed the total marks of a question.

Mark(s)
A.1  Raw material consumed 2.5
 Direct labour 0.5
 Manufacturing overheads 3.5
 Sub-totals 1.5

A.2  0.5 mark for identification of each limitation 2.0


 01 mark for explanation of each limitation 4.0

A.3  Repayment of grant 1.0


 Depreciation 2.5
 Impairment other than computation for value in use 2.0
 Computation of value in use 2.5

A.4 Marks as mentioned on the question paper against each MCQ 9.0

A.5 (a) (i)  Allocation of Rs. 70,000 2.0


 Allocation of Rs. 90,000 2.0

(ii) Allocation of Rs. 104,000 2.0

(b) (i)  For 2019 2.5


 For 2020 1.5

(ii)  For 2019 0.5


 For 2020 0.5

(c)  Definition 2.0


 0.5 mark for each factor 2.0

A.6  Profit before tax 2.0


 Up to 01 mark for each item in ‘Adjustments for’ 3.5
 Up to 01 mark for each item in ‘Changes in working capital’ 4.0
 Interest paid 1.0
 Purchase of property, plant and equipment 2.0
 Other investing activities 1.5
 01 mark for each financing activity 2.0
 Cash and cash equivalent 1.0

Page 1 of 2
Financial Accounting and Reporting - I
Summary of Marking Key
Certificate in Accounting and Finance – Autumn 2020

Mark(s)
A.7 (a)  Cash defalcated from sales 2.0
 Up to 01 mark for each other item 4.0

(b)  Sales 3.5


 Cost of sales 3.5
 Operating expense 2.5
 Other income 1.5

A.8  Presentation of note 2.0


 Additions during the year 4.0
 Disposals during the year 1.5
 Depreciation for the year 4.0
 Revaluation adjustments 4.5
 Other disclosures 4.0

(THE END)

Page 2 of 2
Certificate in Accounting and Finance Stage Examination

The Institute of 3 March 2021


Chartered Accountants 3 hours – 100 marks
of Pakistan Additional reading time – 15 minutes

Financial Accounting and Reporting-I


Instructions to examinees:
(i) Answer all EIGHT questions.
(ii) Answer in black pen only.
(iii) Multiple Choice Questions must be answered in answer script only.

Section A

Q.1 Following information pertains to Astrazenca Limited (AL):

(i) Shareholders' equity as on 1 January 2020:


Rs. in million
Share capital (Rs. 100 each) 250
Share premium 138
Retained earnings 142
Revaluation surplus: Land 25
Buildings 20

(ii) Profit and transfer of incremental depreciation as per the draft financial statements for
the year ended 31 December 2020 amounted to Rs. 45 million and Rs. 5 million
respectively.

(iii) Dividends for the last two years:

For the year ended *Interim cash dividend Final bonus dividend
31 December 2019 10% 20%
31 December 2020 12% 15%
*Declared with half yearly accounts

(iv) AL uses revaluation model for subsequent measurement of its land and buildings
only. The revalued amounts of land and buildings have been assessed at
31 December 2020 but not incorporated in draft financial statements. The relevant
details are as under:
Land Buildings
--- Rs. in million ---
Balances as on 31 December 2020 before revaluation:
Cost 75 240
Accumulated depreciation - 60
Revalued amounts assessed at 31 December 2020 65 158

Required:
Prepare AL’s statement of changes in equity for the year ended 31 December 2020. (08)
(Column for total and comparative figures are not required)
Financial Accounting and Reporting-I Page 2 of 6

Q.2 Describe the behavior of each of the following costs graphically by denoting ‘Per unit cost’
on vertical axis and ‘Level of activity’ on horizontal axis:
(i) Depreciation expense – Depreciation on plant is computed using units of production
method.
(ii) Depreciation expense – Depreciation on plant is computed using straight line method.
(iii) Direct material cost – Bulk discount is available on additional purchases once the total
purchases exceed a certain level.
(iv) Generator rent – A generator has been acquired on rent at an hourly rate; however,
minimum rent for certain hours is payable irrespective of actual usage.
(v) Machine rent – Machines are acquired on a fixed monthly rent. One machine is
required for every 1 million units.
(vi) Direct labour cost – Factory workers are paid at fixed rate per unit. In case production
exceeds target in any month, then workers are paid with double rate for additional
units. (08)

Q.3 On 1 January 2021, Covaxin Telecom (CT) announced a new annual promotional package
for its customers. The package comprises of a mobile phone, full year unlimited on-net calls
and 1,000 minutes per month on other networks. Package price is Rs. 11,550 per quarter
payable in advance on the first day of each quarter. At the end of the contract, the phone
would not be returned to CT.

On the first day of the promotional announcement, CT sold 1,000 packages. Based on the
data available with CT, it is expected that each customer would utilize 10,000 minutes of
other networks with quarterly break-up as under:

Quarter ending Minutes


31 March 2021 2,700
30 June 2021 2,000
30 September 2021 2,900
31 December 2021 2,400

The mobile phone has a retail value of Rs. 34,000, if sold separately. A monthly subscription
for unlimited on-net calls is Rs. 500 while every call on other networks is charged at
Rs. 1.5 per minute, if billed separately.

Required:
Compute the quarterly revenue to be recognised for the quarters ending 31 March 2021 and
30 June 2021. (08)

Q.4 Select the most appropriate answer from the options available for each of the following
Multiple Choice Questions (MCQs).

(i) Which of the following future cash flows should NOT be included in the calculation
of value in use of an asset?

(a) Cash flows on maintaining the asset’s performance


(b) Cash flows on enhancing the asset’s performance
(c) Cash flows from continuing use of the asset
(d) Cash flows from disposal of the asset (01)

(ii) When an impairment review is carried out, an impaired asset is measured at:

(a) fair value less cost to sell (b) value in use


(c) cost (d) recoverable amount (01)
Financial Accounting and Reporting-I Page 3 of 6

(iii) Which of the following would be an external indicator that an asset of an entity may
be impaired?

(a) Increase in central bank discount rates


(b) Decline in economic performance of an asset
(c) Physical obsolescence of an asset
(d) Future restructuring plan of an asset (01)

(iv) Which of the following is NOT a measurement base for assets as referred in the
Conceptual Framework?

(a) Value in use (b) Fulfilment value


(c) Current cost (d) Fair value (01)

(v) The accounting principle applied by IFRS 15 when determining whether or not
revenue should be recognized in respect of a repurchase agreement is:

(a) prudence (b) matching


(c) verifiability (d) faithful representation (01)

(vi) An entity recognises revenue over time if:

(a) entity’s performance does not create an asset with an alternative use
(b) entity’s performance creates an asset whose control will be transferred at the end
of contract
(c) customer simultaneously receives and consumes the benefit provided by the
entity’s performance
(d) entity has an enforceable right to payment for performance completed to-date (01)

(vii) An entity made a profit of Rs. 550,000 for the year 2020 based on historical cost
accounting principles. It had opening capital of Rs. 1,500,000. During 2020, specific
prices indices increased by 15% while general price indices increased by 10%. How
much profit should be recorded for 2020 under physical capital maintenance concept?

(a) Rs. 325,000 (b) Rs. 400,000


(c) Rs. 467,500 (d) Rs. 495,000 (01)

(viii) In order to survive in the long run, a business must generate positive net cash flow
from:

(a) investing activities


(b) operating activities
(c) financing activities
(d) both (a) and (b) (01)
Financial Accounting and Reporting-I Page 4 of 6

Section B

Q.5 A fire broke out in the office of Moderna Sports Club (MSC) and burnt all the accounting
records. The accountant was able to retrieve a burnt copy of financial statements of MSC for
the year ended 31 December 2020. However, few information (as indicated by capital
alphabets) were unreadable. The retrieved copy is as follows:

Balance sheet as on 31 December 2020


Rs. in '000 Rs. in '000
Funds and liabilities Assets
2020 2019 2020 2019
General fund: Fixed assets - net 1,403 1,300
Opening balance A 1,586 Members’ subscription 270 158
Excess of income over expenditure B C Misc. supplies 13 10
Tuck-shop rent E 37
Tennis court fund 260 200 Advance salaries 18 15
Bank F 530
Liabilities:
Members’ subscription 20 25
Salaries 52 41
Utilities 25 D
Annual sports event 10 -

Income and expenditure account for the year ended 31 December 2020
Expenditure Rs. in '000 Income Rs. in '000
Salaries G Members’ subscriptions 919
Utilities 221 Tuck-shop rent 252
Misc. supplies H Donation - sports equipment 70
Members’ subscription written off 12 L M
Annual sports event I
J K
Disposal of fixed assets 8
Repair and maintenance 40
Excess of income over expenditure B

Receipts and payments account for the year ended 31 December 2020
Receipts Rs. in '000 Payments Rs. in '000
Opening balance 530 Salaries 560
N O Fixed assets 92
Tennis court fund P Annual sports event 180
Contribution for annual sports event 49 Misc. supplies 132
Entrance fee - annual sports event 86 Utilities 214
Sale of fixed assets 21 Repair and maintenance Q
Tuck-shop rent 248 Construction of tennis court 131
Scrap sale 15 Closing balance F

Required:
Determine the missing information as indicated by capital alphabets.
(Redrafting of above financial statements is not required) (18)
Financial Accounting and Reporting-I Page 5 of 6

Q.6 Epivac Limited is considering to take some of the following measures during the last week
of the year ending 31 March 2021 in order to show better financial performance;

(i) Pay balance of a major supplier from bank overdraft facility and avail 5% discount.
(ii) Sell slow moving stock items at a price equal to cost.
(iii) Recover debtors’ balances by offering cash discounts of 10%.
(iv) Offer extended credit terms of 90 days which would increase sales at existing margins.
(v) Dispose-off some non-current assets at gain.

Required:
State the effect (increase, decrease, no effect) of each of the above measure on the financial
ratios as per following format:
Ratios (i) (ii) (iii) (iv) (v)
(a) Gross profit margin
(b) Net profit margin
(c) Current ratio
(d) Stock turnover (times)
(e) Return on non-current assets
(f) Quick ratio (17)

Q.7 You have recently joined as the finance manager of Corv Limited (CL). While reviewing the
draft financial statements for the year ended 31 December 2020 prepared by the junior
accountant, you have noted the following:

(i) In January 2020, Government allotted an industrial plot to CL at a prime location


subject to the condition that CL will establish a factory. CL constructed the factory
building which was available for use on 1 October 2020. Due to delay in recruitment
of key factory employees, the production activities will commence on 15 March 2021.

The accountant has not recorded the land as it was given free of cost. While the
factory building is still appearing in capital work in progress as production activities
will commence on 15 March 2021. (06)

(ii) CL acquired a three story building on 1 March 2020. CL uses the ground floor for its
marketing department while remaining two floors were in excess of CL’s need and
therefore were rented out. The first floor was rented out on 1 June 2020 and the
second floor was rented out on 1 December 2020.

The accountant has recorded the building as property, plant and equipment. The
depreciation on ground, first and second floors has been computed from
1 March 2020, 1 June 2020 and 1 December 2020 respectively. (05)

(iii) CL is constructing a power generation plant for its factory. The project started on
1 February 2020 and would complete on 30 November 2021. The work remained
suspended for 3 months. The project is financed through long term loan, acquired
specifically on 1 January 2020. The unutilised amount of loan is kept in a separate
saving account.

The accountant has deducted income of separate saving account from full year’s
interest on loan and presented the net amount as finance cost in the statement of
profit or loss. (05)

The accounting policy of CL is to carry land and building at fair value (wherever permitted
by IFRS).

Required:
Discuss how the above issues should be dealt in the financial statements of CL for the year
ended 31 December 2020 in accordance with the requirements of IFRSs.
Financial Accounting and Reporting-I Page 6 of 6

Q.8 Sputnik Sea Limited (SSL) runs a cruise business across oceans. Following information in
respect of one of SSL’s cruise ship is available:

(i) SSL bought a cruise ship on 1 March 2018. After completing all the required
formalities, the ship was ready to sail on 1 April 2018.
(ii) Details regarding components of the ship are as under:

Estimated
Cost
Component Useful life residual value
(Rs. in million)
(Rs. in million)
Engine 840 50,000 hours 40
Body 535 25 years 35
Dry-docking (overhaul) 60 5 years -

(iii) On 1 May 2019, the ship suffered an accident which damaged its body. Repair work
took 2 months and costed Rs. 26 million. The repair work did not change useful life
and residual values of the components.

(iv) The average monthly sailing of the ship during the last three years are as under:

Year Hours
2018 360
2019 480
2020 600

(v) SSL uses revaluation model for subsequent measurement. SSL accounts for
revaluation on net replacement value method and transfers the maximum possible
amount from the revaluation surplus to retained earnings on an annual basis.

(vi) The revalued amounts of the ship as at 31 December 2019 and 2020 were determined
as Rs. 1,400 million and Rs. 1,000 million respectively. Revalued amounts are
apportioned between the components on the basis of their book values before the
revaluation.

Required:
Prepare necessary journal entries to record the above transaction from the date of acquisition
of the ship to the year ended 31 December 2020. (17)

(THE END)
Financial Accounting and Reporting-I
Suggested Answers
Certificate in Accounting and Finance – Spring 2021

A.1 Astrazenca Limited


Statement of changes in equity
For the year ended 31 December 2020
Share Share Retained Revaluat-
capital premium earnings ion surplus
--------------- Rs. in million ---------------
Balance as at 1 January 2020 (As given) 250 138 142 45
(250×20%
Final bonus dividend @ 20% for 2019 ) 50 (50)
Interim cash dividend @ 12% for (250+50)×12
2020 %) (36)
Total comprehensive income for the year:
Profit for the year 45–7 (W-1) 38
Other comprehensive income (10+15)(W-1) (25)
Transfer of incremental depreciation 5 (5)
Balance as at 31 December 2020 300 138 99 15

W-1: Revaluation of building Building Land


Rs. in million
Revalued amount 158 65
Carrying value (240–60) 180 75
Revaluation loss (22) (10)
Available surplus (20–5) 15 25
Revaluation loss taken to profit or loss 7 -

Page 1 of 7
Financial Accounting and Reporting-I
Suggested Answers
Certificate in Accounting and Finance – Spring 2021

A.2 (i) (ii)

(iii) (iv)

(v) (vi)

Page 2 of 7
Financial Accounting and Reporting-I
Suggested Answers
Certificate in Accounting and Finance – Spring 2021

A.3 Quarter-wise revenue


Quarter ending
Price
31 March 2021 30 June 2021
----------------------------------- Rs. -----------------------------------
Mobile phone 34,000 28,560,000 -
34,000×(46,200÷55,000)×1,000
On-net calls 6,000 1,260,000 1,260,000
6,000×(46,200÷55,000)×3÷12
12×500 ×1,000 1,000×5,040×3÷12
Other networks calls 15,000 3,402,000 2,520,000
10,000×1.5 2,700×1.5×(46,200÷55,000)×1,000 2,000×1.5×(46,200÷55,000)×1,000
Standalone 55,000 33,222,000 3,780,000
Discounted
11,550×4 46,200

A.4 (i) (b) Cash flows on enhancing the asset’s performance


(ii) (d) Recoverable amount
(iii) (a) Increase in central bank discount rates
(iv) (b) Fulfilment value
(v) (d) Faithful representation
(vi) (c) Customer simultaneously receives and consumes the benefit provided by the
entity’s performance
(vii) (a) Rs. 325,000
(viii) (b) Operating activities

A.5 Description Rs. in '000 Working


A Opening general fund - 2020 1,586 +180 (C) / Balancing figure in
1,766
balance sheet of 2020
B Excess of income over expenditure - Balancing figure in Balance sheet /
62
2020 income & expenditure account
C Excess of income over expenditure - 198–18(D) Balancing figure in balance
180
2019 sheet of 2019
D Utilities payable - 2019 18 (W-1)
E Tuck-shop rent receivable 41 (W-2)
Balancing figure in receipt and payment
F Bank balance - 2020 450
account
G Salaries expense 568 (W-3)
H Misc. supplies expense 129 (W-4)
I Annual sports event expense 55 (W-5)
J Depreciation expense
K Depreciation expense 161 (W-6)
L Scrap sale / Other income
M Scrap sale / Other income 15
N Members’ subscriptions received
O Members’ subscriptions received 790 (W-8)
P Tennis court fund received 60 (W-9)
Q Repair and maintenance paid 40

Page 3 of 7
Financial Accounting and Reporting-I
Suggested Answers
Certificate in Accounting and Finance – Spring 2021

W-1: Utilities Rs. in '000


Payment 214 Opening payable (Bal. fig) 18
Closing payable 25 Expense 221
239 239

W-2: Tuck shop rent Rs. in '000


Opening receivable 37 Receipt 248
Income 252 Closing receivable (Bal. fig) 41
289 289

W-3: Salaries Rs. in '000


Opening prepaid 15 Opening payable 41
Payment 560 Expense (Bal. fig) 568
Closing payable 52 Closing prepaid 18
627 627

W-4: Misc. Supplies Rs. in '000


Opening unused supplies 10 Expense (Bal. fig) 129
Payment 132 Closing unused supplies 13
142 142

W-5: Annual Sports event Rs. in '000


Payment 180 Members ‘contribution 49
Closing payable 10 Entrance fee 86
Expense (Bal. fig) 55
190 190

W-6: Fixed Assets - Net Rs. in '000


Opening balance 1,300 Disposal (W-7) 29
Donation 70 Depreciation (Bal. fig) 161
Payment 92 Closing balance 1,403
Tennis court 131
1,593 1,593

W-7: Disposal Rs. in '000


Fixed assets (Bal. fig) 29 sale proceeds 21
Loss on disposal 8
29 29

W-8: Members’ subscription Rs. in '000


Opening arrears 158 Opening advance 25
Income 919 Receipt (Bal. fig) 790
Closing advance 20 Write off 12
Closing arrears 270
1,097 1,097

W-9: Tennis court fund Rs. in '000


Closing fund 260 Opening fund 200
Receipt (Bal. fig) 60
260 260

Page 4 of 7
Financial Accounting and Reporting-I
Suggested Answers
Certificate in Accounting and Finance – Spring 2021

A.6 Measures
S. No. Ratios
(i) (ii) (iii) (iv) (v)
(a) Gross profit % No effect Decrease No effect No effect No effect
(b) Net profit % Increase Decrease Decrease Increase Increase
(c) Current ratio Increase No effect Decrease Increase Increase
(d) Stock turnover (times) No effect Increase No effect Increase No effect
(e) Return on non-current assets Increase No effect Decrease Increase Increase
(f) Quick ratio Increase Increase Decrease Increase Increase

A.7 (i) The accounting treatment adopted by accountant for not recording land is incorrect.
Allotment of land by Government is a transfer of a non-monetary asset and should be
considered as a government grant. Such non-monetary grant may be recorded at fair
value or at a nominal value. As per CL’s policy, fair value of the land should be assessed
and reported in the financial statements under the head property, plant and
equipment (PPE). The grant was made subject to construction of factory so the
resulting deferred income should be recognized in income on a systematic basis over
the useful life of the factory building.

The factory building should also be transferred from capital work in progress to PPE
account as the building is available for use on 1 October 2020. Further depreciation
should also be charged from same date i.e. 1 October 2020.

(ii) The accounting treatment adopted by accountant to record complete building under
PPE head is incorrect. Two floors which have been leased/rented separately so should
be accounted for as investment property. While ground floor used by marketing
department should be recorded as property, plant and equipment under IAS 16 and
depreciated over its useful life.

As per CL policy, investment property should be recorded at fair value and changes in
fair value should be taken to statement of profit or loss. Any depreciation already
charged on these floors should be reversed.

(iii) The accounting treatment adopted by accountant to expense out borrowing cost is
incorrect as some borrowing cost is eligible for capitalization. Power generation plant
falls under definition of qualifying asset as its construction involves substantial period.

Construction of the power plant is financed through specific borrowing so actual


borrowing cost incurred less temporary investment income on the borrowings would
be capitalized. However, the borrowing cost will be capitalized from the date when
construction actually started i.e. 1 February 2020 rather than 1 January 2020. Further,
the capitalization of borrowing costs should be suspended and charged to the
statement of profit or loss during the three months when work was suspended.

In the statement of profit or loss, borrowing cost on loan and interest income earned
from saving account should be presently separately.

Page 5 of 7
Financial Accounting and Reporting-I
Suggested Answers
Certificate in Accounting and Finance – Spring 2021

A.8 Sputnik Sea Limited


General Journal Rs. in million
Date Description Debit Credit
01-03-2018 Cruise ship 1,435.00
Bank 1,435.00
31-12-2018 Depreciation expense (W-1) 75.84
Accumulated depreciation - Cruise ship 75.84
01-05-2019 Repair cost 26.00
Bank 26.00
31-12-2019 Depreciation expense (W-1) 108.80
Accumulated depreciation - Cruise ship 108.80
31-12-2019 Accumulated depreciation - Cruise ship 184.64
Cruise ship 75.84+108.80 184.64
31-12-2019 Cruise ship 149.64
Revaluation surplus 149.64
31-12-2020 Depreciation expense (W-1) 165.82
Accumulated depreciation - Cruise ship 165.82
31-12-2020 Revaluation surplus 165.82–147.20 (W-2) 18.62
Retained earnings 18.62
31-12-2020 Accumulated depreciation - Cruise ship 165.82
Cruise ship 165.82
31-12-2020 Revaluation surplus 149.64–18.62 131.02
Impairment loss (Bal. fig) 103.16
Cruise ship 1,234.18–1,000 234.18

W-1: Rs. in million


Date Description Engine Ship's Body Dry Docking Total
1/3/2018 Acquisition cost 840.0 535.0 60.0 1,435
31/12/18 Depreciation expense (51.84) (15.0) (9.0) (75.84)
(840–40)÷50,000×(360×9) (535–35)÷25×9÷12 60÷5×9/12

31/12/19 Depreciation expense (76.80) (20.0) (12.0) (108.80)


(840– (535–35)÷25 60÷5
40)÷50,000×(480×10)
31/12/19 Carrying value 711.36 500.0 39.0 1,250.36
Revaluation surplus
(Bal.) 149.64
31/12/19 Revalued amount 796.49 559.84 43.67 1,400.00
31/12/20 Depreciation expense (129.81) (22.57) (13.44) (165.82)
(796.38–40)÷(50,000–3240(559.84–35)÷(25–1.75) 43.67÷(5–1.75)
–4,800)×(600×12)
31/12/20 Carrying value 666.68 537.27 30.23 1,234.18

W-2: Depreciation on cost for 2020


Engine (840–40)÷50,000×(600×12) 115.20
Body (535–35)÷25 20.00
Dry docking 60÷5 12.00
147.20

Page 6 of 7
Financial Accounting and Reporting-I
Suggested Answers
Certificate in Accounting and Finance – Spring 2021

(THE END)

Page 7 of 7
INSTITUTE OF CHARTERED ACCOUNTANTS OF PAKISTAN

EXAMINERS’ COMMENTS

SUBJECT SESSION
Financial Accounting and Reporting I Certificate in Accounting and Finance (CAF)
Examination - Spring 2021

Passing %

Question-wise
Overall
1 2 3 4 5 6 7 8
81% 14% 20% 65% 25% 70% 59% 32% 40%

General

An overall passing ratio of 40% is higher than previous session’s 31% but fairly consistent
with average of last 3 sessions. The highest marks obtained were 91.

It was observed that examinees did not study all areas of the syllabus on the assumption
that they would have least possibility of being examined. This was substantiated by the
fact that many examinees secured good marks in two to three questions but failed to obtain
reasonable marks in the remaining questions.

Question-wise common mistakes observed

Question 1

 Profit was not adjusted for revaluation loss.


 Other comprehensive income was not presented.

Question 2

 Graphs were made on the basis of ‘total cost’ instead of ‘per unit cost’ on y-axis.
 Graph (iii) to (vi) were partially incorrect.

Question 3

 Revenue from sale of mobile phone was also recognized over time.
 Standalone price of on-net and other network call was worked out on quarterly basis
instead of annual basis.
 Workings were haphazard and partial marks could not be awarded as trail for the
calculations was not available.

Page 1 of 2
Examiners’ Comments on Financial Accounting and Reporting I – CAF Examination
Spring 2021

Question 4

MCQs at serial (iv) and (vi) were least well answered.

Question 5

 Examinees tried to compute the missing amounts without preparing the relevant
accounts and missed at least one figure in the calculation.
 Description of “N” i.e. “Members’ subscriptions received” was not identified and
consequently not computed.
 Marks for amounts to be determined as balancing figures were lost due to incomplete
answers.

Question 6

 In measure (ii) and (iv), it was wrongly identified that net profit margin and stock
turnover would remain unaffected.
 Some of the examinees did not prepare format.
 Few examinees also offered explanation of effect(s) which was not required.

Question 7

 Answers were correct to the extent discussed but failed to cover all aspects of the given
issues.
 In (i), recognition of grant over life of building was not discussed.
 In (ii), answers were limited to the discussion that the building should be partially
recorded as property, plant and equipment and partially as investment property.
However, consequential adjustments to be made due to this bifurcation were not
discussed.

Question 8

 Depreciation on dry docking and body were not prorated in year 2018.
 Entry for incremental depreciation and its calculation was not presented.
 Depreciation for year 2020 was computed on total life instead of remaining life.

(THE END)

Page 2 of 2
Financial Accounting and Reporting - I
Summary of Marking Key
Certificate in Accounting and Finance – Spring 2021

Note regarding marking scheme:


The marking scheme is given as a guide. Markers also award marks for alternative approaches to a
question and relevant/well-reasoned comments/explanations. Moreover, the available marks in
answer may exceed the total marks of a question.

Mark(s)
A.1  Dividends 2.0
 Presentation of total comprehensive income 2.0
 Correct amount of total comprehensive income 2.0
 Incremental depreciation 1.0
 Opening and closing balances 1.0

A.2  01 mark each for graph (i) and (ii) 2.0


 1.5 marks each for other graphs 6.0

A.3 Quarter wise revenue:


 Mobile phone 2.5
 On-net calls 2.5
 Other network calls 3.0

A.4 Marks as mentioned on the question paper against each MCQ 8.0

A.5  02 marks each for determining the information at ‘K’ and ‘O’ 4.0
 Up to 01 mark each for others 14.0

A.6 Effect on:


 gross profit margin 2.5
 net profit margin 3.0
 current ratio 3.0
 stock turnover (times) 3.0
 return on non-current assets 3.0
 quick ratio 2.5

A.7 (i) Up to 01 mark for each valid discussion point 6.0

(ii) Up to 01 mark for each valid discussion point 5.0

(iii) Up to 01 mark for each valid discussion point 5.0

A.8  0.5 mark each for entries on initial recognition, repair, depreciation and
incremental depreciation 3.0
 01 mark for entries on revaluation 2.0
 2.5 marks each for computation of yearly depreciation and incremental
depreciation 10.0
 Computation of corrected amount of revaluation adjustments 2.0

(THE END)

Page 1 of 1
Certificate in Accounting and Finance Stage Examination

The Institute of 8 September 2021


Chartered Accountants 3 hours – 100 marks
of Pakistan Additional reading time – 15 minutes

Financial Accounting and Reporting-I


Instructions to examinees:
(i) Answer all EIGHT questions.
(ii) Answer in black pen only.
(iii) Multiple Choice Questions must be answered in answer script only.

Section A

Q.1 Following amounts have been extracted from the financial statements of Lithops Limited:

2020 2019
----- Rs. in million -----
Sales 500 450
Cost of sales 378 300
Trade receivables 95 80
Trade payables 72 60
Inventory 93 75
Cash at bank 12 16

All sales and purchases are made on credit.

Required:
(a) Calculate working capital cycle days for 2020. (Assume a 360 day year) (04)
(b) Suggest four possible measures that can be taken to reduce working capital cycle days. (03)

Q.2 The draft financial statements of Barbary Cement Limited (BCL) for the year ended
31 December 2020 include a plant having a carrying value of Rs. 400 million. Due to
technological change, the remaining useful life of the plant has been reduced to 4 years.

Following information has been gathered for impairment testing of the plant:
(i) Inflows from sale of product to be manufactured by the plant for the year 2021 are
estimated at Rs. 200 million. These inflows are subject to 10% decrease in each
subsequent year due to declining demand.
(ii) Outflows from operational cost for 2021 are estimated at Rs. 80 million. These outflow
would increase by 5% in each subsequent year despite decline in demand due to inflation
and increase in plant’s wear and tear.
(iii) BCL’s net profit is subject to income tax of 20%.
(iv) Depreciation on plant is calculated using straight line method.
(v) The plant’s net disposal proceeds at the end of the useful life is estimated at
Rs. 100 million.
(vi) Pre-tax and post-tax discount rates are 12% and 9.6% per annum respectively.
(vii) A technologically advanced plant with similar capacity can be purchased at
Rs. 350 million. BCL has received an offer to buy the existing plant for Rs. 250 million.
BCL will have to incur shipping cost of Rs. 7 million, to dispatch the existing plant to
the purchaser.

Required:
Compute the impairment loss to be recognised as at 31 December 2020. (07)
Financial Accounting and Reporting-I Page 2 of 6

Q.3 On 1 August 2021, Succulent Limited started its manufacturing business. Following
information related to its manufacturing activities for the month of August is available:
(i) Raw materials of Rs. 2.5 million (including 20% indirect material) were acquired, out of
which 40% is still unpaid.
(ii) Total factory payroll for the month amounted to Rs. 4 million, out of which 10% is still
unpaid. 20% of the payroll relates to the indirect labor.
(iii) Other manufacturing overheads were Rs. 3.6 million which included depreciation of
Rs. 0.9 million.
(iv) Manufacturing overheads are applied at the rate of 150% of direct labor.
(v) Cost of physical inventory at month end was as follows:
Rs. in million
Direct material 0.6
Indirect material 0.1
Work in process 1.0

Required:
Prepare necessary journal entries in order to record the production and inventory cost in a
manufacturing environment. (Narrations are not required) (08)

Q.4 Select the most appropriate answer(s) from the options available for each of the following
Multiple Choice Questions.

(i) An asset was purchased on 1 January 2017 for Rs. 100 million with useful life of 6 years
and residual value of Rs. 10 million. On 1 January 2020, it is revalued to Rs. 120 million
with remaining useful life of 3 years and expected residual value of Rs. 15 million. How
much excess depreciation will be charged for the year ended 31 December 2020?
(a) Rs. 15 million (b) Rs. 35 million
(c) Rs. 20 million (d) Rs. 25 million (01)

(ii) A company used to pay its salesman a salary of Rs. 35,000 per month plus
2% commission based on sales. Now he is promoted as assistant manager sales with a
salary of Rs. 50,000 per month plus commission of Rs. 100,000 if sales are Rs. 5 million,
Rs. 200,000 if sales are Rs. 10 million and so on.

Salesman’s commission is the example of:

Before promotion After promotion


(a) Fixed cost Variable cost
(b) Variable cost Fixed cost
(c) Stepped cost Variable cost
(d) Variable cost Stepped cost (02)

(iii) When items of property, plant and equipment are stated at revalued amounts, which of
the following disclosures shall be made?
(a) Any restrictions on the distribution of the revaluation surplus to shareholders
(b) The carrying amount of temporarily idle property, plant and equipment
(c) The gross carrying amount of any fully depreciated property, plant and equipment
that is still in use
(d) All of the above (01)

(iv) Which of the following concepts measures profit in terms of an increase in the
productive capacity of an entity?
(a) Physical capital maintenance
(b) Historical cost accounting
(c) Financial capital maintenance (money terms)
(d) Financial capital maintenance (real terms) (01)
Financial Accounting and Reporting-I Page 3 of 6

(v) Which of the following should be included in the initial cost of investment property?
(a) Cost incurred on opening ceremony to celebrate completion of property
(b) Operating losses incurred before the property achieves the planned level of
occupancy
(c) Abnormal waste of materials incurred in construction of property
(d) Property transfer taxes (01)

(vi) An entity purchased an investment property on 1 January 2018 for Rs. 35 million. The
property had an estimated useful life of 35 years with no residual value. At
31 December 2020, the property had a fair value of Rs. 42 million. On 1 January 2021,
the property was sold for net proceeds of Rs. 40 million. Calculate the profit or loss on
disposal under both the cost and fair value models.
Cost model Fair value model
(a) Gain of Rs. 2 million Gain of Rs. 2 million
(b) Gain of Rs. 8 million Loss of Rs. 2 million
(c) Gain of Rs. 7 million Loss of Rs. 2 million
(d) Gain of Rs. 8 million Gain of Rs. 5 million (02)

(vii) Which of the following is not considered as transaction with owners with reference to
statement of changes in equity?
(a) Issuance of shares at par (b) Issuance of shares at premium
(c) Profit for the year (d) Bonus issue of shares (01)

(viii) Which two of the following factors could cause a company’s gross profit percentage on
sales to be above the expected level?
(a) Over-statement of closing inventories
(b) Sales were higher than expected
(c) Inclusion of disposal proceeds of non-current assets in sales
(d) Decrease in carriage charges borne by the company on goods sent to customers (01)

Section B

Q.5 Financial statements of Parodia Motors Limited (PML) for the year ended 30 June 2021 are
under preparation. While reviewing revenues from contract with customers, following matters
have been identified:
(i) On 1 November 2020, PML sold Car-A to Alpha Limited (AL) for Rs. 5 million. As per
the contract, Rs. 1 million would be paid immediately and the balance would be paid
after 2 years. The accountant has recognized revenue to the extent of the cost of Car-A
i.e. Rs. 3.5 million and remaining revenue would be recognized upon receipt of balance
from AL.

(ii) On 1 January 2021, PML entered into six months’ contract with Beta Limited (BL) to
sell Car-B for Rs. 3.5 million per unit. As per the contract, if BL purchases more than
10 units during the contract period, the price will be retrospectively reduced to
Rs. 3.4 million per unit. At the inception of the contract, PML concluded that BL will
meet the threshold for the discount. BL purchased 11th unit of Car-B on 28 June 2021 for
which no revenue has been recorded. BL has made payments of all units except 11th unit
which will be settled in July 2021.

(iii) On 1 February 2021, PML sold Car-C to Gamma Limited (GL) for Rs. 3 million and
recognized the entire amount as revenue. PML also provided GL a Rs. 0.2 million
discount voucher for any future purchases of spare parts within one year. There is 80%
likelihood that GL will redeem the discount voucher and will purchase spare parts within
one year. By the end of the year, no spare parts were purchased by GL. PML normally
sells Car-C for Rs. 3 million with no discount voucher.
Financial Accounting and Reporting-I Page 4 of 6

(iv) On 20 February 2021, PML sold Car-D to Delta Limited (DL) with one-year free
maintenance services at a lumpsum payment of Rs. 3.6 million. Payment was made on
1 March 2021 upon delivery of Car-D to DL. The revenue of Rs. 1.2 million (i.e. 4/12 of
Rs. 3.6 million) has been recognized. PML normally sells Car-D and annual maintenance
services separately for Rs. 3.5 million and Rs. 0.3 million respectively.

Discount rate of 12% per annum may be used wherever required.

Required:
Prepare correcting entries for the year ended 30 June 2021 in accordance with
IFRS 15 ‘Revenue from Contracts with Customers’. (16)

Q.6 Following are the extracts from the financial statements of Saguaro Limited (SL) for the year
ended 30 June 2021:

Statement of financial position as on 30 June 2021


2021 2020 2021 2020
Assets Equity & liabilities
Rs. in million Rs. in million
Operating fixed assets 820 848 Share capital (Rs. 10 each) 700 500
Accumulated depreciation (300) (262) Share discount (40) -
Capital work in progress 84 - Retained earnings 220 315
Inventories 274 245 Long-term loans 175 210
Trade receivables 177 204 Trade payables 180 130
Insurance claim - 31 Accrued expenses 48 43
Advance to supplier 78 60 Current portion of long-
Cash and bank balances 193 112 term loans 43 40
1,326 1,238 1,326 1,238

Statement of profit or loss for the year ended 30 June 2021


Rs. in million
Sales 757
Cost of sales (485)
Gross profit 272
Operating expenses (310)
Gain on disposal of equipment 17
Loss before interest (21)

Other information:
(i) SL declared a final dividend of 10% on 30 September 2020 which was paid in
December 2020.
(ii) 20 million shares were issued in May 2021.
(iii) Insurance claim was related to plant and machinery destroyed in April 2020. The plant
had cost and book value of Rs. 63 million and Rs. 42 million respectively.
(iv) During the year, SL disposed of equipment having cost and net book value of
Rs. 75 million and Rs. 35 million respectively.
(v) Current portion of long-term loans include accrued interest of Rs. 5 million.
(2020: Rs. 1 million)
(vi) Trade payables include an amount of Rs. 14 million payable against capital work in
progress.

Required:
Prepare SL’s statement of cash flows for the year ended 30 June 2021. (16)
Financial Accounting and Reporting-I Page 5 of 6

Q.7 Following information pertains to non-current assets of Bunny Ear Limited (BEL):

Land:
In January 2019, the government allotted a piece of land to BEL subject to the condition that
BEL will establish a factory building on it. The land was recorded at its fair value of
Rs. 100 million.

Factory building:
On 1 March 2019, BEL started construction of the factory building. The construction work
was completed on 30 June 2020. Payments related to the construction of the factory were as
follows:

Description Date of payment Rs. in million


1 bill of contractor
st
1-Mar-2019 130
2nd bill of contractor 1-Aug-2019 190
3rd bill of contractor 1-Jan-2020 180
Last bill of contractor 1-Jul-2020 100

The project was financed through:


(i) government grant of Rs. 200 million received on 1 February 2019. Unused funds from
government grant were invested in a saving account @ 8% per annum.
(ii) withdrawals from the following running finance facilities obtained from Bank A and
Bank B. The relevant details are:
Bank A Bank B
Obtained on 1 January 2019 1 January 2020
Markup rate 12% 14%
-------- Rs. in million --------
Balance on 31 December 2019 250 -
Markup for 2019 22 -
Balance on 31 December 2020 350 200
Average balance during 2020 300 150
Markup for 2020 36 21

Manufacturing plant:
The manufacturing plant was purchased on 1 August 2020 at cost of Rs. 420 million.
Rs. 240 million was financed through an interest free loan from government. The loan will be
forgiven if the plant is operated for atleast 4 years by BEL. Upon acquisition, there is a
reasonable assurance that BEL will comply with this condition.

Other information:
 BEL uses cost model for subsequent measurement of property, plant and equipment.
 All government grants are recorded as deferred income and a part of it is transferred to
income each year.
 Useful life of the factory building and manufacturing plant has been estimated at 25 years
and 10 years respectively.

Required:
Prepare relevant extracts (including comparative figures) from BEL’s statement of profit or
loss for the year ended 31 December 2020 and statement of financial position as on that date.
(Notes to the financial statements are not required. Borrowing costs are to be calculated on the basis
of number of months) (16)
Financial Accounting and Reporting-I Page 6 of 6

Q.8 The accountant of Cereus Golf Club (CGC) was terminated on charges of fraud and you have
been assigned the task of preparing the accounts for the year ended 31 December 2020. You
have found that the proper books had not been maintained. The management of CGC has
given you the following information:

(i) Cash and bank balances at 1 January 2020 amounted to Rs. 0.5 million and
Rs. 2 million respectively. However, as on 31 December 2020, there was no cash
balance and Rs. 4.2 million in the bank.

(ii) The members are required to pay 3 years’ subscription in advance upon
admission/renewal. Full year subscription is charged from members joining during the
year. Number of subscriptions received are as under:

3 years’ subscription
Year No. of memberships
per member
2018 100 Rs. 60,000
2019 140 Rs. 75,000
2020 160 Rs. 90,000

During 2020, 10 members were awarded membership on special permission but they
had not paid the subscription till year-end.

After year-end, 5 more members informed that they had paid the 3 years’ subscription
amount in 2020. It was found out that the amount was misappropriated by the
accountant.

(iii) CGC had received a donation of Rs. 8 million in 2019 to meet the repair and
maintenance expenditure of its golf course. Out of total donation, the club has spent
Rs. 2.2 million and Rs. 2.8 million in 2019 and 2020 respectively.

(iv) CGC started purchasing golf kits in 2020 for sales as well as for rent purposes. 20% of
the purchases were unpaid at year-end. Two third of the golf kit purchases made in 2020
had been added to inventory of golf kits for sale and remaining had been added directly
to golf kits for rent.

(v) Golf kits are sold for cash at cost plus 40%. Cost of closing inventory of golf kits for sale
amounted to Rs. 1 million. It was decided to transfer half of these kits into golf kits for
rent at 30% of their original cost.

(vi) Some of the receipts and payments during the year were as follows:

Rupees
Rent of golf kits 650,000
Golf kits purchases 4,800,000
Annual insurance (paid till April 2021) 660,000
Salaries (including Rs. 350,000 for 2019) 2,800,000
Other expenses 2,320,000

(vii) CGC has a fidelity insurance policy and any cash deficiency upto a maximum of
Rs. 2 million is recoverable under the policy.

(viii) Fixed assets at 1 January 2020 had a book value of Rs. 25 million. All fixed assets are
to be depreciated at 15% per annum.

Required:
(a) Prepare income and expenditure account for the year ended 31 December 2020. (11)
(b) Prepare statement of financial position as on 31 December 2020. (09)

(THE END)
Financial Accounting and Reporting-I
Suggested Answers
Certificate in Accounting and Finance – Autumn 2021

A.1 (a) Working capital cycle days:

Trade receivables Average debtors 87.5[(95+80)÷2]


A = ×360 = ×360 ⇒ 63 days
collection period Credit sales 500

Inventory holding Average inventory 84[(93+75)÷2]


B = ×360 = ×360 ⇒ 80 days
period Cost of sales 378

Trade payables Average creditors 66[(72+60)÷2]


C = ×360 = ×360 ⇒ 60 days
payment period Credit purchases(W-1) 396

Working capital =A+B − C = 63 + 80 ‒ 60 ⇒ 83 days


cycle

W-1: Calculation of purchases Rs. in million


Cost of goods sold 378
Add: Closing inventory 93
Less: Opening inventory (75)
Purchases (on credit) 396

(b) Measures to improve working capital cycle days:


 Give incentives to customers to pay on time
 Do not transact with customers who have a history of defaulting/late payments
 Automate the monitoring of accounts receivables
 Resolve disputes with customer as early as possible

A.2 Impairment loss: ------ Rs. in million ------


Carrying value 400.0
Recoverable amount
Value in use (W-1) 333.6
Fair value less cost of sell 250‒7 243.0
Higher of above (333.6)
66.4

W-1: Value in use 2021 2022 2023 2024 Total


--------------- Rs. in million ---------------
Inflows from sale 200 180 162 145.80
Operational cost (80) (84) (88.2) (92.61)
Disposal value - - - 100
Cash flows undiscounted 120 96 73.8 153.19
Discount factor @ 12% 0.8929 0.7972 0.7118 0.6355
PV of cash flows /value in use 107.2 76.5 52.5 97.4 333.6

Page 1 of 7
Financial Accounting and Reporting-I
Suggested Answers
Certificate in Accounting and Finance – Autumn 2021

A.3 Succulent Limited


General Journal
Debit Credit
S. No. Description
---- Rs. in million ----
(i) Raw Materials Inventory 2.5
Accounts Payable 2.5×40% 1.0
Cash 1.5
(ii) Factory Payroll 4.0
Wages Payable 4×10% 0.4
Cash 3.6
(ii) Work in Process Inventory 4×80% 3.2
Manufacturing Overhead 4×20% 0.8
Factory Payroll 4.0
(iii) Manufacturing Overhead 3.6
Accumulated depreciation 0.9
Cash / Payable 2.7
(iv) Work in Process Inventory 4.8
Manufacturing Overhead 3.2×150% 4.8
(v) Work in Process Inventory 1.4
Raw Materials Inventory (2.5×80%)‒0.6 1.4
(v) Manufacturing Overhead 0.4
Raw Materials Inventory (2.5×20%)‒0.1 0.4
(v) Finished Goods Inventory 3.2+4.8+1.4‒1 8.4
Work in Process Inventory 8.4

A.4 (i) (c) Rs. 20 million


(ii) (d) Variable cost Stepped cost
(iii) (a) any restrictions on the distribution of the revaluation surplus to shareholders
(iv) (a) Physical capital maintenance
(v) (d) Property transfer taxes
(vi) (b) Gain of Rs. 8 million Loss of Rs. 2 million
(vii) (c) Profit for the year
(viii) (a) Over-statement of closing inventories
(c) Inclusion of disposal proceeds of non-current assets in sales

Page 2 of 7
Financial Accounting and Reporting-I
Suggested Answers
Certificate in Accounting and Finance – Autumn 2021

A.5 Parodia Motor Limited


Correcting entries for the year ended 30 June 2021
Debit Credit
S. Description
------ Rs. in '000 ------
(i) Receivable - AL 4,189(W-1)‒3,500 689
Revenue 689
Receivable - AL 3,189×12%×(8÷12) 255
Interest income 255
(ii) Receivable - BL 3,400‒(100×10) 2,400
Contract / Refund liability- BL 100×10 1,000
Revenue 3,400
(iii) Revenue (W-2) 152
Contract / Refund liability- GL 152
(iv) Contract / Refund liability - DL 2,211
Revenue 3,411(W-4)‒1,200 2,211

W-1: Revenue to be recognized - AL Rs. in '000


Amount received 1,000
Present value of remaining balance 4,000×(1.12)-2 3,189
Total revenue 4,189

W-2: Price allocation – GL


Standalone price Price
Car-C 3,000 3,000×3,000÷3,160 2,848
Discount voucher 160 160×3,000÷3,160 152
(80%×200)
3,160 3,000

W-3: Price allocation – DL


Standalone price Price
Car-D 3,500 3,500×3,600÷3,800 3,316
Annual maintenance 300 300×3,600÷3,800 284
3,800 3,600

W-4: Revenue to be recognized – DL


Car-D (W-3) 3,316
Annual maintenance 284×4/12 95
Total revenue to be recognized 3,411

Page 3 of 7
Financial Accounting and Reporting-I
Suggested Answers
Certificate in Accounting and Finance – Autumn 2021

A.6 Saguaro Limited


Statement of cash flows for the year ended 30 June 2021

Indirect method
Rs. in million
Cash flows from operating activities
Loss for the year 315–50(500×10%)–220 (45)
Adjustments for:
Depreciation on property, plant and equipment 300+40(75–35)–262 78
Interest expense 45–21 24
Gain on disposal of property, plant and equipment (17)
Operating profit before working capital changes 40
Changes in working capital:
Increase in inventory 274–245 (29)
Decrease in trade receivables 177–204 27
Increase in advance to supplier 78–60 (18)
Increase in accrued expenses 48–43 5
Increase in trade payables (180–14) –130 36
21
Cash generated from operations 61

Interest paid 1+24–5 (20)


Net cash flows from operating activities 41

Cash flows from investing activities


Purchase of property, plant and equipment 70(84–14)+47(820+75–848) (117)
Proceeds from disposal of property, plant & equipment 31+52(35+17) 83
Net cash flows used in investing activities (34)

Cash flows from financing activities


Proceeds from issue of shares (700-500)–40 160
Repayment of long term loan (175+43–5)– (210+40–1) (36)
Dividend paid 500×10% (50)
Net cash flows from financing activities 74
Net increase in cash and cash equivalents 81
Cash and cash equivalents at beginning of the year 112
Cash and cash equivalents at the end of the year 193

W-1: Cash paid to suppliers Rs. in million


Cost of goods sold 485
Increase in inventories 274–245 29
Purchases 514
Increase in Advances to suppliers 78–60 18
Increase in Trade payables 180–14–130 (36)
496

W-2: Cash paid to other vendors Rs. in million


Operating expenses net of depreciation 310–78 232
Increase in Accruals 48–43 (5)
Page 4 of 7
Financial Accounting and Reporting-I
Suggested Answers
Certificate in Accounting and Finance – Autumn 2021

227

A.7 Bunny Ear Limited


Extracts from statement of profit or loss for the year ended 31 December 2020
2020 2019
------ Rs. in million ------
Depreciation:
 Factory building 625÷25×6÷12 12.5 -
 Manufacturing plant 420÷10×5÷12 17.5 -

Income from saving account 1.3+2.3(W-2) - 3.6

Grant income:
 Land 100÷25×6÷12 2.0 -
 Factory building 200÷25×6÷12 4.0
 Manufacturing plant 240÷10×5÷12 10.0

Interest expense (36+21–19); (22–6) 38.0 16.0

Extracts from statement of financial position as on 31 December 2020


2020 2019
------ Rs. in million ------
Non-current assets:
Property, plant and equipment
 Land 100.0 100.0
 Factory building 625(W-1)–12.5 612.5 -
 Manufacturing plant 420–17.5 402.5 -
 Capital work-in-process (W-1) - 326.0

Non-current liabilities:
Deferred government grant
 Land 100–2 98.0 100.0
 Factory building 200–4 196.0 200.0
 Manufacturing plant (forgivable loan) 240–10 230.0

Current liabilities:
 Running finance 350+200 550.0 250.0

W-1: Cost of factory building Rs. in million


Payments in 2019 (130+190) 320.0
Borrowing cost capitalized in 2019 (W-2) 6.0
Balance as at 31 December 2019 326.0

Payments in 2020 (180+100) 280.0


Borrowing cost capitalized in 2020 (W-2) 19.0
625.0

Page 5 of 7
Financial Accounting and Reporting-I
Suggested Answers
Certificate in Accounting and Finance – Autumn 2021

W-2: Utilisation of fund


No. of
Date Description Amount Balance Rate Amount
months
1-Feb-19 Grant 200 200 8.00% 1 1.3
1-Mar-19 1st payment 130 70 8.00% 5 2.3
1-Aug-19 2nd payment 190 (120) 12.00% 5 6.0
1-Jan-20 3rd payment 180 (300) 12.67% 6 19.0

W-3: Capitalization rate


36 + 21
× 100 = 12.67%
300 + 150

A.8 Cereus Golf Club


(a) Income and expenditure account for the year ended 31 December 2020
Rs. in '000
Income:
Subscription income (W-1) 10,750
Rent on golf kits 650
Sale of golf kits 3,000×140% 4,200
15,600
Expenditures:
Insurance expense 660–220(660÷12×4) (440)
Salaries expense 2,800–350 (2,450)
Cost of Golf kits sold [6,000(4,800÷0.8)×(2÷3)]–1,000 (3,000)
Loss on golf kits transferred 500×0.7 (350)
Depreciation 300(2,000×15%)+3,750(25,000×15%) (4,050)
Other expenses (2,320)
Loss on misappropriation 4,620(W-2) –2,000 (2,620)
(15,230)
Excess of income over expenditure 370

(b) Statement of financial position as on 31 December 2020


Rs. in '000
Non-current assets:
Fixed assets 25,000–3,750 21,250
Golf kits on rent (6,000×1÷3)+150(500×30%)–300(2,000×15%) 1,850
23,100
Current assets:
Golf kits (1,000–500) 500
Insurance claim 2,000
Prepaid insurance I/E 220
Subscription in arrears (W-1) 300
Cash & bank 4,200
7,220
30,320
General funds:
Opening balance (W-3) 12,350
Excess of income over expenditure I/E 370
12,720
Page 6 of 7
Financial Accounting and Reporting-I
Suggested Answers
Certificate in Accounting and Finance – Autumn 2021

Repair and maintenance fund 5,800-2,800 3,000

Liabilities:
Creditors - golf kits (4,800÷0.8)–4,800 1,200
Subscription in advance (W-1) 13,400
30,320

Workings
W-1: Subscription Rs. in '000
Opening advance
2018: (100×60×1÷3) 2,000
Income (Bal fig) 10,750 2019: (140×75×2÷3) 7,000

Closing advance Receipts (160×90)+(5×90) 14,850


2019: (140×75×1÷3) 3,500
2020: (165×90×2÷3) 9,900 Closing arrears (10×90×1÷3) 300
24,150 24,150

W-2: Bank/cash Rs. in '000


Opening balance 2,500 Repair & maintenance expense 2,800
Subscriptions (W-1) 14,850 Golf kits purchases 4,800
Rent of golf kits 650 Insurance premium paid 660
Sale of golf kits 4,200 Salaries paid 2,800
Other expenses paid 2,320
Misappropriation (bal. fig.) 4,620
Closing balance 4,200
22,200 22,200

W-3: Opening accumulated fund: Rs. in '000


Cash and bank balance 2,500
Fixed assets 25,000
Subscription in advance (W-1) (9,000)
Accrued salaries (350)
Repair and maintenance fund 8,000–2,200 (5,800)
Accumulated fund – opening balance 12,350

(THE END)

Page 7 of 7
INSTITUTE OF CHARTERED ACCOUNTANTS OF PAKISTAN

EXAMINERS’ COMMENTS

SUBJECT SESSION
Financial Accounting & Certificate in Accounting and Finance (CAF)
Reporting I Examination - Autumn 2021

Passing %

Question-wise
Overall
1 2 3 4 5 6 7 8
58% 66% 30% 66% 23% 47% 17% 15% 31%

General:

An overall passing ratio of 31% is lower than previous result of 40%. Decline in result was
mainly due to lower than expected performance in Q2, Q3 and Q5. Around 25% of the
examinees could secure maximum of 1 mark in these questions. The examinees performed
well in section A comprising of short questions but the performance in section B was poor
except for Q6. It was also observed that many examinees secured good marks in two or
three questions but failed to obtain reasonable marks in the remaining questions. It seems
that they did not study all areas of the syllabus on the assumption that these topics would
have least possibility of being examined.

There were many examinees securing marks in the 80s and even as high as 95.

Question-wise common mistakes observed

Question 1

 Cost of sales was used as denominator instead of purchases in calculation of trade


payables payment period.
 Examinees provided general comments e.g. debtor, creditor or sales should be
increased or decreased, instead of providing specific measures which should be taken.

Question 2

Value in use was calculated using post-tax discount rate instead of pre-tax discount rate.

Page 1 of 3
Examiners’ Comments on Financial Accounting & Reporting I – CAF Examination
Autumn 2021

Question 3

 Payment for Factory payroll was recorded but payroll was not transferred to ‘Work in
process’ and ‘Manufacturing overheads’.
 Entries for usage of raw material and completion of finished goods were not presented.

Question 4

 MCQs at serial (ii) and (iii) were not correctly answered by many examinees.
 Some examinees wasted valuable time in reproducing the wordings of correct option
instead of only mentioning the serial of the option.

Question 5

 Examinees did not read the requirement carefully and presented “Correct entries”
instead of “Correcting entries”. Also, workings were often haphazard and partial marks
could not be awarded as trail for the calculations was not available.
 In respect of (ii), revenue for 11 units was recognized instead of 11th unit only.
 In (iii), correction was often made with Rs. 160,000 instead of Rs. 152,000.
 In (iv), revenue from annual maintenance was needed to be recognised for 4 months
which was often not recognised at all or recognised in full.

Question 6

 Instead of following either direct or indirect method, examinees adopted hybrid


approach and computed a few amounts using direct method and other amounts using
indirect method.
 Examinees could not identify that Interest expense for the year need to computed as
difference between loss after interest (balancing amount in retained earnings) and loss
before interest.
 Purchase and depreciation of property, plant and equipment could not be correctly
calculated as examinees merged cost and accumulated depreciation and then got
confused.
 Payable against capital work in progress was not adjusted for presenting Trade
payables in working capital changes or cash paid to supplier.
 Share discount was not adjusted in calculating proceeds from issuance of shares.

Question 7

 Comparative figures for 2019 were not disclosed as required by the question.
 Interest earned on unused funds from grant was deducted from cost of factory building.
 Capitalization rate for 2020 was either not computed or also applied to 2019 as well.
 Portion of deferred grant related to land was not taken to profit or loss.
 Forgivable loan related to manufacturing plant was not treated as government grant.

Page 2 of 3
Examiners’ Comments on Financial Accounting & Reporting I – CAF Examination
Autumn 2021

Question 8

 Examinees could not prepare the subscription account and were mostly confused by
the fact that members were required to pay three years’ subscription.
 Only cash purchases of golf kits were taken as purchases.
 Golf kit for rent were not treated as non-current asset and consequently no depreciation
was charged on them.
 Loss from misappropriation was not reduced by the insurance amount and
corresponding receivable from insurance claim was not presented.
 Repair and maintenance fund was not included in calculation of ‘General fund’.

(THE END)

Page 3 of 3
Financial Accounting and Reporting - I
Summary of Marking Key
Certificate in Accounting and Finance – Autumn 2021

Note regarding marking scheme:


The marking scheme is given as a guide. Markers also award marks for alternative approaches to a
question and relevant/well-reasoned comments/explanations. Moreover, the available marks in
answer may exceed the total marks of a question.

Mark(s)
A.1 (a) 01 mark for calculation of each period/days 4.0

(b) 0.75 mark for each measure 3.0

A.2  Fair value less cost to sell 1.0


 Value in use – cash flows 4.0
 Value in use – discounting 2.0

A.3 01 mark for each entry 8.0

A.4 Marks as mentioned on the question paper against each MCQ 10.0

A.5  Matter (i) 5.0


 Matter (ii) 3.5
 Matter (iii) 3.0
 Matter (iv) 4.5

A.6  Operating profit before working capital changes 4.0


 Working capital changes 3.5
OR
 Cash receipt from customers 1.5
 Cash paid to suppliers 3.5
 Cash paid to other vendors 2.5
 Interest paid 1.0
 Cash flows from investing activities 4.0
 Cash flows from financing activities 3.5

A.7  Depreciation 2.5


 Grant income 2.0
 Interest expense and income 2.0
 Cost of factory building 4.5
 Other property, plant and equipment 2.5
 Deferred government grant 2.5

Page 1 of 2
Financial Accounting and Reporting - I
Summary of Marking Key
Certificate in Accounting and Finance – Autumn 2021

Mark(s)
A.8 (a)  Subscription income 3.0
 Profit and rent from Golf kits 3.5
 Insurance and depreciation 2.0
 Others 2.5

(b)  Non-current assets 2.0


 Current assets 3.0
 Funds 2.5
 Liabilities 1.5

(THE END)

Page 2 of 2
Certificate in Accounting and Finance Stage Examination

The Institute of 15 March 2022


Chartered Accountants 3 hours – 100 marks
of Pakistan Additional reading time – 15 minutes

Financial Accounting and Reporting-I


Instructions to examinees:
(i) Answer all NINE questions.
(ii) Answer in black pen only.
(iii) Multiple Choice Questions must be answered in answer script only.

Section A

Q.1 Bulan Pakistan Limited (BPL) is planning to commence construction of a warehouse on


1 January 2023 and is expecting to complete it by 30 November 2023. The management
wants to ascertain the borrowing costs that can be included in the cost of warehouse.
Relevant details in this respect are as follows:

(i) Expected payments related to the construction of the warehouse will be as follows:

Description Date of payment Rs. in million


1st bill of contractor 1-Feb-23 40
2nd bill of contractor 1-Apr-23 120
3rd bill of contractor 1-Sep-23 100
Last bill of contractor 1-Dec-23 90
350

(ii) The project can be financed through the following sources:


 Specific loan of Rs. 350 million at the rate of 16% per annum to be obtained on
1 January 2023. The principal will be payable in 5 equal annual instalments along
with interest, from 1 January 2024.
 Withdrawals to be made from existing running finance facilities. These facilities
will also be used to finance other needs of BPL. Details of these facilities are as
follows:

Expected average
Limit
Name of bank balance for 2023 Interest rates
------ Rs. in million ------
Bank A 300 220 13.7%
Bank B 350 280 14.6%

(iii) The surplus funds available from the loan will be invested in a saving account at
10% per annum.
(iv) The construction work is expected to be suspended for the entire month of June 2023
due to usual monsoon rains.

Required:
Calculate the borrowing costs to be capitalised in the cost of warehouse in each of the
following independent cases:
(a) if all the payments will be made from the specific loan only. (04)
(b) if all the payments will be made from running finance facilities only. (04)
Financial Accounting and Reporting-I Page 2 of 6

Q.2 Following information pertains to Dahl Limited (DL):

Summarised statement of financial position as at 31 December 2021


2021 2020 2021 2020
Rs. in million Rs. in million
Share capital 11.0 10.0 Property, plant and equipment 18.7 10.6
Retained earnings 32.9 33.8 Working capital other than cash 24.5 17.8
Revaluation surplus 4.0 - Cash 4.7 15.4
47.9 43.8 47.9 43.8

Additional information:
(i) Final dividend was paid in respect of year 2020 amounting to Rs. 3.4 million.
(ii) Additions to property, plant and equipment during the year amounted to
Rs. 14 million.
(iii) Tax expense for the year amounted to Rs. 2.4 million. Tax payable as at
31 December 2021 amounted to Rs. 1 million (2020: Rs. 0.2 million)

Required:
Prepare DL’s statement of cash flows for the year ended 31 December 2021. (08)

Q.3 Following is the trial balance of Mahtab Welfare Hospital (MWH) as on 31 December 2021:

Debit Credit
---- Rs. in million ----
Capital work in progress – hospital building 335
Cash at bank 60
Closing inventory – medicines and supplies 14
Contributions received 281
General fund as at 1 January 2021 332
Medical equipment 320 100
Medicines and supplies used 76
Other expenditures 19
Payables 17
Research cost 33
Restricted fund as at 1 January 2021 180
Salaries 53
Total 910 910

Additional information:
(i) The break-up of restricted fund balance is as follows:

Fund Description Rs. in million


Hospital building Contributions received for the construction of
120
fund hospital building.
As per the resolution of board of trustees,
Research fund MWH is required to allocate 20% of surplus 60
of each year to the research fund.

(ii) Contributions received include Rs. 55 million received for construction of hospital.
(iii) During the year, MWH also received construction materials having fair value of
Rs. 65 million for the hospital building which has not been recorded in books.
(iv) MWH has completed the construction of hospital building on 1 April 2021.
(v) Depreciation is to be charged as follows:
Hospital building 5% – straight line
Other fixed assets 10% – reducing balance
Financial Accounting and Reporting-I Page 3 of 6

Required:
Prepare the following using deferral method:
(a) Statement of income and expenditure for the year ended 31 December 2021 (04)
(b) Statement of financial position as at 31 December 2021 (06)

Q.4 Both IAS 16 ‘Property, Plant and Equipment’ and IAS 40 ‘Investment Property’ deal with
tangible non-current assets of an entity. Discuss any four differences between IAS 16 and
IAS 40. (06)

Q.5 The trial balance of Moon Mart (MM) did not agree as at 31 December 2021 and the
shortage of Rs. 215,000 on the debit side was carried to suspense account. The financial
statements prepared from the trial balance showed net profit of Rs. 1,431,000.

During review, following matters were noted:


(i) A return outward of Rs. 18,000 was posted to the debit of return inward account in
general ledger.
(ii) A sales invoice of Rs. 42,000 was posted twice in sales ledger.
(iii) Balance of accumulated depreciation of equipment was brought forward as
Rs. 641,000 instead of Rs. 461,000 on 1 January 2021.
(iv) Following entries in cash book were not posted to general ledger:
 Receipt of annual rent for the period ending 31 March 2022 amounting to
Rs. 336,000.
 Payment of Rs. 220,000 for equipment purchased on 1 May 2021.
 Cash purchases of Rs. 50,000.

Additional information:
(i) After passing all the adjustments, the remaining amount of suspense account is to be
considered as loss from embezzlement.
(ii) MM uses periodic inventory method. Control accounts are not maintained for trade
receivables and payables. Equipment are depreciated at 15% using reducing balance
method.

Required:
(a) Prepare suspense account. (04)
(b) Compute the corrected net profit. (04)

Q.6 Select the most appropriate answer(s) from the options available for each of the following
Multiple Choice Questions.

(i) A plant has a carrying amount of Rs. 3.3 million as at 31 December 2021. Its fair value
is Rs. 2.4 million and costs of disposal are estimated at Rs. 0.1 million. Cash flows
from the plant for the next 4 years are estimated at Rs. 0.7 million per annum. It will
be disposed of at the end of the 4th year for Rs. 0.6 million. Applicable discount rate is
10% per annum.

What is the approximate impairment loss on the plant to be recognized in the financial
statements for the year ended 31 December 2021?
(a) Rs. 1 million (b) Rs. 2.6 million
(c) Rs. 0.7 million (d) Rs. 1.1 million (02)

(ii) The forgivable loan from government is accounted for as _______________ if there is
no reasonable assurance that the entity will meet the terms for forgiveness of loan.
(a) a liability (b) an income
(c) a government assistance (d) a government grant (01)
Financial Accounting and Reporting-I Page 4 of 6

(iii) Which of the following statements is/are correct?

(I) Cash flows information cannot be manipulated easily, as compared to profit or


loss because it is not affected by different accounting policies.
(II) Cash flows information can be manipulated easily, as compared to profit or loss
because it is affected by different accounting estimates.
(a) Only (I) is correct (b) Only (II) is correct
(c) Both are correct (d) None is correct (01)

(iv) On 1 January 2019, a company purchased an asset for Rs. 5 million against which it
received the government grant of Rs. 0.5 million. The company deducted the grant
from the cost of asset. It is the policy of the company to depreciate such assets using
straight line method over ten years. On 1 January 2021, the government grant became
repayable due to non-fulfilment of conditions. Repayment of grant will result in
increasing:
(a) carrying value by Rs. 0.5 million (b) carrying value by Rs. 0.4 million
(c) expense by Rs. 0.4 million (d) expense by Rs. 0.5 million (02)

(v) As per IAS 20 ‘Accounting for Government Grants and Disclosure of Government
Assistance’, presenting the whole grant as other income in the statement of
comprehensive income or deducting it from a related expense, is the correct treatment
of:
(a) grant related to income
(b) forgivable loan expected to be received in next year
(c) government assistance in the form of free technical advice
(d) grant related to assets (01)

(vi) Which of the following statements is/are correct?

(I) The Conceptual Framework is not an IFRS and nothing in the Conceptual
Framework overrides any specific IFRS.
(II) One of the purpose of Conceptual Framework is to assist IASB to develop IFRSs
that are based on consistent concepts.
(a) Only (I) is correct (b) Only (II) is correct
(c) Both are correct (d) None is correct (01)

(vii) Which of the following may be presented in both statement of comprehensive income
and statement of cash flows?
(a) Purchase of non-current assets (b) Issuance of shares
(c) Repayment of loan (d) Depreciation (01)

(viii) Which TWO of the following are internal sources of assessing whether there is an
indication of impairment?
(a) An expected decline in the asset’s market value
(b) An increase in interest rates
(c) Evidence that the asset is damaged
(d) Evidence that the entity’s performance is worse than expected (01)
Financial Accounting and Reporting-I Page 5 of 6

Section B

Q.7 Qamar Limited (QL) is in the business of consumer goods. Following are the summarized
financial statements of QL for 2021:

Statement of financial position as at 31 December 2021


Assets Rs. in million Equity and liabilities Rs. in million
Fixed assets 550 Share capital 600
Retained earnings 319
Current assets: Long-term loan 350
Inventory 440 Current liabilities:
Trade debtors 350 Trade creditors 150
Short term investment 160 Other payables 70
Cash and bank balances 39 Current maturity of loan 50
1,539 1,539

Statement of profit or loss for the year ended 31 December 2021


Rs. in million
Sales 2,150
Cost of goods sold (1,900)
Gross profit 250
Selling and administrative expenses (93)
Other income 40
Finance cost (35)
Net profit 162

Extracts from management reports submitted to the board of directors:

(i) Ratios for the year 2020:

Gross profit margin 9.5% Net profit margin 3.9%


Interest cover 2.4 times Inventory holding period 90.4 days
Return on non-current assets 16.8% Debtors turnover 7.3 times
Creditor payment period 55.1 days Acid test 0.9 times

(ii) Important financial and operating decisions taken during the year 2021:
 QL renewed a large contract with a customer. In the renewed contract, extended
credit terms were given to the customer.
 A major supplier agreed to reduce the prices by 10% on the condition of cash
purchases only. This reduction helped QL to avoid increase in prices of its
products despite increase in prices by competitors.
 Increasing working capital demands were met by making a share issue. A part of
the proceeds from the issue were also used to prepay a significant portion of the
long term loan.
 QL disposed of its main warehouse in the last month of the year at a gain of
Rs. 25 million. The sale proceeds are temporarily invested in a short term
investment.

Required:
(a) Compute QL’s ratios for 2021 for comparison with 2020. (06)
(b) Keeping in view the financial and operating decisions extracted from management
reports, provide reasons for variation in the ratios computed in (a) above. (09)
Financial Accounting and Reporting-I Page 6 of 6

Q.8 Chand Limited (CL) was incorporated on 1 January 2020 with an authorized share capital of
Rs. 500 million comprising of 50 million shares.
(i) Details of shares issued are as follows:
 On 1 March 2020, CL issued 20 million shares at Rs. 18 each.
 On 1 October 2020, CL issued 15% bonus shares. The market price per share
immediately before the announcement of bonus was Rs. 24 per share.
 On 1 September 2021, CL issued 40% right shares at a premium of
Rs. 12.5 per share. The market price per share immediately before the entitlement
date was Rs. 33 per share.
(ii) Following information has been extracted from CL’s draft financial statements:
2021 2020
Draft Audited
--- Rs. in million ---
Net profit 66 48
Revaluation surplus arising during the year - 20
Transfer of incremental depreciation 4 -

Final cash dividend - 10%


(iii) After the preparation of draft financial statements for the year ended
31 December 2021, it was discovered that installation cost of Rs. 12 million relating to
a plant capitalized on 1 August 2020 was wrongly expensed out. The plant is
subsequently measured using cost model and is being depreciated @ 20% per annum
on reducing balance method.

Required:
(a) Prepare CL’s statement of changes in equity for the year ended 31 December 2021
along with comparative figures. (Column for total is not required) (09)
(b) Compute CL’s basic and diluted earnings per share to be disclosed in the statement of
profit or loss for the years ended 31 December 2021 and 2020. (08)

Q.9 Following information pertains to property, plant and equipment of Tsuki Limited (TL):
Office building Warehouse
Acquisition:
 Date of acquisition 1 July 2017 1 July 2018
 Cost (Rs. in million) 96 156
 Estimated useful life (in years) 16 12
Revalued amount:
 1 January 2019 (Rs. in million) 116 138
 1 January 2021 (Rs. in million) 80 143
Revised useful life on 1 January 2020 (in years) 9 14
Additional information:
(i) TL uses revaluation model for subsequent measurement and accounts for revaluation
on net replacement value method.
(ii) TL transfers maximum possible amount from the revaluation surplus to retained
earnings on an annual basis.
(iii) The revalued amounts were determined by Sagheer Valuers (Private) Limited, an
independent valuation company.

Required:
In accordance with IFRSs, prepare a note on ‘Property, plant and equipment’ (including
comparative information) for inclusion in TL’s financial statements for the year ended
31 December 2021. (Column for total is not required) (18)
(THE END)
Financial Accounting and Reporting-I
Suggested Answers
Certificate in Accounting and Finance – Spring 2022

A.1 (a) Borrowing cost to be capitalised – Specific borrowing


Rs. in
Date Description Utilization Balance Rate Months
million
Interest for 10 months 350 16% 10 46.67
1-Feb 1st bill of contractor 40 310 10% 2 (5.17)
1-Apr 2nd bill of contractor 120 190 10% 5 (7.92)
1-Sep 3rd bill of contractor 100 90 10% 3 (2.25)
31.33

(b) Borrowing cost to be capitalised – General borrowings


Rate Rs. in
Date Description Utilization Balance Months
(W-1) million
1-Feb 1st bill of contractor 40 40 14.2% 2 0.95
1-Apr 2nd bill of contractor 120 160 14.2% 5 9.47
1-Sep 3rd bill of contractor 100 260 14.2% 3 9.23
19.65

W-1: Capitalization rate


Average balance Amount
Rate
Rs. in million Rs. in million
71.02
220 13.7% 30.14 ⇒ ( ) ×100 = 14.2%
280 14.6% 40.88 500
500 71.02

Page 1 of 7
Financial Accounting and Reporting-I
Suggested Answers
Certificate in Accounting and Finance – Spring 2022

A.2 Dahl Limited


Statement of cash flows
For the year ended 31 December 2021
Rs. in million
Cash flow from operating activities
Profit before tax [2.5(32.9+3.4–33.8)+2.4] 4.9
Depreciation charge 18.7–10.6–4.0–14.0 9.9
Working capital other than cash (24.5+1.0)–(17.8+0.2) (7.5)
Cash generated from operation 7.3
Income tax paid 0.2+2.4–1.0 (1.6)
Net cash from operating activities 5.7

Cash flow from investing activities


Purchase of property, plant and equipment (14.0)
Net cash used in investing activities (14.0)

Cash flow from financing activities


Issuance of shares 11.0–10.0 1.0
Dividend paid (3.4)
Net cash from financing activities (2.4)
Net decrease in cash and cash equivalents (10.7)
Cash and cash equivalents at beginning of period 15.4
Cash and cash equivalents at end of period 4.7

Page 2 of 7
Financial Accounting and Reporting-I
Suggested Answers
Certificate in Accounting and Finance – Spring 2022

A.3 Mahtab Welfare Hospital


(a) Statement of income and expenditure for the year ended 31 December 2021
Rs. in million
Income:
Contribution 281–55 226
Hospital contributions 240(120+55+65)×5%×9/12 9
235
Expenditures:
Medicines used 76
Salaries 53
Other expense 19
Depreciation – Hospital 400(335+65)×5%×9/12 15
Depreciation – Other fixed assets (320–100)×10% 22
(185)
Excess of income over expenditure 50

(b) Mahtab Welfare Hospital


Statement of financial position as at 31 December 2021
Rs. in million
Non-current assets:
Hospital building 335+65–15 385
Medical equipment 320–100–22 198
583
Current assets:
Medicines and supplies 14
Cash at bank 60
74
657
Funds:
General fund 332+50–10(50×20%) 372
Research fund 60–33+10 37
409
Non-current liabilities:
Hospital deferred contribution 240(120+55+65)–9–12 219

Current liabilities:
Payables 17
Hospital deferred contribution 240×5% 12
657

Page 3 of 7
Financial Accounting and Reporting-I
Suggested Answers
Certificate in Accounting and Finance – Spring 2022

A.4 IAS 16 IAS 40


(i) Applicable to tangible assets held for use Applicable to property held to earn rental
in business including owner occupied or for capital appreciation
property
(ii) Allows cost or revaluation model for Allows cost or fair value model for
subsequent remeasurement subsequent remeasurement
(iii) Change in fair value are taken to OCI Change in fair value are taken to P&L
and/or P&L only
(iv) Assets under revaluation model are Assets under fair value model are not
depreciated depreciated

A.5 (a) Suspense Rs. in '000


Balance 215 iii) Accumulated depreciation 180
i) Return inward/outward 36 iv) Equipment 220
ii) Account receivable 42 iv) Purchases 50
iv) Rent 336 Loss (Bal fig) 179
629 629

(b) Effect on net profit: Rs. in '000


Given profit 1,431
(i) Return inward/outward 36
(iv) Rent income (336×9÷12) 252
(iv) Purchases (50)
(iv) Depreciation (180×15%)+(220×15%×8÷12) (49)
Loss from embezzlement (179)
Adjusted net profit 1,441

A.6 (i) (c) Rs. 0.7 million


(ii) (a) a liability
(iii) (a) Only (I) is correct
(iv) (b) the carrying value by Rs. 0.4 million
(v) (a) Grant related to income
(vi) (c) Both are correct
(vii) (d) Depreciation
(viii) (c) Evidence that the asset is (d) Evidence that the entity’s performance is
damaged worse than expected

Page 4 of 7
Financial Accounting and Reporting-I
Suggested Answers
Certificate in Accounting and Finance – Spring 2022

A.7 Ratio 2021 Reason


Gross profit 11.6% 2020: 9.5%
margin (250÷2150) ×100 Reduction in cost due to discount by
major supplier

Net profit margin 7.5% 2020: 3.9%


(162÷2150) ×100  Increase in gross profit margin /
Reduction in cost due to discount
by major supplier
 Increase in sales volume achieved
by not increasing prices.
 Profit from disposal of warehouse
 Reduced interest expense due to
repayment of loan

Interest cover 5.6 times 2020: 2.4 times


[(162+35)÷35]  Reduction in finance cost due to
repayment of loan
 Increase in overall profitability

Inventory holding 84.2 days 2020: 90.4 days


period (440÷1900)×365 Due to increase in sales volume
achieved by not increasing prices.

Return on non- 35.8% 2020: 16.8%


current assets (197÷550)×100  Increase in overall profitability
 Disposal of main warehouse

Debtors turnover 6.1 times 2020: 7.3 times


(2150÷350) Due to extended credit period offered
to new customer

Creditor payment 28.8 days 2020: 55.1 days


period (150÷1900)×365 Cash purchases from a major supplier
to avail discount

Acid test 2.03 2020: 0.9 times


(549÷270)  Due to funds generated from sale
of warehouse placed in short term
investments
 Decrease in payables due to cash
purchases

Page 5 of 7
Financial Accounting and Reporting-I
Suggested Answers
Certificate in Accounting and Finance – Spring 2022

A.8 Chand Limited


Statement of changes in equity
For the year ended 31 December 2021
Share Share Retained Revaluat-
capital premium earnings ion surplus
--------------- Rs. in million ---------------
Issuance of shares 200.0 160.0
Interim bonus dividend @ 15% 30.0 (30.0)
Total comprehensive income for the year:
Net profit – Restated 48+12–1 59.0
Other comprehensive income 20.0
Balance as at 31 December 2020 - Restated 230.0 130.0 59.0 20.0

Final dividend @10% (23.0)


Right issue (230×40%) ; ( 92×12.5/10) 92.0 115.0
Net profit 66–2.2(11×20%) 63.8
Transfer of incremental depreciation 4.0 (4.0)
Balance as at 31 December 2021 322.0 245.0 103.8 16.0

EPS for 2020 (Originally reported)


Basic & diluted 48÷19.17 (W-1) Rs. 2.50/share

EPS for 2021


Basic & diluted 63.8÷27.6 (W-2) Rs. 2.31/share

EPS for 2020 (Restated)


Basic & diluted 59÷21.09 (W-3) Rs. 2.80/share

W-1: Weighted average number of shares for 2020


1 March 20×7/12×1.15 13.42 ×1.1 14.76
1 October 23×3/12 5.75 ×1.1 6.33
20×10/12×1.15 19.17 ×1.1 21.09

W-2: Weighted average number of shares for 2021


1 January 23×8/12×1.1 (W-3) 16.87
1 September 32.2×4/12 10.73
(23×1.1)+(9.2×22.5/30×4/12) 27.60

W-3: Right adjustment factor


Ex-theoretical price:
40 × 22.5 = 900
100 × 33 = 3,300
140 4,200 ⇒ 4,200 ÷ 140 = 30

Right adjustment factor = Market price/Ex-theoretical price ⇒ 33/30 = 1.1

Page 6 of 7
Financial Accounting and Reporting-I
Suggested Answers
Certificate in Accounting and Finance – Spring 2022

A.9 Tuski Limited


Notes to the financial statement for the year ended 31 December 2021

1. Property, plant and equipment


2021 2020
Building Warehouse Building Warehouse
------------------------- Rs. in million -------------------------
Gross carrying amount - opening 116.00 138.00 116.00 138.00
Accumulated depreciation (20.00) (21.00) (8.00) (12.00)
(116÷14.5) (138÷11.5)
Opening carrying amount 96.00 117.00 108.00 126.00
Revaluation
- Surplus (16.00) 16.25
(96–80)
- P&L (W-1)9.75
Depreciation for the year (10.00) (11.00) (12.00) (9.00)
(80÷8) (143÷13) (108÷9) (126÷14)
Closing carrying amount 70.00 132.00 96.00 117.00

Gross carrying amount - closing 80.00 143.00 116.00 138.00


Accumulated depreciation (10.00) (11.00) (20.00) (21.00)
(8+12)
Closing carrying amount 70.00 132.00 96.00 117.00

1.1 Building Warehouse


Measurement base Revaluation Revaluation
Useful life 9 years 14 years
Depreciation method Straight line Straight line

1.2 The last revaluation was performed on 1 January 2021 by Sagheer valuation services, an
independent firm of valuers.

1.3 Had revaluations not been made, the carrying value of the buildings and warehouse as on
31 December 2021 would have been Rs. 63 million and Rs. 117 million respectively.

W-1: Revaluation of warehouse 2021 Rs. in million


Carrying value as on 1 January 2019 156–6.5(156/12×6/12) 149.50
Revalued amount 138.00
Revaluation loss taken to P&L 11.50
Additional depreciation that would have been charged in:
2019 11.5/11.5 (1.00)
2020 (11.5–1)/14 (0.75)
Revaluation gain taken to P&L 9.75

W-2: Carrying value on cost basis Building Warehouse


----------- Rs. in million -----------
Book value on 1 January 2020 81.00 136.5
(96×13.5÷16) (156×10.5÷12)
Book value on 31 December 2021 63.00 117
(81×7÷9) (136.5×12÷14)

(THE END)

Page 7 of 7
INSTITUTE OF CHARTERED ACCOUNTANTS OF PAKISTAN

EXAMINERS’ COMMENTS

SUBJECT SESSION
Financial Accounting and Reporting I Certificate in Accounting and Finance (CAF)
Spring 2022

Passing %

Question-wise
Overall
1 2 3 4 5 6 7 8 9
46% 46% 8% 52% 13% 54% 25% 23% 23% 23%

General comments

The overall result of 23% in this session is lower than the previous session’s result of
31%. Result of examinees who attempted this paper after passing Introduction to
Accounting (CAF-1) was 42% as compared to 17% for examinees who appeared in this
paper after obtaining exemption in Introduction to Accounting under the transition rules
of the Education Scheme 2021.

The performance of the examinees significantly varied from one answer script to
another answer script. There were many examinees who secured marks in the 80s and
even as high as 95. Some examinees secured good marks in three to four questions but
failed to obtain reasonable marks in the remaining questions. About one third
examinees did not secure any mark in Q.1, Q.3 and Q.5. It seems that they did not study
these areas of the syllabus on the assumption that these topics would have least
possibility of being examined. Some of the examinees were struggling to obtain the
easy marks available in the paper which could have been achieved with basic
preparation.

Question-wise common mistakes observed

Question 1

 Borrowing cost for June 2023 was not capitalized.


 Examinees often mixed up the concepts of specific and general borrowings.

Question 2

 Amounts of profit before tax and working capital changes were usually incorrect.
 A significant minority tried to solve the question using direct method despite the
fact that sufficient information was not available.

Page 1 of 3
Examiners’ Comments on Financial Accounting and Reporting I Spring 2022

Question 3

Examinees seemed to have no idea of the “deferral method”. This question was mostly
attempted half-heartedly in the last and therefore examinees could not even secure basic
marks available in the question which could have been easily obtained.

Question 4

Examinees mostly discussed difference no. (ii) and (iii) as appearing in the ICAP’s
suggested solution and struggled in identifying four differences.

Question 5

 Examinees prepared correcting entries which were not required. This resulted in
loss of valuable time and also affected the performance in other questions
 Amount of loss from embezzlement was neither calculated in suspense account nor
shown in computation of profit.
 In (a), effect of (ii) was taken to the credit of suspense account while effect of (iii)
was taken to the debit of suspense account.

Question 6

 MCQs at serial (ii) and (iv) were least well answered.


 Some examinees wasted valuable time in reproducing the wordings of correct
option instead of only mentioning the serial number of their preferred option.

Question 7

 About one fourth of the examinees did not secure any mark in the question despite
availability of very easy marks in part (a).
 In part (a), incorrect denominator was used in computing return on non-current asset
and incorrect numerator was used in computing interest cover. Further, debtors’
turnover was computed in ‘days’ instead of ‘times’.
 In part (b), reasons for variation were not linked with financial and operating
decisions provided in the question.

Question 8

 Authorized capital was presented as issuance of shares at the start of statement of


changes in equity.
 The word ‘restated’ was not mentioned for each line item which was different from
the previously reported amount due to impact of correction of error.
 EPS for 2020 was not restated for the impact of correction of error and right issue.
 Examinees could not realize the fact that basic and diluted EPS were the same.

Question 9

 Incomplete schedule of property, plant and equipment was prepared in terms of


presentation.

Page 2 of 3
Examiners’ Comments on Financial Accounting and Reporting I Spring 2022

 Depreciation for 2020 was calculated using ‘original life’ instead of ‘revised life’.
 Adjustment of revaluation of warehouse was not properly presented and/or
incorrectly bifurcated into revaluation surplus and profit or loss.
 Carrying values of assets if the cost model had been used were incorrectly
calculated.

(THE END)

Page 3 of 3
FINANCIAL ACCOUNTING AND REPORTING - I
Summary of Marking Key
Certificate in Accounting and Finance – Spring 2022

Note regarding marking scheme:


The marking scheme is given as a guide. Markers also award marks for alternative approaches to a
question and relevant/well-reasoned comments/explanations. Moreover, the available marks in
answer may exceed the total marks of a question.

Mark(s)
A.1 (a)  Calculation of interest cost 1.0
 Calculation of interest income 3.0

(b)  Calculation of capitalization rate 1.0


 Calculation of borrowing cost 3.0

A.2  Calculation of profit before tax 2.0


 Other items in cash flow from operating activities 3.5
 Cash flow from investing and financing activities 2.5

A.3 (a)  Income 2.0


 Expenditures 2.0

(b)  Assets 1.5


 Equities 4.5

A.4 1.5 marks for each difference 6.0

A.5 (a) Up to 0.75 mark for each posting 4.0

(b) Up to 0.75 mark for each effect 4.0

A.6 Marks as mentioned on the question paper against each MCQ 10.0

A.7 (a) 0.75 mark for calculation of each ratio 6.0

(b) Up to 0.75 mark for each reason 9.0

A.8 (a)  Calculation of corrected profits 3.0


 Presentation of changes in equity for 2020 3.0
 Presentation of changes in equity for 2021 3.0

(b)  EPS for 2020 4.0


 EPS for 2021 4.0

Page 1 of 2
FINANCIAL ACCOUNTING AND REPORTING - I
Summary of Marking Key
Certificate in Accounting and Finance – Spring 2022

Mark(s)
A.9  Presentation of note 2.0
 Opening balances for 2020 3.0
 Depreciation 3.5
 Revaluation adjustments 4.5
 Other disclosures 5.0

(THE END)

Page 2 of 2
Certificate in Accounting and Finance Stage Examination

The Institute of 13 September 2022


Chartered Accountants 3 hours – 100 marks
of Pakistan Additional reading time – 15 minutes

Financial Accounting and Reporting-I


Instructions to examinees:
(i) Answer all NINE questions.
(ii) Answer in black pen only.
(iii) Multiple Choice Questions must be answered in answer script only.

Section A

Q.1 Consider the following statements with reference to ‘Conceptual framework for financial
reporting’:

(i) Physical capital maintenance measures profit in terms of increase in the productive
capacity of an entity.
(ii) In times of rising prices, profits will be overstated and assets will be understated when
financial statements are prepared on the basis of historical cost.
(iii) Income represents all increases in assets or decreases in liabilities that result in increase
in equity.
(iv) To be a perfectly faithful representation, a depiction would have three characteristics.
It would be complete, relevant and verifiable.
(v) In value in use method, assets are measured at the amount that would be paid to
purchase the same or a similar asset currently.
(vi) Current cost and fair value are exit values.
(vii) Requirements of a standard overrides the requirements of conceptual framework.
(viii) Financial capital maintenance is likely to be the most relevant to investors as they are
interested in maximizing the return on their investment and purchasing power.

Required:
Identify whether each of the above statements is TRUE or FALSE. Give reasons for
statements identified as FALSE. (07)

Q.2 Discuss how the following should be dealt with in the current year’s financial statements of
relevant entities in accordance with IAS 20.

(a) Xero Limited (XL) received a government grant to setup a plant in an under-developed
rural area three years ago. One of the conditions of the grant was that XL will maintain
a minimum of 200 employees during the next five years. However, due to worsening
economic conditions, XL failed to maintain 200 employees and the full grant became
repayable immediately in the current year.

XL has been presenting the grant in statement of financial position by deducting the
grant in arriving at the carrying value of the plant. (04)

(b) One Limited received a loan from government in the current year at an interest rate of
5% per annum. The prevailing market interest rate is 12% per annum. The only
condition attached to the loan is that it should be used for acquisition of textile
machinery. (03)
Financial Accounting and Reporting-I Page 2 of 6

Q.3 Oracle Family Club (OFC) was formed in January 2021. The following information is
available in respect of the first year of operations:

Receipt and payment account for the year ended 31 December 2021
Receipts Rs. in '000 Payments Rs. in '000
Subscriptions for: Salaries 640
 2021 2,800 Rent 990
 2022 1,360 Equipment 2,560
Joining fees 2,100 10% Fixed deposit 2,020
Canteen sales 720 Construction of building 1,500
Life-time memberships 1,840 Canteen purchases 700
Closing balance 410
8,820 8,820

Income and expenditure account for the year ended 31 December 2021
Expenditures Rs. in '000 Incomes Rs. in '000
Salaries 700 Subscription 3,450
Rent 760 Interest on fixed deposit 150
Depreciation of equipment 200 Life-time memberships 360
Surplus 2,330 Profit from canteen 30
3,990 3,990

Additional information:
(i) OFC also operates a canteen. All sales and purchases of canteen are made for cash.
(ii) Salary of canteen’s salesman amounted to Rs. 90,000 is included in payments.

Required:
Prepare OFC’s statement of financial position as on 31 December 2021. (10)

Q.4 During the review of accounting records and financial statements for the year ended
30 June 2022 of Tally Traders, following errors were highlighted:

(i) Sales included an outstanding balance of Rs. 500,000 for which a customer would need
to pay Rs. 485,000 only if payment is made within 30 days. The customer is expected
to pay within 30 days.
(ii) An item was included in closing inventory at its net realizable value of Rs. 490,000.
However, the item had a cost of Rs. 450,000.
Periodic inventory method is used to record the inventory transactions.
(iii) A sub-total of Rs. 234,000 was carried forward in the purchase day book as Rs. 432,000.
Control accounts are not maintained for Debtors and Creditors.
(iv) A credit note issued to a customer of Rs. 128,000 was recorded as credit note received
from supplier.
(v) An office machine costing Rs. 3,540,000 with a carrying value of Rs. 2,040,000 as on
1 July 2021 was disposed of on 28 February 2022 for Rs. 1,860,000. The sale proceeds
were credited to accumulated depreciation account and full year’s depreciation was
provided on the machine.
Office machines are depreciated at 10% per annum using reducing balance method.

Required:
Prepare journal entries to correct the above errors. (Narrations are not required) (08)
Financial Accounting and Reporting-I Page 3 of 6

Q.5 On 1 March 2017, Zarmoney Limited imported an automatic plant for Rs. 130 million. The
commissioning of the plant was completed on 1 January 2018 at a cost of Rs. 10 million. The
economic life of the plant was estimated as 12 years and useful life of the plant was estimated
as 8 years. The plant is being depreciated at 20% per annum using reducing balance method.

Due to declining demand for the product manufactured from this plant, an impairment test
was carried out at 31 December 2021. Following information has been gathered for
impairment testing of the plant:
(i) The current selling price of a similar plant in the local market is Rs. 50 million. The
present decommissioning cost of the plant is estimated at Rs. 2 million.
(ii) The plant’s net disposal proceeds at the end of the useful life is estimated at
Rs. 4 million.
(iii) The current market risk-free rate of interest is 8% per annum, however, an investor
would ask additional return of 2% for bearing the uncertainty inherent in such a plant.
(iv) A junior accountant has calculated following net cash flows from operating the plant:

Year 2022 2023 2024 2025


Net cash inflow (Rs. in million) 11 7 3 1

However, a review of accountant’s working has revealed the following:


 Depreciation of the plant has been included as an outflow in each year.
 Tax payments of Rs. 2 million has been included as an outflow in each year.
 Inflows from plant in 2022 include receipts from sale of existing inventory
amounting to Rs. 3 million

Required:
Compute the impairment loss (if any) in the value of the plant to be recognised on
31 December 2021. (Show all necessary workings) (08)

Q.6 Select the most appropriate answer(s) from the options available for each of the following
Multiple Choice Questions.

(i) Which of the following is NOT a limitation of ratio analysis?

(a) Use of different accounting policies and estimates


(b) Use of different formulas for calculating ratios
(c) Different inflation rates in different years
(d) Companies pursuing different strategies (01)

(ii) If the existing current ratio of a company is more than 1, what would be the impact of
a credit purchase of inventory on the current ratio?

(a) Current ratio would increase


(b) Current ratio would decrease
(c) Current ratio would decrease but would remain higher than 1
(d) Current ratio would remain same (01)

(iii) Which of the following statements are correct?

(I) Giving incentives to customer to pay on time would result in decrease in debtor’s
turnover in times.
(II) If all debtors pay their debts within the credit period, the average collection period
would be Nil.

(a) Only (I) is correct (b) Only (II) is correct


(c) Both are correct (d) None is correct (01)
Financial Accounting and Reporting-I Page 4 of 6

(iv) Which TWO of the following would improve gearing ratio of a company?

(a) Issuance of shares at discount


(b) Repayment of a bank loan
(c) Issuance of bonus shares
(d) Disposal of a land at its carrying value (01)

(v) Which of the following changes would be considered as change in accounting policy?

(I) Changing the subsequent measurement model for property, plant and equipment
from cost model to revaluation model.
(II) Changing the inventory valuation method from FIFO to Weighted average.

(a) Only (I) is change in accounting policy


(b) Only (II) is change in accounting policy
(c) Both are change in accounting policy
(d) None is change in accounting policy (01)

(vi) On 1 January 2021, a company borrowed Rs. 20 million @ 9% per annum for the
purpose of constructing an asset. The company started construction on 1 February 2021
and paid Rs. 8 million on 1 March 2021 and Rs. 12 million on 1 July 2021. The asset
was ready to use on 1 September 2021. Surplus funds were invested @ 6% per annum.

The borrowing cost that can be capitalized is:

(a) Rs. 660,000 (b) Rs. 710,000


(c) Rs. 900,000 (d) Rs. 1,050,000 (02)

(vii) Which of the following statements are correct?

(I) Investment income on the temporary investment of unused funds of general


borrowings is taken to profit or loss.
(II) Capitalisation of borrowing cost always commences as soon as construction of a
qualifying asset begins.
(a) Only (I) is correct (b) Only (II) is correct
(c) Both are correct (d) None is correct (01)

(viii) Which TWO of the following would be shown as a deduction from the column of
retained earnings in statement of changes in equity?

(a) Transfer of incremental depreciation


(b) Issuance of shares at discount
(c) Cash dividend
(d) Transfer to general reserves (01)

(ix) Which TWO of the following situations would require prior year adjustment as per
IAS 8?
(a) Changing the depreciation method from straight line basis to the reducing balance
basis in respect of a building held for the last 10 years.
(b) Changing the measurement model for Investment property from cost model to
fair value model.
(c) Incorporating the effects of a material understatement found in last year closing
inventories due to incorrect formula in excel sheet.
(d) Adopting the requirements of IAS 20 for a government grant received by an entity
for the first time. (01)
Financial Accounting and Reporting-I Page 5 of 6

Section B

Q.7 Following is the statement of financial position of Quicken Limited (QL) as at 30 June 2022:
2022 2021 2022 2021
Rs. in million Rs. in million
Share capital 480 400 Land and building 748 526
Revaluation surplus 135 - Vehicles 118 96
Retained earnings 337 325 Inventories 365 444
Long-term loan 335 460 Trade and other receivables 212 185
Trade and other payables 160 142 Cash and bank balances 73 111
Advance from customers 69 35
1,516 1,362 1,516 1,362

Additional information:
(i) During the year, land and building were revalued for the first time, resulting in a surplus
of Rs. 150 million and incremental depreciation of Rs. 15 million.
(ii) Depreciation on building charged to profit or loss amounted to Rs. 72 million.
(iii) During the year, vehicles having book value of Rs. 8 million were sold for
Rs. 11 million received in cash. Further, sale proceeds of Rs. 6 million of another
vehicle (book value Rs. 7 million) disposed of in May 2021 were received in
August 2021.
(iv) Vehicles costing Rs. 51 million were purchased during the year of which Rs. 12 million
is still unpaid.
(v) Inventories as at 30 June 2022 included work in process inventories of Rs. 96 million
(2021: Rs. 80 million) which are not available for sale.
(vi) Interest on loan for the year amounted to Rs. 48 million of which Rs. 14 million was
capitalised in the cost of a building constructed during the year.
(vii) Following dividends were announced for the year ended 30 June 2022 and 2021:
2022 20% interim bonus shares and 15% final cash dividend
2021 5% interim bonus shares and 10% final cash dividend

Required:
Prepare QL’s statement of cash flows for the year ended 30 June 2022. (15)

Q.8 Peach Tree Limited (PTL) was incorporated on 1 July 2020. Following information has been
extracted from its financial statements for the year ended 30 June 2022:
2022 2021
---- Rs. in million ----
Net profit 250 210
Revaluation surplus arising during the year 30 50
Total comprehensive income 280 260

Details of shares and bonds issued by PTL since incorporation are as follows:
(i) On 1 July 2020, 50 million ordinary shares having par value of Rs. 10 each were issued
at Rs. 14 each.
(ii) On 1 July 2020, 10 million 12% redeemable preference shares having par value of
Rs. 50 each were issued at Rs. 64 each. Each preference share is convertible into
3 ordinary shares after 5 years.
(iii) On 1 February 2021, further 20 million ordinary shares having par value of Rs. 10 each
were issued at prevailing market price of Rs. 16 each.
(iv) On 1 October 2021, 40% right shares were issued at a premium of Rs. 10 per share. The
market price per share immediately before the entitlement date was Rs. 30 per share.
(v) On 1 November 2021, 3 million convertible bonds having par value of Rs. 100 each
were issued. The bonds carry interest @ 10% per annum payable on 31 October each
year. Each bond is convertible into 7 ordinary shares after 3 years.
Financial Accounting and Reporting-I Page 6 of 6

Required:
Compute basic and diluted earnings per share to be disclosed in PTL’s financial statements
for the years ended 30 June 2021 and 2022. (Show comparative figures) (15)

Q.9 Following information pertains to non-current assets of GnuCash Limited (GL):

(i) GL purchased a manufacturing plant for Rs. 340 million on 1 January 2021. On that
date, the plant had an estimated useful life and residual value of 13 years and
Rs. 60 million respectively. The revalued amounts and residual value were as follows:
Revalued amount Residual value
----------- Rs. in million -----------
30 June 2021 304 54
30 June 2022 315 44

(ii) A warehouse owned by GL was given on rent on 1 January 2022. Previously, the
warehouse was in use of GL.

The warehouse was acquired by GL on 1 July 2019 at a cost of Rs. 200 million and is
being depreciated @ 10% per annum on reducing balance method.

Fair value of the warehouse on various dates are as follows:

Rs. in million
1 January 2022 206
30 June 2022 214

Rentals earned for the year ended 30 June 2022 amounted to Rs. 10 million out of
which Rs. 6 million is still outstanding.

(iii) GL acquired a property comprising of three similar showrooms at a total cost of


Rs. 900 million on 1 October 2021. 40% of the cost of property is attributable to the
value of land. Each of the showroom can be leased out separately and has a useful life
of 15 years with no residual value.

GL is using one showroom for its own products while the other showrooms were held
to be leased out. On 1 March 2022, the two showrooms were given on monthly rent of
Rs. 4 million.

The fair value of each showroom is increasing by Rs. 3 million each month.

Other information:
 Cost model is used for subsequent measurement of all property, plant and equipment
except for manufacturing plant for which revaluation model is used.
 Maximum possible amount is transferred from the revaluation surplus to retained
earnings on an annual basis.
 Fair value model is used for subsequent measurement of all investment properties.

Required:
Prepare notes on ‘Property, Plant and Equipment’ and ‘Investment Property’, for inclusion
in GL’s financial statements for the year ended 30 June 2022.
(Comparative figures and column for total are not required) (20)

(THE END)
Financial Accounting and Reporting-I
Suggested Answers
Certificate in Accounting and Finance – Autumn 2022

A.1 (i) True


(ii) True
(iii) False
Income does not include those increase in equity which are relating to contributions
from holders of equity claims.
(iv) False
The three characteristics are complete, neutral and free from error.
(v) False
Value in use is the present value of the cash flows, or other economic benefits that an
entity expects to derive from the use of an asset and from its ultimate disposal.
(vi) False
Current cost is an entry value while fair value is an exit value.
(vii) True
(viii) True

A.2 (a)  When a government grant becomes repayable it is accounted for as a change in
accounting estimate.
 As the grant was presented as deduction from related plant, its repayment would
be recognized by increasing the carrying value of the plant
 The cumulative additional depreciation that would have been recognized in profit
or loss to date in the absence of the grant must be recognized immediately in profit
or loss.
 Also the circumstances giving rise to repayment of the grant might indicate the
possible impairment of the new carrying amount of the plant.

(b)  The benefit of the government loan at a below market rate of interest is treated as
a government grant. The loan shall be recognised and measured as per IFRS 9.
 Government grant should be recorded as the difference between the initial carrying
amount of the loan and the proceeds received.
 As the primary condition for the loan is acquisition of textile machinery, the grant
should be considered as grant related to asset and should be recognized in profit
or loss over the life of the machinery.
 The grant may be presented in the statement of financial position by setting up the
grant as deferred income or by deducting the grant in arriving at the carrying value
of the machinery.

Page 1 of 7
Financial Accounting and Reporting-I
Suggested Answers
Certificate in Accounting and Finance – Autumn 2022

A.3 Statement of financial position as on 31 December 2021


Rs. in '000
Non-current assets:
Equipment 2,560–200 2,360
Capital work in progress - Building 1,500
3,860
Current assets:
Canteen inventory (W-1) 100
Subscription in arrears 3,450–2,800 650
Prepaid rent 990–760 230
Fixed deposit 2,020
Interest receivable 150
Cash 410
3,560
7,420
General funds:
Opening balance -
Excess of income over expenditure 2,330
2,330

Joining fees fund 2,100

Liabilities:
Deferred life membership 1,840–360 1,480
Salaries payable (700+90)–640 150
Subscription in advance 1,360
7,420

W-1: Canteen inventory Rs. in '000


Purchases A 700
Cost of goods sold:
 Sales 720
 Gross profit 30+90 (120)
B 600
A-B 100

Page 2 of 7
Financial Accounting and Reporting-I
Suggested Answers
Certificate in Accounting and Finance – Autumn 2022

A.4 Correcting Entries


Debit Credit
S. # Description
----- Rs. in '000 -----
(i) Revenue/Sales/Discount allowed 15
Receivables 15
(ii) Cost of goods sold/Trading account 490–450 40
Closing inventory 40
(iii) Suspense 432–234 198
Purchases 198
(iv) Sales return / Return inward 128
Purchase return / Return outward 128
Accounts Payables 128
Account Receivables 128
(v) Depreciation Expense (W-1) 118
Accumulated depreciation 118
Accumulated Depreciation (3,540–2,040)+136+1,860 3,496
Loss on disposal (2,040–136)–1,860 44
Machine 3,540

W-1: Computation Rs. in '000


Correct depreciation 2,040×10%×8÷12 136
Depreciation charged (2,040–1,860)×10% (18)
118

A.5 Computation of impairment of plant ------- Rs. in million -------


Carrying value 140(130+10) ×(0.8)4 57.3
Less: Recoverable amount
Value in use (W-1) 52.6
Fair value less cost of sell 50–2 48.0
Higher of above 52.6
Impairment 4.7

W-1: Value in use 2022 2023 2024 2025


--------------------- Rs. in million ---------------------
Net cash inflow 11.0 7.0 3.0 1.0
Depreciation 11.5 9.2 7.4 5.9
Income tax 2.0 2.0 2.0 2.0
Sale of inventory (3.0) - - -
Disposal value - - - 4.0
Cash flows undiscounted 21.5 18.2 12.4 12.9
Discount factor @ 10% 0.909 0.826 0.751 0.683
PV of cash flows /value in use 19.5 15.0 9.3 8.8 52.6

Page 3 of 7
Financial Accounting and Reporting-I
Suggested Answers
Certificate in Accounting and Finance – Autumn 2022

A.6 (i) (b) Use of different formulas for calculating ratios


(ii) (c) Current ratio would decrease but would remain higher than 1
(iii) (d) None is correct
(iv) (a) Issuance of shares at discount
(b) Repayment of a bank loan
(v) (c) Both are change in accounting policy
(vi) (a) Rs. 660,000
(vii) (a) Only (I) is correct
(viii) (c) Cash dividend
(d) Transfer to general reserves
(ix) (b) Changing the measurement model for Investment property from cost model to
fair value model.
(c) Incorporating the effects of a material understatement found in last year closing
inventories due to incorrect formula in excel sheet.

A.7 Quicken Limited


Statement of cash flows
For the year ended 30 June 2022
Rs. in million
Cash flow from operating activities
Profit 12(337-325)+40+80–15 117
Adjustments for:
Depreciation expense 72+21(96+51–8–118) 93
Interest expense 48–14 34
Gain on disposal of vehicle 11–8 (3)
124
Operating profit before working capital changes 241
Changes in working capital:
Decrease in inventories 365–444 79
Increase in trade and other receivables 27(212–185)+6 (33)
Increase in trade and other payables 18(160–142)–12 6
Increase in advances from customers 69–35 34
86
Cash generated from operations 327
Interest paid 48–14 (34)
Net cash flows from operating activities 293
Cash flow from investing activities
Purchase of Land and Building (526–748)+150–72 (144)
Purchase of Vehicles 51–12 (39)
Sale proceeds of vehicle 11+6 17
Net cash used in investing activities (166)
Cash flow from financing activities
Repayment of loan 335–460 (125)
Dividend paid 400×10% (40)
Net cash used in financing activities (165)
Net decrease in cash and cash equivalents (38)
Cash and cash equivalents at beginning of period 111
Cash and cash equivalents at end of period 73

Page 4 of 7
Financial Accounting and Reporting-I
Suggested Answers
Certificate in Accounting and Finance – Autumn 2022

A.8 EPS for 2021 (Originally reported)

Basic EPS ⇒ 210


= Rs. 3.60/share
58.34(W-1)

Diluted EPS ⇒ 210+60(500×12%) 270


= = Rs. 3.05/share
58.34+30(10×3) 88.34

EPS for 2022:

Basic EPS ⇒ 250


= Rs. 2.69/share
92.84(W-2)

Diluted EPS (after ⇒ 250+20(300×10%×8÷12) 270 Rs. 2.53/share


= =
including bond) 92.84+14(3×7×8÷12) 106.84 Dilutive

Diluted EPS (after 270+60(500×12%) 330


⇒ Rs. 2.41/share
including preference 106.84+30(10×3) = 136.84 =
Dilutive
shares)

EPS for 2021 (Reported as comparative in 2022)

Basic EPS ⇒ 210 210


= = Rs. 3.26/share
58.34×(30/27.14) 64.48

Diluted EPS ⇒ 210+60(500×12%) 270 Rs. 2.86/share


= =
64.48+30(10×3) 94.48 Dilutive

W-1: Weighted average number of shares for 2021


1 Jul 50×7/12 29.17
1 Feb 70×5/12 29.17
58.34

W-2: Weighted average number of shares for 2022


1 Jul (70×3/12)×(30/27.14) (W-3) 19.34
1 October 98(70×1.4)×9/12 73.50
92.84

W-3: Right adjustment factor

Right adjustment factor ⇒ Market price 30


=
Ex-theoretical price 27.14

Ex-theoretical price:
40 × 20 = 800
100 × 30 = 3,000
140 3,800 ⇒ 3,800 ÷ 140 = 27.14

W-4 : Ranking of potential ordinary shares


Increase in Increase in Incremental
Description Rank
earnings shares EPS
Rs. in million In million
Preference shares 60 30 Rs. 2/share 2
Convertible bonds 20 14 Rs. 1.43/share 1

Page 5 of 7
Financial Accounting and Reporting-I
Suggested Answers
Certificate in Accounting and Finance – Autumn 2022

A.9 GnuCash Limited (GL)


Notes to the financial statements for the year ended 30 June 2022
1 Property, plant and equipment
Manufacturing Showrooms
Warehouse
plant Land Building
---------------------- Rs. in million ----------------------
Gross carrying amount – opening 304.0 200.0 - -
Accumulated depreciation - (38.0) - -
(20+18)
Opening carrying amount 304.0 162.0 - -
Addition 120.0 180.0
(300×40%) (300×60%)
Depreciation for the year (20.8) (8.1) - (9.0)
(304–44)÷12.5) (162×10%×6÷12) (180÷15)×9÷12
Revaluation
- P&L (W-1)23.0
- Surplus 8.8 52.1
Reclassification (206.0)
Closing carrying amount 315.0 - 120.0 171.0
Gross carrying amount - closing 315.0 - 120.0 180.0
Accumulated depreciation - - (9.0)
Closing carrying amount 315.0 - 120.0 171.0

1.1 Manufacturing
Warehouse Showroom
plant
Measurement base Revaluation Cost model Cost model
Useful life/depreciation rate 12.5 years 10% 14 years
Depreciation method Straight line Reducing balance Straight line

1.2 Had revaluations not been made, the carrying value of the plant as on 31 December 2022 would
have been Rs. 306.2 (W-2) million.

1.3 The last revaluation was performed on 30 June 2022 by an independent firm of valuers.

2 Investment property Warehouse Showroom


------ Rs. in million ------
Opening carrying amount - -
Additions - 600.0
(900×2/3)
Transfer from property, plant and equipment 206.0 -
Fair value adjustment 8.0 54.0
(214–206) (3×2×9)
Closing carrying amount 214.0 654.0

2.1 Measurement basis


All assets in investment property are subsequently measured at fair value.

2.2 The revaluation was performed by an independent firm of valuers.

2.3 Rental income


The rental income for the year ended 30 June 2022 is Rs. 26(10+4×4) million.

W-1: Revaluation of plant 2022 Rs. in million


Carrying value as on 30 June 2021 329
Revalued amount 304
Revaluation loss taken to P&L 25
Additional depreciation that would have been charged in 2022 [22.8(W-2)–20.8] (2)
Revaluation gain taken to P&L 23
Page 6 of 7
Financial Accounting and Reporting-I
Suggested Answers
Certificate in Accounting and Finance – Autumn 2022

W-2: Book value of plant under cost basis Rs. in million


Cost as on 1 January 2021 340
Depreciation for 2021 (340–54)÷13×(6÷12) (11)
Depreciation for 2022 (329–44) ÷12.5 (22.8)
Book value as on 30 June 2022 (315–8.8) 306.2

(THE END)

Page 7 of 7
INSTITUTE OF CHARTERED ACCOUNTANTS OF PAKISTAN

EXAMINERS’ COMMENTS

SUBJECT SESSION
Financial Accounting and Reporting I Certificate in Accounting and Finance (CAF)
Autumn 2022

Passing %

Question-wise
Overall
1 2 3 4 5 6 7 8 9
55% 12% 27% 23% 54% 35% 48% 17% 19% 25%

General comments

The current result of 25% is consistent with the previous result of 23%. The result of
examinees who attempted this paper after passing Introduction to Accounting was 40%
as compared to 21% for examinees who attempted this paper after obtaining exemption
in Introduction to Accounting due to the transition to Education Scheme 2021.

Many examinees secured marks in the 80s and even as high as 88. It was commonly
noted that many examinees secured good marks in three to four questions but failed to
obtain reasonable marks in the remaining questions. The element of the selective study
was evident from the fact that a number of examinees secured a maximum of 1 mark in
Q2, Q3, Q4, and Q8 whereas numerous other examinees secured full marks in these
questions. The examinees struggled to obtain the easy marks available in the paper
which could have been achieved with just basic preparation of the topic.

Question-wise common mistakes observed

Question 1

 Statements at serial (iii) and (vi) were least well answered.


 Reasons for false statements were not mentioned as required by the question.

Question 2

 Examinees just reproduced the wording of the question.


 Entries and calculations were presented that were not required by the question.

Question 3

 Overall approach to handling the question was missing and the performance was
lower than expected in such an easy question. Examinees made basic mistakes; for
example, reported equipment without deducting depreciation, computed opening
general fund and omitted interest receivable and cash balance.

Page 1 of 2
Examiners’ Comments on Financial Accounting and Reporting I Autumn 2022

 Examinees could not work out the amount of closing inventory on the basis of the
canteen trading account.

Question 4

 In error (ii), inventory was recorded by Rs. 450,000 instead of reducing the
inventory by Rs. 40,000.
 In error (iv), corrections to trade receivables and trade payables were omitted.

Question 5

 Discount rate of 8% was applied instead of 10% for calculating the value in use.
 Cash flows were not adjusted for depreciation and tax while computing value in
use.

Question 6

 MCQs at serial (ii) and (vi) were least well answered.


 Some examinees wasted valuable time in reproducing the wordings of the correct
option instead of mentioning the serial of the option.

Question 7

 Transfer of incremental depreciation was not considered in computing “profit”.


Further, the incremental depreciation was shown in “adjustments for”.
 Loss on disposal of the vehicle disposed of in the previous year was also shown in
‘adjustments for’.
 Examinees could not identify that the impact of work in process inventories was not
required to be excluded while calculating the change in inventories.

Question 8

 Examinees were found lacking even the elementary concepts of calculating basic
earnings per share.
 Calculations for diluted earnings per share were mostly altogether incorrect or were
left unattempted. Also, workings were haphazard and partial marks could not be
awarded as the trail for the calculations was not available.

Question 9

 Schedule of property, plant and equipment was incomplete in terms of presentation.


 Adjustment of revaluation of the warehouse was not properly presented and/or
incorrectly bifurcated into revaluation surplus and profit or loss.
 The revaluation of the warehouse before reclassification into investment property
was not presented in the note of property, plant and equipment.
 Note on investment property was not presented at all.

(THE END)

Page 2 of 2
FINANCIAL ACCOUNTING AND REPORTING - I
Summary of Marking Key
Certificate in Accounting and Finance – Autumn 2022

Note regarding marking scheme:


The marking scheme is given as a guide. Markers also award marks for alternative approaches to a
question and relevant/well-reasoned comments/explanations. Moreover, the available marks in
answer may exceed the total marks of a question.

Marks
A.1  True statements 3.0
 False statements 4.0

A.2 (a) 01 mark for each point 4.0

(b) Up to 01 mark for each point 3.0

A.3  Non-current assets 1.5


 Current assets 4.5
 Funds 2.0
 Liabilities 2.0

A.4  01 mark each for error (i), (ii) and (iii) 3.0
 Error (iv) 2.0
 Error (v) 3.0

A.5  Computation of carrying value 1.0


 Computation of fair value less cost to sell 1.0
 Computation of value in use:
– Determining cash flows 4.0
– Calculating present value 2.0

A.6 Marks as mentioned on the question paper against each MCQ 10.0

A.7  Cash flow from operating activities:


– Profit 1.5
– Adjustments for 3.5
– Changes in working capital 3.5
 Cash flow from investing activities 3.0
 Cash flow from financing activities 2.0
 Presentation 1.5

A.8  EPS for 2021 originally reported in 2022:


– Basic EPS 2.0
– Diluted EPS 2.5
 EPS for 2022:
– Basic EPS 4.0
– Diluted EPS 5.0
 EPS for 2021 restated for reporting in 2022 1.5

Page 1 of 2
FINANCIAL ACCOUNTING AND REPORTING - I
Summary of Marking Key
Certificate in Accounting and Finance – Autumn 2022

Marks
A.9  Property, plant and equipment:
– Appropriate form of the schedule 2.0
– Opening balances 1.5
– Additions and reclassification 2.0
– Depreciation 2.5
– Revaluation adjustments 3.5
– Other disclosures 2.5
 Investment property:
– Appropriate form of the schedule 1.0
– Additions and reclassification 2.0
– Fair value adjustments 2.0
– Other disclosures 1.0

(THE END)

Page 2 of 2
Certificate in Accounting and Finance Stage Examination

The Institute of 14 March 2023


Chartered Accountants 3 hours – 100 marks
of Pakistan Additional reading time – 15 minutes

Financial Accounting and Reporting-I


Instructions to examinees:
(i) Answer all NINE questions.
(ii) Answer in black pen only.
(iii) Multiple Choice Questions must be answered in answer script only.

Section A

Q.1 Following information relating to Akkadian Limited (AL) has been gathered for the purpose
of calculating earnings per share:

(i) Outstanding ordinary, preference and potential shares of AL as at 1 January 2021:


 6 million ordinary shares having par value of Rs. 10 each.
 2 million irredeemable preference shares having par value of Rs. 20 each carrying
cumulative dividend of 14% per annum.
 3 million share options for ordinary shares having exercise price of Rs. 36 each.
(ii) On 1 September 2021, AL announced 40% right shares to its ordinary shareholders at
Rs. 18 per share. The entitlement date of right shares was 1 October 2021. The market
price per share immediately before the announcement date and entitlement date were
Rs. 30 and Rs. 32 respectively.
(iii) The average market price of ordinary shares during the years 2021 and 2022 were
Rs. 30 and Rs. 45 per share respectively.
(iv) Profits for the years 2021 and 2022 amounted to Rs. 24 million and Rs. 34 million
respectively.
(v) No dividend was declared in the years 2021 and 2022.
(vi) Share options were not exercised in the years 2021 and 2022.

Required:
Compute AL’s basic and diluted earnings per share to be disclosed in the statement of profit
or loss for the years ended 31 December 2021 and 2022. (10)

Q.2 Discuss how the following should be dealt with in the financial statements of relevant entities
according to IAS 20:

(a) A government grant of Rs. 25 million was received by an entity in 2022 for the damage
to its head office building caused by the flood in December 2021. As a result of damage,
an impairment loss of Rs. 21 million was recognised in 2021. (02)

(b) A manufacturing entity established a plant in an area with high illiteracy rate and
received a government grant of Rs. 40 million. The grant received was equivalent to
two years’ salaries of the 50 local persons employed by the entity. The grant is repayable
in full if the number of these employees falls below 50 at any time during the next
five years. It is highly probable that the entity will comply with the condition attached
to the grant. (03)

(c) Government built an alternate road to the industrial zone, in which an entity’s factory
is situated. The new road has reduced the distance to the market and would result in an
annual saving of transportation costs of Rs. 3 million for the entity. (03)
Financial Accounting and Reporting-I Page 2 of 6

Q.3 You are working as the finance manager of Hittite Limited (HL). A new CFO has joined HL
and has recommended changes to accounting policies related to assets to improve HL’s
financial ratios in the next financial statements. The CFO has suggested the following changes
to the policies:

(i) Subsequent measurement of investment property from cost model to fair value model.
(ii) Subsequent measurement of property, plant and equipment from cost model to
revaluation model.
(iii) Cost formula for inventory from weighted average to FIFO method.

You may assume that:


 fair values / cost / prices of all assets would increase over the time.
 the maximum possible amount from the revaluation surplus to retained earnings would
be transferred on an annual basis.
 periodic inventory system is followed by HL.

Required:
State the effect (increase, decrease, no effect) of each of the above changes on the ratios in the
next financial statements. (Note: Use the following format)

Change in policy of
Ratios investment property, plant
inventory
property and equipment
Net profit to sales ratio
Return on assets
Return on capital employed
Debt equity ratio
Current ratio (10)

Q.4 On 1 July 2019, Sumerian Limited (SL) purchased a manufacturing plant for Rs. 570 million.
The plant is being depreciated at a rate of 15% per annum using the reducing balance method.
On 31 December 2021, the remaining life of the plant was estimated at 4 years resulting in an
increase of 5% in depreciation rate.

SL carried out impairment testing of the plant on 31 December 2021 and also on
31 December 2022 using the following estimates:

31 Dec 2021 31 Dec 2022


----- Rs. in million -----
Annual inflows from the sale of product 245 263
Annual outflows for operations 167 174
Annual interest on loan obtained for plant acquisition 14 14
Net sales proceeds at the end of useful life in current condition 142 140
Additional sale proceeds at the end of useful life if plant is
125 125
modified at cost of Rs. 50 million
Current fair value less cost to sell 300 280

Applicable discount rate 12% 10%

Required:
Calculate the carrying value of the manufacturing plant as at 31 December 2021 and 2022. (08)
Financial Accounting and Reporting-I Page 3 of 6

Q.5 On 1 March 2022, Inca Empire Limited (IEL) commenced business with a capital of
Rs. 60,000 which was used to purchase two items of inventory. Details of their cost and sales
for the year ended 28 February 2023 are as follows:

Cost Sale
----------- Rupees -----------
Product A 25,000 55,000
Product B 35,000 70,000

Additional information:
(i) General inflation during the year is 8%.
(ii) Inflation specific to product A during the year is 12%.
(iii) Replacement cost of the product B at the end of the year is Rs. 45,000.

Required:
Prepare the statement of profit or loss and the statement of financial position (equity portion
only) of IEL according to the concept of ‘Physical Capital Maintenance’. (04)

Q.6 Select the most appropriate answer(s) from the options available for each of the following
Multiple Choice Questions.

(i) Alpha Limited made a profit before tax of Rs. 80,000 in the year just ended after
charging depreciation of Rs. 75,000. There was a gain of Rs. 25,000 on disposals of
property, plant and equipment. Net working capital excluding cash increased by
Rs. 19,000. Income tax paid during the year was Rs. 24,000.

What is the amount of cash generated from operations?

(a) Rs. 87,000 (b) Rs. 111,000 (c) Rs. 125,000 (d) Rs. 148,000 (02)

(ii) A company’s cash balances have increased from last year. Which of the following
events could account for this?

(a) Delayed payments by debtors


(b) Shortening of the credit period by the creditors
(c) Acquisition of a long-term loan at high interest rate
(d) A decrease in final dividend proposed by the directors (01)

(iii) Which of the following statements is/are correct?

(I) Statement of cash flows is useful in assessing the ability of the entity to generate
cash and cash equivalents.
(II) Historical cash flows are often a fairly reliable indicator of the amount, timing
and certainty of the future cash flows.

(a) Both are correct (b) Only (I) is correct


(c) Only (II) is correct (d) None is correct (01)

(iv) An entity reported a positive earnings per share in previous year. Which of the
following would result in increase in earnings per share of previous year due to
restatement?

(a) Right issue (b) Bonus issue


(c) Share split (d) Share consolidation (01)

(v) Quick and current ratios of a business as on 31 December were 1:1 and 1.25:1. If
inventories at that date amounted to Rs. 45 million, then current liabilities were:

(a) Rs. 144 million (b) Rs. 225 million


(c) Rs. 180 million (d) Rs. 135 million (01)
Financial Accounting and Reporting-I Page 4 of 6

(vi) After the preparation of the draft financial statements, it was discovered that inventory
items lost in a fire incident were ignored altogether. If the entity follows periodic
inventory system, what would be the effect of the correction?

Gross profit Net profit


(a) Increase No impact
(b) Decrease Decrease
(c) No impact No impact
(d) Increase Decrease (01)

(vii) Which of the following falls under the definition of investment property?

(a) Owner occupied property awaiting disposal


(b) Property occupied by an employee
(c) Land held for undetermined use
(d) Property held for future development and subsequent use as owner-occupied
property (01)

(viii) Which of the following statements is/are correct?

(I) Earnings per share amounts should not be presented if they are negative i.e. losses
per share.
(II) Earnings per share amounts calculated for discontinued operations must be
presented on the face of the statement of profit or loss.

(a) Both are correct (b) Only (I) is correct


(c) Only (II) is correct (d) None is correct (01)

(ix) Which TWO of the following may appear in the operating cash flows?

(a) Increase in depreciation expense


(b) Interest received
(c) Dividend paid
(d) Sale proceeds from disposal of property, plant and equipment (01)

Section B

Q.7 Roman Limited (RL) has extracted the following information for the purpose of preparation
of statement of changes in equity for the year ended 31 December 2022:

2022 2021 2020


Draft Audited Audited
--------- Rs. in million ---------
Net profit 285 195 177
Revaluation surplus arising during the year - 115 (78)
Transfer of incremental depreciation 30 26 28

Additional information:
(i) On 1 February 2021, a bonus issue of 10% was made as final dividend for 2020.
(ii) On 15 May 2021, RL issued right shares for Rs. 20 per share. Right shares were issued
in a proportion of 1 right share for every 4 ordinary shares held. Transaction cost of
Rs. 0.5 per share was also incurred.
(iii) On 1 May 2022, an item of property, plant and equipment was disposed of at its
carrying value. An amount of Rs. 75 million was remaining in the revaluation surplus
account in respect of this item’s previous revaluations.
(iv) On 1 July 2022, 50 million irredeemable preference shares having par value Rs. 10 each
were issued at Rs. 15 per share.
(v) In October 2022, an interim 5% cash dividend on all shares was made.
Financial Accounting and Reporting-I Page 5 of 6

(vi) The revalued amount of RL’s head office building was determined as Rs. 400 million
as on 31 December 2021. However, revaluation was not incorporated as the change in
revalued amount was considered to be temporary by RL’s management. The head
office building had a carrying value of Rs. 350 million on 31 December 2021 and had
a remaining useful life of 10 years. A revaluation loss of Rs. 24 million was recorded
on 31 December 2019 on its previous revaluation.
(vii) Share capital and reserves as at 1 January:
2021 2020
------ Rs. in million ------
Ordinary share capital (Rs. 10 each) 800 800
Retained earnings 715 510
Revaluation surplus 399 505

Required:
Prepare RL’s statement of changes in equity for the year ended 31 December 2022 along with
comparative figures. (Column for total is not required) (15)

Q.8 Aztec Sports Club (ASC) was formed on 1 January 2021 when a founding member sold a
piece of land to ASC having fair value of Rs. 4,000,000 for the purpose of establishing a sports
club, for Rs. 1,000,000 only. The following information is available for the preparation of
financial statements of ASC for the year ended 31 December 2022:
(i) Balances of some assets and liabilities as on 1 January 2022:
Rs. in '000
Cash and bank balances 223
Fixed assets (other than land) 6,450
Prepaid insurance 274
Accrued other expenditures 865

(ii) Payments made during the year:


Rs. in '000
Fixed assets (on 1 May 2022) 6,000
Annual insurance (valid till 31 March 2023) 1,404
Other expenditures 2,788

(iii) Annual membership fee for the years 2021, 2022 and 2023 was Rs. 8,000, Rs. 10,000
and Rs. 12,000 respectively. However, members joining in second half of year are
charged only half fee for that year. Each member is required to pay the membership fee
for the current year and the next year at the time of admission. The numbers of
members admitted during the years 2021 and 2022 are as follows:

2021 2022
1st half 2nd half 1st half 2nd half
150 270 220 105

(iv) Contributions received during the year:


 A member contributed Rs. 1,400,000 for the purchase of a tractor for ground’s
maintenance. The tractor will be purchased in the year 2023.
 Another member contributed Rs. 1,100,000 without specifying any restriction.
(v) On 1 April 2021, an area was given on rent for operating a canteen in the club at an
annual rent of Rs. 840,000. However, to facilitate the tenant for setting up the canteen,
it was agreed that the rent for 2 years will be paid in 2023.
(vi) On 1 September 2022, some fixed assets having book value of Rs. 3,000,000 on
1 January 2022 were disposed of for Rs. 3,300,000.
(vii) Depreciation is charged on all fixed assets (other than land) using reducing balance
method at a rate of 20% per annum.
Financial Accounting and Reporting-I Page 6 of 6

Required:
Prepare the following using the deferral method:
(a) Statement of income and expenditure for the year ended 31 December 2022 (09)
(b) Statement of financial position as at 31 December 2022 (09)

Q.9 Following information pertains to non-current assets of Mesopotamia Limited (ML):

(i) On 1 July 2019, ML acquired a warehouse at a cost of Rs. 300 million and was
immediately given on rent to a third party. On 1 January 2022, ML commenced the
development work on its warehouse with a view to put it in own use. The development
work was completed on 31 March 2022 at a cost of Rs. 50 million. ML started using
the warehouse for its inventory on 1 May 2022. Fair value of the warehouse on various
dates are as follows:
31 Dec 2020 31 Dec 2021 31 Mar 2022 31 Dec 2022
Rs. in million 316 344 352 366
Depreciation is charged on warehouse at a rate of 10% per annum using the reducing
balance method.

(ii) On 1 January 2020, ML purchased a heavy duty vehicle for Rs. 360 million. On
purchase date, the vehicle had an estimated useful life and residual value of 5 years
and Rs. 72 million respectively.
During 2022, ML has decided to change the depreciation method for vehicles from
reducing balance to straight line.
(iii) On 1 June 2021, ML started construction of an office building. The building was
available for use on 1 October 2022 and was immediately put into use. Details of the
construction costs incurred are as under:

Payment date Rs. in million Sources (See below)


1 May 2021 140 A
1 January 2022 *100 A&B
1 April 2022 70 C
1 August 2022 160 D
470
*The bill from the contractor was received on 1 December 2021.
These payments were financed through the following sources:
(A) A short term loan of Rs. 200 million obtained on 1 April 2021 from Bank A at the
rate of 16% per annum. The surplus funds available from the loan were invested
in a saving account at 10% per annum. On 1 March 2022, ML repaid the loan
using the proceeds received from a right issue of shares.
(B) Excess cash available with ML in current bank accounts.
(C) Withdrawals from its short term investments earning a profit of 12% per annum.
(D) Withdrawals from a running finance facility from Bank B carrying interest at
14% per annum. The facility is also used for working capital needs.

Depreciation is charged on office building using straight line method over the
estimated useful life of 20 years.

Additional information:
 Cost model is used for subsequent measurement of all property, plant and equipment.
 Fair value model is used for subsequent measurement of all investment properties.

Required:
Prepare relevant extracts (including comparative figures) from ML’s statement of profit or
loss for the year ended 31 December 2022 and statement of financial position as on that date. (17)
(THE END)

You might also like